Sei sulla pagina 1di 327

Heyson’s Notes for 

History Taking, Clinical Examinations and Case Studies 
By Heyson Chan 

COMPILED BY ICU 2013 
History Taking Contents 
ABDOMINAL PAIN ..................................................................................................................................................... 6 
ASTHMA .................................................................................................................................................................... 8 
BLEEDING TENDENCY ............................................................................................................................................... 13 
CHEST PAIN ............................................................................................................................................................. 14 
CHRONIC RENAL FAILURE ......................................................................................................................................... 17 
COUGH .................................................................................................................................................................... 21 
DIABETES ................................................................................................................................................................. 25 
DIARRHEA ............................................................................................................................................................... 29 
DIPLOPIA ................................................................................................................................................................. 32 
DIZZINESS ................................................................................................................................................................ 34 
DYSPHAGIA ............................................................................................................................................................. 36 
EDEMA .................................................................................................................................................................... 38 
EPILEPSY .................................................................................................................................................................. 40 
FALL ........................................................................................................................................................................ 44 
FEVER ...................................................................................................................................................................... 46 
HAEMATURIA .......................................................................................................................................................... 48 
HAEMOPTYSIS ......................................................................................................................................................... 50 
HEADACHE .............................................................................................................................................................. 53 
HEMIPLEGIA ............................................................................................................................................................ 56 
HYPERTENSION (SECONDARY) ................................................................................................................................. 60 
JAUNDICE ................................................................................................................................................................ 64 
JOINT PAIN .............................................................................................................................................................. 66 
MALAISE .................................................................................................................................................................. 68 
OSA ......................................................................................................................................................................... 70 
PALPITATION ........................................................................................................................................................... 72 
PTOSIS ..................................................................................................................................................................... 74 
RHEUMATOID ARTHRITIS ......................................................................................................................................... 76 
SOB ......................................................................................................................................................................... 79 
SYNCOPE ................................................................................................................................................................. 82 
TREMOR .................................................................................................................................................................. 84 
UGIB ........................................................................................................................................................................ 86 
VERTIGO .................................................................................................................................................................. 89 
WEAKNESS .............................................................................................................................................................. 90 
WEIGHT LOSS .......................................................................................................................................................... 94 

Page 2 By Heyson Chan


Clinical Examination Contents 
ABDOMEN EXAMINATION ....................................................................................................................................... 96 
BREAST EXAMINATION .......................................................................................................................................... 104 
CARDIOVASCULAR EXAMINATION ......................................................................................................................... 112 
EXAMINATION OF HANDS ...................................................................................................................................... 122 
HERNIA EXAMINATION .......................................................................................................................................... 126 
LN EXAMINATION .................................................................................................................................................. 131 
LUMP AND BUMP .................................................................................................................................................. 133 
O&T PHYSICAL EXAM ............................................................................................................................................. 135 
SHOULDER ....................................................................................................................................................................... 135 
ELBOW ........................................................................................................................................................................... 138 
HAND & WRIST ............................................................................................................................................................... 139 
HIP ................................................................................................................................................................................ 141 
KNEE .............................................................................................................................................................................. 143 
FOOT & ANKLE ................................................................................................................................................................ 146 
SPINE ............................................................................................................................................................................. 150 
TUMOR ........................................................................................................................................................................... 154 
X‐RAY ............................................................................................................................................................................ 155 
PAROTID GLANDS .................................................................................................................................................. 158 
PROSTATE EXAMINATION ...................................................................................................................................... 160 
RESPIRATORY EXAM .............................................................................................................................................. 161 
SCROTUM EXAMINATION ...................................................................................................................................... 172 
THYROID EXAMINATION ........................................................................................................................................ 173 
VASCULAR EXAMINATION ..................................................................................................................................... 183 

Page 3 By Heyson Chan


Case Studies Contents 
ACOUSTIC NEUROMA ............................................................................................................................................ 191 
ACROMEGALY ....................................................................................................................................................... 192 
ACUTE CORONARY SYNDROME .............................................................................................................................. 194 
AORTIC REGURGITATION ....................................................................................................................................... 196 
AORTIC STENOSIS .................................................................................................................................................. 198 
ARRHYTHMIA ........................................................................................................................................................ 200 
ASCITES ................................................................................................................................................................. 205 
ATRIAL FIBRILLATION ............................................................................................................................................. 207 
BRONCHIECTASIS ................................................................................................................................................... 209 
BROWN‐SEQUARD SYNDROME .............................................................................................................................. 211 
BULLOUS ERUPTION .............................................................................................................................................. 212 
CEREBELLAR DISEASE ............................................................................................................................................. 214 
CHARCOT MARIE TOOTH DISEASE .......................................................................................................................... 216 
CN3 PALSY ............................................................................................................................................................. 217 
CN6 PALSY ............................................................................................................................................................. 219 
CN7 PALSY ............................................................................................................................................................. 220 
CNS INFECTION ...................................................................................................................................................... 222 
CONN’S SYNDROME .............................................................................................................................................. 225 
COPD ..................................................................................................................................................................... 226 
CT BRAIN ............................................................................................................................................................... 229 
CUSHING’S SYNDROME .......................................................................................................................................... 231 
DIABETES INSIPIDUS .............................................................................................................................................. 234 
DM RETINOPATHY ................................................................................................................................................. 235 
NOTES ON ECG ...................................................................................................................................................... 238 
FIBROSING ALVEOLITIS .......................................................................................................................................... 240 
GOUT .................................................................................................................................................................... 242 
HEART FAILURE ..................................................................................................................................................... 244 
HEPATOMEGALY .................................................................................................................................................... 247 
HYPERCALCEMIA ................................................................................................................................................... 250 
HYPERKALEMIA ..................................................................................................................................................... 253 
HYPERLIPIDEMIA ................................................................................................................................................... 255 
HYPERNATREMIA .................................................................................................................................................. 257 
HYPOCALCEMIA ..................................................................................................................................................... 258 
HYPOKALEMIA ....................................................................................................................................................... 259 
HYPONATREMIA .................................................................................................................................................... 260 

Page 4 By Heyson Chan


L5 ROOT COMPRESSION ........................................................................................................................................ 262 
LATERAL MEDULLARY SYNDROME ......................................................................................................................... 264 
LUNG COLLAPSE .................................................................................................................................................... 266 
LUNG MASS ........................................................................................................................................................... 268 
MITRAL REGURGITATION ....................................................................................................................................... 269 
MITRAL STENOSIS .................................................................................................................................................. 270 
MIXED AORTIC VALVE LESION ................................................................................................................................ 271 
MIXED MITRAL VALVE LESION................................................................................................................................ 272 
MOTOR NEURON DISEASE ..................................................................................................................................... 273 
MYASTHENIA GRAVIS ............................................................................................................................................ 274 
MYOSITIS .............................................................................................................................................................. 276 
NEUROLOGICAL LOCALIZATION .............................................................................................................................. 278 
LIMB WEAKNESS .................................................................................................................................................... 278 
OLD POLIO ............................................................................................................................................................. 280 
OPTIC ATROPHY .................................................................................................................................................... 282 
PARKINSONISM ..................................................................................................................................................... 285 
MULTIPLE SYSTEM ATROPHY ................................................................................................................................. 287 
PLEURAL EFFUSION ................................................................................................................................................ 290 
PHYSICAL EXAMINATION ....................................................................................................................................... 290 
PNEUMONIA ......................................................................................................................................................... 292 
PNEUMOTHORAX .................................................................................................................................................. 295 
POLYCYSTIC KIDNEY DISEASE ................................................................................................................................. 297 
PROLACTINOMA .................................................................................................................................................... 299 
PSORIASIS ............................................................................................................................................................. 302 
PULMONARY FIBROSIS .......................................................................................................................................... 303 
RETINITIS PIGMENTOSA ......................................................................................................................................... 307 
RHEUMATOID ARTHRITIS ....................................................................................................................................... 308 
SCLERODERMA ...................................................................................................................................................... 310 
SPINAL CORD COMPRESSION ................................................................................................................................. 312 
SPLENOMEGALY .................................................................................................................................................... 314 
STROKE ................................................................................................................................................................. 316 
TRANSPLANTED KIDNEY......................................................................................................................................... 320 
TRICUSPID REGURGITATION .................................................................................................................................. 322 
URINARY TRACT INFECTION ................................................................................................................................... 323 
VALVULAR REPLACEMENT ..................................................................................................................................... 325 
VSD ....................................................................................................................................................................... 326 
 

Page 5 By Heyson Chan


Abdominal Pain 
HPI
Abdominal pain
 Onset
 Progression
 Duration (on and off or continuous? For how long?)
 Site (show me the region)
 Radiation
 Character
 Aggravating factors (e.g. oily food)
 Relieving factors (use of analgesics / rest)
 Severity: how severe it was? Does it wake the patient from sleep?
 Any previous episodes before? Frequency and duration

Associate symptoms
 Any jaundice (biliary obstruction)
 Any tea-colored urine / clay-colored stool (biliary obstruction)
 Any fever / chills / rigors (sepsis e.g. from biliary; penumonia)
 Any diarrhea / vomiting (IBD, GE, pancreatitis)
 Any change of bowel habits (colorectal CA); if acute constipation: BO
 Any blood / pus in stool (IBD, CA)
 Any weight loss (IBD, CA, chronic infection)
 Any coffee ground / tarry stool (GIB)
 Urinary symptoms: frequency, color of urine, dysuria, loin pain (UTI / renal stone)
 In female: LMP and sexual history (gynecological e.g. ruptured ectopic)
 Any cough / sputum (lower lobe pneumonia)
 Any SOB (metabolic acidosis e.g. DKA)
 Any recent trauma / injury (musculo-skeletal)
 TOCC history

PMH
 History of gallstones, PU, AAA
 Chronic renal failure requiring CAPD (CAPD peritonitis)
 Hepatitis vaccine
SHx
 Smoking, drinking
 Occupation
 Sexual history
FHx
 Hepatitis status / complications e.g. cirrhosis, HCC
DHx
 NSAID / aspirin (specifically ask the patient if suspect GIB)
 Other medication

Page 6 By Heyson Chan


DDx
 GI: PU / hepatitis / cholangitis / cholecystitis / IBD / pancreatitis / BO / diverticulitis /
constipation / peritonitis
 Cardiac: MI / angina / AAA
 Respi: lower lobe pneumonia
 Musculo: injury / herpes zoster
 Psychological

Page 7 By Heyson Chan


Asthma
HPI
Acute asthmatic attack
SOB
 Onset
 Progression
 Aggravating factors: exercise / change of weather / allergens
 Relieving factors: response to bronchodilator
 Severity: can you speak a whole sentence?

Associate symptoms (SOB)


 Any cough / sputum / fever (chest infection)
 Any chest pain (pneumothorax)
 Any noisy breathing (Asthma / airway obstruction)
 Any recent weight loss / night sweat
 Any palpitation / edema / PND / orthopnea (heart failure)
 Any postural hypotension / malaise / potential source of bleeding (anemia)
 TOCC history

Previous history
 Age of first attack
 Progression
 Aggravating factors
 Any relationship with works

Control
 Frequency of attacks / hospitalization
 Exercise tolerance
 Regular home PEFR monitoring
 Nocturnal cough
 Frequency of use of bronchodilators
 Any exercise induced attacks

Treatment, side effects and knowledge


 Knowledge on simple management plan during acute attack
 Knowledge on the difference between reliever and controller and their side effects
 Frequency and method of administration
 Technique of inhalation
 Bronchodilator – tremor / palpitation
 Steroid– oral thrush / hoarseness of voice

Environmental history (enquire knowledge on risk factors and assess whether allergen
avoidance ispossible)
 Pets
 Carpets
 2nd hand smoke
 Incant burning
 Nearby construction site / roads with heavy traffic

Page 8 By Heyson Chan


PMH
 Atopic history – allergic rhinitis / eczema
SHx
 Smoking
 Occupation  any relation with works?
 Environmentalhistory– pets / carpets / 2nd hand smoke / incant burning

FHx
 Atopic history
DHx
 Allergy (food and drug) / regular use of medication

Questions
1 What is your immediate management plan for a patient presented with acute asthma?
 ABC, bed rest
 O2 supplement, keep SaO2 >90%
 Monitor BP/P/T; SaO2
 Check RFT, ABG, PEFR
 CXR (to look for pneumothorax)
 Bronchodilator (Salbutamol 5 mg) 5 puff
 Oral steroid (prednisolone 30-60mg po) – taper off gradually after acute attack
 Consider intubation and ICU admission if life-theratening

2 What are the signs suggesting severe / life-threatening acute attack?

Moderate Severe Life-threatening


Talking In full sentence Can’t speak full Can’t speak at all
sentence
RR >25 >30 >30
SaO2 >90% <90% <90%
Pulse >110 >120 <60
BP Normal Normal Hypotension
PEFR 60-80% <60% <33%
Cyanosis Absent Absent Present
Auscultation Wheeze Wheeze Silent chest
Respiratory Increased Severely Feeble
effort increased (use of
accessory muscle)
Confusion / coma Absent Absent Present
/ exhaustion
Chest movement Normal Normal Paradoxical
ABG Normal pH Normal pH Low pH

Page 9 By Heyson Chan


3 What is your management plan for chronic control of asthma?
For all steps: education, avoid or control triggers at all times
Features Management
Step 4  Continuoussymptoms  Inhaled short-acting
Severe persistent limiting physical beta-2 agonist prn
activity  Daily inhaled steroid
 Frequent night time plus inhaled LABA
symptoms  Long-term oral steroid
 PEFR >60% or >30%
variability
Step 3  Daily symptoms  Inhaled short-acting
Moderate persistent  Daily use of beta-2 beta-2 agonist prn
agonist  Daily inhaled steroid
 Attacks affect activity plus inhaled LABA
 >1 night time symptoms  Consider anti-
/ week leukotrienes for
exercise-induced asthma
or aspirin sensitivity
Step 2  >1 time / week but <1  Inhaled short-acting
Mild persistent time / day beta2-agonist prn
 >2 night time symptoms  Daily inhaled steroid
/ month  Anti-leukotrienes may
 >80% predicted within be used
day variability 20-30%
Step 1  <1 time / week  Inhaled short-acting
Intermittent  <2 night time symptoms beta2-agonist prn
/ month  Inhaled short-acting
 Normal PEFR between beta2-agonist before
attacks exercise / allergen
exposure

Step-down: review treatment every 3-6 months. If control has been sustained >3 months,
consider a gradual stepwise reduction
Step-up: if control is not achieved, consider stepping up after reviewing patient’s medication
technique, compliance and environmental control (avoidance of allergens / trigger factors)

GINA 2006
Alternative reliever treatments include inhaled anticholinergics, short-acting oral
2-agonists, some long-acting 2-agonists, and short-acting theophylline. Regular
dosing with short and long-acting 2-agonist is not advised unless accompanied by
regular use of an inhaled glucocorticosteroid.

4 What investigations would you perform in a patient with suspected occupational asthma?
 Confirm relationship of attacks with works (PEFR on working and rest days)
 IgE antibody assay
 Skin prick test
 Bronchial hyper-responsiveness demonstrated by inhaled histamine bronchial challenge test (also
performed if equivocal clinical features and in research)

Page 10 By Heyson Chan


5 Compare and contrast early-onset and late-onset asthma.
Early-onset Late-onset
Atopic history Positive Negative
Steroid dependence Less steroid dependent More steroid dependent
Severity Less severe More severe

6 Compare and contrast COPD and asthma.


Asthma COPD
Age of onset Young Older
Atopic history Present Absent
Smoking history Absent Present (chronic smoking)
Response to Response >20% and greater Response <20%
bronchodilator than 0.2L on lung function
(reversibility) test
Family history Positive Negative

7 According to GINA 2006, define different level of asthma control.


Characteristic Controlled Partly Controlled Uncontrolled
(All of the (Any measure
following) present in any
week)
Daytime None (twice or More than Three or more
symptoms less/week) twice/week features of partly
Limitations of None Any controlled
activities asthma
present in any
Nocturnal None Any week
symptoms or
awakening
Need for reliever/ None (twice or More than
rescue treatment less/week) twice/week

Lung function Normal <80% predicted or


(PEF or FEV1) personal test

Exacerbations None Once or more / year One in any week

8 What is the treatment guideline from GINA 2006?


Step 1 – As-needed reliever medication
 Patients with occasional daytime symptoms of short duration
 A rapid-acting inhaled β2-agonist is the recommended reliever treatment (Evidence A)
 When symptomsare more frequent, and/or worsen periodically, patients require regular
controller treatment (step 2 or higher)

Step 2 – Reliever medication plus a single controller


 A low-dose ICS is recommended as the initial controller treatment for patients of all ages
(Evidence A)
 Alternative controller medications include leukotriene modifiers (Evidence A) appropriate
for patients unable/unwilling to use ICS

Page 11 By Heyson Chan


Step 3 – Reliever medication plus one or two controllers
 For adults and adolescents, combine a low-dose ICS with an inhaled long-acting β2-agonist
(LABA) either in a combination inhaler device or as separate components (Evidence A)
 Inhaled LABA must not be used as monotherapy
Step 4 – Reliever medication plus two or more controllers
 Selection of treatment at Step 4 depends on prior selections at Steps 2 and 3
 Where possible, patients not controlled on Step 3 treatments should be referred to a health
professional with expertise in the management of asthma
 Medium- or high-dose ICS combined with a LABA (Evidence A)
 Medium- or high-dose ICS combined with leukotriene modifiers (Evidence A)
 Low-dose sustained-release theophylline added to medium- or high-dose ICS combined with
a LABA (Evidence B)
Step 5 – Reliever medication plus additional controller options
 Addition of oral glucocorticosteroids to other controller medications may be effective
(Evidence D) but is associated with severe side effects (Evidence A)
 Addition of anti-IgE treatment to other controller medications improves control of allergic
asthma when control has not been achieved on other medications (Evidence A)

9 When would you consider stepping up / stepping down the treatment?

Stepping down
 When controlled on medium- to high-dose ICS: 50% dose reduction at 3 month intervals
(Evidence B)
 When controlled on low-dose ICS: switch to once-daily dosing (Evidence A)
 When controlled on combination ICS and LABA, reduce dose of inhaled glucocorticosteroid
by 50% while continuing the LABA (Evidence B)
 If control is maintained, reduce to low-dose ICS and stop LABA (Evidence D)

Stepping up
 Rapid-onset, short-acting bronchodilator or LABA provide temporary relief.
 Need for repeated dosing over more than one/two days signals need for possible increase in
controller therapy
 Use of a combination rapid and LABA (e.g., formoterol) and an ICS (e.g., budesonide) in a
single inhaler both as a controller and reliever is effecting in maintaining a high level of
asthma control and reduces exacerbations (Evidence A)
 Doubling the dose of ICS is not effective, and is not recommended (Evidence A)

Page 12 By Heyson Chan


Bleeding Tendency 
HPI
Bleeding Tendency
 Onset
 Progression
 Site
 Petechiae
 Ecchymosis
 Epistaxis
 Menorrhagia
 Gum bleeding
 Haematuria
 Melena
 Bleeding into joint
 Bleeding into muscles
 Type of bleeding
 Immediate
 Delayed bleeding (e.g. when does it bleed when you have a cut)
 Severity
 Extent of bruising
 History of stroke
 Anemic symptoms
 Any fever / shock (DIC)
 Any malaise / weight loss (leukemia)
 Any cough / sputum / night sweat (TB bone)
PMH
 History of bleeding tendency
SHx
 Smoking, drinking
 Occupation
 Hobbies (especially contact sports)
FHx
 Family history of bleeding tendency
DHx
 Anti-coagulants – enquire about compliance and diet
 Allergy

Page 13 By Heyson Chan


Chest Pain 
HPI
Chest pain
 Onset
 Progression
 Duration
 Site
 Radiation
 Character
 Aggravating factors (e.g. exertion – quantify e.g. how many flight of stairs?)
 Relieving factors (use of analgesics / rest – for how long?)
 Severity: how severe it was? Does it wake the patient from sleep? How does it affect the patient’s
daily activities (functional status)?
 Any previous episodes before? Frequency and duration

Associate symptoms
 Any sweating / nausea / vomiting (sympathetic overdrive)
 Any palpitation
 Any edema / PND / orthopnea (heart failure)
 Any cough / sputum / fever (chest infections)
 Any SOB – quantify ET (pneumothorax, heart failure, chest infections)
 Any reflux / heartburn, association with meals (GERD)
 Any tarry stool / coffee-ground (PU)
 Any recent trauma / injury (musculo-skeletal)
 Any rash (herpes zoster)
 Tender on touch? (costochondritis)

PMH
 Previous hemorrhagic stroke / use of thrombolytics / intracranial tumor (prepare for thrombolytic
treatment)
 Cardiac risk factors (HT / DM / dyslipidemia – control), cardiac disease
 Recent infection (pericarditis)
 TB

SHx
 Smoking, drinking
 Diet, exercise
 Occupation – family income
 Any financial difficulties (implicated if CABG needed)

FHx
 CVD and age of onset

DHx
 Allergy / regular use of medication

Page 14 By Heyson Chan


Objectives
1 Presenting complaints
2 Severity (ET, SOB, functional status)
3 Associate symptoms (cardiac, respi, GI, MS)
4 Cardiac risk factors
5 Prepare for thrombolytics
6 Social set-up

DDx
 Cardiac: MI, angina, aortic dissection, pericarditis
 Respi: pneumonia, pneumothorax, PE
 GI: PU, GERD
 Musculo: injury / # rib / costochondritis / herpes zoster
 Psychological

Questions
1 What is your management plan in a patient presented with ST-elevated myocardial
infarction?

General
 Admit CCU
 Close monitor vitals
 O2 supplement with 4-6L/min
 Relieve anxiety with adequate explanation an support
 CBC, L/RFT, lipid profile (taken within 24 hours), CXR
 Serial ECG and cardiac markers (TnT, CPK) for 3 days
 Clotting profile to prepare for thrombolysis
Specific
 Give adequate anagelsic (morphine 2-5mg iv with RR monitor)
 Give stool softener
 Give aspirin 300 mg chewed
 Give ACEI captopril 6.25mg Q8H or enalapril 5mg bd or acertil 2mg daily
 Give beta-blocker (avoid in inferior infarct due to possible involvement of AV node as AV
node is supplied by right coronary artery)
 Give statin
 Consider streptokinase 1.5 MU if ST elevated MI presented within 3 hours
 Consider CABG if thromoblytics contraindicated

2 What are the contraindications of thrombolysis?


 Active internal bleeding (incl haemorrhagic CVA, active GIB)
 Suspected aortic dissection or pericarditis
 Prolonged or traumatic CPR
 Recent head trauma or intracranial tumour
 Haemorrhagic eye condition (e.g. DM haemorrhagic retinopathy)
 Blood pressure > 200/120 mmHg

Page 15 By Heyson Chan


3 What is the management plan for patients with unstable angina?

General
 Admit CCU
 Close monitor vitals
 O2 supplement with 4-6L/min
 Relieve anxiety with adequate explanation an support
 CBC, L/RFT, lipid profile (taken within 24 hours), CXR
 Serial ECG and cardiac markers (TnT, CPK) for 3 days
 Correct underlying causes (e.g. anemia)
 Consider IABP, urgent coronary angiogram / revascularisation in cases refractory to medical
therapy
Specific drug treatment:
 Nitrates (Oral first, iv if unstable esp. with CHF or marked HT)
Begin with lowest dose, step up till pain is relieved
Watch BP/P, keep SBP > 100 mmHg
 ß-blockers (if no C/I)
Metoprolol (Betaloc) 25-100 mg bd
Atenolol (Tenormin) 50-100 mg daily
 Aspirin (soluble or chewed) 160 mg stat & daily
Ticlopidine 250mg bd or Clopidogrel 75mg daily if aspirin is contraindicated
 Calcium Antagonists ( third line agent, non-dihydropyridine type is preferred)
Verapamil 40-120 mg tds
Nifedipine (Adalat) 5-20 mg tds (to be considered only if patient is already on ß-blockers)
 Heparinisation (if pain is not controlled)
 SC low molecular weight (LMW) heparin

4 What are the high-risk features?


 Ongoing or recurrent rest pain
 Hypotension, bradycardia or tachycardia, VT
 ST segment changes >0.1 mV; new bundle branch block
 Elevated Troponin > 0.1 mg/mL)
 CHF

5 What is the after-care for uncomplicated MI?


 Advise on risk factor modification and treatment(Smoking, HT, DM, hyperlipidaemia,
exercise)
 Stress test (Pre-discharge or symptom limited stress 2-3 wks postMI)
 Angiogram if + ve stress test or post-infarct angina
 Drugs for Secondary Prevention of MI
 Beat-blocker: Metoprolol 25-100 mg bd
 Aspirin: 75-300 mg daily
 ACEI (esp for large anterior MI, recurrent MI, impaired LVsystolic function or CHF)e.g.
Lisinopril 5-20 mg daily; Ramipril 2.5-10 mg daily;Acertil 2-8 mg daily
 Lipid lowering drugs (statin) – keep LDL <2.6

Page 16 By Heyson Chan


Chronic Renal Failure 
HPI
Chronic renal failure
Presenting complaints
 Nausea / anorexia / vomiting
 Weight loss
 Malaise
 Pruritis
Renal symptoms
 Urinary pattern: frequency / amount / nocturia
 Obstructive symptoms: urgency / hesistency / terminal dripping / polyuria
 Character of urine: color / haematuria / frothy urine
 Signs of fluid overload – edema, / abdominal distention / SOB
Complications – systemic review
 CVS – chest pain / SOB / BP
 Respi – SOB / chest infection
 GI – nausea / vomiting / dyspepsia / anorexia / gastritis
 GU – libido / menstral history
 Neurological – headache / blunting of affect / weakness (HT  CVA)
 Hematological– anemic symptoms / easy bruising (platelet dysfunction)
 Musculoskeletal – bone pain / fractures (hyperparathyroidism) / weakness / joint pain (gout)
 Skin – itching / pallor / pigmentation
Disease progress and treatment received / possible treatment modalities
 CAPD; cx e.g. peritonitis (frequency)
 Hemodialysis; cx e.g. vascular access
 Results of recent renal function test
 Explore the possibility of hemodialysis
 Impact of dialysis on the patient’s daily living
 Assessment for possibility of kidney transplant
 Blood group
 Family set-up (potential donor)
 Chronic infection

PMH
 Childhood illness / UTI / persistent enuresis
 Previous renal problems / recurrent UTI
 History of hypertension
 Recent sore throat / hematuria
 Systemic disease – DM / nephritis
SHx
 Social set-up
 Occupation
 How does the disease / treatment affect your daily activities / work?
FHx
 Renal disease
 Deafness (Alport’s syndrome)
 DM / metabolic disease
DHx
 Use of analgesics / NSAID / herbs

Page 17 By Heyson Chan


Objectives
1 Presenting complaints
2 Renal symptoms
3 Complications – systemic enquiries
4 Causes – PMH
5 Progress, treatment received and possible treatment modalities
6 Functional status

Causes
 Diabetes
 Glomerulonephritis
 Hypertension and renal vascular disease
 Obstructive nephropathy
 Infection / reflux nephropathy
 Myeloma and other systemic disease
 Others: e.g. polycystic kidneys, hereditary nephropathies
 Renal: GN / infection / reflux
 Vascular: HT / renal vascular disease
 Urolithiasis and obstructive
 Systemic disease: DM / myeloma / hypercalcemia / hyperuricemia
 Hereditary: polycystic kidney / agenesis

Questions
1 What are the common causes of renal failure in Hong Kong?
 Diabetes
 Glomerulonephritis
 Hypertension and renal vascular disease
 Obstructive nephropathy
 Infection / reflux nephropathy
 Myeloma and other systemic disease
 Others: e.g. polycystic kidneys, hereditary nephropathies

Page 18 By Heyson Chan


2
2 Compare haemodialysis and peritoneal dialysis.

Haemodialysis Peritoneal dialysis


Requirements  Good vascular access  Good peritoneal access
 Stable haemodynamics  Patient or helper to perform
CAPD
Advantages  Lower teachnique failure  More biocompatible
rate  Fewer incidence of
 In center monitoring hypotension
 Higher solute clearence (Higher conc of dialysate 
 Good for patients with poor more fluid withdraw e.g in
peritoneal access or CHF patienst)
respiratory reserve  Better control of HT
 Anemia less frequent
 Drugs can be given
intraperitoneally
 More normal lifestyle
Disadvantages  Need frequent visits  Malnutrition due to loss of
 Thrombosis and infection of proteins
vascular access  Peritonitis as major
 More rapid loss of renal complication
function  Higher incidence of
 Activation of complements inadequate dialysis
and cytokines  Catheter problems

3 What are the complications of haemodialysis?


Acute
 Dialysis related hypotension
 Dialysis disequilibrium syndrome leading to cerebral edema
 Arrhythmia
 Acute hemorrhage secondary to heparin
 Technical error e.g. contamination, air emoblism
Chronic

Persistent renal failure


 Hypertension
 Muscle cramps
 Anemia
 Cadiomyopathy
 Peripheral neuropathy
 Dyslipidemia

Vascular Access
 Vascular access may be thrombosed or infected

Page 19 By Heyson Chan


Dialysis
 Infections: hepatitis B or C
 Dialysis associated amyloidosis
 Malnutrition
 Fluid overload

Social
 Impaired quality of life and depression

4 What are the complications of peritoneal dialysis?


Complications as a result of dialysate in abdomen
 Dialysate leakage
 Hydrothorax-pleuroperitoneal communication
 Hernia
 Abdominal pain or back pain
Mechanical complications
 Inadequate drainage
 Perforation of the visceral organs
Medical complications
 Infection: peritonitis, exit-site infections
 Metabolic: protein malnutrition, dyslipidemia, hyperglycemia, loss of water soluble vitamins
 Related to complications of persistent renal failure: anemia, renal osteodystrophy, neuropthy,
hypertension

5 What is the management plan for CAPD peritonitis?


Definition (2 out of 3): cloudy peritoneal effluent WCC>100, demonstration of bacteria,
symptoms of peritonitis (e.g. abdominal pain); relapse = another episode by same organism
within 4 weeks of stopping ABi
 Flush with 2-3 bags of PD fluid
 First line antibiotics for CAPD patients
Vancomycin 1 g/2L ip or iv on day 1 and day 7 + Netilmicin 80 mg/bag ip as loading, then
40 mg/2L PD fluid ip overnight for 9 more doses
Cephazolin or cephalothin 500 mg/l loading dose, maintenance 125 mg/l ip and Tobramycin,
or gentamicin, or netilmicin : 0.6 mg/kg BW ip once daily, or Amikacin 2.0 mg/kg BW ip
once daily
 Heparin ip 100-500 units/L if fibrin present
 Duration of treatment 10 days
 Give parenteral antibiotics if patient has septicaemia
 Monitor glucose control for DM patient
(Organisms: 70% G+ve e.g Staph aureus, epidermidis and viridans; 20% G-ve e.g. E coli,
Enterobacter, pesduomondas associate with catheter loss and damage to peritoneal membrane,
10% others, including fungal)

Page 20 By Heyson Chan


Cough 
HPI
Cough
 Onset
 Progression
 Frequency and duration
 Character of cough
 Aggravating factors
 Relieving factors (ask for use of bronchodilator and response, if any)
 Severity: how severe it was? Does it wake the patient from sleep?
 Any diurnal variation (early signs of inadequate asthma control)
Associate symptoms
 Any sputum (amount, color, haemoptysis) (chest infections, TB, CA, bronchiectasis)
 Any SOB – quantify ET (COPD, asthma, pulmonary fibrosis)
 Any fever (chest infections, COPD)
 Any chest pain (pneumonia)
 Any recent weight loss (TB / CA)
 Any night sweat (TB)
 Any systemic upset: nausea, vomiting, diarrhea (may be present in pneumonia)
 Any palpitation / edema / PND / orthopnea (heart failure)
 Any post-nasal drip (URTI / rhinitis)
 Any reflux feeling and relation with meals (GERD)
 TOCC history

PMH
 COPD / asthma (atopic history as well) / heart failure
 Quantity severity by enquiring number of admissions, exacerbations and types of medications
(including O2 therapy for COPD)

SHx
 Smoking
 Occupation (pneumoconiosis)
 Environmentalhistory for asthma
FHx
 Contact
DHx
 Any use of ACEI / beta-blocker
 Current medication and compliance for asthma / COPD / heart problem, if any
 Allergy / regular use of medication

DDx
Acute
 Foreign body, infection (URTI, pneumonia, COPD exacerbation)
Chronic (>3 weeks)
 Respi: TB / pulmonary fibrosis / asthma / COPD / rhinitis / CA lung / bronchiectasis
 Cardiac: heart failure
 GI: GERD
 Drugs: ACEI

Page 21 By Heyson Chan


Objectives
1 Presenting complaints – including sputum
2 SOB – ET (severity)
3 Respiratory symptoms
4 Cardiac symptoms
5 Constitutional symptoms

Questions
1 What are the causes of bronchiectasis?
Post-infective
 Pneumonia
 Tuberculosis
 Viral
 Measles
 Pertuisis
Post-obstructive
 Tumor
 Foreign body
 Post-TB endobronchitis
 LN compression
Congenital
 Cystic fibrosis
 Primary ciliary dyskinesia
 Kartagener syndrome (associate with dextrocardia and sinus invertus)
 RA, Sjogren syndrome

2 What are the common organisms responsible for acute exacerbation of bronchiectasis?
 H influenza
 Strept pneumoniae
 Staph aureus
 Pseudomonas aeruginosa
 Moraxella catarrhalis

3 What is your treatment plan for acute exacerbation of bronchiectasis?


Investigations
 CBC, L/RFT
 ABG if severe airway obstruction
 Save sputum x C/ST
 CXR
 Diagnosis by high-resolution CT thorax
Treatment
 Chest physio-therapy including postural drainage
 Short acting beta-agonist (salbutamol)
 Antibiotics during early phase of acute exacernation (need anti-Pseudomonal)
 According to previous sensitivity results, or
 3rd generation cephalosporin (cefoperazone) + quinolones (levofloxacin)
 If severe haemoptysis  BAE + urgent bronchoscopy
 Inhaled steroid (if airway reversibility following bronchodilator)

Page 22 By Heyson Chan


4 What are the common organisms responsible for community-acquired pneumonia?
Typical
 Strept pneumoniae
 Staph aureus
 H influenza
 G-ve organisms
Atypical
 Myciplasma pneumiae
 Legionellae
 Chlamydia
 Viral (including SARS)
Specific risk factors for specific pathogens
 Penicillin resistant penumococcal: >65 years old, alcoholics, immunocompromised, medical
co-morbidities, recent use of antibiotics
 Enteric gram-negative organisms: nursing home, cardiopulmonary
 Pseudomonas aerogenosa: structural lung disease, chronic steroid use, malnutrition

5 What are your criteria for determining whether the patient needs admission?
Low risk patient if
 Age <50
 Absence of the following conditions
 Active neoplastic disease at the time or within 1 year of presentation
 Congestive heart failure
 Cerebrovascular disease
 Chronic renal disease
 Chronic liver disease
 Absence of the following abnormal physical signs
 Altered mental state
 Pulse >125
 Respiratory rate >30
 Systolic blood pressure <90
 T <35 or >40
 Absence of the following features in investigations
 Arterial pH <7.35
 Blood urea
 Na <130
 Glucose >14
 Hematocrit <30%
 PO2 <8kPa
 Pleural effusion

Page 23 By Heyson Chan


6 What investigations would you like to perform for a patient presented with pneumonia?
 CXR
 Complete blood count/Diff WCC
 Renal/Liver function tests
 Sputum MCS/Tb
 SaO2 or blood gases
 Atypical serology
 NPA for flu, RSV, etc. If flu A +ve: H5 by RT-PCR ormonoclonal antibody (IF)
 SARS work-up (NPA, serum RNA, stool, etc for CoV)
 Blood culture
 Urinary antigens for pneumococcus/legionella
 Bronchoscopy
 Chest CT
 Tracheal aspirate/ lung bx

7 What is your choice of antibiotics?


Outpatient pneumonia: non-smokers of any age with nocardiopulmonary disease
 Augmentin+/-Macrolide
Outpatient pneumonia with cardiopulmonary disease / smokers
 Rx: Augmentin +/-Macrolide
 Those with DRSP risk# - Rx: Augmentin/Unasyn +Amoxil +/-Macrolide OR
Fluoroquinolone
Hospitalised patients with mild to moderate infection (these
patients have risk factors requiring hospitalisation)
 Without DRSP risk# - Rx: augmentin +/-Macrolide OR Fluoroquinolone
 With DRSP risk# - Rx: Augmentin/Unasyn + Amoxil +/- Macrolide OR
Cefotaxime/Ceftriaxone +/- Macrolide
Severe hospitalised community-acquired pneumonia
(Either 1 out 3 major OR 2 out of 6 minor)
Major criteria: a) ARF, b) Septic shock, c) Require MV
Minor criteria: a) RR>30/min, b) PaO2/FiO2<250,c) SBP<90 mmHg or DBP<60 mmHg, d)
Urea>7 mmol/L,e) Mental confusion, f) Multilobar involvement
 Without pseudomonas risk:
Rx: Cefotaxime/Ceftriaxone +/-Macrolide or Piperacillin/Tazobactam +/- Macrolide
 With pseudomonas risk
Rx: 2 antipseudomonal agents +/- Macrolide, Fluoroquinolone + antipseudomonal agents
* Modify antibiotics according to C/ST when available

Page 24 By Heyson Chan


Diabetes 
HPI
Diabetes
Onset and Diagnosis
 How was DM diagnosed?
 When was DM diagnosed?
 What investigations had been performed? What were the results?
 Symptoms at onset: polyuria / polydipsia / weight loss / loss of appetite
Disease progress and treatment received
 Knowledge of the disease
 Any monitoring of glucose level
 Previous and current treatment methods
 Compliance and side-effects on drug treatment
 Follow-up
Complications
a) Macrovascular
 Cardiac: chest discomfort / SOB (angina / ACS)
 Vascular: calf pain (limb claudication)
 Neurological: limb weakness / numbness (CVA / TIA)
 Autonomic dysfunction: postural hypotension / impotence / erectile dysfunction
b) Microvascular
 Renal: urine output / frothy urine (nephropathy)
 Eye: blurred vision (retinopathy / cataract)
 Foot: poor ulcer healing / athlete foot / foot care (neuropathy)
c) Others
 Hypoglycemic attack frequency, symptoms and precipitating factors
 DKA / hyperosmolar coma
 Recurrent infection / oral thrush / UTI (immunocompromised)
 Injection site: liposclerosis
Evaluation of cardiovascular risk factors
 Hypertension / hyperlipidemia and its control
 Diet
 Smoking / drinking
 Exercise
 Family history of cardiovascular events

PMH
 Cardiovascular events
 Exclude secondary causes: Cushing’s, haemachromatosis, acromegaly, chronic pancreatitis
SHx
 Smoking, drinking
 Occupation

FHx
 Diabetes / other CVD and age of onset
 Family support
 If patient has poor drug compliance  educate family members to remind the patient to take
medications

Page 25 By Heyson Chan


DHx
 Steroid / thiazide (diabetogenic)
 Allergy / regular use of medication

Questions
1 What would you like to pay particular attention to during your physical examination?
 Body weight, height and waist circumference
 Foot exam for dermopathy, ulcers and signs of infection
 Insulin injection site for lipodermasclerosis
 Lower limb examination for sensory and loss of ankle jerk
 Limb examination for power (CVA)
 Cardiovascular examination, including measurement of postural blood pressure
 Peipheral vascular examination for distal pulses
 Visual acuity
 Fundi examination
 Urine for multistix

2 How would you like to investigate the patient?


 Blood glucose
 CBC
 Lipid profile
 Liver function test
 Renal function test
 Urine for microalbuminuria
 HbA1c
 ECG and CXR if indicated

3 How would you follow-up the patient?


 Self-monitoring by home H’stix
 Lifestyle modification + diet adjustment
 Check BP every 3 months
 Check body weight every 3 months
 Check blood glucose every 3 months
 Lipid profile every 6 months
 Renal function test and BC every 6 months
 HbA1c every 6 months for long-term control
 Review technique every year
 Complete assessment every year

Page 26 By Heyson Chan


4 What is the treatment plan for patients with diabetes?
General
 Exercise
 Diet
 Education
 Foot care
 Regular home monitoring
 Control other risk factors (aim BP<130/80; BMI<23; glucose 4-8)
Pharmacological
a) Oral hypoglycemic agents
 Sulphyurea
 Metformin
 Acarbose
b) Insulin

5 Name some hypoglycemic agents. Explain their mechanism of actions and name some
possible side effects.
 Sulphyurea
 Increase insulin secretion
 May promote weight gain (not the first-line for obese patient)
 Given before meal or once / twice daily
 Risk of hypoglycemia
 Contraindicated in moderate to severe renal impairment
 Metformin
 Increase insulin sensitivity, inhibits gluconeogenesis
 Also decrease TG and cholesterol
 Good for over-weight patients (decrease risk of death, MI, CVA)
 GI side-effects, e.g. nausea, vomiting, abdominal bloating, diarrhea
 May leads to lactic acidosis in patient with chronic renal failure
 Contraindicated in hypoxic patients, liver failure
 Acarbose
 Decrease rate of glucose absorption
 Must be taken immediately before meal

6 What is the insulin regime?


Insulin regime should be individualized, possible regime including:
 Twice daily injection of intermediate insulin (monotard); 2/3 in the morning, 1/3 in the
evening
 If high post-pranial glucose level; add short acting insulin (actrapid); in a ratio of 3:7 with
intermediate acting insulin
 If frequent hypoglycemia; use short acting insulin before meal

7 What are the diagnostic criteria for DKA?


 Plasma glucose > 14 mmol/L
 Arterial pH < 7.3
 Plasma bicarbonate < 15 mmol/L
 Moderateketonuria or ketonemia

Page 27 By Heyson Chan


8 What is the management plan for a patient presented with DKA?
Investigations
 CBC for Hb
 RFT for Na, K, Urea, Creatinine
 Urine & Blood glucose
 Urine + plasma ketones
 PO4, Mg
 Arterial blood gas (ABG)
If indicated:
 CXR
 ECG
 Blood & urine cultureand sensitivity
 Urine & serum osmolality
 PT, APTT
Treatment
 Hydration (1L NS over 1 hour)
 Insulin (0.15U/kg bolus, then insulin infusion pump and titrate against blood glucose level)
 Potassium supplement (aim at postassium level between 4-5)
 Sodium bicarbonate (only if pH<7.0)
 Treat the underlying precipitating factors

9 What are the diagnostic criteria for hyperglycemic hyperosmolar coma?


 Blood glucose > 33 mmol/L
 Arterial pH > 7.3
 Serum bicarbonate > 15 mmol/L
 Effective serum osmolality ((2xmeasured Na) + glucose) > 320 mOsm/kg H2O
 Mild ketonuriaor ketonemia, usually in association with change in mental state

10 Explain the acute management plan for patients presented with hyperglycemic
hyperosmolarcoma.
 Management principles are similar to DKA
 Fluid replacement is of paramount importance as patient isusually very dehydrated
 If plasma sodium is high, use hypotonic saline
 Watch out for heart failure (CVP usually required for elderly)
 Serum urea is the best prognostic factor
 Insulin requirement is usually less than that for DKA, watchout for too rapid fall in blood
glucose and overshothypoglycaemia

11 What is Whipple’s triad?


It is the triad of hypoglycemia
 Symptoms (sympathoadrenic + neuroglycopenic)
 Documentation of hypoglycemia
 Symptoms improved after glucose corrected

Page 28 By Heyson Chan


Diarrhea 
HPI
Diarrhea
 Onset (acute vs chronic)
 Progression
 Any aggravating factors
 Any relieving factors
 Severity – any signs of dehydration (urine output / postural hypotension) / malnutrition (weight
and appetite – quantify e.g. how many bowl of rice)

Bowel symptoms
 Frequency (compare with previous bowel habit)
 Consistency and color of stool
 Any pus / mucus
 Any constipation between episodes (cancer)

Associated symptoms
 Any abdominal pain
 Any weight loss (IBD / cancer)
 Any fever
 Any nausea / vomiting
 Any change of bowel habit / tenesmus (cancer)
 Any oral ulcer / anal tags (Crohn’s)
 Any joint pain / eye problem (extra-GI manifestation of IBD)
 Any cough / sputum / night sweat (TB)
 Any rash
 Any weight loss / heat intolerance / sweating / tremor (thyrotoxicosis)
 Any anemic symptoms – malaise / SOB / palpitation (cancer / IBD)
 Any signs of infection (for acute diarrhea) – cough / sputum / RN / haematuria / frequency /
dysuria
 Previous investigations and treatment
 TOCC history
 Detailed food history

PMH
 TB (TB peritonitis / ileus / rule out for anti-TNF treatment in IBD)
 Bowel surgery
 Renal failure / DM / hyperthyroidism

SHx
 Smoking, drinking
 Occupation, recent stressors (IBS)
 How does it affect your daily activities / social functioning?

FHx
 Colorectal cancer
 Colonic polyps
 IBD

Page 29 By Heyson Chan


DHx
 Recent use of antibiotics
 “Weight-losing” medication – e.g. laxatives / thyroxine
 Any use of over-the-counter medication / herbs
 Any medication taken and its side effects

DDx
Acute diarrhea
 Infection:
 Bacterial: E coli / Campylobacter / V cholera / Salmonella / Shigella
 Viral: hepatits A / rotavirus / astrovirus
 Parasite: amebic
 Food poisoning
 Drugs: laxative / antibiotics / Mg-containing antacid
 Pseudomembranous colitis (antibiotic associated)
Chronic diarrhea
 Lactose intolerance
 Malabsorption syndrome
 Irritable bowel syndrome
 Colonic polyps / polyposis
 Inflammation: inflammatory bowel disease (Crohn’s or UC)
 Endocrine: thyrotoxicosis

Questions
1 What investigations would you like to perform in a patient presented with chronic diarrhea?
 CBC for anemia
 LFT for nutrition status
 RFT for dehydration
 TFT for hyperthyroidism
 Stool analysis for culture, microscopy, identification of ova, viral antigen and toxins
 CXR, AXR, CT
 Radiological studies (e.g. Ba enema if indicated)
 Colonoscopy for morphology and biopsy
If indicated, consider:
 Serum biochemistry for serology, plasma peptides and ACTH
 Urine biochemistry for 5-HIAA and histamine
 Second line stool analysis for Mg and pH

2 What is your management plan for a patient with inflammatory bowel disease?
Acute
Therapy should be guided by disease activity
 Mild [< 4 stool daily, no blood & systemic disturbance, normal ESR]
 Sulphasalazine 2-6 g/d (i.e. 0.5-1.5g qid)
 Prednisolone enema 20-100mg 1-2x/d
 Moderate [> 4 stool daily, ± blood, no systemic disturbance]
 Sulphasalazine 2-6 g/d (i.e. 0.5-1.5g qid)
 Prednisolone enema 20-100mg 1-2x/d
 Prednisone 40-60mg/d

Page 30 By Heyson Chan


 Severe [> 6 stool daily, bloody diarrhoea, fever, tachycardia, anaemia]
 NPO
 Hospitalization with fluid/electrolyte infusion +/- TPN
 Sulphasalazine 2-6 g/d (i.e. 0.5-1.5g qid) or
 Mesalamine 1.5-4.5g/d (i.e.0.5-1.5g tds)
 Prednisolone enema 20-100mg 1-2x/d
 IV Hydrocortisone 100mg q6h
 Surgical consultation
 Empirical broad-spectrum iv antibiotics to cover forenteric organisms
 Daily AXR to monitor toxic megacolon if present
 Metronidazole for Crohn’s Disease with perianalcomplications or fistula
Maintenance
 Sulphasalazine 2-4 g/d (i.e. 0.5-1g qid)
 Azathioprine and 6-MP 50-100 mg/d

3 How would you monitor the disease progress of patients with inflammatory bowel disease?
Clinical signs and symptoms
 Number of bowel openings
 Blood and mucus in stool
 Systemic upset (including fever, tachycardia)
 Appetite
 Weight change / nutritional status
Investigations
 CBC for anemia
 CRP, ESR for inflammation
 LFT for albumin level (nutritional status)

Page 31 By Heyson Chan


Diplopia 
HPI
Diplopia
 Onset (sudden vs gradual) how does the patient notice?
 Progression
 Duration (how long does it takes to get back to normal?)
 Maximal at which direction?
 All direction: MG
 All except abduction of affected side: 3rd nerve palsy
 Abduction: 6th nerve palsy
 Character: how does the images appeared (side by side / vertical)
 Any precipitating factors
 Any relieving factors: does it go away when you cover one eye?
 Severity: complete / partial ptosis
 Any diurnal variation (MG)
Associated symptoms
 Any muscle weakness / fatigue
 Pattern of weakness (hemi – stroke; general – MG)
 Any precipitating factors
 Any relieving factors
 Any diurnal variation
 Any headache (ophamoplegic migraine)
 Any fatigability (MG)
 Any ptosis (third nerve palsy)
 Any blurred vision / disturbance to vision
 Any vertigo
 Any weight loss / sweating / palpitation / heat intolerance / proptosis (Grave’s)
 Any recent trauma
 Any signs of other cranial nerve involvement (e.g. hearing)
 Any nose bleeding / recurrent otitis media (NPC  6th nerve palsy)

PMH
 HT (risk factors for third / sixth nerve palsy)
 DM (risk factors for third / sixth nerve palsy)

SHx
 Smoking, drinking
 Occupation

FHx
 Similar conditions
DHx
 Allergy / regular use of medication

Page 32 By Heyson Chan


DDx
Constant
 Third nerve palsy
 Sixth nerve palsy
 Extra-ocular muscle problem
 Myasthenia gravis (less likely)
Intermittent
 Myasthenia gravis
 Migraine
 Grave’s disease

Page 33 By Heyson Chan


Dizziness 
HPI
Dizziness
 Differentiate between light headedness, vertigo and LOC
 Onset (sudden vs gradual)
 Progression
 Frequency and duration
 Any precipitating factors (e.g. exercise, postural change)
 Any specific conditions leading to dizziness (e.g. micturation / strain / cough / prolonged
standing / postural change)
 If suspect vasovagal, specifically ask whether dizziness will come when patient is lying down
(should be negative if it is vasovagal)
 Any relieving factors (e.g. rest)
 Severity: how severe it was? Any LOC (important!)?
 Any previous episodes, any difference from this episode e.g. duration
Associate symptoms (for light headedness)
 Any SOB (quantify ET) / palpitation (cardiac cause / anemia)
 Any chest discomfort (ACS, arrhythmia)
 Any malaise / weight loss (anemia / chronic illness)
 Any tarry stool / coffee ground / menorrhagia in female (anemia)
 Any previous investigations and results (e.g. low BP, ECG)
 Any hunger feeling / sweating / tremor / mental confusion / nausea (hypoglycemia)
 Any weight gain or weight loss / heat or cold intolerance (thyroid)
 Any previous head injury (HI)
 Any visual / hearing disturbance
 Any hyperventilation / panic sensation (psychogenic)

PMH
 Cardiac disease, IHD, known valvular lesions / rheumatic heart disease
 HT, DM
 CVA

SHx
 Smoking, drinking
 Diet in DM patients (when did he last eat before the attack?)
 Occupation and accidental risk assessment (especially in elderly)
 Functional status

FHx
 Cardiac disease
DHx
 Any use of anti-hypertensive
 Use of oral hypoglycemic agents / insulin
 NSAID / aspirin (risk of GIB)
 Use of sedatives / alcohol / recreational drugs

Page 34 By Heyson Chan


DDx
 Cardiac: arrhythmia (bradycardia, heart block, SVT, AF), aortic stenosis, acute coronary
syndrome
 Vascular: hypotension, postural hypotension, anemia
 Endocrine: hypoglycemia
 Psychogenic
 Head injury
 Drugs: anti-hypertensive, oral hypoglycemic drugs, sedatives, alcohol

What is your management plan for a patient presented with dizziness?


 BP/P/T Q4H (postural BP if indicated)
 CBC w/ D/C
 L/RFT
 TFT
 Blood glucose
 ABG
 CXR, ECG
 CT brain (if indicated)

Page 35 By Heyson Chan


Dysphagia 
HPI
Dysphagia
 Onset
 Is it related to swallowing? Are you completely well between meals (rule out globus)
 Progression
 Character: difficulty in swallowing solid / liquid / both
 Level of obstruction – any holding up sensation? Where?
 Any pain (odynophagia)
 Severity – can the patient eat at all?
 Does it comes and goes or is it continuous?
 Aggravating factors
 Relieving factors
 Any previous investigations e.g. OGD
Associated symptoms
 Any weight loss (malignancy / just due to poor eating secondary to dysphagia)
 Any malaise / anemic symptoms (malignancy)
 Any nasal regurgitation / choking / symptoms appeared immediate after swallowing
(oropharyngeal)
 Any hoarseness of voice (malignancy)
 Any neck swelling (large goiter)
 Any cough / hamoptysis (lung cancer)
 Any chronic regurgitation (GERD  peptic stricture)
 Any muscle weakness (stroke / polymyositis / dermatomyositis / GBS / MND)
 Any tremor (Parkinson’s disease)
 Any chest infection (aspiration pneumonia  complication of dysphagia)

PMH
 Previous malignancy requiring radiotherapy e.g. neck irradiation (post-radiotherapy fibrosis)
 Ingestion of corrosive (corrosive stricture)

SHx
 Smoking, drinking
 Occupation

FHx
 GI malignancy

DHx
 Tetracycline / NSAID / KCl (may cause local damage)
 Any medication including herbs
 Allergy history

Page 36 By Heyson Chan


DDx
Obstruction
 Intrinsic: Esophageal cancer, corrosive stricture, peptic stricture, esophageal ring
 Extrinsic: Lung cancer, lymphoma, vascular diseases (e.g. aneurysm, dysphagia lusoria)
Dysmotility
 Primary: Achalasia, diffuse esophageal spasm
 Secondary: caused by systemic diseases

Questions
1 How would you examine the patient?
 Cognitive assessment
 Speech assessment for nasal speech / dysphonia
 Oropharyngeal assessment for swallowing test / drooling of saliva
 Neurological assessment: CN9-12 / tongue wasting / tremor / limb weakness / facial
weakness
 Thyroid status
 Skin for rash / scleroderma
 Neck examination for LN / goiter / mass / scar / post-RT changes
 Respiratory for signs of respiratory infection
 Nutritional assessment for BMI / tricep skin fold

2 What investigations would you like to perform for the patient?


 CBC, L/RFT (urine Cr index)
 Albumin level
 Iron profile for transferrin level
 TFT (thyroid myopathy)
 CPK (if suspect polymyositis)
 ACh receptor antibody (if suspect MG)
 Plasma glucose
 CXR (for extrinsic compression or chest infection)
 Barium swallow
 OGD

Page 37 By Heyson Chan


Edema 
HPI
Edema
 Onset – how did you discover
 Progression
 Unilateral / bilateral
 Severity – level of edema, any puffy face / scrotal swelling (for male) / abdominal distention /
SOB
 Does it comes and goes or is it continuous?
 Aggravating factors
 Relieving factors
 Any previous investigations / treatment e.g. diurectics
Urinary symptoms
 Urinary pattern – frequency and nocturia
 Urine output
 Any frothy urine
 Any haematuria
 Other urinary symptoms – dysuria
Associated symptoms
 Blood pressure reading (nephritic syndrome)
 Any fever (UTI / DVT)
 Any redness / hotness over affected limbs (DVT / cellulites)
 Any pain when pressing on the affected limbs (DVT / cellulites)
 Any SOB (severe edema / heart failure)
 Any orthopnea / PND / chest discomfort / palpitation (heart failure)
 Any signs of chronic liver disease: known cirrhosis / chronic hepatitis; malaise / weight loss /
easy bruising / jaundice (chronic liver failure)
 Any diarrhea (IBD / protein losing enteropathy)
 If unilateral  ask specifically for recent long-haul flight / surgery / use of OC pills / HRT

PMH
 Chronic hepatitis (especially chronic Hep B, associate with liver failure and membranous
glomerulopathy) (also note: steroid and cyclophosamide is contraindicated in chronic Hep B)
 SLE (associate with membranous glomerulopathy)
 Chronic renal failure
 Congestive heart failure / HT / DM / hyperlipidemia / arrhythmia
 Recent operation – esp pelvic surgery (DVT)
SHx
 Smoking, drinking
 Occupation
 Are you able to put on shoes?
 Does it affect your daily activities?

FHx
 Hepatitis
 History of liver disease / cirrhosis / HCC

Page 38 By Heyson Chan


DHx
 Recreational drugs (nephrotic syndrome)
 Any NSAID use (acute renal failure)
 Any medication including herbs
 Allergy history

DDx
 Unilateral: DVT / cellulites
 Bilateral: nephrotic syndrome / hypoalbuminemia / renal failure / chronic liver failure /
congestive heart failure / fluid overload / malnutrition

Summary
1 Chief complaint
2 Causes
3 Severity

Questions
1 What is the typical triad of nephrotic syndrome?
 Heavy proteinuria
3.5 g/day; or > 40 mg/m2/hr inchildren
 Hypoalbuminemia
 Edema
Note: hypercholesterolemia is common

2 What are the causes of nephrotic syndrome?


 Minimal change nephropathy
 Focal glomerulosclerosis
 Membranous nephropathy
 Mesangiocapillary glomerulonephritis
 Diabetic glomerulosclerosis
 Amyloidosis
Note: almost all causes are glomerular in origin and a reanl biopsy is indicated unless the
diagnosis is immediately obvious

3 What are the investigations that you would like to perform in a patient presented with
nephroticsyndrome?

4 What is your management plan?

Page 39 By Heyson Chan


Epilepsy 
HPI
Epilepsy
 Onset
 Progression
 Frequency of attacks
 How was the disease diagnosed?
 Any investigations performed
 Any treatment given? Compliance? Side effects?
Describe a typical attack:
Before the event
 Exact circumstances before the event (what exactly were you doing – describe in detail.)
 Any aura
 Any palpitation / chest discomfort (rule out cardiac cause)
 Any hunger feeling / tremor / other neurological symptoms (r/o hypoglycemia)
 Any source of bleeding, e.g. tarry stool, menorrhagia (r/oanemia)
The event
 Onset
 Duration of event
 Any loss of consiousness / aware of what is happening (seizure)
 Any twitching of limbs – describe in detail (one sided or generalized at onset?)
 Any biting of tongue / up-rolling eyeball
 Any incontinence
 Any fall / head injury / weakness
 Any witness
After the event
 How did the event ceased?
 Any injury noticed
 Any amnesia / clouding of consciousness (do you know what happened?) –how long did it take
to recover?
 Color of face (pale / red)
Any localizing symptoms
 Any visual or hearing disturbance
 Any facial asymmetry
 Any weakness / tiredness
 Any sensory disturbance

PMH
 Stroke / tumor / head injury / childhood fever  identify possible secondary causes

DHx
 Anti-epileptics – compliance and side effects!
 Antibiotics (may interact with anti-epileptics)
 Recreational drugs

Page 40 By Heyson Chan


SHx
 Smoking, drinking
 Occupation, hobbies, brief functional assessment
 Risk assessment (e.g. driving, swimming alone, bathing)
 Sleep pattern – do you have enough sleep (precipitating an attack)
 Identify the stressors
 View on the disease, medications and side-effects
 Give a brief education for the patient
FHx
 Epilepsy

Summary
1 Typical attack
2 Control, including frequency, admission
3 Medications including compliance and side-effects
4 Neurological symptoms
5 Functional assessment

Page 41 By Heyson Chan


Questions
1 What investigations would you like to perform in a patient presented with first epileptic
attack?
 CBC with D/C
 Renal function test for electrolyte disturbance
 H’stix or random glucose
 ABG, urea, anti-convulsants level
 CT brain (MRI brain for temoral lobe epilepsy– look for temporal sclerosis)
 EEG (repeated EEG examinations with seizure provoking measures including sleep
deprivation and flash light to increase sensitivity– around 75%)

2 When would you consider starting treatment in a patient with epileptic attacks?
Consider starting anti-epileptics if patient has:
 Single unprovoked seizure with speific risk factors (e.g. presence of structural brain lesions,
head injury, meningitis)
 Status epilepticus at onset
 Recurrent seizures

3 Name some common anti-epileptics, state the type of epilepsy that they work best on
anddescribe some common side effects.
General side effects
 Drowsiness
 Allergic reactions (e.g. rash)
Specific side effects
a) Valporate (short variable half life, for absence and generalized seizure)
 Weight gain
 Alopecia
 Liver toxicity (drug-induced hepatitis)
 Thrombocytopenia
 Acne
 Tremor
 GI upset
b) Carbamazapine (short half life, for partial seizure)
 Nausea
 Leukopenia
 Arrthymia
 Hyponatremia
 Diplopia
c) Phenytoin (long half life, for partial seizure)
 Hirsutism
 Gum hypertrophy
 SLE
 diplopia
d) Phenobarbitone
 Cognitive impairment

Page 42 By Heyson Chan


4 What is your immediate management plan for a patient presented with status epilepticus?
 Establish ABC, administer oxygen
 Ensure good oxygenation and IV access
 Check glucose and h’stix, electrolytes (include Ca and Mg),ABG, urea, anticonvulsant level
 Give D50 50 ml iv and/or 100 mg thiamine iv whereappropriate. Treat acidosis if severe
 Suppress clinical seizures rapidly with iv lorazepam 4mg (or1mg/kg) over 2 minute, up to
8mg. Alternative: iv diazepam10 mg (or 0.15mg/kg) over 1-2 minutes, up to 20 mg.
 Give simultaneously long acting anti-epileptic drug:
 Phenytoin–iv loading dose 15mg (elderly) to 20mg (adult) perkg, at rate of 50mg per
minute. Maintenance dose 5mg per kgper day. (Beware of cardiorespiratory depression,
hypotensionand arrhythmias)
 If above agents unsuccessful or seizures persist after 30-45minutes from onset, ICU
admission advisable for ventilatoryassistance and second line agents eg. Thiopentone,
midazolamor propofol, with EEG monitoring.
 Monitor BP/P,RR, ECG and document fits. Continue intensivetreatment for 12-24 hrs after
last clinical or EEG seizure.
 Diagnose and treat any underlying cause e.g. head injury, CNSinfection, intracranial
haemorrhage, metabolic disturbances,alcoholism, drug intoxication, abrupt
anticonvulsantwithdrawal. Identify and treat medical complications.
 When a patient fails to gradually recover after the convulsivemovements stop, an urgent EEG
is necessary

5 How would you educate a patient with epilepsy?


 Avoid risk taking behavior
 Don’t drive
 Don’t bath, take shower instead
 Don’t swim / hike alone
 Sleep adequately
 Side-effects of medications
 Compliance is essential
 Educate on the possible side-effects accordingly
 Medication can be gradually tapered off if the patient has been seizure free for 2-3 years
 Contraception
 Possible teratogenic effects of antiepileptics (6% against background risk of 3%)
 Advise on contraception during and at least 6months after stopping anti-epileptics
 Knowledge on epilepsy
 Acute management of epileptic attack
 Remove nearby danger
 Put patient in lateral position
 Don’t put any thing into patient’s mouth

Page 43 By Heyson Chan


Fall 
HPI
Fall and gait disturbance
 Onset
 Frequency
This episode
 Time / place / person
 Any preceding event before the episode (e.g. dizziness / palpitation / LOC / transient neurological
symptoms)
 Describe the event (high / low energy fall)
 Any LOC / near-syncope
 Any injury
Previous episodes
 Briefly describe
 Any hospitalization required
 Any injury
 Any investigations / causes identified
Associated symptoms
 Any dizziness / LOC especially on postural change (postural hypotension)
 Any palpitation (show me the rhythm) / chest discomfort (cardiac disease)
 Any limb weakness (stroke)
 Any gait disturbance (cerebellar ataxia)
 Any tremor (Parkinson’s disease / cerebellar disease)
 Any visual disturbance (eye disease)
 Any previous trauma / joint pain (musculo-skeletal problem)
Functional status and caring issue
 Can the patient look after himself?
 If not, can anyone help?
 Can the patient feed / bath / cook / go out by himself?
 Does the patient require any walking aids?
 Explore the living environment (lighting, living area, storage area)
 Use of personal emergency link service (平安鐘)
 Explore the social / family support of the patient

PMH
 CVA / Parkinson’s disease / dementia / cardiac disease
SHx
 Smoking, drinking
 Environmental history (e.g. living environment, lighting)
 Caring issue
 Social support
 Usual daily activity
FHx
 Family support
DHx
 Anti-hypertensive / DM medications – compliance and side-effects
 Sedative
 Allergy / regular use of medication

Page 44 By Heyson Chan


Questions
1 What are the possible causes of recurrent fall?
Intrinsic
 Neurological disease: CVA / Parkinsonism / peripheral neuropathy / cerebellar ataxia
 Musculoskeletal: muscle wasting from arthritis / unstable knee joints /
 Poor vision
 Medication: side effects or sedatives
 Alcoholism
 Poor nutritional state
 Electrolyte disturbance
Extrinsic
 Inadequate lighting
 Isles not clear of objects

2 How would you examine the patient?


 Stand with feet together, observe for body sway – balance
 Observe patient’s gait; use of walking aid, if necessary
 Limb power and sensation
 Joint examination
 Mental state examination
 Visual acuity and hearing
 Postural blood pressure

3 What are the possible ways of minimizing the possibility of recurrent fall?
 Correct vision
 Correct extrinsic danger (e.g. adequate lighting, clear isles)
 Exercise to train balance and muscle strength
 Vitamin D
 Bisphosphanate to reduce hip fracture
 Hip protector
 Walking aids prescription
 Emergency alarm call at home

Page 45 By Heyson Chan


Fever 
HPI
Fever
 Onset
 Duration
 Pattern of fever
 Degree of fever
 Method of measurement
 Any chills / rigors
 Any use of anti-pyrectics
Associated symptoms
 Respiratory: any cough / sputum / haemoptysis / SOB / pleuritic chest pain
 GI: any vomiting / diarrhea / abdominal pain / jaundice
 Urinary: any haematuria / dysuria / frequency / loin pain
 Neurological: any headache / neck stiffness / photophobia / rash
 Joint: any redness / swelling / painful joint
 Constitutional symptoms: any weight loss / malaise
 Hematological: any bruises / petechiae
 General: any rash
 Any unilateral leg swelling (DVT)
 Any joint pain associate with morning stiffness (auto-immune cause)
 TOCC history
 Any investigations done
 Any medications taken? Does it help?

PMH
 Recent dental surgery (SBE)
 Use of IV drugs (SBE)
 Known cardiac disease / valvular replacement
 Vaccination history

SHx
 Smoking, drinking
 Occupation

FHx
 Any one with fever / infection recently
DHx
 Recent change of medication
 Any known allergy

Page 46 By Heyson Chan


DDx
 Infections –
 Respiratory –pneumonia / TB / URTI
 Renal – pyelonephritis / cystitis
 GI – hepatitis / GE / cholangitis / cholecystitis
 Neuro – meningitis / encephalitis
 Joint – septic arthritis / osetomyelitis
 CVS – endocaritis / pericarditis
 General –sepsis / malaria
 Connective tissue disorder – e.g. SLE / RA
 Malignancy – primary or secondary (hematological or solid)
 IBD – Crohn’s / UC
 Drug reaction
 Heat stroke
 Factitious fever

Page 47 By Heyson Chan


Haematuria 
HPI
Haematuria
 Onset
 Intermittent / episodic
 Character – fresh blood or old blood
 Early stream / end stream / whole stream
 Severity
 Any blood clot
 Any anemic symptom e.g. dizziness / postural hypotension / SOB / palpitation
 Precipitating factors
 Relieving factors
 Any previous episodes before

Other urinary symptoms


 Urinary pattern – frequency / nocturia / volume
 Any dysuria
 Any frothy urine
 Any obstructive symptoms –hesitancy / weak stream / terminal dripping

Associate symptoms
 Any fever (UTI)
 Any abdominal pain / loin pain (renal stone / pyelonephritis)
 Any vaginal symptoms – vaginal discharge / PV bleeding
 Constitutional symptoms – weight loss / malaise / loss of appetite / SOB
 Any easy bruising / bleeding elsewhere (bleeding tendency)
 Any vomiting / abdominal pain (hypercalcemic symptoms)
 Any itching / lethargy / numbness (uremix symptoms)
 Any recent trauma / biopsy / investigations
 Any skin rash / joint pain
 Recent URTI
 Systemic review of respiratory / cardiac / GI system

PMH
 H Pylori status
 IHD / CVA / angina requiring aspirin
 Chronic pain / joint problem e.g. gout / SLE / RA requiring NSAID
 Heart disease (AF / valvular disease) requiring warfarin
 Hepatitis vaccine

SHx
 Smoking, drinking
 Occupation

FHx
 Renal disease
 Deafness (Alport’s syndrome)

Page 48 By Heyson Chan


DHx
 Warfarin
 Use of analgesics
 Other medication / herbs

DDx
 General: bleeding tendency / vaginal bleeding / trauma
 Local
 Kidney: RCC / GN / pyelonephritis / injury / cysts
 Ureter: stone / tumor
 Bladder: trauma / cystitis / stone / tumor
 Prostate: CA / BPH / prostitis
 Urethra: trauma / stone / CA / stricture

Page 49 By Heyson Chan


Haemoptysis 
HPI
Haemoptysis
 Onset
 Intermittent / episodic
 Character – fresh blood or old blood
 Severity
 Amount
 Any blood clot
 Any anemic symptom e.g. dizziness / postural hypotension / SOB / palpitation
 Rule out pseudo-heamoptysis
 Are you sure you cough them up?
 Do you have recent GIB / nose bleeding?
 Can you throw them up easily?
 Precipitating factors
 Relieving factors
 Any previous episodes before
Associate symptoms
 Any cough
 Any sputum
o Amount – estimate by number of bowl
o Color
o Purulent?
 Any SOB – quantify the exercise tolerance and enquire for any limitations to his daily activities,
including work
 Any fever (chest infection)
 Any night sweat (TB)
 Any chest pain (chest infection / CA / pulmonary infarction)
 Constitutional symptoms – weight loss / malaise / loss of appetite (TB / CA)
 Any palpitation (pulmonary embolism)
 Any joint pain
 Any tarry stool / abdominal pain (GIB leading to pseudo-haematemesis)
 Any easy bruises (bleeding tendency)
 Any PND / orthopnea (heart failure)
 Any rash / joint pain (connective tissue disorder)
 Any diplopia / facial asymmetry / neck swelling / nose-bleeding (NPC)
 Any runny nose / episodes of nose bleeding (epistaxis with post-nasal drip)
 Any visual disturbance (prepare for ethambutol)
 Any liver dysfunction / hepatitis (prepare for anti-TB treatment)
 Any renal disease (prepare for ethambutol)
 TOCC history

PMH
 Past history of TB / bronchiectasis
 Valvular heart disease
 History of liver / renal disease / vitamin deficiency (prepare for TB treatment)
 Risk factors for DVT / PE

Page 50 By Heyson Chan


SHx
 Smoking, drinking
 Occupation – how does the SOB affect his work?
 Living / working environment
FHx
 TB / infection / other lung disease
DHx
 Warfarin
 Use of analgesics
 Other medication / herbs

DDx

Questions
1 What are the causes of haemoptysis?
Common
 TB
 Pneumonia
 Bronchiectasis
 CA lung
 NPC (in the context of far east)
 Chronic rhinitis with epistaxis and post-nasal drip
Less common
 Congestive heart failure / mitral stenosis
 Connective tissue disorder
 Pulmonary embolism / infarction

2 What investigations would you like to perform if CA lung is suspected?

3 What are the treatment options for CA lung?


Treatment of CA lung depends on the histological type and staging of the disease

4 What investigations would you perform in a patient with suspected tuberculosis?


Haematological
 CBC with D/C
 L/RFT (prepare for anti-TB treatment)
Radiological
 CXR for infiltrative shadows, hilar lymphadenopathy, calcification and cavitations
 CT scan is more sensitive
 MRI if bone and joint TB or intracranial TB suspected
Microbiological
 Early morning sputum x 3 for G stain, ZN stain and C/ST
Note: ZN stain +ve smear indicates significant infectivity
 Early morning urine x 3 for ZN stain and C/ST

Note: PCR technique may be employed to increase sensitivity
Histological
 Pleural biopsy for granuloma and C/ST

Page 51 By Heyson Chan


5 What is the treatment regime for tuberculosis?
 Report to the Department of Health
 Start treatment under DOTS

6 Name some common side-effects from anti-TB treatment.

7 What is your immediate action if the ALT of a patient on anti-TB treatment is 4 fold above
theupper normal limit?
 Stop the treatment if ALT >3 fold above upper limit or bilirubin 2 fold above upper limit
 Reintroduce drugs whenALT returns to normal or baseline or less than 2 times the upper limit
of normal
 Non-hepatotoxic interim regimen may be employed

Page 52 By Heyson Chan


Headache 
HPI
Headache
 Onset
 Progression / episodic
 Duration
 Frequency
 Site
 Radiation
 Character
 Severity – grading and how does it affect the patient? Does it disturb the patient from sleep /
work / daily activities?
 Aggravating factors
 Relieving factors

Previous episodes
 First time – when?
 Frequency of attacks
 Any investigations done
 Any medications taken? Dosage, side-effects and compliance
 Any difference from previous episodes?

Before an attack
 Any aura
 Any other warning signs

Associated symptoms
 Any photophobia / phonophobia (meningitis / migrane)
 Any fever (CNS infection)
 Any rash (meningococcal infection)
 Any neck stiffness (CNS infection / cervical spondylosis)
 Any early morning headache / vomiting (raised ICP)
 Any visual disturbance (raised ICP / glaucoma / migrane)
 Any limb weakness / sensory disturbance (SOL)
 Any facial tenderness / nasal blockage / discharge (sinusitis)

Note:
If meningitis is suspected, enquire into the possible causes and complications
 Causes: sinusitis, endocarditis, otitismedia, skull fracture, immunocompromise
 Complications: deafness, seizures, cranial nerve palsy
If brain tumor is suspected, search for possible primary site

PMH
 Risk factors of stroke

Page 53 By Heyson Chan


SHx
 Smoking, drinking
 Occupation
 Disturbance of daily activities (work / sleep)

FHx
 Migrane / headache

DHx
 Use of analgesic
 Allergy history

DDx
 Acute: SAH / head injury / glaucoma
 Sub-acute: meningitis / encephalitis / sinusitis / giant cell arteritis
 Chronic: tension headache / raised ICP / SOL / cluster headache
 Episodic: migraine / analgesic rebound headache / cervical spondylosis

Questions
1 If a patient presented to the A&E with acute onset headache. How would you approach the
patient?
2 Name some precipitating factors of migraine.
3 What is the prevalence of migraine?
4 What is the male to female ratio of migraine?
5 What are the treatment options for migraine?
6 What is the management plan for a patient with bacterial meningitis?
 CBC with D/C
 Blood for C/ST (before or with the commencement of anti-biotics)
 CT brain
 LP (after ruling out SOL and raised ICP)
 Penicillin G 4 MU iv q4h (if risk of listerosisanticipated) +Cefotaxime 1.5-2 g iv q4h OR
Ceftriaxone 2 g q12hiv
 Add metronidazole for brain absecess
 Dexamethasone in complicated TB meningitis, brain abscess with significant cerebral edema
and bacterial meningitis may decrease complication rate
 Duration of Rx for meningitis usually 10-14 days, for brainabscess 6-8 weeks
 Treatment should be given for at least 7days for H. influenzae,10-14 days for S. pneumoniae,
14-21 days for L.monocytogenes and S. agalactiae, and 21 days for Gramnegative bacilli.
DO NOT change to oral therapy.
Specific treatment for other types of meningitis:
 Cryptococcal meningitis: Amphotericin B 0.5-0.8 mg/kgiv infusion over 4-6 hrs +5-
Flucytosine 37.5 mg/kg q6h po for 6 weeks(immunocompetent patients)
 Viral encephalitis: acyclovir 10mg/kg iv Q8H
 TB meningitis: need prolonged anti-TB treatment
 Consider prophylaxis for contacts in cases of meningococcalmeningitis: Rifampicin 600 mg
bd (10 mg/kg bd) for 2 days

Page 54 By Heyson Chan


7 Compare and contrast the LP picture of different types of meningitis.
Normal Viral Bacterial TB /
cryptococcal
Appearance Clear Clear Turbid Slightly turbid
/ viscous
Cell count <5 10 – 100 <50 100 – 300
PMN Nil Nil 200 – 3000 0 – 200
Protein 0.2 – 0.4 0.4 – 0.8 0.8 – 2.0 0.8 – 3.0
(g/L)
CSF/blood >1/2 >1/2 <1/2 <1/2
Glucose

8 What are the causes of aseptic meningitis?


Infective
 Viral meningitis
 TB meningitis
 Partially treated bacterial meningitis
 Cryptococcal meningitis
Non-infective
 Malignant meningitis

9 What are the common pathogens for meningitis?


10 What are the contraindications for lumbar puncture?

Page 55 By Heyson Chan


Hemiplegia 
HPI
Hemiplegia
 Onset (sudden vs gradual) how does the patient notice?
 Progression
 Duration
 Any precipitating event?
 Severity: how severe it was? Can the patient walk? Can you move the limbs at all?
Localization of lesion
 Any sensory disturbance / numbness
 Any disturbance in vision
 Any diplopia
 Any facial asymmetry
 Any dysphagia
 Any slurring of speech
 Any headache
 Any nausea / vomiting
 Any loss of consciousness
Risk factors / pre-morbid
 ADL – bathing / feeding / shopping
 Walks unaided / with stick / with frame
 Functional status
 Living environment
 Family / social support

PMH
 HT / DM / hyperlipidemia (severity, treatment and compliance)
 IHD / AF / other cardiac disease
 TIA / CVA (number of previous episodes, severity and FU)

SHx
 Smoking, drinking
 Environmental history (e.g. living environment, any lift, toilet facilities e.g. safety rails)
 Care taking issue, family support

FHx
 CVA and other CVD (age of onset)

DHx
 Aspirin / anti-coagulants
 Allergy / regular use of medication

Page 56 By Heyson Chan


Questions
1 What is your management plan for a patient presented with acute onset hemiplegia?
Investigations
 CBC with D/C, L/RFT, TFT, random glucose, lipid profile, clotting profile
 CXR
 ECG
 Urgent non-contrast CT brain
 Transcranial Doppler
 Carotid duplex ultrasound
 MR / CT angiography, if indicated
Management
 Admit to stroke unit
 BP/P/T Q4H, neuro observation
 Monitor and control hyperglycemia, but avoid correcting BP unless systolic BP >220 or
diastolic BP > 120 (use iv labetolol in this case)
 Nil by mouth until patient has passed the swallowing test, then allow puree diet
 Early physio / occupational and speech therapy

For ischemic stroke


 Start aspirin if not contraindicated
 May consider thromoblysis if presented within 3 hours
 Anti-coagulate with warfarin in special cases
 Consults neurosurgery for craniotomy if large infarct with midline shift

For intracranial haemorrhage


 Urgent consult neurosurgery if posterior fossa or cortical haemorrhage with progressive
deterioration
 Avoid use of anti-coagulants in the future

2 What are the measures for secondary prevention of stroke?


 Aggressive control of blood pressure (use diurectics or ACEI) after the acute period (if no
arterial stenosis/occlusion)
 Control DM, lipids (statin, maintain LDL <2.6)
 Stop smoking; regular exercise
 Antiplatelet (aspirin 80-325mg/day or clopidogrel 75mg/day) for ischaemic stroke
 Warfarin for atrial fibrillation (keep INR 2-3) if not contraindicated
 Consider endarterectomy/angioplasty if symptomatic extracranial carotid stenosis>60-70% in
a center with low perioperative risk

3 State some clinical features of a middle cerebral artery infarct.


 Contralateral hemiplegia (leg relatively spared)
 Contralateral hemianopia
 Aphasia (dominant)
 Neglect of contralateral limb, dressing difficulty (non-dominanat)

Page 57 By Heyson Chan


4 State some localizing features of anterior and posterior circulation.
Anterior circulation
 Monocular blindness
 Dysphasia (Broca’s in frontal lobe, Wernicke’s in parietal-temoral area)
 Gerstmann syndome (finger agnosia, acalculia, agraphia – lesion in dominant parietal lobe)
Posterior circulation
 Ataxia
 Nystagmus
 Vertigo
 Cranial nerve palsy (excluding I and II and UNM VII)
 Internuclear ophthalmoplegia
 Horner’s syndrome

5 What is your immediate management plan for a patient with atrial fibrillation, on
warfarinpresented with hemiplegia, with CT brain confirming intracranial hemorrhage
and INR 2.0?
 Re-check clotting profile
 Stop warfarin
 IV FFP to reverse warfarin action
 Consult neurosurgery

6 What are the common sites for hypertensive hamorragic stroke?


 Basal ganglia
 Cerebellar
 Thalamic
 Potine

Page 58 By Heyson Chan


7 What is your immediate management plan for a patient presented with acute onset of
hemiplegia?
 Admit ASU
 NPO until patient has passed the swallowing test (give puree diet)
 Monitor vitals, GCS, neuro obs
 Urgent non-contrast CT brain
 CBP, R/LFT
 Clotting profile
 Bloodglucose, lipid
 CXR, ECG
 If indicated, consider: Echocardiography, Duplex study ofcarotid arteries, Transcranial
Doppler (TCD), cerebralangiography, hyper-coagulopathy, autoimmune screening.
 Positioning splinting toavoid aspiration, contractures, pressure nerve palsy,
shouldersubluxation, pressure sores, etc
 Ensure good hydration and nutrition
 Meticulous control of blood sugar & pyrexia
 Aspirin 75mg to 325 mg daily within 48 hours of onset ofacute ischaemic stroke
 Anticoagulation should be considered for acute ischaemicstroke in:
- Arterial dissection
- Documented cardiac or intra-arterial source of embolism
- Progressing stroke due to large artery thrombosis

Neurosurgical consultation:
 Cerebellar haematoma or large cerebellar infarct withsignificant mass effect
 Large cerebral haematoma (> 30ml) with significant masseffect
 Impending or established hydrocephalus
 Subarachnoid haemorrhage
 Malignant MCA syndrome

Secondary prevention:
 Risk factor modification for all types of stroke
 Oral anticoagulation in cardiogenic embolism (includingnon-valvular AF) and anti-
phospholipid syndrome
 Carotid endarterectomy for symptomatic carotid arterystenosis (between 70-99%)
 Aspirin 80-300 mg daily for ischaemic stroke ifanticoagulation not indicated, consider
thienopyridines ifintolerant to aspirin
 Keep LDH <2.6

8 What are the complications of stroke?


 Cerebral edema (starting from day 2-3, peaks at day 4-5)
 Hemorrhagic transformation (usually in 1 week, 3 weeks for cerebellar stroke)
 Seizure – in 10% of cases, usually presented in first 48 hours
 Depression
 Vascular dementia – in 30% of patient

Page 59 By Heyson Chan


Hypertension (Secondary) 
HPI
Secondary Hypertension
Onset and Diagnosis
 How was HT diagnosed?
 When was HT diagnosed?
 When was your BP last known to be normal?

Disease progress and treatment received


 Any monitoring of blood pressure level
 Previous and current treatment methods
 Compliance and side-effects on drug treatment
 Diuretics: polyuria, weak erections, worsening of BPH symptoms
 ACEI: dry cough
 Beta-blocker: syncope due to bradycardia
 Follow-up

Complications
 Cardiac: chest discomfort / SOB (angina / ACS)
 Cardiac: PND / orthopnea / edema (heart failure) – quantify exercise tolerance and enquire any
limitation of activities due to the symptoms
 Neurological: limb weakness / numbness (CVA / TIA)
 Renal: frothy urine (nephropathy)
 Eye: blurred vision (retinopathy)
 Headache

Evaluation of cardiovascular risk factors


 DM / hyperlipidemia and their control
 Diet
 Smoking / drinking
 Exercise
 Family history of cardiovascular events

Exclude secondary causes


 Any snoring / early morning headache / unrefreshing sleep (OSA)
 Any change in shoe size / ring size (acromegaly)
 Any proximal weakness / easy bruising / striae / acne (Cushing’s)
 Any episodic flushing / palpitation / sweating / anxiety (pheochromocytoma)
 Any haematuria / proteinuria / recurrent UTI / renal atone / GN (renal cause)
 Any weight gain / cold intolerance (hypothyroidism)
 Any pregnancy complication (Sheehan syndrome)
PMH
 Gout (contraindications for diuretics)
 Obstructive urinary symptoms (hesitancy, terminal dripping, nocturia) / impotence / weak
erection (contraindications for diuretics)
 Bradycardia / heart block (contraindications for beta-blocker)
 COPD / asthma (contraindications for beta-blocker)
 Renal diseases (contraindications for ACEI / angiotensin II blocker)

Page 60 By Heyson Chan


 Possibility of preganacy in lady of child bearing age (contraindications for ACEI and angiotensin
II blocker)
 DM (affects threshold of drug treatment)
 Other cardiovascular disease e.g. CVA / TIA / angina / heart failure

SHx
 Smoking, drinking
 Occupation

FHx
 Hypertension / other CVD and age of onset
 Renal disease (polycystic kidney)
DHx
 Estrogen / excessive salt intake / sympathomometics (causing HT)
 Chronic use of steroid (Cushing’s)
 Allergy / regular use of medication

Counseling
 Explain the need of lifestyle modification
 In moderate to severe HT / HT with DM or other complications, explain the need of
pharamocological treatment, on top of lifestyle modification
 If Cr is raised, enquire into urinary symptoms. (it may be a consequence of HT or a secondary
cause of HT)
 If patient has a high normal blood glucose level, explain the likelihood of impaired glucose
tolerance or minor DM and the need of further investigations

Questions
1 What are the causes of secondary hypertension?
 OSA
 Pheochromocytoma
 Primary adrenal insufficiency
 Cushing’s
 Acromegaly
 Hypothyroidism
 Renal disease (intrinsic or renal artery stenosis)

2 How would you examine a patient with hypertension?

3 What investigations would you like to perform?

4 When would you consider starting treatment in a patient with hypertension?

Page 61 By Heyson Chan


5 What are the treatment options for hypertension? How would you choose between them?
Name some common side effects.
ACEI
 Indications: DM, CHF, post-MI, LV dysfunction cardiomegaly
 Contraindications: renal failure, hyperkalemia, pregnancy, bilateral renal artery stenosis
 Side-effects: cry cough (10%), hyperkalemia, angioedema, first-dose hypotension

Angiotensin II blocker
 Indications: Patients not being able to tolerate ACEI due to dru cough, DM, CHF, post-MI,
LV dysfunction cardiomegaly
 Contraindications: renal failure, hyperkalemia, pregnancy, bilateral renal artery stenosis
 Side-effects: hyperkalemia

Beta-blocker
 Indications: angina, post-MI, tachycardia
 Contraindications: heart block, COPD, asthma, dyslipidemia
 Side-effects: bradycardia, dylipidemia, impaired glucose

Calcium channel blocker


 Indications: angina, elderly with systolic hypertension, PVD
 Contraindications: heart block (block AV node); heart failure (-ve inotropic)
 Side-effects: headache, facial flushing, edema, dizziness, palpitation

Diuretics
 Indications: CHF, elderly with systolic hypertension
 Contraindications: gout, impotence, BPH, sexually active male
 Side-effects: frequency, electrolyte disturbance, impaired glucose, postural hypotension

Note:
 Consider combination therapy before maximizing dose of first drug
 Combination: A+B, B+C, A+D, C+D

6 What are the complications of hypertension?

7 What is the target BP for hypertensive patients?


 Normal patient: 140/90
 DM patient: 130/85
 Renal patient: 125/80

Page 62 By Heyson Chan


8 What is the treatment plan for malignant hypertension, hypertensive emergency and
hypertensive urgency?
General
 Recheck BP at least twice
 Look for target organ damage (neurological, cardiac)
 Complete bed rest, Low salt diet (NPO in HT emergency)
 BP/P q1h or more frequently, monitor I/O (Close monitoringin CCU/ICU with intra-arterial
line in HT emergency)
 Check CBP, R/LFT, cardiac enzymes, aPTT/PT, CXR, ECG,urine x RBC and albumin
 Aim: Controlled reduction (Rapid drop may ppt CVA / MI)
Specific
 Malignant BP >220/120 mmHg + Grade III/IV fundal changes
 Emergeny– Malignant HT with acute target organ damage
 Malignant or severe HT + ICH, dissecting aneurysm,APO, encephalopathy,
phaeochromocytoma crisis,eclampsia
 End organ damage due to HT versus riskof organ hypoperfusion due to rapid BP drop
 Need immediate reduction of BP to target levels(Initial phase drop in BP by 20-25% of
baseline)
 Resume usual medications if previously on anti-HT
 If previously not on anti-HT or failure to control BP with usual medications in 4-6 hours,
consider:
 Metoprolol 50-200 mg bd / Labetalol 200 mg po stat, then 200 mgtds
 Captopril 12.5-25 mg po stat, then tds po (if phaeo suspected)
 Long acting Calcium antagonists (Isradipine 5mg/Felodipine 5mg)
 If not volume depleted, lasix 20mg or higher in renal insufficiency
 Aim: Decrease BP to 160/110 over several hours
(Sublingual nifedipine may precipitate ischaemic insult dueto rapid drop of BP)
 Urgency – Malignant HT without acute target organ damage
 HT associate with bleeding (post-surgery, severepistaxis, retinal haemorrhage, CVA etc.)
 Severe HT + pregnancy / AMI / unstable angina
 Catecholamine excess or sympathomimeticoverdose (rebound after withdrawal of
clonidine /methyldopa; LSD, cocaine overdose; interactionswith MAOI)
 BP reduction within 12-24 hours to target level
 Labetalol 20 mg iv over 2 mins. Rept 40 mg iv bolus ifuncontrolled by 15 mins, then
0.5-2 mg/min infusion in D5(max 300 mg/d), followed by 100-400 mg po bd
 Na Nitroprusside 0.25-10 mcg/kg/min iv infusion (50 mg in100 ml D5 = 500 mcg/ml,
start with 10 ml/hr and titrate todesired BP)
o Check BP every 2 mins till stable, then every 30 mins
o Protect from light by wrapping, Discard after every 12 hrs
o esp good for acute LV failure, rapid onset of action
o Do not give in pregnancy or for > 48 hrs (risk of thiocyanideintoxication)

Page 63 By Heyson Chan


Jaundice 
HPI
Jaundice
 Onset – how did you discover
 Progression
 Severity –yellowing of skin / sclera / both
 Does it comes and goes or is it continous?
 Aggravating factors
 Relieving factors
 Any previous investigations e.g. ERCP / treatment

Associated symptoms
 Any abdominal pain – site and characteristic
 Any back pain (pancreatitis)
 Any tea-colored urine / pale stool (post-heaptic)
 Haematuria – suspect haemolytic anemia  enquire about severity (postural dizziness, SOB)
 Any fever / chills / rigors (cholangitis / cholecystitis)
 Any steatorrhea / pruritis (pancreatitis)
 Any chronic liver stigmata: abdominaldistention / edema / easy bruising / history of variceal
bleed / easy fatigue / itchiness (chronic hepatitis)
 Any symptoms suggesting underlying malignancy: change of bowel habit / haemoptysis / weight
loss / malaise (liver secondary)
 Any other bleeding orifice / easy bruising (bleeding tendency)
 TOCC history
 Detailed food history if suspecting hepatitis A
 Sexual history

PMH
 Hepatitis B/C; vaccination history
 Gallstone
 DM / diarrhea of recent onset (pancreatitis / CA pancreas)
 History of immunization
 Haemolytic anemia

SHx
 Smoking, drinking
 Sexual history
 Birth history (born in HK / China, when)
 Transfusion history
 Occupation

FHx
 Hepatitis
 History of liver disease / cirrhosis / HCC
DHx
 Anti-coagulants
 Any medication including herbs
 Allergy history

Page 64 By Heyson Chan


DDx
 Pre-heaptic: haemolysis
 Hepatic: hepatitis A/B/C / HCC / secondaries / drugs (paracetamol / anti-TB e.g. HRZ / ABi e.g.
erythromycin) / alcohol / autoimmune (female predominance)
 Post-hepatic: CA head of pancreas / gallstone / cholangitis / cholangiocarcinoma / stricture of
CBD / primary sclerosis cholangitis

Remarks
For mildly elevated ALT in a patient with metabolic syndrome, think of fatty liver disease (enquire
about drinking history, drug history, risk factors of chronic hepatitis); note: acute hepatitis A is less
likely in a patient with only mild elevated ALT.

There is no proven treatment for MASH. Education and reversal of risk factors, including control of
HT, DM, weight reduction and regular exercise should be encouraged.

For patient with elevated ALP, think of gallstone, biliary stricture, amupillary or pancreatic head CA.
Also bear in mind the possibility of primary biliary sclerosis.

Hepatitis serology

Page 65 By Heyson Chan


Joint Pain 
HPI
Joint Pain
 Onset
 Progression
 Pattern of joint involvement – show me joint by joint
 Symmetrical / asymmetrical
 Mono-arthritis / pauci-arthritis / asymmetrical
 Small / large joints
 Frequency of attack
 Duration
 Are you completely well between attacks?
 Any aggravating factors – exercise (mechanical)
 Any relieving factors – exercise (inflammatory)
 Severity – grading, how does it affect daily activities
 Any diurnal variation
Previous attacks
 Onset – when?
 Frequency of attacks
 Duration of typical attack
 Describe the disease course chronologically
 Any difference from previous episodes
 Any previous investigations
 Any treatment and its efficacy and side-effects (e.g. NSAID – GIB)
 Any physiotherapy / occupational therapy received?
Joint symptoms
 Any redness / hotness / swelling of the joint
 Any morning stiffness – for how long?
 Any joint deformity
Associated symptoms
 Any anemic symptoms – postural hypotension / malaise / palpitation
 Any SOB
 Any rash / skin changes (SLE / psoriasis)
 Any weight loss
 Any fever
 Any trauma
 Any mood problem
 Any bleeding tendency (easy bruising)
 Any oral ulcers / photosensitivity (SLE)
 Any chest pain / SOB (serositis)
 Any frothy urine / frequency / haematuria (renal sx of SLE)
 Any seizure / headache / previous CVA / hallucinations (neuro sx of SLE)
 Any abdominal pain / ascites (GI sx of SLE / IBD)
 Any recent diarrhea (IBD / Reiter’s / STD)

Page 66 By Heyson Chan


Functional status
 Basic ADL – dining, bathing, toileting
 Advance ADL – shopping, doing housework, cooking
 Walking ability – distance and any walking aids needed?
 Living environment (which floor, any lift, type of housing)
 Working environment and job nature
 Walking ability (walks with stick / frame)
 Other daily activities – e.g. cooking / housework / eating
 Family status
 Family / social support
 Financial status (especially for patients who may not able to work)

PMH
 Chronic renal disease (gout)
 TB / other active infections (prepare for treatment by biologic agents)

SHx
 Smoking, drinking (gout)
 Diet (gout)
 Occupation, recent stressors
 How does it affect your daily activities / social functioning?

FHx
 Rheumatological / other auto-immune disorders

DHx
 Anti-hypertensive – diurectics (gout)
 NSAID / steroid / disease modifying agents / other medications and its side-effects for joint
problems

DDx
Monoarthritis
 Gout
 Septic arthritis
 Osteoarthritis
Polyarthritis
 RA
 SLE
 Psoriasis
 AS
 HSP
 Leukemia
 Bleeding disorders leading to haemathrosis
 Haemachromatosis

Page 67 By Heyson Chan


Malaise 
HPI
Malaise
 Onset (sudden vs gradual)
 Progression
 Frequency and duration
 Character: clarify what the patient meant (poor concentration? Tiredness? Weakness? Sensory
disturbance?)
 Any precipitating factors (e.g. exercise)
 Any relieving factors (e.g. rest)
 Severity: how does it affect the patient’s daily activity?
 Any previous episodes

Associate symptoms
 Any cold intolerance / weight gain / goiter / constipation (hypothyroidism)
 Any anorexia / weight loss (malignancy)
 Any GIB / changes of bowel habit / haemoptysis / neck lump (malignancy)
 Any fever / night sweat / rash (chronic infection)
 Any tarry stool / coffee ground / menorrhagia in female (anemia)
 Any snoring / early morning headache (OSA)
 Any previous investigations and results (e.g. low BP, ECG)
 Any mood problem / insomnia (depression)
 Any recent increase in physical demand (e.g. change of job nature)

If suspecting hypopituitarism
 Any visual disturbance (crainophargioma)
 Low BP (ACTH)
 Growth (GH)
 Menstrual disturbance / infertility (LH/FSH)
 Libido / impotence (LH/FSH)
 Cold intolerance / weight gain / goiter / constipation(TSH)
 Urinary pattern – amount and frequency (ADH – if posterior pituitary also involved)

PMH
 Malignancy
 NPC with radiation / pregnancy complication (suspect hypopituitarism)
 Ask about pregnancy history if suspecting hypopituitarism (sex hormone)

SHx
 Smoking, drinking
 Occupation
 Identify recent stressors and worries
 Functional status
FHx
 Malignancy

DHx
 Any use of anti-epileptics

Page 68 By Heyson Chan


Questions
1 What are the differential diagnoses?
2 What investigations would you like to perform?
3 If the patient has hypopituitarism, what is your management plan?
4 What are the causes of hypopituitarism?

Page 69 By Heyson Chan


OSA 
HPI
OSA
Presenting complaints
 Loud snoring
Associated symptoms
 Witnessed apnea during sleep
 Frequent turning during sleep
 Early morning headache (due to CO2 retention)
 Unrefreshing sleep
 Poor daytime concentration – any accident? How does it affect the patient?
 High blood pressure
 Nocturia
 Sexual dysfunction
 Mood change
 Atypical chest pain (due to breathing against closed airway)
 Dry mouth
Sleep pattern
 Usual time for bed
 How long does it take for you to get into sleep
 Usual time of waking up
 Sleep position (supine / lateral)
 Shift work required?
Complications of OSA
 Poor daytime concentration
 Accidents (secondary to poor daytime concentration)
 Early morning headache (CO2 retention)
 Blood pressure (hypertension)
 Atypical chest pain (due to effort breathing against closed airway)
 Diminished libido and impotence
Assessment of risk factors
 Body weight and height (for BMI)
 Weight circumference
 Neck circumference
 Any facial deformity
 Collar size
 Cranial facial abnormalities
 Any weight gain / cold intolerance / goiter (hypothyroidism)
 Any recent change in shoe size / ring size (acromegaly)
Disease progress, investigations and treatment received
 Sleep study
 AHI = no of apnea and hyponea per hour of sleep (>5 = OSA; >20 in elderly)
 RDI = no of apnea, hyponea and reporatory effort related arousal per hour of sleep (>30 or
SaO2 <70% = severe; 15-30 or SaO2 70-85% = moderate; 5-15 or SaO2 >85% = mild)
 CPAP – side effects and tolerance
Cardiovascular risk factors
 Hypertension / hyperlipidemia and its control
 Diet

Page 70 By Heyson Chan


 Smoking / drinking
 Exercise
 Family history of cardiovascular event

PMH
 Hypothyroidism / acromegaly (associate with OSA)
 Allergic rhinitis (difficult to cope with CPAP)
 LMP in female (menopause – risk factor of OSA)
 Other cardiovascular risk factors – DM, HT, hyperlipidemia (syndrome X)

SHx
 Smoking (worsens OSA) / drinking
 Occupation – does the patient need to drive (esp. lorry driver)?
 How does the disease (esp poor concentration) affect your daily activities / work?
 How does the disease affect family relationship (e.g. loud snoring may disturb the wife from
having a good sleep)

FHx
 OSA
 Cardiovascular events, including age of onset
 DM / metabolic disease

DHx
 Use of sedatives (risk factors for OSA)

Risk factors
 Male / postmenopausal female
 Aging
 Obesity
 Use of alcohol
 Use of sedatives
 Craniofacial abnormalities
 Congenital disorders

Complications
 Pulmonary hypertension
 Systemic hypertension
 Accident due to impaired concentration
 Other cardiovascular events (e.g. ACS, stroke, heart failure)

Questions
1 How would you examine this patient?
2 What investigations would you like to perform?
3 What are the treatment options for OSA? Describe the pros and cons for each option.
4 How would you educate the patient?
5 What are the complications of OSA?

Page 71 By Heyson Chan


Palpitation 
HPI
Palpitation
 Onset
 Progression / episodic
 Duration
 Frequency
 Character – regular / irregular? Fast / slow? Show me by tapping on the table
 Aggravating factors – e.g. anxiety / coffee / excercise
 Relieving factors – e.g. valsavar
 How does an episode stop?

Previous episodes
 First time – when?
 Frequency of attacks
 Any investigations done
 Any medications taken? Dosage, side-effects and compliance
 Any difference from previous episodes?

Associated symptoms
 Any chest discomfort
 Any dizziness / LOC
 Any SOB
 Any postural hypotension / malaise / weight loss / source of bleeding (anemia)
 Any feeling of hunger / confusion / tremor during an attack (hypoglycemia)
 Any heat intolerance / sweating / diarrhea / tremor / goiter (thyrotoxicosis)
 Any PND / orthopnea / edema (heart failure)
 Any episodic flushing / tremor / headaches (pheochromocytoma)
 Constitutional symptoms: malaise / weight loss / sleep
 Chance of pregnancy in woman of reproductive age group

PMH
 DM / HT / hyperlipidemia
 IHD / angina / LOC
 Known cardiac disease
SHx
 Smoking
 Coffee / tea / alcohol
 Occupation, stress
FHx
 Known cardiac disease
DHx
 Use of anti-arrythmics
 Use of anti-hypertensives
 Over-the-counter medication
 “Weight-losing” medication (thyroxine)
 Allergy history

Page 72 By Heyson Chan


DDx
 Physiological: exercise / stress / pregnancy
 Pharmacological: anti-arrhythmic / thyroxine / coffee / alcohol
 Pathological: anemia / thyrotoxicosis / hypoglycemia / PE / vasovagal / infection
 Physical deconditioning
 SVT: AF / AVNRT / AVRT / WPW
 VT
 Bradycardia: intermittent heart block / SSS / drugs
 Psychological

Page 73 By Heyson Chan


Ptosis 
HPI
Ptosis
 Onset (sudden vs gradual) how does the patient notice?
 Progression
 Unilateral / bilateral
 Duration (how long does it takes to get back to normal?)
 Any precipitating factors
 Any relieving factors
 Severity: complete / partial ptosis
 Any diurnal variation (MG)

Associated symptoms
 Any double vision
 Maximize at which direction
All direction: MG
All except abduction of affected side: 3rd nerve palsy
 Any precipitating factors
 Any muscle weakness / fatigue
 Pattern of weakness
 Any precipitating factors
 Any relieving factors
 Any diurnal variation
 Any loss of sweating on affected side (Horner’s syndrome)
 Any SOB / dysphagia (myasthenia crisis)
 Speech
 Any motor / sensory disturbance (CVA involving third nerve)
 Any cough / haemoptysis / SOB (CA lung causing Horner’s syndrome)

PMH
 HT (risk factors for third nerve palsy)
 DM (risk factors for third nerve palsy)
 Lung cancer (Horner’s syndrome)

SHx
 Smoking, drinking
 Occupation

FHx
 Similar conditions

DHx
 Allergy / regular use of medication

Page 74 By Heyson Chan


DDx
Unilateral
 Third nerve palsy
 Myasthenia gravis
 Horner’s syndrome
 Congenital or idiopathic

Bilateral
 Myasthenia gravis
 Dystrophia myotonica
 Ocular myopathy
 Congenital
 Bilateral Horner’s syndrome (as in syringomyelia)

Page 75 By Heyson Chan


Rheumatoid Arthritis 
HPI
Joint Pain
 Onset
 Progression
 Pattern of joint involvement – show me joint by joint
 Frequency of attack
 Duration
 Are you completely well between attacks?
 Any aggravating factors – rest (inflammatory)
 Any relieving factors – exercise (inflammatory)
 Severity – grading, how does it affect daily activities
 Any diurnal variation

Joint symptoms
 Any redness / hotness / swelling of the joint
 Any morning stiffness – for how long? (>1 hour per day)
 Any joint deformity (rheumatoid nodule)
 Range of motion
Previous attacks
 Onset – when?
 Frequency of attacks
 Duration of typical attack
 Describe the disease course chronologically
 Any difference from previous episodes
 Any previous investigations
 Any treatment and its efficacy and side-effects (e.g. NSAID – GIB)
 Any physiotherapy / occupational therapy received?

Associated symptoms
 Any anemic symptoms – postural hypotension / malaise / palpitation
 Any weight loss
 Any fever
 Any skin problem
 Any change in bowel habit
 Any mood problem

Functional status
 Basic ADL – dining, bathing, toileting
 Advance ADL – shopping, doing housework, cooking
 Walking ability – distance and any walking aids needed?
 Living environment (which floor, any lift, type of housing)
 Working environment and job nature
 Walking ability (walks with stick / frame)
 Other daily activities – e.g. cooking / housework / eating
 Family status
 Family / social support
 Financial status (especially for patients who may not able to work)

Page 76 By Heyson Chan


PMH
 TB / other active infections (prepare for treatment by biologic agents)

SHx
 Smoking, drinking
 Occupation
 How does it affect your daily activities / social functioning?
 Living environment
 Working environment

FHx
 Rheumatological / other auto-immune disorders
 Family status

DHx
 NSAID / steroid / disease modifying agents / other medications and their side-effects

Questuins
1 How would you diagnose RA?
Patient must have four out of the following sever
 Morning stiffness lasting at least for 1 hour*
 Simutaneous polyarthritis (three or more joints)*
 Arthritis of hand joints (DIP usually spared)*
 Symmetrical arthritis*
 Rheumatoid nodules
 Abnormal rheumatoid factor
 Typical changes on X-ray of hands and wrists
* Must be present for at least 6 weeks

2 What is a rheumatoid factor?


 Rheuamtopid factor consists of autoantibodies of IgM type that is directed against the Fc
portion of IgG
 Presence in 80% of RA patients (if absent = seronegative RA)
 Also present in SBE, COPD, liver cirrhosis, SLE, infection

Page 77 By Heyson Chan


3 What is the management plan for RA?

First-line
 NSAID
Ibuprofen 1200mg Q8H or indomethacin 200mg Q8H
Use minimal dose or consider omeprazole/misoprostol for PU prophylaxis
May also consider COX2 inhibitor which has less GI s/e (same renal s/e)
S/E: elevated liver enzyme, renbal insufficiency, gastric bleeding

DMARDS
 Methrotrexate
Weekly injection, excreted by kidney, response noted within 3-6 weeks (rapid onset with
sustained effects)
S/E: nausea, vomiting, rashes, pancytopenia, fatal pneumonitis (3%), teratogenic
 Sulfasalzine
Metabolized to yield 5ASA and sulfapyridine, response seen in 4-6 weeks
Well-tolerated with minimal GI side effect
C/I in G6PD deficiency
 Corticosteriod
Use low dose before DMRSA takes effect
Reserved for patients who have very active disease and prolonged morning stiffness which
affects morning functions
 Biologic agents (e.g. TNF antagonist - rule out infection before use!)
 Etanrecept (monotherapy as effective as methrotrexate in improving joint site
destruction; s/e: injection site reaction, infection, malignancy)
 Infliximab (anti-TNF alpha, significant response in combination with methrotrexate;
adverse effect due to increased risk of infection)

4 What are the risk factors for NSAID-induced GIB?


 Chronically disabled
 Age > 60 years
 Previous history of proven peptic disease
 Co-administration of prednisolone > 30 mg/day
 Higher dosage of NSAIDs
 Extent of inflammatory disease for which NSAIDs are prescribed

5 What are the typical radiological appearance of RA?


 Peri-articular soft tissue swelling
 Juxta-articular osteoporosis
 Uniformed joint space narrowing
 Central and marginal erosions
 Synovial cysts formation
 Joint deformities and subluxation
 Symmetrical small joint distribution

Page 78 By Heyson Chan


SOB 
HPI
SOB
 Onset
 Progression
 Duration
 Aggravating factors
 Relieving factors
 Severity: qualify by exercise tolerance / able to speak full sentence; does it affects daily activities?
Associate symptoms
 Any cough / sputum (including haemoptysis) / fever (chest infections, COPD)
 Any chest pain (pneumothorax, pneumonia)
 Any noisy breathing (Asthma / airway obstruction)
 Any recent weight loss (TB / CA) / night sweat (TB)
 Any trauma (pneumothorax)
 Any palpitation / edema / PND / orthopnea (heart failure)
 Any postural hypotension / malaise / potential source of bleeding (anemia)
 Any unilateral leg swelling / long haul flight / prolonged sitting (DVT / PE)
 TOCC history

PMH
 COPD / asthma (atopic history as well) / heart failure
 Quantity severity by enquiring number of admissions, exacerbations and types of medications
(including O2 therapy for COPD)
SHx
 Smoking
 Occupation
 Environmentalhistory for asthma
FHx
 Contact
DHx
 Allergy / regular use of medication

DDx
 Acute: pneumothorax / heart failure / PE / foreign body / panic attack
 Sub-acute: pneumonia / COPD / asthma exacerbation / heart failure / metabolic acidosis /
collapsed lung / respiratory muscle weakness (GBS)
 Chronic: TB / anemia / CA lung / pleural effusion / pneumoconiosis / pulmonary fibrosis / chest
wall deformity / neuromuscular disease

Questions
1 What is the management plan for a patient presented with acute pulmonary edema?
 Complete bed rest, prop up
 Monitor BP/P, I/O, SaO2, CVP, RR, clinical status every 30-60 mins
 Chart I/O, body weight daily
 Oxygen (may require high flowrate / concentration)
 Low salt diet + fluid restriction (1L/day)
 Investigations: CBC with D/C, TFT, L/RFT, CXR, ECG, echocardiogram

Page 79 By Heyson Chan


 If unstable BP, giveInotropic agents (dopamine 2.5-10 g/kg/min or dobutamine 2.5-15
g/kg/min)
 Frusemide(Lasix) 40-120 mg ivfor symptomatic relief
 IV nitrate e.g. isoket 2-10 mg/hr
 Morphine 2-5 mg slow iv
 If the patient has arrhythmia, use amiodarone or consider implantable defibrillator
 Consider ventilatory support in case of desaturation, patient exhaustion, cardiogenic shock

2 What are the causes of acute exacerbation of APO?


 Arrhythmia (e.g. AF, atrial flutter) – common
 IHD (especially silent MI in elderly) – common
 Uncontrolled HT– common
 Anemia – common
 Chest infection– common
 Thyrotoxicosis
 Poor compliance to medication

3 What is the chronic management plan for a patient with heart failure?

General
 Gentle exercise according to capacity
 Weight reduction
 Low salt diet

Pharmacological
 Diurectics – for symptomatic relieve
Frusemide (judicious use in diastolic failure)
 ACEI – improves signs and symptoms, improves ET, reduce mortality
Captopril 6.25mg, enalapril 2.5mg
S/E: symptomatic hypotension, deterioration of renal function (check renal function before
giving ACEI), hyperkalemia, cough (common in Chinese)
Check RFT 2-3 weeks after starting ACEI
 Aldosterone antagonist – reduce mortality and reduce salt and water retention
Spironolactone, safe in combination with ACEI but potential hyperkalemia
 Beta-blocker – improves survival, imporves LC ejection fraction
Carvedilol, metoprolol, bispolol – start at very low dose, improves slowly
 Vasdilator – nitrates for APO
 Digoxin – reduce sympathetic activity, mild positive inotropic agent
 Amiodarone – only antiarrythatics suitable for heart failure patients
 Inotropic support for cardiogenic shock

Non-pharmacological
 Surgical
 Implantable defibrillator
 Biventricular pacing
 Cardiac transplantation

Page 80 By Heyson Chan


4 What is the acute management for COPD exacerbation?
 Controlled low dose oxygen administration (start with 24% Venturi mask or 1-2L/min by
nasal prongs).
 Check ABGs within 30-60 mins of starting oxygen, modify flow rate according to PaO2 and
pH
 Other Ix: CBC with D/C, L/RFT, sputum for C/ST, ECG
 CXR (pay attention to any penumothorax)
 2 agonist (salbutamol 5mg Q4H puff) and ipratropium bromide alone or in combination
 If no response, consider iv aminophylline
 Corticosteroids (hydrocortisone 100 mg iv Q6-8 hours or Prednisolone 30-40 mg orally per
day). Steroid should be quickly discontinued after the acute episode and long term use
determined by steroid trial when patient is stable
 Prescribe an antibiotic (augmentin 1mg po) if two or more of the followings:
i) Increased breathlessnesss;
ii) Increased sputum volume;
iii) Development of purulent sputum
 If pH is < 7.26 and PaCO2 is rising, consider ventilatory support.

5 What are the criteria for use of NIPPV?


 Moderate to severe SOB with use of accessory muscles
 Moderate acidosis and hypercapnia
 Respiratory rate >25 per minute
(Contraindications and complications are written in “COPD”)

6 What are the criteria for initiating home oxygen therapy?


 PaO2< 7.3 kPa or SaO2< 88% on room air at rest, or
 In the presence of cor pulmonale when PaO2 7.3-7.9 kPa or SaO2 > 89%, andcor pulmonale
in ECG, haematocrit > 55%, congestive heart failure
 In some specific situations, when PaO2> 8.0 kPa or SaO2> 90% in the presence of lung
disease and other clinical needs, such as sleep apnoea with nocturnal desaturation not
correctable by CPAP

7 What is the chronic management plan for COPD?


Principle of chronic management for COPD can be summarized as below:
 Smoking cessation - slow disease progression
 Vaccination – influenzae, pneumococcus
 Drugs:
 Short acting bronchodilator
 Long acting bronchodilator
 Inhaled corticosteroids
 Pulmonary rehabilitation
 Long term oxygen therapy – improves survival
 Surgical treatments
For detailed GOLD guideline management, please refer to “COPD”

Page 81 By Heyson Chan


Syncope 
HPI
Syncope
Before the event
 Exact circumstances before the event (what exactly were you doing – describe in detail?)
 Any aura (epilepsy)
 Any palpitation / chest discomfort (cardiac cause)
 Any hunger feeling / tremor / other neurological symptoms (hypoglycemia)
 Any source of bleeding, e.g. tarry stool, menorrhagia (anemia)
The event
 Onset (give the exact timing of event)
 Duration of event
 Any loss of consiousness / aware of what is happening (seizure)
 Any twitching of limbs – describe in detail (one sided or generalized at onset?)
 Any biting of tongue / up-rolling eyeball (seizure)
 Any incontinence (seizure)
 Any fall / head injury / weakness
 Any witness
After the event
 How did the event ceased?
 Any injury noticed
 Any amnesia / clouding of consciousness (do you know what happened?) –how long did it take
to recover?
 Any weakness / tiredness (post-seizure or CVA)
 Any sensory disturbance (CVA / TIA)
 Any visual or hearing disturbance (CVA / TIA)
 Color of face (pale / red)
Previous episodes, if any
 First episode– when? Briefly describe
 Frequency of attacks
 Any difference from this attack?
 Any investigations done?
 Any current medications for this condition? Side-effects and compliance
 Recent precipitating factors e.g. sleep deprivation / lack of compliance / stress / recent infection /
change of medication

PMH
 Epilepsy –frequency of attack, current medication and s/e, compliance
 Neurological disease, e.g. brain tumor, stroke, previous head injury
 Known cardiac disease, including arrhythmia, pacemaker
 DM

DHx
 Anti-epileptics – compliance!
 Oral hypoglycemic agents / insulin
 Anti-hypertensive
Recreational drugs

Page 82 By Heyson Chan


SHx
 Smoking, drinking
 Occupation
 Risk assessment (e.g. driving, swimming alone, bathing)

FHx
 Cardiac disease, sudden death

DDx
 Neurological: epilepsy, CVA, TIA
 Cardiac: vasovagal, hypotension, arrhythmia, aortic stenosis
 Hypoglycemia
 Anemia
 Psychological: panic attacks, hyperventilation

Questions
1 What is your immediate management plan for a patient presented with syncope?
 Maintain ABC
 H’stix to rule out hypoglycemia
 BP/P/T, GCS, respiratory pattern, fundi, evidence of trauma,meningism, brainstem reflexes,
alcohol smell
 Blood sugar with h’stix
 L/RFT, TFT
 ABG
 Blood andurine toxicology
 CXR, ECG
 CT brain
 If indicated: CSFexamination, EEG, cortisol, serumosmolality

2 Briefly comment on the specific treatment for different causes of syncope.


 D50 40 ml iv for hypoglycaemia
 Thiamine 100 mg iv for alcoholic or malnourished patient
 Naloxone (narcan) 0.8 mg to 2 mg iv stat, then every 2mins prn up to 10 mg for suspected
narcotic overdose
 Flumazenil (Anexate) 0.2 mg followed by 0.3 mg at 1min , then 0.5 mg every 1 min to a total of
3 mg forsuspected benzodiazepine overdose
 Antidote or specific therapy (if available) for other drugoverdose
 Definitive treatment for the cause of coma

Support measures:
 Close monitoring of vital signs and neurological status
 Proper positioning and turning to avoid aspiration, pressurenerve palsy, contracture, pressure
sore
 Bladder catheterisation
 Adequate hydration and nutrition
 Chest and limb physiotherapy
 Hypromellose eyedrops and secure eyelids if nospontaneous blinking

Page 83 By Heyson Chan


Tremor 
HPI
 Tremor
 Onset (sudden vs gradual) how does the patient notice?
 Progression
 Episodic / continuous
 Unilateral / bilateral
 Duration
 Severity: Can the patient write / drinks? Can he walk without aid? How does it affect his daily
activity?
 Aggravating factors: under what circumstances? (resting / intentional)
 Relieving factors
 Any previous investigations

For previous treatment,


 Response (L-dopa responsive  PD)
 Time needed for drug to be effective
 Duration of drug effect
 Side-effects
 Peak-dose dyskinesia / wearing off for L-dopa
 Nausea / hypotension for dopamine agonist
 Visual hallucination and anti-ACh side-effects for Artane (anti-ACh)

Features of Pakinsonism
 Bradykinesia – slowness in motion
 Rigidity
 Postural instability
 Freezing - difficulty in getting up from sitting position / initiating gait
 Mood / sleep disturbance / sensation of restlessness / constipation / sexual dysfunction / loss of
facial expression

Associated symptoms
 Any weakness (CVA)
 Any fall / gait disturbance (cerebellar / MSA / normal pressure hydrocephalus)
 Any vertigo (cerebellar)
 Any change on his speech (scanning speech / monotone)
 Any dementia (Lewy-body dementia)
 Any visual disturbance (SNP)
 Any incontinence / postural hypotension (MSA / normal pressure hydrocephalus)
 Any respiratory stridor (SND)
 Any head injury (HI)
 Any weight loss / heat intolerance / proptosis / sweating (thyrotoxicosis)
 Any hunger feeling / palpitation / neurological disturbance (hypoglycemia)

Page 84 By Heyson Chan


PMH
 Psychiatric illness (use of anti-psychotics)
 COPD / asthma requiring bronchodilator
 Head injury
 Previous suicidal attempt (CO poisoning)
SHx
 Smoking
 Coffee / tea / alcohol
 Environmental history (e.g. living environment, any lift)
 Caring issue
 Occupation (e.g. Boxer)
FHx
 Wilson’s disease
 Essential tremor
DHx
 Anti-psychotics
 Drug abuse (amphetamine)
 Beta-agonist (e.g. bronchodilators)
 “Weight-losing” medication (thyroxine)
 Allergy / regular use of medication

DDx
 Parkinsonism (primary or secondary)
 Cerebellar disease (due to stroke / tumor / multiple sclerosis)
 Hypoglycemia
 Thyrotoxicosis
 Essential tremor
 Drugs

Questions
1 Name some anti-Parkinson’s medication and give their common side-effects.
2 What are the features of Parkinson’s disease?
3 What is Parkinson’s plus? Give some features.

Page 85 By Heyson Chan


UGIB 
HPI
UGIB
 Onset
 Character: coffee-ground / fresh haematemesis
 Severity
 Number of episodes
 Estimate amount by number of “rice bowl”
 Any blood clot
 Any anemic symptom e.g. dizziness / postural hypotension / SOB / palpitation
 Precipitating event – e.g. vomiting
 Any previous episodes before
 Any relation to meal

Associate symptoms
 Any abdominal pain
 Any tarry stool
 Any chronic liver stigmata – abdominal distension / edema / jaundice
 Any symptoms suggesting underlying maliganacy – dysphagia / change of bowel habit
 Constitutional symptoms – weight loss / malaise / loss of appetite / SOB
 Any bleeding tendency – easy bruising / bleeding elsewhere
 Any respiratory symptoms – cough / fever / sputum / haemoptysis

PMH
 H Pylori status
 IHD / CVA / angina requiring aspirin
 Chronic pain / joint problem e.g. gout / SLE / RA requiring NSAID
 Heart disease (AF / valvular disease) requiring warfarin
 Hepatitis vaccine

SHx
 Smoking, drinking
 Occupation
 Sexual history

FHx
 Hepatitis status / complications e.g. cirrhosis, HCC

DHx
 NSAID / aspirin (specifically ask the patient if suspect GIB)
 Warfarin
 Other medication

DDx
 General: bleeding tendency / exclude pseudo-haematemesis
 Local: PU / variceal bleed / esophagitis / gastritis / MW tear / drug-related / malignancy

Page 86 By Heyson Chan


Questions
1 What is your immediate management plan for a patient with acute UGIB?
Emergency Management (Consider ICU care if severebleeding)
 Nil by mouth
 Insert large bore IV cannula
 Closely monitor BP, Pulse, I/O, CVP if BP < 90 mmHg
 Infuse full rate normal saline
 Emergency cross match, clotting profile; transfuse if necessary (usually if failed to restore BP
after 1L of normal saline)
 Cuffed ET tube to prevent aspiration if massivehaematemesis, nasogastric tube if massive
haematemesisor signs suggestive of GI obstruction or perforation
 Look out for and treat any medical decompensationsecondary to GIB
 IV proton-pump inhibitor may be beneficial
 Arrange emergency endoscopy after initial stabilization
 Consider emergency operation if
 Arterial bleeding not controlled by endoscopic treatment
 Transfusion > 8 units
 Rebleeding after apparently successful endoscopic therapy

2 What are the contraindications for Endoscopy?


 Suspected intestinal perforation (e.g. free gas under diaphragm)
 Suspected intestinal obstruction (e.g. bile stained vomitus)
 Dysphagia without delineation of level of obstruction
 Unstable cardiac or pulmonary status (e.g. shock, cardiac ischemia)

3 What is your subsequent management for a patient with gastric ulcer bleed?
 After endoscopic treatment of patients with activelybleeding ulcer or ulcer with visible vessel,
bolusomeprazole 80 mg IVI stat followed by infusion of 8mg/hr for 3 days reduced the risk of
rebleeding (by increasing the pH, thus stabilizing the clot)
 Take a gastric biopsy at the antrum for rapid urease test; if H Pylori test +ve, start anti- H
Pylori treatment
 Give triple therapy for 1 week
 Omeprazole 20 mg bd or lansoprazole 30 mg bd orpantoprazole 40 mg bd
 PLUS Amoxycillin 1 g bd or metronidazole 400 mg bd
 PLUS Clarithromycin 500 mg bd

 Warn the patient about GI upset with triple therapy


 Give ulcer-healing dugs
 H2-antagonists for 8 weeks (Cimetidine 400 mg bd or 800 mg nocte)
 PPI for 4 - 6 weeks (Omeprazole or esomeprazole 20 mg om)
 Sucralfate 1 g qid for 6 - 8 week (not recommended for renal failure)

 If the patient is on aspirin / NSAID, balance the risk and benefits of stopping the medication
 Re-check H Pyloric status 8 weeks after triple therapy (by CLO test if GU and by urease
breath test if DU)
 Re-scope 8 weeks after and obtain gastric biopsy (at ulcer edge) to rule out CA stomach if
gastric ulcer (not necessary for duodenal ulcer)

Page 87 By Heyson Chan


4 What is your immediate management plan for a patient with variceal bleed?
 Resuscitation as in any other case of Upper GI bleeding
 Investigations
 CBP, LFT, RFT
 PT, APTT & platelet
 Serology for HBV and HCV
 AFP
 Abdominal ultrasound
 Vasoactive Agents
 Octreotide 50g iv bolus, then50 g/h iv infusion
 Somatostatin 250 g iv bolus, then250 g/h iv infusion
 Vasopressin 0.4 units/min iv infusion(Watch out for cardiovascular complications)
 Anti-encephalopathy regimen
 Correct fluid and electrolyte disturbance
 Lactulose 10-20 ml q4H-q8H to induce diarrhea
 Low protein and low salt diet

5 What is your subsequent management plan for a patient with variceal bleed?
 Sclerotherapy / band ligation for visible varices
 Consider TIPS / surgery if re-bleeding after 2nd endoscopic treatment

6 If emergency endoscopy is not readily available, what can you do to control bleeding
varices?
 Vasoactive Agents
 Octreotide 50g iv bolus, then50 g/h iv infusion
 Somatostatin 250 g iv bolus, then250 g/h iv infusion
 Vasopressin 0.4 units/min iv infusion(Watch out for cardiovascular complications)
 Balloon tamponade if
 Urgent endoscopy not available
 Vasoactive agents fail to control bleeding
 Recurrent bleeding after endoscopy

Page 88 By Heyson Chan


Vertigo 
HPI
Vertigo
 Differentiate between light headedness, vertigo and LOC
 Onset (sudden vs gradual)
 Progression
 Frequency and duration
 Any precipitating factors (e.g. recent URTI, motion sickness)
 Any relieving factors (e.g. rest)
 How does an event end?
 Severity: how severe it was? Any LOC (important!)?
 Any previous episodes, any difference from this episode e.g. duration
Associate symptoms (for light headedness)
 Any hearing disturbance / tinnitus (Meniere’s disease)
 Any nausea (BPPV) / vomiting
 Any tremor / truncal ataxia (cerebellar lesion)
 Any walking difficulty / gait disturbance / fall (cerebellar lesion)
 Any dysarthia (cerebellar lesion)
 Any neck stiffness / limited range of movement (cervical spondylitis)
 Any headache (migrane)
 Any head / neck injury (HI)
 Any convulsion / LOC / involuntary movement (partial seziure)
 Screen for neurologicalsymptoms– weakness, sensory disturbance, headache, LOC, dysphagia,
visual disturbance, facial asymmetry (neurological cause)

PMH
 Recent URTI (vestibular neuronitis / acute labyrinthitis)
 HT, DM, hyperlipidemia, TIA, CVA (risk factors for cerebellar stroke)

SHx
 Smoking, drinking
 Occupation and accidental risk assessment (especially in elderly)
 Functional status

FHx
 Neurofibromatosis, brain tumor

DHx
 Ototoxic drugs (e.g. gentamicin, anti-convulsants)

DDx
 Seconds: BPPV
 Minutes: TIA / vestibular insufficiency
 Hours: Meiner’s disease / migrane
 Days: vestibular neuronitis / acute labyrinthitis / cerebellar stroke
 Constant: neurological disorder (MS, acoustic neuroma, temporal lobe epilepsy) / recovery of
vestibular failure / psychogenic

Page 89 By Heyson Chan


Weakness 
HPI
Weakness
 Onset (sudden vs gradual) how does the patient notice?
 Progression
 Pattern of weakness
 Proximal: difficulty in getting up from bed / walking upstairs / raise from sitting position /
combing hair
 Distal: difficulty in holding chopsticks / writing
 Generalized
 Paraplegia
 Duration
 Any precipitating event – e.g. sweating, heavy meal, vigorous exercise
 Severity: Can the patient walk? Any dysphagia / SOB (severe!)
 Any diurnal variation (MG)
Localization of lesion
 Any sensory disturbance (neuropathy, spinal cord lesion)
 Any diplopia / ptosis (MG)
 Any facial asymmetry
 Any thyroid symptoms (TPP / endocrine)
 Any chronic use of steroid (steroid induced myopathy)
 Any rash (dermatomyositis)  if positive, enquire possible malignancy
 Any injury (head / neck / spine injury)
 Any urinary symptoms / on diuretics (frequency  electrolyte disturbance)

PMH
 Recent infection (respiratory tract / diarrhea – GBS)
 Thyroid disease (TPP)
 HT (use of diuretics)
 DM (DM myopathy)

SHx
 Smoking, drinking
 Environmental history (e.g. living environment, any lift)

FHx
 Muscular dystrophy

DHx
 Steroid, diuretics
 Allergy / regular use of medication

Objectives
1 Presenting complaints – characteristics
2 Pattern of weakness
3 Severity – functional disturbance
4 Other neurological symptoms
5 Possible causes

Page 90 By Heyson Chan


DDx
Weakness with sensory disturbance
 Neuropathy (distal) e.g. GBS (acute), Charcot Marie Tooth (chronic)
 Spinal cord lesions (with para- / tetraplegia and sensory level): trauma
Weakness without sensory disturbance
 Inherited (proximal): muscular dystrophies
 Inflammation (proximal): polymyositis, dermatomyositis, polymyalgia rheumatica
 Endocrine disorders (proximal): thyroid problem, Cushings, DM
 Electrolyte disorders (generalized): hypoklameia, hypocalcemia, hypomagnesemia
 Toxic / drug-induced (proximal): steroid, statins, alcohol
 Neuromuscular (variable): MG, myopathy, motor neuron disease, muscle dystrophy
 Others: paraneoplastic syndrome

Questions
1 What are the causes of spinal cord compression?
Traumatic
 Injury to the spinal cord
Non-traumatic
 Spinal cord compression (e.g. due to tumor)
 Inflammatory (e.g. due to multiple sclerosis)
 Infective

2 What investigations would you like to perform in patients with lower motor neuron lesions?
Routine
 L/RFT, glucose for electrolyte disturbance
 TFT for metabolic myopathy
 CPK for myositis
 CXR for CA lung causing paraneoplastic syndrome
 ECG for cadiomyopathy associated with myopathy
 FVC for monitoring lung function (respiratory reserve)
Specific
 EMG
 Nerve conduction study
 Muscle biopsy
 Nerve biopsy
 MRI for muscle or nerve swelling and inflammation

3 How would you manage a patient with polymyositis / dermatomyositis?


 Bed rest at acute stage
 Investigations as above
 Prenisolone 1mg/kg daily for 5-8 weeks (then maintenance dose for ~1year)
 If steroid resistant, consider methotrexate, azathioprine and high dose IVIG
 Treat underlying malignancy

Page 91 By Heyson Chan


4 How would you manage the patient with acute spinal cord syndrome?
 Correct any compromised airway, breathing and circulation
 Immobilize relevant level of spine in case of traumaticspinal cord injury or spine instability.
 Initiate appropriate treatment for specific spinal cordlesions:
 Neurosurgical / orthopaedic consultation for structural lesions
 Antimicrobial therapy for abscess or other infections
 Methylprednisolone 1 gm intravenously over one hour daily for 4 days, may be useful in
non-infectious inflammatory myelitis
 Mono-traumatic acute cord syndrome, start steroid within 8hrs: methylprednisolone 30
mg/kg iv in 15 min, followed by 45-minute pause, then 5.4 mg/kg/hr iv for 23 hrs

 Institute general supportive care:


 Proper positioning & splinting
 Adequate hydration and nutrition
 Bladder catheterization
 Regular monitoring of vital signs
 Close monitoring of respiratory function (FVC, respiratoryrate) in case of high cord lesions

5 What is the management plan for a patient with acute cord compression due to tumor?
 Immediate decompression by
 Emergency surgery, or
 Immediate radiotherapy

6 How would you manage a patient with myasthenia crisis?


 Admit ICU
 Watch out for respiratory failure in any patient withprogressive weakness
 Regularly monitor FVC (peak flow rate, SaO2, ABG not useful)–Intubate and initiate
mechanical ventilation if FVC < 15-20ml/kg or patient exhausted
 Stop anticholinesterase
 Start prednisolone 50-70 mg/day, early steroid-induceddeterioration may occur.
 Perform plasma exchange 50 ml/kg daily or on alternate daysuntil adequateresponse achieved
(usually after 2-5exchanges) or administer IVIG 0.4 g/kg/day for 5 days
 Resume anticholinesterase at a smaller dose 48-72 hours afterstabilization and titrate
according to response.
 Identify and treat any precipitating conditions

Page 92 By Heyson Chan


7 What is the general management plan for myasthenia?
Depends on mode of presentation
Pure ocular MG
 Anti-cholinesterase (pyrodpstigmine)
 Steroid may help
Mild generalized MG
 Anti-cholinesterase (pyrodpstigmine)
 Young patient may benefit from thymectomy in long term
Moderately severe MG
 Anti-cholinesterase are usually inadequate
 Steroid and azathoprine are usually required
 Early thymectomy recommended
Fulminating MG
 Elective ventilation
 Plasma exchange
 IVIG
 Thymectomy

8 What drugs may precipitate myasthenia?


 Aminoglycosides
 Quinine
 Quinidine (“gin-tonic” effect)
 Procainamide
 Beta-blockers
 Musclerelaxants
 Penicillamine

Page 93 By Heyson Chan


Weight Loss 
HPI
Weight Loss
 Onset (sudden vs gradual)
 Progression
 Severity: what is your usual weight? What is your weight now? What is the rate of weight
reduction?
 Is this intentional? What do you think about your weight now? (anorexia nervosa) – quantify
how much you eat in a meal

Associate symptoms
 Any heat intolerance / goiter / diarrhea / proptosis / visual disturbance (thyrotoxicosis)
 Any polyuria / polydipsia / loss of appetite (DM)
 Any anorexia / malaise (malignancy)
 Any GIB / changes of bowel habit / haemoptysis / neck lump (malignancy)
 Any fever / night sweat / rash (chronic infection)
 Any tarry stool / coffee ground / menorrhagia in female (anemia)
 Any previous investigations and results (e.g. low BP, ECG)
 Any mood problem / insomnia (depression)
 Any recent increase in physical demand (e.g. change of job nature)

PMH
 Malignancy
 Chronic infection (e.g. TB)

SHx
 Smoking, drinking
 Occupation – any recent change in job nature, increase in physical demand
 Identify recent stressors and worries (depression)
 Assess diet (malnutrition)

FHx
 Malignancy
 DM
 TB

DHx
 “Weight losing” medication (diurectics / thyroxine)
 Any use of anti-epileptics
 Herbs

Page 94 By Heyson Chan


DDx
 Malignancy
 Hyperthyroidism
 DM
 Chronic infection (e.g. TB / AIDS)
 Chronic disease
 Malnutrition
 Depression
 Intentional weight loss
 Drug induced (e.g. thyroxine, diuretics)

Objectives
1 Presenting complaints
2 Any signs of malignancy
3 Any signs of chronic infections (e.g. TB)
4 Any signs of DM
5 Any signs of hyperthyroidism
6 Drug history

 
 

Page 95 By Heyson Chan


Abdomen Examination 
Routine Introductions
1 Introduce yourself and ask for consent from the patient
2 Proper position: lye the patient flat on one pillow with hands on the side
3 Proper exposure: from nipple to mid-thigh
4 Comment on the general condition of the patient
- Conscious and alert
- Oxygen supplement
- Any heparin block / iv infusion

Inspection
 Be systematic. Start the inspection from the hand, then from the head, down to the trunk, the
abdomen, the groin and finally the lower limbs.
 Observe from the end of the bed and ask the patient to take a deep breath. The examiner may
stand at the end of the bed and block your way, remember to ask him to give you a way politely.
 * = Stigmata of chronic liver disease
 # = Alcoholism

1 Inspection of the hands


a) Clubbing
 Examine the fingers at eye level
b) Palmar erythema *
c) Dupylture contracture #
 Feel the thickening of palmar fascia before you comment
d) Hepatic flap *
 Ask patient to stretch out arms with the wrist cocked up
e) Leukonychia
 Signifies hypoalbuminaemia
f) Scratch marks
g) Tattoos / needle punctures
 Risk of hepatitis B / C infection
h) Pulse
i) Any bruises *
j) AV fistula
2 Inspection of the head
a) Eye
 Pallor
 Jaundice *
 Cyanosis
b) Mouth for glossitis
c) Enlargement of parotid glands #
 Palpate the parotid gland in front of the ears
d) Enlargement of Virchow’s nodes
 Tell the examiner that you would like to examine from the back
 Left supraclavicular nodes
 Drains organs below the diaphragm (e.g. stomach)

Page 96 By Heyson Chan


3 Inspection of the trunk and axilla
a) Loss of axilliary hair *
b) Gynaecomastia *
 Show the examiner by palpating the breast
c) Spider navi *
 Only seen in SVC drainage area
 Confirm by pressing on it – it blenches and will refill from the central
4 Inspection of the abdomen
Observe from the end of the bed and ask patient to take a deep breath and cough
a) Scars
b) Abdominal distension
c) Dilated veins (caput medusae)
 Determine the direction of flow below the umbilicus
 Caput medusae flow away from the umbilicus
d) Any abnormal mass
e) Stoma
 Ask the patient to cough to inspect for parastomal hernia
 Feel for the surrounding skin
f) Expansile pulsation in aortic aneurysm (seldom seen in exam)
5 Inspection of the groin
Ask patient to cough
 Any cough impulse
6 Inspection of the lower limb
You may leave this till the end of the examination
 Edema
Palpation and Percussion
 Kneel down or sit when you are palpating the patient, palpate with your palm and the flat of your
fingers, keep your forearm level with the abdominal wall
 Warn the patient to inform you if he feels any discomfort (never hurts your patient)
 Look at the patient when you are palpating
 Be systematic. Palpate quadrant by quadrant
 Warm your hand before you palpate – show your sympathy

1 Light palpation for tenderness and rebound tenderness


 Ask the patient if he experience any sore in the abdomen before you proceed
 Start as far as possible from the tender spot
 Should the patient experience any pain, ask the patient whether pain was felt when the you
press down or when you remove the hand quickly
2 Deep palpation for possible organomegaly
 Inform the patient you are going to palpate deeper
 If you can feel for any mass, ask the patient to look at his toes (to flex his head) for any
attachment to the abdominal muscle
3 The liver
 Start low (RIF). Instruct the patient to inspire and expire deeply
 Move your hand upward when patient expires
 Percuss and define the liver boarder (upper and lower)
(Note: always percuss from a resonant area to a dull area)
 Measure the liver span along the mid-clavicular line (show the examiner that you are
measuring along the MCL by palpating the clavicles and define its mid-point)
 Normal liver span = 13cm at the MCL

Page 97 By Heyson Chan


4 The spleen
 Start low, beginning in the right iliac fossa
 Palpate along the Gardner’s line, instruct the patient to inspire and expire deeply
 If the spleen is not palpable, turn the patient and palpate again (leave it until you perform
shifting dullness – so you need to turn the patient once only)
 Percuss along the Gardner’s line  should be resonant all along
5 The kidneys
 Ballot with one hand at transpyloric plane, the other at costovertebral angle
 Move one hand at a time only
6 Ascites
 Shifting dullness test
 Remember to wait for a few seconds before percussing again after you have turned the patient
into a lateral position
 Always percuss with your finger parallel to the level of fluid
 Also take this opportunity to palpate and percuss for splenomegaly again

Auscultation
Do not miss auscultation!
1 Bowel sound
 Sluggish in ileus
 Hyperactive in intestinal obstruction
2 Bruits
a) Abdominal aorta
 At the umbilicus
b) Renal bruit
 Next to the umbilicus
c) Any mass
 Liver bruit in alcoholic hepatitis, primary or secondaries
 Abdominal venous hum in portal venous hypertension

Routine Conclusion
1 Always tell the examiner that you would like to conclude the examination by performing a Per
Rectal examination and the examination of the external genitalia
2 Cover the patient up and make sure he feels comfortable
3 Summarize your findings and suggest possible differential diagnosis
Note: normal surrogate may be used in MED3 OSCE

Page 98 By Heyson Chan


Questions
Important Questions (must know by heart)
1 What are the differential diagnoses for a sole hepatomegaly?
Tumors
Irregular boarder, firm to hard, nodular, may associate with bruit
High in the list of DDx if there is lymph node involvement
 Primary: hepatocellular carcinoma
 Secondary
 Lymphoma, leukemia

Infections
May be tender in acute hepatitis (stretch on its capsule due to recent enlargement)
 Viral: Hepatitis B / C infections
 Bacterial: liver abscess

Alcoholic liver diseases


Non-tender, firm
 Fatty liver
 Cirrhosis (Early stage, firm, non-tender, may associate with a bruit)

Metabolic disease
 Haemachromatosis – iron metabolism problem
 Wilson’s disease – copper metabolism problem, K-F ring

Congestive heart diseases


Smooth, firm, tender liver
 Right heart failure
 Tricuspid regurgitation (pulsatile liver – ask the patient to hold the breath)

“Pseudo-hepatomegaly”
 Hyper-expanded chest

2 What are the differential diagnoses for a sole splenomegaly?


Massive (beyond umbilicus)
 Myeloproliferative disorder (e.g. myelofibrosis, CML)
 Malaria
 Kala-azar
Moderate
 Myeloproliferative disorder, e.g. myelofibrosis, chronic myeloid leukemia
 Lymphoproliferative disorder, e.g. lymphoma, leukemia
 Haemolytic anemia
 Portal hypertension secondary to liver cirrhosis
 Infective: SBE, infectious hepatitis

3 What are the differential diagnoses for a heaptomegaly associated with splenomegaly?
 Lymphoproliferative disorder, e.g. lymphoma, leukemia (important)
 Portal hypertension secondary to cirrhosis (important)
 Infectious hepatitis
 Systemic disease: amylordosis, sarcoidoisis

Page 99 By Heyson Chan


4 What are the differential diagnoses for enlarged kidneys?
Congenital
 Polycystic kidney (bilateral but asymetrical)
- Patient may present with hypertension, loin pain and haematuria
- Associate with berry aneurysm --> stroke, colonic diverticulae, MR
 Hypertrophic single functioning kidney (unilateral)
Acquired
 Renal carcinoma (unilateral or bilateral)
- haematuria, mass, loin pain
 Hydronephrosis (unilateral or bilateral)
 Renal cyst (unilateral or bilateral)
 Pyonephrosis
 Perinephric abscess

5 How can you differentiate between a splenomegaly and an enlarged kidney?

Splenomegaly Enlarged kidneys


Cannot reach over boarder Can reach over upper boarder
Dull on percussion Resonant on percussion
Moves with respiration along Not moves with respiration
Gardner’s line
Notches can be felt (medial notch) No notches
Not ballottable Ballottable

6 What are the possible causes of ascites?


Definition: pathological accumulation of fluid in the peritoneal cavity
Exudate (protein > 25g/l)
 Liver cirrhosis
 Abdominal malignancy
Transudate (protein < 25g/l)
 Congestive heart failure
 Hypoalbuminaemia
 Liver failure
 Nephrotic syndrome
Note: send ascitic tap fluid for cytology, protein and culture

7 Define clubbing. Give the differential diagnosis of clubbing.


Clubbing is defined as the loss of nail bed angle, it is seen in:
GI system
 HCC, chronic liver disease, ulcerative colitis, Crohn’s disease
Respiratory system
 Chronic suppurative lung disease, CA lung
CVS
 Cyanotic heart disease
Endocrine
 Thyrotoxicosis (also known as acropachy)

8 What are the possible causes for a mass in the supra-pubic area?
 Distended bladder
 In female: pregnant uterus, ovarian cysts and uterine fibroids

Page 100 By Heyson Chan


9 What are the possible causes for a mass in the left iliac fossa?
 Fecal loaded sigmoid colon
 CA colon
 Gynecological causes (e.g. ovarian mass)
 Transplant kidney
- Peripheral signs: anaemia, AV fistula in the hand
- Scar over the iliac fossa, swelling seen on inspection
- Mass is usually superficial, placed outside the peritoneum
- Palpate lightly!

10 What are the possible causes for a mass in the right iliac fossa?
Arising from the GI system
 CA caecum (more common)
 Crohn’s disease (common)
 Appendix mass (rarer)
 Ileoceacal TB (rarer)
Arising from the gynaecological system
 Ovarian mass
 Fibroid uterus
Arising from the urological system
 Transplant kidney
Arising from the skin and soft tissue
 Sebaceous cysts
 Lipoma

11 What is Dupuytren’s contracture? What conditions are associated with it?


Definition: thickening and shortening of palmar fascia, particularly along the medial aspect. It is
associated with,
 Age / idiopathic
 Trauma
 Alcoholism
 Cirrhosis
 Diabetes mellitus
 Anti-epileptic therapy
Note: this deformity is usually well tolerated because it exaggerates the normal position of
function of the hand.

12 What does tenderness and rebound tenderness signifies?


 Tenderness: peritoneum under or the underlying organ is inflamed
 Rebound tenderness: underlying peritoneum is inflamed

13 Why is it important to percuss the upper boarder of the liver when the liver is palpable
under thecostal margin?
 Since the liver may be pushed downwards by emphysematous lung and hence palpable
without genuine enlargement, it is important to determine the liver span and check if the
palpable liver is due to an enlargement or not

Page 101 By Heyson Chan


14 What are the causes of edema in the leg?
Increased hydrostatic pressure
 Cardiac failure
 Fluid overload
 Renal failure
Decreased oncotic pressure
 Malnutrition
 Nephritic syndrome
 Cirrhosis
Increased capillary permeability
 Sepsis
 Burns

15 What are the possible causes for abdominal distension?


 Fluid – to be detected by fluid thrill or shifting dullness
 Fetus – especially women in reproductive age
 Feces – lies in the distribution of colon, often multiple separate masses
 Fat – usually deposited in the lower half of the abdomen
 Flatus – abdomen will be hyper-resonant

16 How to differentiate a dilated vein due to portal hypertension or inferior cava obstruction?
Determine the flow of vein below the level of umbilicus
 Portal hypertension: flow away from the umbilicus
 Inferior vena cava obstruction: flow towards the umbilicus

17 What is the nervous system signs in alcoholism?


 Peripheral neuropathy
 Proximal myopathy
 Cerebellar syndrome
 Bilateral sixth nerve palsy as in Wernicke’s encephaloathy
 Recent memory loss
 Confabulation as in Korsakoff’s psychosis

18 What are the causes for gyanecomastia?


Presence of glandular tissue in male breast
 Physiological: newborn, adolescence, ageing
 Chronic liver disease
 Thyrotoxicosis
 Neoplasm: CA lung, testicular tumor, hepatoma
 Drugs
- Antibiotics: isoniazide, metronidazole
- Cardiovascular drugs: digoxin, methyldopa, spironolactone, verapamil
- Antiulcer drugs: cimetidine, omeprazole
- Psychoactive drugs: diazepam, tricyclic anti-depressants

Page 102 By Heyson Chan


19 How can you tell the difference between an ileostomy and a colostomy?

Ileostomy Colostomy
Site Right iliac fossa Left iliac fossa

Surface Spout (contents are corrosive Flush with skin


and can damage local skin)

Muscosal surface Mucosa more pinkish Mucosa less pinkish

Contents Watery – small bowel content Faeculent

Function Continuous Intermittent

20 What are the complications of stoma?


Early
 Obstruction
 Ischemia
 Retraction
 High output  fluid and electrolyte imbalance
Late
 Stenosis
 Parastomal hernia
 Dermatitis
 Infection
 Prolapse

21 What is the likely operation that is performed in a patient with loop ileostomy?
 Low anterior resection for CA rectum
 To protect the anatsomosis and reduce the complication should leakage occur

22 What is the likely operation that is performed in a patient with end colostomy?
 Anus present: Hartmann’s procedure (temporary)
 Anus absent: APR (anterior peroneal resection)

23 What are the indications of loop colostomy?


 In emergency situation e.g. divertculitis

24 What are the complications of endoscopy?


OGD
ERCP
Colonscopy

References:
1 Passing Surgery MB by Dr. Paul Lai
2 Clinical cases and OSCEs in Surgery by Ramachandran and Poole
3 Lecture Notes on General Surgery by Ellis, Calne and Watson
4 Surgical Finals – Passing the Clinical by Kuperberg and Lumley
5 250 Cases in Clinical Medicine by Baliga

Page 103 By Heyson Chan


Breast Examination 
Routine Introduction
1 Introduce yourself
2 Ask for permission to examine the patient and request for a chaperon
3 Proper position: sit the patient up in the bedside for inspection, lye on bed for palpation of the
breast; sit the patient up again for palpation of axilla
4 Expose the whole upper trunk of the patient
5 Ask chaperon

Inspection
1 Inspect both breasts: sitting and arm rest on her side
- Asymmetry: size, contour, colour
- Nipple: symmetrically everted, flat, inverted
: if unilateral flattening or nipple inversion, ask patient is recent or long-standing
appearance
: Male: flat
: aerola skin: eczema point to Paget’s disease
: any spontaneous discharge
: Destruction, Depression, Discolouration, Displacement, Deviation, Discharge,
Duplication
- Skin puckering
- Obvious lumps
- Previous scar
- Skin changes
- Prominent vein
- Edema: peau d’orange (lymphatic blockage)
2 Ask the patient to do the following maneuver
a) Raise hand above head: can accentuate any asymmetry
- strains the ligaments of Astley Cooper and may result in a skin dimple or inversion of the
nipple if there is CA
b) Lean forward
c) Put her hand on her hips and Press hard on the hip with both hands: tighten suspensory
ligament and pect major, exaggerate the contour and highlight abnormaility: observe the
breast for dimples and inversion of the nipple
3 inspect the axilla, arms, supraclavicular fossa: LN

Palpation
Patient lie flat on bed with hand on both side or held under head
Ask the patient to point to the lump
1 Palpation of the normal breast first: since breast tissue varies between women and stage of
menstrual cycle
 Quadrant by quadrant
- use palmar surface of the middle three fingers
- with rotary movement
- compress GENTLY towards the chest wall: can’t feel small soft mass if too strong pressure
- if breast is too large, can use one hand to steady the breast on its lower border while palpating
- texture: variable, with menstrual cycle. Help to tell by examine both breast

Page 104 By Heyson Chan


 then centrally around the nipple: bimanually
 Don’t forget about the axillary tail of Spence
- rest arm below her head
- feel tail between thumb and finger
- from URQ towards the axilla
 Note any discharge from the nipple, ask patient to help you by gently milk the nipple: note
colour and viscosity of the fluid, from how many duct
 Male: gynaecomastia => disc of breast tissue can be felt under areola, lobular texture
: fat => soft

2 Palpation of the affected breast


If you feel any lump, try to
Feel and measure the lump
Feel the texture
Move the skin overlying the lump
Ask the patient to rest her hand lightly against her hip and move the lump
Ask the patient to press hard against her hips and move the lump again

3 Palpation of the axilla


Palpate the right axilla with your left hand
Rest the patient’s hand near the elbow on your non-palpating hand’s forearm
Palpate the lymph nodes group by group
i) Medial: lie against the chest wall high in the axilla midway between the anterior and
posterior axillary folds
ii) Anterior: inside surface of the anterior axillary fold, along the posterior border of the
pectoralis major muscle
iii) Lateral: lie against upper humerus
iv) Posterior: lie deep to the posterior axillary fold along the lateral border of the scapula
v) Apical:
vi) Central: level 2 LN behind the pect minor therefore need not to tense. 綺夢 post (hand
on head)
4 Palpation of supraclavicular and infraclavicular LN

Describe the lump


Site: name the quadrant and
Size: measure the size approximately
Shape
Consistency – soft / firm / hard
Color change / necrosis over the skin
Contour and boarder of the lump – well / poorly defined
Any fixation to the skin: try to squeeze the skin beyond the mass and observe any indentation
Any fixation to the underlying muscle: press on the hip => if lesion become less mobile, it is fixed
Any tenderness
Any temperature change

Page 105 By Heyson Chan


Routine Conclusion
1 Palpate the cervical and supraclavicular lymph nodes
2 Feel the spine for any bone pain, spine tenderness by asking the patient to  
3 Percuss the lung base for any dullness (pleural effusion)
4 Feel the liver for any heaptomegaly
5 Check for brain
6 Cover the patient up and thank the patient
7 Report your findings to the examiner

Typical Cases
CA Breast
1 Age: usually >35
2 Positive family history
3 Lump:
Surface: irregular / nodular
Edge: poorly defined
Consistencey: firm, sometimes hard
Tenderness: non-tender
Fluctuation: non fluctuant
Mobility: Immobile, may be fixed to the skin / underlying chest wall
May involve the nipple (inverted / distorted / peu d’ orange / discharge)
May have lympadenopathy

Fibroadenoma
1 Age: usually 15-35 (peak at 15-25)
2 Lump:
Usually small
Surface: nodular
Edge: well-defined, smooth boarder
Consistency: firm rubbery hard
Tenderness: non-tender
Mobility: very mobile – known as breast mice

Fibroadenosis
1 Age: common in women of reproductive age, peak at 35-45 years old
2 Presentation:
a) Single lump (solid / cystic)
b) Multiple lumps or generalized nodularity
c) Cyclical breast pain
d) Nipple discharge (clear, white or clean)

Breast Cyst
1 Age: most common between 40-55 years old (peri-menopausal)
2 Lump
Distended, involuted lobules
Usually fluctuant
May be tender
Well defined and smooth
Can be multiple

Page 106 By Heyson Chan


Questions
1 What history would you like to elicit from the patient?
Breast lump
Onset – when and how did you notice that?
Progress – any changes?
Location
Tenderness
Any changes in relation to menstrual cycle?
Family history
Any previous history of CA breast?
Any previous investigations done?
Nipple discharge
Any discharge from both nipples?
Color of the discharge, if any
Hormonal factors
Age of menarche
Regularity of menstruation
Number of pregnancies (full pregnancy)
Any use of OC pills / HRT
Symptoms suggesting metastasis
Weight loss, bone pain, jaundice, SOB

2 How would you further investigate this patient?


I would like to do a triple assessment, which includes
Clinical assessment (Hx + P/E)
Imaging (USG / mammography)
- craniocaudal and oblique views
- characteristically a white asymmetrical speculated lesion containing microcalcification
- mammogram usually for those after 35 because too glandular before that age
Cytology (FNAC / Trucut biopsy)

3 How would you choose between ultrasound and mammography?


Ultrasound preferred in patients <35 years old (dense breast tissue)
Mammography for patients >35 years old

4 What is the TNM staging? What is the importance?


a) Tumor (T)
T0: no evidence of primary tumor
T1: tumor <2cm
T2: tumor <5cm but >2cm
T3: tumor >5cm
T4: tumor of any size with skin or chest wall involvement
b) Lymph node (N)
N0: no regional LN involvement
N1: movable ipsilateral axillary nodes
N2: fixed ipsilateral axillary nodes
N3: ipsilateral internal mammary nodes
c) Metastasis (M) (supraclavicular LN belongs to metastasis)
M0: no distant metastasis
M1: distant metastasis present

Page 107 By Heyson Chan


5 What is the importance of axillary lymph node?
In the absence of distant metastasis, the presence or absence of axillary lymph node is the best
indicator of prognosis
The staging of the axillary lymph nodes can be done by sentinel node biopsy
(the first node where the breast drains to) (can be identified by injecting a dye containing
radioisotope into the tumor and identifying the node it spreads to by use of a Geiger counter)

6 What is the treatment of choice in the following condition?


Breast cysts
Aspiration (use 20G syringe)
If residual lesion – core needle biopsy or excisional biopsy (use 13G)
Fibroadenoma
Excisional biopsy
Carcinoma in-situ <-- check!!!!!!!!
Wide local exicision, no RT needed if clear margin

7 What is the treatment plan for breast cancer?


Loco-regional
Surgery
Radiotherapy
Systemic
Hormonal therapy
Chemotherapy

8 What are the different choices of mastectomy?


a) Simple mastectomy
Removal of the breast alone
b) Modified radical mastectomy
Removal of the breast, pectoralis minor and the axillary structures
c) Radical mastectomy
Removal of the breast, pectoralis minor and major and axillary contents
d) Extended radical mastectomy
 As for radical mastectomy but also removing the internal mammary nodes (between 2nd
and 4th anterior intercostals space)

9 What are the complications of mastectomy?


a) Early
 Wound infection
 Wound haematoma
 Damage to the intercostobrachial nerve
b) Late
 Lymphoedema of the upper limb (secondary to axillary clearance of RT)
 Breast seroma
 Frozen shoulder

10 What are the available breast reconstruction therapy?


a) Subcutaneous prosthesis
b) Submuscular implant
c) Tissue expander

Page 108 By Heyson Chan


d) Myocutaneous flap
i) Transverse rectus abdominis myocutaneous flap (TRAM)
ii) Latissimus dorsi flap (LD flap)

11 What are the advantages and disadvantages of using an implant?


Advantages:
Simpler technique
Place under the pectoralis muscles to reduce the incidence of contraction of the capsule
Can be performed at the time of mastectomy or at a later date

Disadvantages
Cosmetic results less satisfactory
Requires plenty of available skin after surgery
Lies above the natural inframammary fold, leaving the breast higher than the other one

12 What are the advantages and disadvantages of using myocutaneous flap?


Advantages
Useful where remaining skin and muscle in short supply
Better cosmetic results
Suitable for use in post-mastectomy
Suitable for salvage after local recurrence

Disadvantages
Greater blood loss
Greater complication rate and longer operation
Use of rectus abdominus may be impossible if the patient has had previous abdominal surgery
Late complications include flap necrosis and infection

13 What is the role of systemic therapy in CA breast?


This can be adjuvant or neo-adjuvant (to decrease the tumor size before surgery)
a) Radiotherapy
 Post-operatively to improve prognosis: give to the breast if a wide local excision is
performed; given to the axillar if axillary sampling or sentinel node biopsy is done and
cancer is detected
 Pallipative pain control in advanced disease
b) Chemotherapy (CMF – cyclophosphamide, methrotrexate, 5-fluorouracil)
 Positive axillary nodes
 Recurrent disease
 Large tumors
 High grade tumors
 All post-menopausal women
c) Hormonal therapy
 Estrogen receptor antagonist or selective estrogen receptor modulator
 1st line: tamoxifen (partial agonist of estrogen at its receptor)
- Note: not to be used >5 years since increased risk of endometrial CA
 2nd line: aromatase inhibitor, e.g. anastrazole

Page 109 By Heyson Chan


14 How would you follow up a patient with CA breast?
Annual mammography for the affected breast which undergo conservation surgery
Contralateral breast for mammography every 2 years

15 What are the features in mammography suggesting malignancy?


Irregular boarder
Pleomorphism
Distortion of normal breast architecture
Linear, branching masses
Cluster of microcalcification
2 views: medial-lateral-oblique (pect major seen) and craniocaudal

16 DDx of breast lump


95% is one of the following
1. CA breast
2. cyst
3. fibroadenoma (mobile)
4. fibroadenosis (tender lumpy breast)
benign: fibroadenoma, fibroadenosis, breast cyst, fat necrosis

5. Comparison of the clinical features of 4 common breast lumps


Type of lump Age Pain Surface Consistence Axilla
Solitary cyst 35-55 occasional Smooth Soft to hard normal
Nodularity 20-55 occasional Indistinct Mixed normal
Fibroadenoma 15-55 Smooth and Rubbery normal
bosselated
carcinoma >35 irregular Stony hard Glands may
be palpable

6. algorithm
- define surface and shape and then define the consistence
- Irregular and indistinct: if hard => CA
If rubbery => nodularity
- smooth and well defined: if hard => cyst
: if rubbery => fibroadenoma
7. define is soft or hard/firm
- soft: lipoma, cyst, galactocele
- hard/firm: well defined: fibroadenoma, Phylloides tumor (very similar to fibroadenoma),
malignancy (low grade), cyst
: ill-defined: malignancy, fibrocystic, fat necrosis, inflammatory changes, cyst

8. sign of malignancy:
- nipple eversion, discharge
- edema
- axillary LN
- involve muscle
- skin tethering
- mass

Page 110 By Heyson Chan


Risk factors
1. age
2. gender
3.Family Hx: first degree relative; relative had breast CA b4 50 or with bilateral breast CA at any age
4.Exposure to oestrogen: early menarche, late menopause, no pregnancy, pregnancy after 35, no
lactation (ask abt menarche, parity and age at birth of children, menstrual cycle and regularity, age
of menopause)
5.OCP: use of OCP for >4 yrs in younger women before first pregnancy increase the risk of
postmenopausal breast CA
6.HRT: million-women study has demonstrated that oestrogen-only HRT carries a small risk but
oestrogen-progesterone HRT double the risk
7.previous benign breast disease such as atypical epithelial hyperplasia
8.others: obesity in post-menopausal, diet rich in saturated fat and high alcohol intake

Page 111 By Heyson Chan


Cardiovascular Examination 
- usually mitral and/or aortic valve disease

Routine Introductions
5 Introduce yourself and ask for consent from the patient
6 Proper position: lye the patient at 45° on one pillow
7 Proper exposure: expose the whole upper trunk
8 Comment on the general condition of the patient
- Conscious and alert
- Any respiratory distress
- Body build (obese or not)
- cyanose
- Oxygen supplement

Inspection
 Be systemic. Start from the hands, to the head, the neck and the trunk.
1 The hand
a) Cold
b) Peripheral perfusion
c) Clubbing
d) Stigmata of infective endocarditis (especially if patient is febrile)
i) Splinter haemorrhage
ii) Oshler’s nodes (red, raised tender nodules on the pulps of fingers)
iii) Janeway lesions (non-tender erythematous maculopapular lesions on the palms or pulps
of the fingers)
iv) Rmb the Roth’s spot (fundoscopy)
e) Radial pulse (NOT for volume and character)
i) Rate
- Normal: 60 – 100 beats per minute
ii) Regularity (volume and rhythm)
- Irregularly irregular in atrial fribillation  if present, pay particular attention to any
mitral stenosis or mitral regurgitation
iii) Radial-radial delay
- Due to large arterial occlusion by atherosclerotic plaque or aneurysm
iv) Radial-femoral delay
- Coarctation of the aorta
f) Collapsing pulse (not equal to bounding pulse)
- Use the palm (less sensitive) of your hand to feel, not the fingertips
- Use the other hand as a supporting hand to raise the hand Quickly
- If present, suspect aortic regurgitation
g) Blood Pressure
- Tell the examiner that you would like to know the blood pressure of the patient
- Wide pulse pressure (>100mmHg) in aortic regurgitation
- Narrow pulse pressure in aortic stenosis

2 The head
a) Pallor
b) Jaundice

Page 112 By Heyson Chan


c) Cyanosis
d) Malar flush
e) Xanthelasma
f) ear lobe: diagonal crease in the lobule of the auricle (Frank’s sign). Statistically associated
with coronary artery disease

3 The neck
a) Carotid pulse
Medial to the sternomastoid, at the level of thyroid cartilage
i) Character
 Slow rising in AS
 Collapsing in AR
 Corrigan sign in AR (carotid pulsation)
 Bisferiens pulses in combined aortic stenosis and regurgitation
 Pulsus alternas: LV failure
ii) Volume
iii) Conditions of the vessel wall

b) JVP
Internal jugular vein is medial to the sternomastoid (lateral to SMC is EJV), lateral to carotid
pulse
- Ask the patient to turn his head to the left. With the hand well support by the pillow
otherwise tense up the SMC
- Show the examiner that you know the JVP has two wave forms by palpating the radial /
carotid pulse
- Stand a bit toward the end of bed not exactly at the side
- The pulsation should be indrawing in character
- Find the angle between the 2 heads of the SMC
- Use hepatojugular reflex to confirm (Ask any pain)
- If too high, sit patient up
- If cannot determine is carotid from JVP: first feel it, then ask patient to sit up to observe
any changes
- Report: base of heart, middle of the neck, angle of jaw, ear lobe Or by cm Or finger
breath
i) Height
 Raised if higher than 5 cm from sternal angle  right heart failure
ii) Character
 Cannon a waves in complete heart block
 Giant a waves in pulmonary hypertension (mitral valve disease, cor pulmonale or
pulmonary stenosis (rare)
 Large v waves in tricuspid regurgitation
 *Absent a waves in atrial fibrillation: a wave should come before the carotid pulsation

c) Hepatojugular reflux
- Inform the patient that you are going to press on his abdomen and tell him that it may be
uncomfortable (Don’t hurt your patient!)
- Press firmly on the abdomen for 15 seconds
- Normally, the JVP should rise transiently and return to the original position within 2
seconds
- Failure for the JVP in returning to its original position confirmed right heart failure

Page 113 By Heyson Chan


4 The trunk
a) Scars
- Show the examiner that you are inspecting for scars (i.e. exaggerate your movement,
especially when inspecting the lateral aspect)
- Thoractomy scar – possible valvular replacement  comment on any metallic heart
sound heard (even without a stethoscope)
- Sternotomy (valve replacement)
- Submammary scar (mitral valve replacement): in female patients, check the base of the
breast avoid missing any scars
- Infraclavicular (pace-maker)
- Femoral catherization
b) Pacemaker
- Actively palpate the upper chest to avoid missing a pacemaker (usually under the pectoral
muscles, with infraclavicular scar
- Defibrillators are usually larger than the pacemaker
c) any visible pulsation

Palpation
1 Apex beat (site, location, force, time, tapping)
a) Position
- Lowest and most lateral part of the pulsation
- Show the examiner by pin-pointing the position of apex beat with one finger
- Palpating the clavicles and define its mid-point, show the position of the apex beat in relation
to MCL to the examiner; also palpate the sternal angle in order to define the position of apex
beat in relation to the intercostals space
- Normal position: 5th intercostals space, lateral to mid-clavicular line
- If cannot feel the apex beat, turn to left decubitus position.
- If still cannot feel the beat, RMB to feel for the right side before u comment
b) Character
Different descriptions were used in different textbook which made things complicated.
According to Prof Joseph Sung, we are only required to differentiate between tapping,
hyperdynamic and hypervolemic apex beat.
i) Tapping
 Palpable first heart sound
 Mitral stenosis: loud first HS
 Like the mechanical heart valve
ii) Hyperdynamic (pressure overload: forceful, SUBSTAINED time)
 Aortic stenosis
iii) Hypervolemic (volume overload: displace, forceful, diffuse)
 Mitral regurgitation
 Aortic regurgitation
2 Parasternal heave (movement)
- Associate with pulmonary hypertension (may have loud S2)
- Right ventricular hypertrophy or LA hypertrophy pushing RV
- If feel this, quickly recall and rechck wither there is a GIANT a wave (pulmonary HT,
pulmonary stenosis) or v wave (TR, CHF)
3. at pulmonary area for palpable second heart sound (pulmonary HT)
5 Thrills (at apex, parasternal, base of heart)
- Defined as palpable murmur
- When present  makes the murmur grade 4 or above

Page 114 By Heyson Chan


Auscultation (make up your mind what u r going to look for)
 Comment on i) whether the heart sound is dual; ii) any murmurs; iii) any added sound
 If a murmur is present comment on i) phase of the mumur; ii) where is the murmur heard and best
heard; iii) any maneuver that makes the murmur louder; iv) any radiation; v) grade of murmur
 Press lightly when you are using the bell for low-pitched murmurs
 Palpate the carotid pulse with a thumb during auscultation
 The sequence of auscultation is as follow:
1 Use the bell to auscultate the apex (for MS)
2 Turn the patient to the Left lateral position (for MS)
3 Use the diaphragm to auscultate the apex (for MR)
4 Use the diaphragm to auscultate the axilla area (for MR radiation)
5 Use the diaphragm to auscultate the tricuspid area (for TR: feel for the liver as well)
6 Use the diaphragm to auscultate the left lower sternal boarder. Loudest at the third or fourth left
intercostals spaces(for AR)
7 Ask the patient to lean forward and hold his breath at full expiration (for exaggerate AR)
8 Whilst the patient is still leaning forward, auscultate the base of lung: inspiratory crepitation (for
CHF)
9 Auscultate the aortic area with the diaphragm (for AR/AS)
10 Auscultate the neck for carotid bruit or any radiation of murmur (for AS)
11 If can’t hear the MS but with peripheral signs, then ask for auscultation after exercise

Palpate
1. liver: if see a large v wave + pansystolic murmur over tricuspid area
2. edema

Routine Conclusion
1 Examine the patient’s lower limb for any edema (for CHF)
2 Examine the peripheral pulses (see separate notes for examination of peripheral pulses)
3 If you have not measure the blood pressure (esp AS: low systolic and narrow pulse pressure)
and do Fundoscopy of the patient, tell the examiner that you would like to do so
4 If a pan-systolic murmur was heard, ask for the permission to perform an abdomen exam for
pulsatile liver (ask patient to hold his breath at inspiration), a hepatomegaly can also be felt in
congestive heart failure; while a splenomegaly can be felt in infective endocarditis
5 Cover the patient up and make sure he feels comfortable
6 Summarize your findings and suggest possible differential diagnosis

Summary
Insepct for: clubbing, SBE, pulse (rate + rhythm)  P, J, C  JVP + carotid pulse  scars /
pacemaker
Palpate for: apex beat (position + character)  parasternal heave  thrills
Auscultation (heart + lung base), Edema

Page 115 By Heyson Chan


Frequent Asked Questions
1 How can you differentiate a JVP from a carotid pulsation?
JVP are:
 2 waveform per cardiac cycle (except AF)
 Affected by hepatojugular reflux
 Generally not palpable (except in TR)
 Alters with changes in respiration and position
 Obliterated by finger pressure on the vein

2 What are the causes for raised JVP?


Raised JVP if higher than 5 cm from the sternal angle
 Right heart failure
 Fluid overload
 Tricuspid regurgitation or stenosis (rare!)
 Heart block (cannon waves)
 Superior vena cava obstruction (no wave form)
 Cardiac tamponade or constrictive pericarditis

3 Give a typical presentation for the following conditions. (Extremely important!)


a) Mitral regurgitation
- symptoms: dyspnea (due to pulmonary congestion), fatigue (decrease CO), palpitation (due to
low AF or LV dysfunction), IE
 *AF common
 *Pulse: normal or sharp upstroke
 *Displaces, diffuse, hyperdynamic apex beat (volume overload)
 *left parasternal heave (LA enlargement behind the RV): pulmonary HT
 *Soft first heart sound
 *Pan-systolic murmur at the apex radiating to the axilla
 *Best detected on expiration (DDx from TR)
 Loud P2
 S3: rapid LV filling in early diastole
 Causes: MVP, rheumatic heart, LV dilatation, IE
b) Mitral stenosis
 Malar flush (don’t cross nose bridge)
 Peripheral cyanosis
 AF common (large LA)
 JVP: prominent a wave if with pulmonary HT. NO a wave in AF
 **Tapping, undisplaced apex beat (papable S1)
 parasternal heave: sever LA enlargement
 **Loud S1, opening snap (indicating that the leaflets are pliable)
 Rumbling, low-pitched, mid-diastolic murmur, best heard at expiration on left lateral
position
 Causes: rheumatic, congenital
c) Tricuspid regurgitation
 Giant v waves  atrial filling
 Pansystolic murmur, best heard at lower sternal edge in inspiration
 Pulsatile liver

Page 116 By Heyson Chan


d) Aortic regurgitation
 Collapsing pulse
 Wide pulse pressure: check blood pressure
 Corrigan’s sign (carotid pulsation); Quincke’s sign (pulsating nails); head nodding (De
Musset’s), booming sound heard over femoral (Traube’s sign), pulsation of uvula
(Muller’s sign)
 Displaced, hyperdynamic apex beat
 Loud P2 (pulmonary hypertension)
 High-pitched early diastolic murmur at left sternal border (best heard at expiration with
patient leaning forward)
 Look for Argyll Robertson pupils
 Causes: Marfan (high arched palate, arm span greater than height, AS, RA, bicuspid aortic
valve, syphilis (rare)
e) Aortic stenosis
 Symptoms: exertional dyspnea, exertional chest pain, exertional syncope
 Slow rising pulse with narrow pulse pressure
 Hyperdynamic undisplaced apex beat
 Aortic thrill common
 Systolic ejection click (early systolic high pitch sound)
 Ejection systolic murmur (heard at the base, left sternal edge and the aortic area, in full
expiration and lean forward), radiating to the carotid
 Causes: rheumatic, calcified bicuspid aortic valve, degenerative calcification

4 Both tricuspid regurgitation and mitral regurgitation give rise to pan-systolic murmur,
how canyou differentiate between the two?

Tricuspid regurgitation Mitral regurgitation


Pulse normal Jerky or normal
Intensity Increased with inspiration Increased with expiration

JVP Raised, giant v waves Normal

Abdomen Pulsatile liver Normal

Radiation No Radiate to the axilla

5 What are the grading for murmurs?


It would be advisable to grade a murmur of grade 3 (w/o thrill) or grade 4 (w/ thrill) in an
examination.
 Grade 1 – just audible when the room is quiet and the patient holding his
breath;
 Grade 2 – audible but faint or quiet;
 Grade 3 – readily audible but not accompanied by a thrill;
 Grade 4 – easily audible and accompanied by a thrill
 Grade 5 – very loud;
 Grade 6 – loud enough to be heard without a stethoscope; the examiner only has to
put his ear close to, but not on, the patient’s chest.

Page 117 By Heyson Chan


6 What are the common causes for the following valvular lesions? (very important)
General reasons
 Applied to regurgitations: infective endocarditis and chronic rheumatic fever
Specific reasons
 Mitral regurgitation – general reasons + papillary muscle dysfunction / rupture, e.g. post-MI;
functional, e.g. LV dilatation; mitral valve prolapse (w/ mid-systolic click); connective tissue
disorder (e.g. Marfan’s); trauma
 Mitral stenosis – chronic rheumatic fever (most common), RA / SLE (rare)
 Aortic stenosis – senile calcification (common), congenital e.g. bicuspid aortic valve (rarer)
 Aortic regurgitation – general reasons + aortic root dilatation (e.g. HT, trauma, aortic
dissection); trauma
 Tricuspid regurgitation – pulmonary hypertension (with prominent a and v waves associated
with raised JVP, parasternal heave)

7 What are the causes of pulmonary hypertension (cor pulmonale)?


Lung Disease
 COPD
 Bronchiectasis
 Pulmonary fibrosis
Pulmonary vascular disease
 Pulmonary emboli
 Primary pulmonary hypertension
Neuromuscular disease
 Myasthenia gravis
 Motor neuron disease
Thoracic cage abnormality
 Kyphosis
 Scoliosis
Hypoventilation
 Sleep apena
 Stroke

8 What are the differential diagnosis of collapsing pulse?


 Aortic regurgitation
 Pregnancy
 Thyrotoxicosis
 AV fistula
 Patent ductus arteriosis (continuous murmur best heard under clavicle)

9 What are the possible causes for splinter hemorrhage?


 Trauma e.g. manual worker
 Infection: Infective endocaritis, spesis
 Vasculitis: Rhaeumatoid arthritis
 Hematological malignancy

10 Why did you inspect for jaundice in cardiovascular examination?


Jaundice is present in the following situations
 Congested liver due to heart failure
 Prosthetic valve induced haemolysis

Page 118 By Heyson Chan


11 What are the differential diagnosis of non-palpable apex beat?
 Thick chest wall
 Pleural effusion
 emphysema
 Pericardial effusion
 Haemothorax
 Pneumothorax
 Dextrocadia
 Cardiac arrest (never mention this in the exam!)

12 What are the causes for atrial fibrillation?


 Mitral valve disease
 Ischemic heart disease
 Thyrotoxicosis
 Conrtictive pericarditis
 Chronic pulmonary disease

13 What does the first and second heart sound represents?


 The first heart sound or S1 originates from mitral valve closure occurring at the beginning of
ventricular systole. The tricuspid valve closes at the same time but tricuspid valve closure is
usually quiet because of the low pressures in the right heart
 The second heart sound or S2) comes from aortic and pulmonic valve closure at the end of
ventricular systole
 During inspiration A2 comes earlier than P2 and S2 appears split, because of the increase in
venous return occurring during inspiration
 Fixed splitting of S2 (both during expiration and inspiration) seen in atrial septal defect
 Reverses splitting P2 due to delayed ventricular depolarization (LBBB), delayed ventricular
emptying (aortic stenosis or coarctation of the aorta), or increased left ventricular volume (e.g.
patent ductus arteriosus)

14 What are the causes of hypertention?


a) Primary (idiopathic): 95%
b) Secondary (RENAL: menomics)
i) Kidney
 Look for renal bruit and polycystic kidneys
ii) Adrenal
 Pheochromocytoma – sweating, flushing, anxiety
iii) Coarctation of the aorta
iv) Metabolic
 Conn’s – Na increase, K decrease
 Cushing’s
 Hyperparathyroidism

15 What are the differential diagnosis of chest pain?


a) Cardiovascular
 Ischemic heart disease
 Pericarditis
 Aortic dissection
 Aortic stenosis
 Pulmonary hypertension

Page 119 By Heyson Chan


b) Respiratory
 Pleuritis
 Pulmonary embolism
 Pneumothorax
c) Gastrointestinal
 Peptic ulcer
 GERD
 Biliary colic / pancreatitis
d) Musculo-skeletal
e) Neurological
 Peripheral numbness due to hyperventilation
 Zoster pain in the elderly

16 What are the differential diagnosis of syncope?


a) Cardiovascular
 Arrythmia (most common)
 Vasovagal syncope
 Postural hypotension
 Carotid stenosis (increase sensitivity of the carotid body  bruit)
b) Neurological
 Epilepsy
c) Drug induced
 Beta-blocker
 Parietal lobe anticholinergics
d) Metabolic
e) Hypoglycemia

Ear lobe crease => coronary artery disease


Corneal arcus => hyperlipidemia, aging

Added sound in diastolic phase


1. murmur: rumbling in MS
2. rub: pericarditis, restrictive myopathy
3. S3: low pitch, gallop rhythm, when dilated LV with rapid LV filling => poor LV function (HF)
or physiological: pregnancy or younger than 30
4. S4: late, high pitch, atrial effective contraction against ventricle. Therefore not in AF
5. split S2
6. opening snap: high pitch, MS

Added sound in systolic phase


1. systolic ejection click: high pitch, AS/PS
2. non-ejection systolic click: high pitched, ASD
3. mid-systolic click: MVP

Loud first HS
1. MS/ TS:remain wide open at the end of diastole and shut forcefully with onset of ventricular
systole
2. hyperdynamic: pregnancy or thyrotoxicosis,tachychardia: decrease PR interval

Page 120 By Heyson Chan


Soft S1
1. prolonged diastolic filling eg first degree HB
2. delay onset of LV systole: LBBB
3. Failure to close: MR

Loud A2
1. systemic HT: forceful aortic valve closure secondary to high aortic pressure
2. AS

Loud P2
1. pulmonary HT: depends on pulmonary pressure such as septal defect, MS

Soft S2
Aortic valve calcified therefore movement decrease eg AR
DDx of pansystolic mumur
7. MR
8. TR
9. VSD
10. aortic pulmonary shunt

Malar facies
4. MS
5. hypothyroidism

pulsus paradoxus:
BP drop >10mmHg in inspiration
3. pericardial effusion
4. constrictive pericarditis
5. severe astham

Bisferiens (2 peaks)
AS + AR

CXR of CHF
A: alveolar edema (bat’s wing sign)
B: Kerley’s B line
C: cardiomegaly
D: dilated prominent upper lobe vessel
E: pleural effusion

References
1 250 cases in clinical medicine
2 Clinical Examination by Nicholas J Telly
3 Clinical Examination by Epstein, Perkin, Cookson and de Bono
4 Heart Examination by Prof David Chung
5 Medicine at a Glance by Davey

Page 121 By Heyson Chan


Examination of hands 
- Rheumatoid arthritis: proximal joint swelling, spindling of the finger, ulnar deviation, nodules
- Scleroderma: sclerodactyly with tapering of the fingers, sometimes with gangrene of the
fingertips, tight, shiny, adherent skin, calcified nodules etc
- Wasting of small muscle: observe dorsal guttering
- Psoriasis: pitting of the nail, terminal interphalangeal arthorpathy, scaly rash
- Ulnar nerve: claw hand, muscle wasting spares the thenar eminence, sensation loss
- Clubbing
(they make up 75% of the cases)
Routine
1. introduce yourself
2. ask any pain, stiffness
3. get consent
Exposure
1. at least roll up till the elbow (1 joint below and 1 joint above)
2. use a pillow to rest the hands on

Inspection (look): first dorsal of the hand and then palmar side
1. posture
2. face:
- expressionless facies with adherent shiny skin, sometimes with telangiectasis: systemic
sclerosis
- cushingoid facies: steroid changes in RA patient
- exophthalmos: thyroid acropachy
- xanthelasma etc
3. joint deformity
- Swan-neck deformity
- Boutonniere deformity
- Z deformity of the thumb
- Ulnar deviation of the finger
- Heberden’s node (DIP)
- Bouchard’s node
- Gouty tophi
4. nail
- nail fold infarcts: vasculitis usually associated with RA
- nail pitting, onycholysis, ridging, hyperkeratosis, discoloration (psoriasis)
5. skin:
- colour: Palmar erythrema (RA), any pigmentation of the skin crease
- consistency: tight and shiny in scleroderma; papery thin with purpuric patches in steroid
therapy, thick in acromegaly
- lesions: psoriasis, vasculitis, tophi, neurofibromata etc
6. muscle wasting
- small muscles of the hand: both dorsum and palm
- thenar wasting: median
- generalized wasting esp first dorsal interosseous but not thenar: ulnar
- generalized wasting from a T1 lesion eg motor neuron disease (with prominent
fasciculation), syringomyelia (dissociated sensory loss), charcot-marie-tooth disease (with
pes cavus), cord compression

Page 122 By Heyson Chan


7. elbow for sign of RA, gout, psoriatic plaque
Palpation and Percussion (Feel)
- Ask any pain
1. feel temperature: use dorsum of hand, from proximal to distal (active inflammation)
2. feel tenderness: look at patient face
3. 4 fingers techniques: feel for joint swelling over DIP and PIP (can either be fluid or synovial
thickening)
- 3 finger: MCPJ
- 2 finger: waist (esp ulnar)
(important to check the deformity is fixed or mobile because involve different surgery)
4. feel for Dupuytren’s contracture: RA
5. tap over flexor retinaculum: tinel’s sign for median nerve entrapment

Sensation
- test the pattern of sensory loss defect of ulnar and median nerve
- test dermatome

Movement:
- first active then passive: if limited, then need to determine is fixed joint problem or is
weakness
- test power and range
1. Tone of all joints
2. waist: flexion and extension
3. MCPJ: flexion (observe with waist in supination position) then finger grip and extension => C8,
T1
4. adduction: hold a piece of paper (DAB: dorsal abduct; PAD: palmar adduct) => ulnar
5. abduction => ulnar
6. abduction of thumb: abductor pollicis brevis => median nerve
7. oppoisition
8. functional status eg unbuttoning of clothes, writing

Finally, check pulse and capillary filling

Remember hand composed of (skin, muscle, bone, joint, nerve, vascular)


1. arthropathy
2. myopathy
3. neuropathy
4. peripheral nerve lesion
5. vascular disease

Rheumatoid arthritis (symmetrical polyarthritis + systemic disease)


1. deformity
- subluxation at MCPJ
- swan-neck deformity
- boutonniere deformity
- Z thumb
- Dorsal subluxation of the ulnar at the carpal joint
- Ulnar deviation of finger: due to sublaxation and dislocation at the MCPJ
- Spindling of finger due to soft tissue swelling of the PIPJ and MCPJ

Page 123 By Heyson Chan


2. skin lesion
- nail-fold infarct and vasculitis skin lesin: immune complex
- palmar erythema
- wasting of small muscles
3. use is restricted due to weakness, deformity, pain
4. carpal tunnel syndrome
5. cushing’s
6. rheumatoid nodules: elbow, flexor and extensor tendon of the hand, sacrum, Achilles tendon,
sclera, lung, myocardium
7. diagnostic criteria for RA: 4 out of 7
- morning stiffness for at least 1 hour for duration of 6 weeks or more
- swelling of at least three joint for 6 weeks or more
- swelling of wrist, MCPJ, PIPJ for 6 weeks or more
- symmetry of swollen joint area for 6 weeks or more
- subcutaneous nodules
- positive RF
- radiographic features typical of RA eg erosion and periarticular osteopenia
8. anemia:
- anemia of chronic disease (normochromic normocytic)
- GI bleeding related to NSAIDs
- Bone marrow suppression: gold, indomethacin, MTX
- Megaloblastic anemia: folic deficiency or associated pernicious anaemia
- Felty’s syndrome: sever RA + splenomegaly, anemia, leucopenia, thrombocytopenia
(hypersplenism) and leg ulcers
9. systemic involvement (extra-articular)
- anemia
- nodules
- lymphadenopathy
- vasculitis
- carpal tunnel syndrome
- multifocal neuropathies
- splenomegaly (either alone or with Felty)
- eyes: episcleritis, scleritis, keratoconjunctivitis sicca
- chest: pleural effusion, fibrosing alveolitis, pleurisy
- cardiac: pericarditis, increased risk of IHD
- Osteoporosis
- Amyloidosis
- Other autoimmune disease: eg thyroid
10. Mx
- general: education, exercise, physio
- NSAIDs: symptoms relief
- DMARDS (disease modifying antirheumatic drugs): MTX, sulphasalazine, gold,
hydroxychloroquine, combination => limited use due to SE
- Corticosteroids
- Biological agents (anti-TNF alpha): infliximab (chermic anti-TNF Ab), etanercept (TNF
receptor), adalimumab (human anti-TNF mAb)

Page 124 By Heyson Chan


Psoriatic arthropathy
1. asymmetrical arthropathy
2. mainly DIPJ
3. nail: pitting of finger nail, onycholysis, hyperkeratosis (thick scale)
- 80% involve nail
4. skin lesion: elbow, extensor, scalp, behind ear, knee, intragluteal cleft
- psoriatic plaque: circular with well-defined edges, red with silvery scaly surface
5. sausage shaped finger due to tenosynovitis
6. pattern of joint involvement
- asymmetrical DIPJ
- symmetrical joint involvement as seen in RA
- sacroiliitis: differ from AS since most syndesmophytes tends to arise from lateral and
anterior surface of the vertebral bodies and not at the margin of the bodies
- arthritis mutilans: complicated by telescoping of digits

Scleroderma
1. smooth, shiny, tight skin over face and joints
2. puffy hands and feet
3. sclerodactyly: finger pulp atrophy
4. beaking of nail: pseudoclubbing
5. atrophic nail
6. telangiectasia of face
7. subcutaneous calcinosis: fingers, elbow, extensor aspect of forearm
8. vertigo or pigmentation
9. eye: dry eye
10. skin: raynaud’s phenomenon, pigmentation, telangiectasia, ulceration
11. MSK: arthritis, myositis, myopathy, bonce ischemia with resorption of phalanges
12. GI: dysphagia, reflux esophagitis
13. Lung: fibrosis, atelectasis, pulmonary hypertension, pneumonia
14. kidney: glomerulonephritis, malignant hypertension (ACEi can improve renal function)
15. heart: myocardial fibrosis
16. CREST syndrome: calcinosis, Raynaud’s phenomenon, oesophageal dysmobility, sclerodactyly,
telangiectasia => benign type of systemic scleroderma

Page 125 By Heyson Chan


Hernia Examination 
Physical Examination
Aim
1 Confirm a hernia
2 Differentiate an inguinal hernia (direct / indirect) from a femoral hernia
3 Determine the urgency of operation (i.e. whether the hernia is reducible)

Routine Introduction
1 Introduce yourself and greet the patient
2 Ask for permission and a chaperon
3 Proper exposure: Expose the groin and external genitalia
4 Proper position: if there’s obvious swelling  lie down; if not  stand the patient

Inspection
1 Ask the patient to cough and observe (to check if it’s hernia by cough impulse)
 Should be done when the patient is standing first
2 Inspect for any surgical scars which may indicate previous surgery (recurrence)
3

Palpation
While patient is standing
1 Feel for the cough impulse (bilaterally)
2 Check if the mass can be felt separately from the testis
3 Briefly describe the characteristics, e.g. tenderness, temperature and tension
Note: you can never get above the upper boarder hernia

While patient is lying on bed


4 Define the anatomical landmarks (i.e. ASIS and PT), show that the hernia arises above the
inguinal ligament
5 Check if it is femoral/ inguinal
 Inguinal: above and medial to the pubic tubercle
 Femoral: below and lateral to the pubic tubercle
6 Ask patient to reduce the mass by himself, if failed, help the patient to do so but never force in!
(You may hurt the patient!)
7 If inguinal, press on the deep ring and ask patient to cough.
 Direct inguinal hernia: not controlled by the deep ring compression test
 Indirect inguinal hernia: the mass controlled by deep ring compression test and will come out
when you release the finger and ask patient to cough again.
8 Check for the opposite side
9 Check for the external genitalia
 Palpate one testis at a time
 When palpating the testis, place the fingers of one hand behind the testis, supporting it, while
examining the surface of the testis with the thumb

Page 126 By Heyson Chan


Routine Conclusions
1 Tell the examiner that you would like to complete by
a) Abdominal – for ascites
b) Respiratory – for chronic obstructive pulmonary disease
c) PR examination – for benign prostate hyperplasia
2 Cover up the patient
3 Wash your hands
4 Report your findings

Questions
1 Define hernia.
 The protrusion of the whole or part of a viscus, from its normal position, through an opening
in the wall of its containing cavity

2 What are the differential diagnosis for lump in the groin?


Hernia – cough impulse + can’t get over upper boarder
 Inguinal hernia
 Femoral hernia – intestinal obstruction more likely (narrow femoral ring)
Skin, soft tissues
 Lipoma
 Sebaceous cyst
Lymphadenopathy
 Enlarged lymph nodes
Testis
 Ectopic testis
 Undescend testis
Spermatic cord
 Hydrocele – transilluminatable + testis cannot be felt separately
 Lipoma
Vascular
 Saphena varix – very soft + may associate with varicose vein
 Femoral aneurysm– expansile + bruit
Muscles
 Psoas abscess – soft + fluctuation between parts of abscess above and below inguinal
ligament

3 What are the anatomical boundaries for the inguinal canal?


 Roof: lowest fibers of the internal oblique and transersus abdominis joint together as conjoint
tendon
 Floor: inguinal ligament reinforced medially by the lacunar ligament
 Anterior: external oblique aponeurosis, internal oblique in the lateral third
 Posterior: conjoint tendon and lacunar ligament in the medial; inguinal ligament in the lateral,
transversalis fascia throughout

4 What are the anatomical boundaries for the femoral canal?


 Medially: lacunar part of the inguinal ligament
 Laterally: femoral vein
 Supero-anteriorly: inguinal ligament
 Infero-posteriorly: pectineal ligament
5 What are the causes of abdominal hernia?

Page 127 By Heyson Chan


 An anatomical weakness where structures pass through the abdominal wall, muscles fail to
overlap or where there are no muscles
 An acquired weakness following a trauma
 High intra-abdominal pressure (coughing, stringing or abdominal distention)
 Patent processus vaginalis – indirect inguinal hernia

6 What is the lymph node drainage of the groin?


 Retroperitoneal organs (e.g. testis) – para-aortic nodes
 Skin of the scrotum or penis – inguinal nodes

7 What is the treatment option for hernia?


 Sac excision with repairment of weakened abdominal wall by MESH
(works by inducing fibrosis)
 Treatment is important to prevent the complication of hernia, namely,
i) Obstruction (constriction at the neck of the hernia sac leading to obstruction of the loops
of bowel within it)
ii) Strangulation (constriction prevents venous return causing venous obstruction, arterial
occlusion and gangrene)

8 Distinguish direct and indirect inguinal hernia clinically.


Direct Indirect
Extends to scrotum No Possible

Direction of reduction Straight back Up and lateral

Controlled by deep ring No Yes


compression test
Direction of reappearance To original position Down and medial
after reduction

But ultimate differentiation can only be made during operation. Indirect inguinal hernia with its sac
pass lateral to the inferior epigastirc artery (demarcates the medial edge of the internal ring) while
that of direct inguinal hernia will pass medial

9 Where are the pubic tubercle, deep ring, and superficial ring?
 Pubic tubercle: the most inferior and lateral point of the pubic crest, it is also the point for the
insertion of the adductor longus tendon (which can be felt by asking the patient adduct against
your fist which is put against them)
 Deep ring: two cm above the mid-point between the pubic tubercle and ASIS
 Superficial ring: lies just above and medial to the pubic tubercle

10 What are the contents of the inguinal canal?


 Spermatic cord / round ligament
 Ilioinguinal nerve

Page 128 By Heyson Chan


11 What are the contents of the spermatic cord?
Three arteries
 Artery to vas deferens (from inferior vesicular artery)
 Testicular artery (from aorta)
 Cremasteric artery (from inferior epigastric artery)
Three nerves
 Ilioinguinal nerve (on the front of the cord)
 Nerve to cremaster (from genitofemoral nerve)
 Autonomic nerves (sympathetic fibers from T10)
Three other structures
 Vas deferens
 Pampiniform plexus of veins (R  IVC; L  renal vein)
 Lymphatics (to para-aortic nodes)

12 What can be contained in the sac of an inguinal hernia? (check!!)


 Small bowel – most common
 Omentum
 Ascending colon

13 What can be contained in the sac of a femoral hernia?


 Omentum – most common
 Small bowel

14 What are the risk factors of incisional hernia?


Incisional hernia – extrusion of peritoneum and abdominal content through a weak scar or
accidental wound on the abdominal wall, representing a partial wound dehiscence where skin
remains intact.
Pre-operative
 Age
 Obesity
 Malignancy
 Abdominal distention
 Immunocompromise state
Operative
 Poor technique of closeure
 Placing drains through the wound
 Longitudinal incision
Post-operative
 Wound haematoma
 Wound infection
 Early mobilization
 Post-operative atelectasis and chest infection

Page 129 By Heyson Chan


15 What are the complications of hernia surgery?
Early
 Damage to the vas deference
 Bleeding (damage to the testicular / inferior epigastric artery)
 Wound complications
 Urinary retention
Late
 Recurrence
 Chronic pain (secondary to ilioinguinal nerve entrapment)

16 How would you perform a hernia repair?


 Hernia repair can be performed as a day case
 It can be performed under LA / SA or GA
 It can be performed laparoscopically or through open procedure
 The recommended methods of repair by the Royal College of Surgeons
- Lichtenstein mesh repair (bowel reduced, deep ring tightened and prolene mesh applied to
reinforce lax posterior wall)
- Shouldice repair: posterior wall is split and edges overlapped and tightened with prolene
sutures

17 What general advices will you give to the patient post-operatively?


 Early mobilization from 10 days
 Keep the area clean and wash carefully
 Need to be off work for 6 weeks if their job involves lifting
 Avoid prolonged coughing
 Take laxatives if they get constipated

18 What are the differential diagnoses of a scrotal swelling after a hernia surgery?
 Haematoma (most common)
 Ischemic orchitis – scrotum will be very tender

19 What are the differences between hernioplasty, herniorrhaphy and herniotomy?


 Hernioplasty – excision and repair of the hernia sac
 Herniorrhaphy
 Herniotomy – excision of the sac completely, suitable for children only

20 What is incarcerated hernia and strangulated hernia?


 Incarcerated hernia – an irreducible, non-obstructed hernia, caused by adhesions forming
around the sac
 Strangulated hernia – a hernia containing ischaemic bowel

References
1 Lecture Notes on General Surgery by Ellis, Calne and Watson
2 Surgical Finals – Passing the Clinical by Kuperberg and Lumley
3 Fast Track Surgery – general, vascular & urology by Trinidade & Ramachandran

Page 130 By Heyson Chan


LN examination 
1. involve inspection and palpation
2. normal node cannot palpate
3. if feel node
- size (length and width)
- consistency: soft, firm, rubbery, hard
- regular or irreglular
- tenderness
- mobility to surrounding nodes or tissue
- skin condition
- check the drainage area in an attempt to find the source

Cervical LN
- better from the back if is surgery examination because they concern more
- use finger pulp
- rotate movement
- drains the superficial and deep structures of H/N
1. start from submental group
2. then submandibular
3. then tonsillar at the angle of jaw
4. up to per-auricular (in front of ear)
5. back to post-auricular at mastoid process
6. up at the back of occipital LN
7. slide down along the anterior border of trapezius for the posterior cervical
8. go up again along SMC muscle for the superficial cervical LN (slight flex the neck to relax the
strap muscle)
9. go down again for the deep cervical chain by palpating more firmly along the body of SMC
- upper, middle, lower
10. finally for the suprascapular LN (the left one is known as Virchow’s node and is drained by
abdominal organ and highly related to stomach)
(Submental – submandibular – pre-auricular – postauricular – occipital – higher cervical – middle
cervical – lower cervical – supraclavicular – post triangle)

Inguinal node
- drain the lower limb, lower trunk, penis, scrotum, perineum, lower vagina and anus
- lie patient down
- 2 chains: horizontal and vertical
1. run below the inguinal ligament
2. then run along the saphenous vein
3. relax posterior popliteal fossa by passively flexing the knee

Axillary nodes
- incl. anterior (border of pect major), posterior (ant border of lact dorsi), central, lateral
(humeral), and brachial (apical)
- lateral part of breast usually drains towards the axillary group
- medial part towards internal mammary chain (deep node)
- either lying or sitting
- press firmly against the chest wall

Page 131 By Heyson Chan


1. feel for anterior group along the post border of the ant axillary fold
2. post group along post axillary group
3. palpate along the medial border of the humerus to check for the lateral gourp
4. axilla is the central LN, palpate against lateral chest wall
5. go up to apical region
6. inspect for infraclavicular and supraclavicular LN

(for right axilla, I will first use my left hand to grip her elbow and rest her wrist on my right arm.
Then use my right hand to feel for lateral and post LN. Then change position by holding her wrist
with my right arm and rest her elbow on my left arm. Further feel for ant, central and apical by my
left hand)

(for left axillar, I will first use my left hand to grip her wrist and rest her arm on my right arm. Then
use my right hand to feel for the ant, central and apical LN. then change position to hold her elbow
by my right hand and rest her wrist on my left arm. Then try to feel for the post and lateral LN.)

Page 132 By Heyson Chan


Lump and bump 
4S - Site, Size, Shape, Surrounding (skin, LN)
4C - Contour (edge), Colur, Consistency, Compressibility (fluctuance)
4T - Transillumination, tenderness, temperature, tecthering + Tinel

Solid mass
1. reduce
2. mobility
3. fluctuate
4. get below / above it
5. bruit
6. border

Vascular mass
1. empty
2. bruit
3. pulsatile / expansile

lipoma
Most commonly occur in the subcutaneous layer of skin
- discoid or hemispherical swelling
- lobulated: with gentle pressure apply, even more lobulated
- soft and large in size
- slip sign: lipoma tends to slip away from the examining finger on gentle pressure because it’s soft
- maybe subcutaneous or intramuscular
- not fixed to skin or deeper structure

Sebaceous cyst
This follow obstruction to the mouth of a sebaceous duct
Any where except sole and palm with no sebaceous glands
Round
Soft
Attach to skin
Central puctum maybe visible
With cheesy material and maybe infected

Ganglion
Cystic swelling most commonly seen on dorsum of the wrist
Debatable origin but probably a cystic mucoid degeneration of the joint capsule or tendon sheath
- painless
- can disappear spontaneously (though bash with Bible is the traditional treatment)
- smooth
- fluctuant
- fixed to deeper structure but not to skin
- commonly recur
- transilluminable

Page 133 By Heyson Chan


lumps in the neck
- lumps in the neck anterior to the anterior border of SMC are within anterior triangle of the neck
(boundaries are SMC, midline, lower border of mandible)
1. branchial cyst
2. branchial sinus or fistula
3. carotid tumor if with pulsation / aneurysm
4. thyroid origin
5. LN
6. salivary glands
7. SMC tumor
- lumps posterior to the anterior border of the SMC are within the posterior triangle of the neck
(SMC, trapezius, clavicle). Base of posterior triangle is more commonly referred to
supraclavicular fossa

1. cystic hygroma: in infancy. In lower part of posterior triangle but can also extend into axilla.
Also known as cavernous lymphangioma (historically consists of multiple lymph-filled spaces)
clinically, is soft and poorly compressible. Increase in size when the child cries or coughs.
Brilliantly translucent when transilluminated
2. phargngeal pouch: not palpable usually
3. subclavian artery aneurysm
- lump under the jaw are submandibular region
- midline lumps are invariably of thyroid origin
- LN can occur in any region
 

Page 134 By Heyson Chan


O&T Physical Exam 
Shoulder 
1. Inspection
I. Front
i. Deltoid muscle
ii. Acromioclavicular joint
iii. Pectoris major
II. Side
i. Deltoid
ii. Anterior shoulder swelling
III. Back
i. Supraspinatus
ii. Infraspinatus
iii. Deltoid
iv. Inferior spine of scapular
- asymmetry  scapular winging
2. Palpation
I. Sternoclavicular joint
II. Clavicle
III. Acromioclavicular joint
IV. Acromial process
V. Long head of bicep tendon
- tenderness biceptal tendinitis
VI. Coracoid process
VII. Supraspinatus insertion
- tenderness  maybe supraspinatus tear
3. Movement
I. Active ROM
i. Flexion
A. Front
B. Side

ii. External rotation


- hand behind head

iii. Internal rotation


- hand behind back (document the level of spinous process reached)

iv. Adduction
- cross chest adduction

v. Abduction

vi. Scapular humeral rhythm


- abduction& seen at the back
A. Glenohumeral joint
B. Scapulothoracic joint
C. Inferior spine of scapula

Page 135 By Heyson Chan


II. Passive ROM
i. Flexion
- put your hand on pt’s shoulder  crepitus?
ii. External / Internal rotation

4. Special test
I. Shoulder impingement
i. Palpation
- supraspinatus insertion over greater tuberosity

ii. Neer impingement sign


- passive forward flexion

iii. Hawkin sign


- forward flexion, internal rotation

iv. Painful arc


- 70 - 120

II. Bicep tendon


i. Palpation
- long head of bicep

ii. Speed test


A. Supinated, extended elbow
B. Pt. resists pressing on forearm
C. Pain  bicep tendon problem

iii. Yergason test


A. Flexed elbow
B. Supinate against resistance

III. Acromioclavicular joint


i. Palpation
ii. Cross chest adduction
- stress the acromiolavicular joint

IV. Rotator cuff


i. External rotation against resistance
- infraspinatus, teres minor (external rotators)
A. Elbow flexed w/ arm facing forward
B. Resist your inward force

ii. Empty beer can test


- supraspinatus (abductor)
A. Elevate arm forward to shoulder level
B. Internal rotate shoulder
C. Resist downward force

Page 136 By Heyson Chan


iii. Lift off test
- subscapularis (internal rotator)
A. Hand at the back
B. Resist forward force

V. Shoulder instability
i. Load + shift test
A. Stand at pt’s back
B. One hand: scapula
The other hand: humeral head
C. Anterior force to humeral head
D. Degree of humeral head translation

ii. Apprehension test


- Look at the pt’s face
A. External rotate the shoulder
B. Apprehension  anterior instability
C. Backward force applied (Relocation test)
D. Alleviation of apprehension  +ve relocation test
- This test can also be done in supine position, w/ shoulder just beyond the
edge of the bed

iii. Sulcus sign


A. Pull arm downward
B. Look for sulcus appeared between lateral border of acromium &
humeral head
C. +ve  inferior instability

iv. Posterior apprehension test


A. forward flex shoulder
B. Posterior directed force on elbow
C. Apprehension  posterior instability

VI. Scapular winging


i. Push against the wall
ii. Winging  long thoracic nerve injury

Page 137 By Heyson Chan


Elbow 
1. Inspection
I. Deformity
II. Swelling
III. Mass
IV. Muscle wasting
V. Alignment
i. Measuring carrying angle
A. M: 11
B. F: 13
2. Palpation
I. Anatomical landmarks
i. Olecranon
Flexion: Equilateral triangle
ii. Medial epicondyle
Extension: Straight line
iii. Lateral epicondyle
iv. Radial head
- rotate forearm  radial head movement beneath the skin
3. Movement
I. Active ROM
i. Flexion: 145
ii. Extension
iii. Supination: 90
Elbow fixed to the sides of the pt.
iv. Pronation: 80
II. Passive ROM
4. Special test
I. Valgus stress test
i. Medial collateral ligament laxity open up of medial joint space
II. Varus stress test
i. Lateral collateral ligament laxity open up of lateral joint space
III. Ulnar nerve
i. Palpation
A. Swelling
B. Thickening
C. Tenderness
ii. Subluxation
A. Move the elbow by flexion & extension
B. Ab○ N: ulnar nerve moves back & froth around epicondyle
iii. Tinel sign
A. Elbow extended
B. Tap ulnar nerve at cubital tunnel
C. +ve  lightning sensation down ulnar nerve distribution
IV. Tennis elbow
i. Palpate lateral epicondyle  tenderness
ii. Resisted active wrist dorsiflexion  tenderness around lateral epicondyle
iii. Press on the finger while elbow extended
stretch extensor tendon
tenderness around lateral epicondyle

Page 138 By Heyson Chan


Hand & Wrist 
1. Inspection
I. Palm
i. Discoloration
ii. Scars
iii. Deformity
iv. Muscle wasting
A. Ulnar nerve: adductor pollicis (dorsum)
B. Median nerve: thenar eminence (palm)
v. Resting posture
II. Dorsum
Hand
2. Movement
I. Making a fist
i. quick assessment of integrity of finger & thumb flexors

II. Finger extension + abduction

III. Finger Abduction / Adduction

IV. Wrist extension + Finger extension

V. Flexor Digitorum Superficialis (FDS)


- immbolize the fingers that are not tested

VI. Flexor Digitorum Profundas (FDP)


- fingers are immobilized at middle phalanx

VII. Thumb abduction


- Direction:  plane of fingers
i. Abduction against resistance
ii. Abduction pollicis brevis: can be seen and palpated on resistant abduction

VIII. Flexor Pollicis Longus (FPL)


i. Flexion against resistance
ii. Flexion across the palm of the hand

IX. Opposition of thumb to little finger (Opponens pollicis)


- Opposition: abduction, pronation, flexion
i. Opposition against resistance: Palmaris longus tendon can be seen and
palpate

X. Abductor Digiti Minimi


i. Power: little fingers press against each other
ii. Abduct against examiner’s thumb, palpate
i. Tendon & muscle bulk of abductor digiti minimi
ii. Tendon of flexor carpi ulnaris

Page 139 By Heyson Chan


3. Special Test
I. Cross finger test
i. Finger adduction & abduction of MPJ
II. Finger adduction
- ensure the PIPs & DIPs extending
III. Froment’s test
- +ve: flexion of thumb interphalangeal joint
IV. Extensor Pollicis Longus (EPL)
- Tendon can be seen & palpated
V. O sign
- Flexor pollicis longus, Flexor digitorum profundas
- supplied by anterior interosseous nerve

Wrist
4. Movement
I. Radial / Ulnar deviation
II. Extension
III. Flexion
IV. Anatomical snuff box
- Extensor pollicis longus, Extensor pollicis brevis
- floor: radial artery, scaphoid
5. Palpation
I. Carpal tunnel
i. Radial: Scaphoid tubercle, Trapezium
ii. Ulnar: Hook of hamate, pisiform
II. Extensor pollicis longus tendon
- run around ulnar side of Lister's tubercle of distal radius
III. Anatomical snuffbox
IV. Watson’s test
i. Stablize the scaphoid tubercle
ii. Flex & radial deviate the wrist
iii. Pain? Clicking?
V. Pisotriquetral shear test
i. Stablize pisiform
ii. Flex & ulnar deviate the wrist
iii. Pain: arthritis of pisotriquetral joint
VI. Wrist dorsum
- radial side  ulnar side
i. Lister's tubercle of distal radius
ii. Extensor pollicis longus tendon
iii. Distal radial ulnar joint (DRUJ)
VII. Distal radial ulnar joint
i. Stablize the wrist (distal end of radius)
ii. Ulnar head is stressed in volar & dorsal direction
iii. Laxity or end point?
iv. Test in both pronation & supination position
VIII. Luno-triquetral ballotment
i. Luno-triquetral joint is just distal to DRUJ
ii. Stablize the lunate
iii. Triquetrum is balloted against the lunate (move in volar & dorsal direction)

Page 140 By Heyson Chan


Hip 
1. Inspection
I. Front
i. Leg length discrepancy
ii. Pelvic tilting
iii. Rotational Deformity
iv. Muscle Wasting
v. Scars
vi. Sinuses

II. Side
i. Increase lumbar lordosis  Fixed flexion deformity of the hip

III. Back
i. Scoliosis
ii. Gluteal Muscle Wasting
iii. Scars
iv. Sinuses

IV. Gait
i. Short limb gait
ii. Antalgic gait
iii. Trendelenberg gait

V. Trendelenberg Test
One leg stand (○ N side first, then affected side)

Gluteal muscle paralysis  Drop on unsupported side  Trendelenberg test +ve


or Cannot hold steady for 30s
Alternatively, look at the sacral dipple
One leg stand
N : sacral dimple on opposite side 

+ve: sacral dimple on opposite side 

VI. Leg length measurement


- ensure both ASIS are at the same level
i. Apparent
- umbilicus medial malleolus
ii. True:
- ASIS  medial malleolus

Significant shortening presentsdetermine the defect is above or below the knee


A. Flex the knee w/ heels hold together  difference in the knee level?
B. Direct measurement of limb length
a. Below knee
- medial joint line of knee  medial malleolus
b. Above knee
- ASIS  medial joint line of knee

Page 141 By Heyson Chan


c. Pathology lies above knee  measure between the level of ASIS and
greater trochanter
- pathology above greater trochanter  shortening of this distance
2. Palpation
Around hip joint for local tenderness
I. ASIS
II. Inguinal ligament
III. Hip joint
- deep palpationat the position of femoral pulse
IV. Pubic symphysis
V. Tip of greater trochanter
VI. Ischial tuberosity

3. Movement
I. Thomas test
II. ROM
i. N  130
Flexion: ○
- obliterate the lumbar lordosis

ii. Abduction: ○N  40


- fixed the pelvis

iii. Adduction: ○N  25


- fixed the pelvis

iv. Internal & External Rotation


A. Supine
Flex the knee and hip to 90
B. Prone
Flex the knee to 90

v. Extension
A. Knee flexed at 90
B. Buttock raised from the couch  fixed flexion of the hip

Page 142 By Heyson Chan


Knee 
1. Inspection
I. Stand
i. Front & Back
A. alignment
B. skin changes
C. scars
D. swellings e.g. popliteal cyst
ii. Side
A. Full extension of the knee?
II. Gait
- swing&stance phase
i. Side & Front
A. Rhythm
B. Abnormalities e.g. Antalgic, Trendelenberg, Short limb gait
III. Knee joint
i. Hollows
A. Medial
B. Suprapetallar
Fluid in the knee  bulge formed
2. Palpation
I. Temperature

II. Fluid in the knee


i. Fluid displacement test
A. Empty suprapetallar pouch
B. Empty medial side
C. Empty lateral side
D. Any fluid bulging out on medial side?

ii. Patellar tap test


A. Empty the suprapetallar pouch
B. Tap on the petallar to look for floating sensation

III. Anatomical Landmarks


i. Anterior
A. Tibial tuberosity
B. Patellar tendon
C. Patellar

ii. Medial
A. Medial tibial condyle
B. Medial femoral condyle
C. Medial joint line
- tenderness medial menisical tear
D. Semitendinosus & Semimembranosus tendon

Page 143 By Heyson Chan


iii. Lateral
A. Lateral tibial condyle
B. Fibular head
C. Lateral femoral condyle
D. Lateral joint line
- tenderness lateral menisical tear
E. Long head of bicep tendon
- attached to fibular head
F. Iliotibial band
- definite thick band-like structure
G. Lateral collateral ligament
a. Hip& knee flexed at 90
b. Apply varus force

3. Movement
I. Active ROM
i. Flexion
A. Golionmeter
B. Heel to buttock distance
ii. Extension
II. Passive ROM
- Note any crepitus
III. Prone heel height
- Look for subtle loss of knee extension

4. Special test
I. ACL
- ensure PCL is intact first
 posterior sag & anterior step down (just above medial tibial plateau)
∵ ruptured PCL can also give a “+ve” anterior drawer test

i. Lachman test
- Knee 15 flexion
A. Lack of end point?
B. Excessive laxaity?

ii. Anterior drawer test


A. Hamstring relaxed
B. Tibia is pulled anteriorly
C. Excessive laxaity?

iii. Pivot shift test


A. At least one finger placed on the joint line
B. Internal rotation
C. Valgus force
D. Axial loading (move the tibia towards femur)
E. Flexion: 10 - 20
F. Any clicking sensation on the joint line?

Page 144 By Heyson Chan


II. PCL
i. Posterior drawer test
A. Excessive laxaity?
B. Lack of end point?

ii. Posterior sag test


A. Pt. lying down
B. Hip & knee flexed at 90
C. ○
N : tibial tuberosity is slightly superior (anterior) to the petallar

D. PCL ruturelower tibial tuberosity

III. Collateral ligaments


i.Valgus/Varus stress test
A. Straight knee
a. Valgus force: MCL
Varus force: LCL
b. Also testing integrity of capsular structures
B. Knee flexed at 30
- only testing the collateral ligaments
IV. Meniscus
i. Mcmurray test
A. Fully flex the knee
B. Put one hand on the joint line
C. Rotation
- to bring the posterior horn of meniscus forward
a. External rotation: medial meniscus
b. Internal rotation: lateral meniscus
D. Varus: medial meniscus
Valgus: lateral meniscus
E. Extension
F. ‘Pop’ sound?Pain?
- Anterior horn injury  local tenderness along the joint line is sufficient for
diagnosis
ii. Apley’s grinding test
A. Patient in prone position
B. Knee flexed
C. Downward axial force
D. Rotate the foot
E. Meniscal tear  pain: lateral / medial side of the joint

Page 145 By Heyson Chan


Foot & Ankle 
1. Inspection
I. Walking (Gait cycle)
- Stance phase & Swing phase Heel strike, Foot flat, Heel off, Toe off
i. Side
A. Rhythm
ii. Front
A. Abnormal swing
B. Centre of gravity movement
C. Balance of iliac crest & shoulders

II. Standing
i. Front
A. Whole foot
a. Sign of infection
b. Skin conditions
c. Swelling
d. Mass
e. Wound
B. Fore foot
a. Varus / Valgus
C. Mid foot
a. Skin condition
b. Sign of arthritis
c. Swelling
D. Hind foot
a. Medial malleolus
b. Lateral malleolus
c. Ankle joint
d. Subtalal joint (seen from lateral)
E. Knee & Hip: deformity upstream can affect Foot & Ankle
ii. Back
A. Tibial calcaneal angle
-○N : a few degrees of valgus
iii. Medial
A. Medial Logitudinal Arch
-○N : a space below it
B. Windlass mechanism
a. Pt. w/ no arch or low arch
b. Stand on tip toe
c. Hyperextend the MTPJ exaggerate medial arch
iv. Lateral

III. Sitting
i. Foot hanging free
A. Plantar callosity
B. Ulcers
C. Web spaces

Page 146 By Heyson Chan


2. Palpation
I. Bone
i. Medial
A. 1st metatarsal head
B. Navicular tuberosity
C. Head of talus
D. Medial malleolus

ii. Lateral
A. Fibula
B. Calcaneus
C. Cuboid
D. Base of 5th metatarsal & styloid
E. 5th metatarsal head

iii. Plantar surface


A. 1st – 5th metatarsal head
B. Calcaneal tuberosity – tender in plantar fasciitis

II. Joint
i. Ankle joint
A. Tenderness
B. Swelling
ii. Syndesmosis
- e.g. anterior talofibular ligament
iii. Sinus tarsi
- a depression that is anteroinferior to fibula
- between talus & calcaneus
iv. Tarsometatarsal joint
A. mid-dorsum of foot
B. 1st – 5th are all palpable
v. Metatarsophalangeal joint

III. Ligament & Tendon


i. Tibialis anterior
- active ankle dorsiflexion

ii. Extensor hallucis longus


- active big toe dorsiflexion

iii. Extensor digitorum longus


- active lesser toes dorsiflexion

iv. Peroneus longus & brevis


- active eversion brings it out
A. Peroneus longus: plantar surface of 1st metatarsal base
B. Peroneus brevis: 5th metatarsal base

IV. Muscle

Page 147 By Heyson Chan


3. Movement
I. Active ROM
i. Dorsiflexion / Plantarflexion
ii. Hind foot + Mid foot motion
- a combined motion
- appear as eversion & inversion

II. Passive ROM


i. Hind foot
A. Dorsiflexion / Plantarflexion
B. Inversion / Eversion, subtalar joint
- hold the tibia & calcaneus

ii. Mid foot


N  minimal movement
- ○
- hold the calcaneus & metatarsal
A. Flexion / Extension
B. Abduction / Adduction

iii. Fore foot


A. Flexion / Extension, MTPJ
- extension: important for ○
N gait
B. Flexion / Extension, IPJ
-○N : no extension

4. Special Tests
I. Acute inversion / eversion injury
i. Lateral
A. Lateral malleolar #
B. Lateral longitudinal ligament (ligamental injury)
C. Syndesmosis
D. 5th metatarsal base #
E. Peroneus tendon

ii. Medial
A. Medial malleolar #
B. Deltoid ligament (Medial longitudinal ligament)
C. Navicular
D. Posterior tibial tendon
E. Achilles tenson

II. Ankle instability stress tests


- Recurrent ankle sprain  some people: ankle instability
i. Talar tilt test
A. Hold the tibia& calcaneus
B. Varus tilt the ankle joint
C. Repeat the test in slight plantarflexion
ii. Anterior drawer test
A. Hold the tibia& calcaneus
B. Sustained anterior force on calcaneus

Page 148 By Heyson Chan


III. Achilles injury
i. Single leg stance
Intact Achilles tendon
ii. Single leg heel raise (supporting leg)
iii. Thomson test
A. Squeeze the gastronemius
B. ○ N : plantarflexion

IV. Achilles contracture


- limited dorsiflexion
i. Knee extended
A. Passive RO dorsiflexion
ii. Knee flexed
- relax gastronemius attachment to femoral condyle
A. Passive RO dorsiflexion
B.  dorsiflexion  gastronemius is tight
C.  / ○ N dorsiflexion gastronemius & soleus is tight

V. Vascular
VI. Neurological Examine when necessary
VII. Leg length
VIII. Shoe
- Ab○ N wear pattern  Ab○ N biomechanics
i. Wearing pattern
ii. Heel counter
- ○N : wear more on outer surface
iii. Wear over metatarsal area

Page 149 By Heyson Chan


Spine 
Throacic & Lumbar Spine
1. Inspection
I. Front
II. Side
III. Back
- Scoliosis
- Muscle wasting

2. Palpation
I. Spinous Processes
i. Tenderness
ii. Deviation from the midline
II. Paraspinal muscles
i. Muscle spasm
ii. Asymmetry?

3. Movement
I. Lateral flexion
II. Rotation
- stabilize the pelvis
III. Shoeber test
- forward flexion range
i. start: tape measuring10cm above S2
N : 5cm
ii. after forward flexion: ○
Alternative method:
Distance = pt’s hand  ground

4. Neurological Examination
I. Tone
- Muscle tightness?
i. Roll the LL
ii. Move the knee
II. Power
i. Hip flexion (L2) + abduction
ii. Knee extension (L3)
iii. Ankle dorsiflexion (L4) + plantarflexion (S1)
iv. Toe dorsiflexion (L5)
III. Reflex
i. Knee Jerk / Clonus
ii. Ankle Jerk / Clonus
iii. Plantar reflex
IV. Sensation
i. Dermatomes

Page 150 By Heyson Chan


5. Special Test
I. Straight leg raise
- Demostrate tension in sciatic nerve
- Always look at the pt’s face
i. Tension sign
A. Dorsiflex the ankle stretch the sciatic nerve
B. Radiating pain down the leg big toe
ii. Bowstring sign
A. Flex the knee when the leg is still raised
B. Use the thumb to press into the popliteal fossa: lateral medial
C. Pain whensciatic nerve is palpated?

Cervical Spine
1. Inspection
I. Front
i. Torticollis
II. Side
III. Back
i. Asymmetry
ii. Muscle wasing

2. Palpation
I. Spinous Processes
i. Tenderness
ii. Upper part: Ligamentum nuclei
Lower part: Spinous processes
II. Paraspinal muscles, incl. trapezius
i. Muscle spasm
ii. Asymmetry?

3. Movement
I. Rotation
- ○N : 90
II. Lateral flexion (Side flexion)
III. Forward flexion
IV. Extension

4. Neurological Examination
I. Power
i. Shoulder Abduction – deltoid (C5)
ii. Elbow flexion (C5) / extension (C7)
iii. Wrist dorsiflexion (C6) / plantarflexion
iv. Finger abduction (T1) / adduction (T1)
II. Reflex
i. Bicep jerk
ii. Tricep jerk
iii. Brachioradialis jerk
III. Sensation
i. Dermatomes

Page 151 By Heyson Chan


5. Special Test
I. Hoffman sign
i. Flex the DIPJ of one of the fingers, usually middle finger, w/ PIPJ extended
ii. Flip it
iii. Contraction of other fingers?
- usually thumb or little finger

II. Abdominal reflex


i. Scratch the 4 quadrants lightly
ii. Equal contraction in rectus abdominii in 4 quadrants?

III. Spurling test


i. Lateral flex pt’s neck w/ an axial force
ii. Radiating pain down to the ULs?
Tilting cervical spine Neural foramina narrowed  Irritation of nerve root

IV. Lhermitte’s test


- detect UMNL
i. Flex the spine
ii. Shielding sensation down the UL?

V. Reverse Lhermitte’s test


- detect UMNL
i. Extend the spine
ii. Shielding sensation down the UL?

VI. Ten second test


- testing cervical myelopathy
i. Open & close fingers as many times as possible in 10s
ii. N : at least 12 times

iii. Ab○ N : cervical myelopathy  spastic muscle  rate of muscle contraction

VII. Finger escape test


- testing cervical myelopathy
i. Pt closes eye
ii. Fingers hold together for 30s
iii. Ab○N : drifting of any finger
unable to adduct fingers tightly

Page 152 By Heyson Chan


Things to note:
Cord Compression
3 elements must be examined
1. Motor
2. Sensory
3. Sphincter
I. PR examination
i. Anal tone

Other Reflexes Available


1. Cremasteric reflex
I. Lightly touch the medial thigh
II. Elevation of scrotum

2. Anal reflex
I. Press on the anal sphincter during PR exam
II. Sphincter contraction

3. Bulbospongiosus muscle reflex


I. ♂: pull the penis
♀: pull the Foley’s cathether
II. ♂: erection
♀: contraction of internal urinary sphincter

Page 153 By Heyson Chan


Tumor 
1. Inspection
I. Swelling
II. Muscle wasting
III. Skin changes
i. Inflammatory changes
ii. Ulceration
iii. Abnormal Pigmentation
IV. Size
- Measurement in at least 2 dimensions

2. Palpation
Palpate the ○
N side first
I. Consistence
II. Tenderness
III. Border & Shape
IV. Attachment to surrounding tissues
V. Differential movement
VI. Regional lymphadenopathy

3. Other Relevant Examinations


I. Neurological Examinations
i. Muscle Power
- MRC grading
ii. Sensation
iii. Reflexes
iv. Gait
A. Front
B. Side

Page 154 By Heyson Chan


X‐Ray 
General
1. Region, structures shown
2. View
3. General Bone Density
II. Rarefaction
i. Osteoporosis
-  bone content ∵ bone matrix 
ii. Osteomalacia
- uncalcified bone matrix
III. Sclerosis
4. Local Density Changes
5. Cortex
- continue?
6. Medulla
7. Joint space
8. Soft tissue

Fracture
1. View
I. AP
II. Lateral
III. Ankle mortis
2. Site
- also decide intra-articular / extra-articular
3. Structures shown
4. Type
I. Transverse
II. Oblique
III. Spiral
5. Comminution
6. Open?
7. Angulation & Displacement
- 2 views are need to exclude them
8. Soft tissue
I. Swelling?
9. Joint
I. Widened joint space?
II.  opacity in the joint space  overlapping w/ displaced bone
10. Extent
I. Neighbouring bone injury?
II. Epiphysis involvement?
11. Pathological?
I. History
II. Blurred # line
III. Bone density

Page 155 By Heyson Chan


Cervical Spine
1. Curvature
I.  /  lordosis
II. Its cause?

2. Anterior & Posterior margin


I. Incontinuity  subluxation?
II. Vertebral lipping  osteophyte
III. Posterior lipping  compression of nerve root?
∵ localized lipping in the uncovertbral joint
- ○L &○
R oblique views are better in showing lipping in the joint of Luschka
IV. # of osteophyte?

3. Prevertebral soft tissue


I. N <¼ of diameter of vertebral body

II. Any soft tissue compression?

4. Vertebral body
I. Rarefaction / Sclerosis?
II. Collapsed?

5. Disc space
I. /
II. Fusion?

6. Neural foramina
I. Smaller?

7. Canal
- ○
N (lateral view): depth of spinal canal = depth of vertebral body

8. Spinous process
I. #?

Page 156 By Heyson Chan


Thoracic & Lumbar Spine
1. Curvature
Thoracic: kyphosis
Lumbar: lordosis
I.  / curvature?
II. Sharp alteration?

2. Anterior & Posterior margin


I. Spondylolithesis
II. Lipping

3. Vertebral body
I. Rarefaction / Sclerosis?
II. Wedging?
III. Collapse?
IV. Bamboo spine?

4. Disc space
I. /
II. Fusion?
III. Central disc herniation?
IV. Irregular disc margin?

5. Canal
I. Canal to body ratio
- N  depth of spinal canal  interpedicular distance

: depth of vertebral body  width of vertebral body
= 1:2 – 1:4.5

6. Superior & inferior articular process


I. Defect in pars interarticularis
II. Upward displacement of inferior articular process

7. Spinous process
I. Impingement? ( ‘kissing spines’ )
II. Ossification of interspinous ligament?

8. Soft tissue shadow


I. Psoas shadow
II. Fusiform abscess shadow  tuberculosis

Page 157 By Heyson Chan


Parotid glands 
1. any mass from parotid gland is parotid tumor unless proven otherwise
2. commonest infection of parotid glands is mumps
3. DDx of parotid swelling

Gland itself Other anatomical structure in vicinity


Parotitis Lipoma
Pleomorphic adenoma Sebaceous cyst
Adenolymphoma Enlarged peri-auricular/post-auricular
LN
CA Neuroma of facial nerve

4. Anatomy
- lie posterior and inferior to angle of mandible and anterior to the external auditory meatus.
- Over the SMC and masseter
- Parotid duct open into mouth near the second upper molar teeth
- Facial nerve runs through the parotid gland (5 branches)
- Wrap around the vertical ramus of the mandible
- Deep portion in between this bone and mastoid process
-
5. Branches of facial nerves
6. Five major facial branches (in parotid gland) - from top to bottom: (Two Zebras Bit My Cat)
Temporal branch
Zygomatic branch
Buccal branch
Marginal mandibular branch
Cervical branch: platysma

7. causes of enlargement
Unilateral Bilateral
Mixed parotid tumor Mumps
Tumor infiltration (painless usually) Lymphoma
Duct blockage eg calculus Alcohol-associated
malnutrition
Mikulicz syndrome: bilateral painless
enlargement of all three salivary glands.
Probably the early stage of Sjogren’s
syndrome

Inspection

1. general condition
2. any lump in parotid area
- location
- size
- skin changes
- shape

Page 158 By Heyson Chan


3. ask the patient to open mouth to examine the opening of the parotid duct by massaging the gland
- any pus/blood
- can ask the patient to thk of sth sour => saliva will come out
->correlate it with any painful sensation
4. uvula deviation
- if deep lobe bulge out, tonsil and pillar of fauces will be pushed medially (pleomorphic
adenoma or tumor)

Palpation
1. palpate for the lump:
- site
- size
- shape
- surface texture
- edge
- temperature
- consistency
- mobility
- relationship
2. ask the patient to clench the teeth => masseter muscle become palpable
- normal: cannot palpable gland
- enlarged: best felt behind the masseter muscle and in front of the ear
3. tender, warm, swollen suggest parotititis
4. palpate the deep lobe bimanually with glove (mention it only)
5. palpate for fauces: parotid tumor may bulge into the pharynx
6. local LN: metastasis
7. submental and submandibular salivary glands

Special tests
1. test the integrity of facial nerve: intact in benign swelling vs paralyzed in malignant
- cervical: platysma
- look up
- close eye
- smile
- blow cheek

9. Sacroiliac joint
I. Unilateral / Bilateral
II. Sclerosis & destruction?
III. Fusion?

Page 159 By Heyson Chan


Prostate examination 
-comment on
1. size: finger breath
2. texture
3. pain
4. nodular / smooth
5. median groove
6. symmetrical
7. consistency: firm, rubbery, homongenous

Page 160 By Heyson Chan


Respiratory Exam 
Routine Introductions
1 Introduce yourself and ask for consent from the patient
2 Proper position: lye the patient at 45° on one pillow / sit each bed side
3 Proper exposure: expose the whole upper trunk
4 Comment on the general condition of the patient
- Conscious and alert
- Any respiratory distress
- Tachypnoea
- Cyanose
- Pursed lips
- Any oxygen supplement (e.g. nasal cannula)
- Any bronchodilators on bedside
- Any chest drain: find the chest drain box, observe the colur
- Any sputum bottle
- Ask for temperature chart

Inspection
 Be systemic. Start from the hands, to the head, the neck and the trunk.
 Ask the patient if he experience any pain before you proceed

1 The hand
a) Clubbing
b) Tar stain
c) Small muscle wasting
- first dorsal web
- thenar
d) Oesteotrophicarthropathy
e) Dysplastic nail (ugly)
f) Peripheral cyanosis
g) Signs of CO2 retention
 Warm and sweaty periphery, bounding pulse, and flapping tremor

2 The waist
a) bounding pulse
b) HPO
c) Flapping tremor

3 The head
a) Eyes
 Look for pallor and Horner’s syndrome (meiosis)
b) Tongue
 Sign of central cyanosis (cor pulmonale, fibrosing alveiolitis, bronchiectasis)
c) pursed lips (chronic small airway obstruction)

Page 161 By Heyson Chan


4 The neck
a) accessory neck muscle
b) distend vein (SVOC obstruction) => plethora
c) any neck mass and the apex area
d) JVP: rise in cor pulomonale
- COPD, braonchietasis, fibrosis

5 The chest: stand at the end of bed


a) Chest expansion
 Pathologies always lie on the side with diminished chest movement
 Symmetrical movement
b) Scars
 Look out for scar underneath the breast in females
 Sternectomy, thoracotomy
 Radiation marks
a) Deformity: barrel chest, pigeon chest (pectus carinatum), funnel chest (pectus excavatum),
Harrison’s sclcus
b) Any kyphoscoliosis
c) Any indrawing of intercostals muscles
d) Any supraclavicular fossa hyperinflation
e) Respiratory rate and rhythm
Normal rate: 10—20/min

6 peripheral edema

Palpation
1 Trachea: upper zone pathology
- Warn the patient about the choking feeling
- Ask the patient to slight flex his head
- Palpate the trachea above and behind the suprasternal notch with your middle finger, put your
ring finger and index finger at the medial end of the clavicles
 Determine whether it is in the mid-line or deviated to one side by determining the distance of
the trachea with the clavicle
 Determine any deviation and tracheal tug (draw inward when chest expands)
 May check notch-cricoid distance: length of trachea from suprasternal notch to the cricoid
cartilage is normally three or more finger breaths. Shortening of this distance is a sign of
hyperinflation

2 Apex beat: lower zone pathology


- Determine any mediastinal shift of the lower mediastinum
- Difficult to localize if hyperinflated lung

3. Chest expansion
- Instruct the patient to breath in and out deeply
- Note: any decrease in chest expansion signifies an underlying pathology
a) Upper chest and apex
- Rest the palm on the upper chest and apex bilaterally
- ask patient to 呼盡吸盡
- Note any difference in the rise and fall movement of the chest (indicated by the position
of your thumb)

Page 162 By Heyson Chan


b) Middle chest and Lower chest
- Clasping the chest using both hands with the extended thumbs pointing at each other
when the patient in full expiration: 呼盡 -> grasp ->吸盡
- Discrepancy in the distance between the thumbs during full expiration and full inspiration
is an estimation of chest expansion
- normally: 5 cm
- one at middle zone and one at lower zone

c) Posterior
- ask patient to sit up and give him a pillow to hold: swing the scapula away
- Stand facing the patient’s back squarely
- Clasping the chest using both hands with the extended thumbs pointing at each other
- Repeat in upper, middle and lower zone of the posterior: 各一

d) Cervical lymph nodes


- Palpation of the cervical lymphadenopathy should be done from the back
- You may tell the examiner that you would like to reserve the palpation of cervical lymph
nodes till the end of the examination
- CA, TB, lymphoma, sarcoidosis

Percussion
1. Anterior
- Start at the apex of the lungs
- Percuss the clavicle
- percuss along the mid-clavicla line: for comparison
- Continue downward until liver dullness is reached on the right side and stomach tympani is
reached (try to stay lateral to avoid the cardiac dullness)
e.g. hyperinflate lung will displace the cardiac dullness
- I will try to percuss all the intercostals space

2. Lateral
- Percuss the lateral chest wall along the mid-axillary line and descend in steps until dullness is
reached
- start from axilla

3. Posterior
- Ask the patient to fold both arms in front of him so as to SWING the scapula laterally when
percussing the chest wall posteriorly
- Percuss downward within the medial border of the scapulae and beyond until dullness is
reached
- dun percuss too medially since it has muscles
- from medial to lateral (in 梯形)

Findings:
Dullness: pleural effusion (stony dullness) or consolidation or lobar collapse
Hyper-resonance: pneumothorax

Page 163 By Heyson Chan


Auscultation
1. Breath sounds
- listen to breath sound with unaided ear
- quality (vesicular or bronchial), intensity (normal or reduced), added sound
- vesicular breath sound: louder and longer on inspiration than on expiration and there is no gap
between the inspiratory and expiratory sounds (though expiratory phase composed of 2/3 of
the cycle but it cannot be heard only)
- Bronchial breath sound: due to turbulence in large airway without being filtered by the alveoli.
Audible throughout expiration and with a gap between inspiratory and expiratory
- Key: compare both side

Steps:
- Ask the patient to breath in deep and slowly with mouth
- Use the bell to auscultate the lung apices
- Use the diaphragm to auscultate the rest of the lung
- Note: try to stay lateral to avoid the cardiac dullness
- Whenever crackles are heard – ask the patient to cough and re-auscultate to see if the crackles
are clear up after coughing
->if yes, then is bronchietasis

Findings:
- Normal: vesicular breath sound
- Bronchial breath sound: consolidation (lobar pneumonia), localized pulmonary fibrosis, or
lung collapse
- Wheeze crackles: COPD / asthma
- Fine crackles: early congestive heart failure and pulmonary fibrosis
- Coarse: secretions in larger airways, bronchietasis (clear up by coughing)

2. Vocal resonance
Steps:
- Follow the pattern of auscultation
- Key: educate the patient and ask to perform once before asculatate
- Ask the patient to speak softly the words “三, 五, 三” (key: nasal tone)
Findings:
- Increased (solid): lobar pneumonia, extensive fibrosis, or pulmonary collapse
- Decreased (separate): pleural effusion, pneumothorax, or pleural thickening
3. may take the opportunity to listen to the second pulmonary sound: loud in pulmonary
hypertension

The examiner may ask you to examine the patient’s respiratory system from the back. This may
serves as a hint that physical signs may be detected from the lower zone of the lung (e.g.
broncheatasis or pleural effusion, LL collapse)
In examining the respiratory system from the back, one should not forget to palpate the trachea,
lateral side and the peripheral signs

To conclude, bedside functional: peak flow meter, temperature chart, sputum pots (purulent or
haemoptysis), nebulizer therapy, inhaler therapy, oxygen supplement (rate per minute)

Page 164 By Heyson Chan


Common Respiratory Conditions
Bronchiestasis
- chronic necrotizing infection of bronchi and bronchiole leading to abnormal, permanent dilatation
of airway
Signs
Note the copious sputum mud – copious sputum, intermittent haemopytsis, foul-smelling
(Hx of chronic cough and purulent sputum since childhood is diagnostic)
Clubbing
Lung base dull on percussion
late or pan-inspiratory coarse crackles, clear up on coughing: on affected lobe
Systemic: fever, cachexia, sinusitis

Causes
Chronic infection of the bronchi and bronchioles leading to permanent dilatation of these airways
(e.g. H influenza, Strept pneumoniae, Staph aureus, pseudomonas)
Congenital: cystic fibrosis, primary ciliary dyskinesia (Kartagener’s syndrome: immobile cilia
syndrome)
Post infection: TB, pneumonia, whooping cough
Bronchial obstruction: tumor, foreign body
Ix
- high resolution CT
- sputum culture
- CXR
- bronchography
Mx
- postural drainage
- Abx
- Bronchodilator
- Surgery in selected cases

Pleural effusion
- Collection of fluid in pleural space
- blood (haemothorax), chyle (chylothorax; lymph), pus (empyema)
Signs
May with biopsy or aspiration needle marks or chest drain for pleurodesis
Look for signs of underlying malignancy (e.g. clubbing / Horner’s)
Trachea or apex beat may be deviated to the unaffected side in large effusion
Decrease chest expansion on the affected side
Stony dull percussion note on the affected side
Diminished (nearly absent) breath sound on the affected side, with bronchial BS above the area of
dullness
Vocal resonance decreased on the affected side

Causes
Need diagnostic aspiration (pleuraltesis)
Exudate (protein > 30g/L): CA lung, pneumonia, TB, secondaries of the pleura
Transudate (protein <30g/L): nephrotic syndrome, cardiac failure, liver failure
Haemothorax: trauma
Chylothorax: trauma to thoracic duct, CA or lymphoma involving thoracic duct
Empyema: pneumonia, lung abscess, bronchiectasis, TB

Page 165 By Heyson Chan


Ix
CXR
Pleural tap
Pleural Bx
CT chest
Mechanisms
1. abnormality of the pleura itself such as inflammation a/c increased permeability
2. disruption of the integrity of a fluid-containing structure within the pleural cavity such as blood
vessel with leakage of the content into the pleural space
3. abnormal hydrostatic or osmotic force operating on an otherwise normal pleural surface and
producing transudate

Lung fibrosis
- diffuse fibrosis of the lung parenchyma impair gas transfer and cause V/Q mismatch
Causes
- focal fibrosis: inhale mineral dust, radiation
- replacement of lung tissue following disease which damage the lungs: TB etc
- interstitial disease: fibrosing alveolitis
or
- upper lobe: SCHART
 S: silicosis (progressive massive fibrosis), sarcoidosis
 C: coal workers’ pneumoconisosis (progressive massive fibrosis)
 H: histiocytosis
 A: AS, allergic bronchopulmonary aspergillosis
 R: radiation
 T: TB

- lower lobe: RASCO


 R: RA
 A: asbestosis
 S: scleroderma
 C: Cryptogenic fibrosing alveolitis
 O: other eg drugs such as hydralazine, MTX

Signs: usually apical


General: exertional dyspnea, central cyanosis, clubbing
Deviated trachea to affected side
Flattened chest on affected side
Lung expansion decrease on affected side
Percussion note normal or dull on affected side
fine, late or pan-inspiratory crackles not cleared on coughing but quieter when leaning forward
Vocal fermitus normal
Signs of associated connective tissue disease: rheumatoid arthritis, SLE, scleroderma, Sjogren’s
syndrome, polymyositis and dermatomyositis

Page 166 By Heyson Chan


COPD
- spectrum of abnormalities
- bronchitis: Mucous gland hypertrophy resulting hypersecretion of mucus with chronic cough
and sputum. Defined as daily production of sputum for 3 months a year for at least 2
consecutive years
- emphysema (destructing lung parenchyma, loss elasticity, closure small airway therefore
abnormal enlargement of airway and trapping mucus)
-
Signs
Tachypnea, use of asscessory muscles
Hyper-inflation
Downward displaced hepatic and cardiac dullness
Decrease chest expansion
Expiratory wheeze
Barrel chest
Decrease breath sound
Cor pulmonale therefore increase a wave and increase v wave if with TR
Causes
Chronic smoking
Patients with anti-alpha-trypsin 1 deficiency are particularly prone

Signs of emphysema
1. dyspnoeic and sometimes called pink puffer due to hyperinflation
2. reduced suprasternal notch to cricoid distance: hyperinflation
3. barrel-shaped chest: increase AP diameter
4. pursed lip breathing: expiration through partly closed lips increases the end-expiratory pressure
and keeps airways open, helping to minimize air trapping. Prolonged expiration
5. use of accessory muscle
6. indrawing in lower intercostals muscles with inspiration: due to flattened diaphragm:
hyperinflation
7. trachea tug: hyperinflation
8. palpation: reduced expansion and hyperinflated chest
9. percussion: hyperresonant with decreased liver dullness
10. BS: decreased
11. no wheeze: but usually won’t have pure emphysema therefore may with wheeze caused by
bronchial disease
12. no cyanosed

Causes of emphysema
smoking
alpha-antitrypsin deficiency

Signs of chronic bronchitis (due to hypersecretion + airway obstruction)


1. loose cough and sputum (mucoid or mucopurulent)
2. cyanosis: blue bloaters because of cyanosis present in the latter stages and because of associated
oedema from right ventricular failure
3. palpation: hyperinflated chest with decreased expansion
4. indrawing in lower intercostals muscles with inspiration: due to flattened diaphragm:
hyperinflation
5. trachea tug: hyperinflation

Page 167 By Heyson Chan


6. use of accessory muscle
7. expansion reduced
8. percussion: increased resonance
9. BS: reduced with end-expiratory high or low-pitched wheezes and early inspiratory crackle
10. signs of RV failure due to cor pulmonale:
- increased JVP (a and v wave)
- CVS: left parasternal heave, RV gallop rhythm, loud P2 and loud ejection click, pansystolic
murmur of TR, early diastolic Graham Steell murmur
- Hepatomegaly, edema
Causes:
- smoking (major), recurrent bronchial infection may cause progression
Emphysema Chronic bronchitis
Pink puffer Blue bloater
Cyanosis Absent Prominent
Dyspnea ++ +
Hyperinflation ++ +
Cor pulmonale - Common
Respiratory drive High low

Lung collapse
If bronchus is obstructed, the air in the part of the lung supplied by the bronchus is absorbed and the
affected part of lung collapses

Signs
Mediastinum or trachea: displaced towards affected side
Chest expansion reduced on affected side
Percussion dull on affected side
Breath sounds decreased on affected side with or without bronchial breathing above the area of
collapse
Vocal fremitus decreased on affected side (but not as marked as pleural effusion)

Causes
1. intraluminal: mucus, FB, aspiration
2. mural: bronchial carcinoma
3. extramural: peribronchial lymphadenopathy, aortic aneurysm
If the lobe is not ventilated, the air within it is absorbed by the blood and the lung collapse
Neoplasm: central bronchial carcinoma
Infective: TB / bronchietasis
Foreign body

Consolidation (lobar pneumonia)


1. expansion: reduced on affected side
2. vocal fermitus: increased on affected side
3. percussion: dull
4. BS: bronchial
5. added sound: medium, later or pan-inspiratory crackles as the pneumonia resolves
6. vocal resonance: increased
7. pleural rub: may be present

Causes: pneumococcal (90%), H. influenzed, staphylococcal

Page 168 By Heyson Chan


Asthma
- paroxysmal recurrent attacks of wheezing due to airway narrowing which changes in severity
over short period of time
Signs:
1. wheezing
2. tachypnoea
3. dry or productive cough
4. tachycardia
5. prolonged expiration
6. prolonged FET: decreased peak flow, decreased FEV1
7. use of accessory muscle
8. hyperinflated lung: increased AP diameter with high shoulder and on percussion, shifting of the
cardiac and liver dullness
9. inspiratory and expiratory wheezing

Signs of severe asthma


1. exhaustion
2. cannot speak
3. drowsiness due to hypercapnia
4. cyanosis
5. tachycardia: with significant hypoxaemia
6. pulsus paradoxus: BP drops more than 10mmHg during inspiration
7. reduced BS or silent

Cases of pneumonia and pneumothorax are seldom used for OSCE. Think twice before you try to
make such a differential diagnosis. Usually with apical fibrosis, bronchietasis, PE, fibrosing
alveolitis

Type I respiratory failure


- decrease O2 and CO2 is normal.
- Pneumonia, PE, pulmonary edema, asthma, emphysema, fibrosing alveolitis, ARDS

Type II respiratory failure


- decrease O2 and increase CO2
- Pulmonary disease, decrease respiratory drive, NM disease, thoracic wall disease

Causes of bronchial breath sounds


Common: lung consolidation (lobar pneumonia etc)
Uncommon: localized pulmonary fibrosis, pleural effusion, collapsed lung

Causes of reduced breath sound


- chronic airflow limitation (esp emphysema)
- pleural effusion
- pneumothorax
- pneumonia
- large neoplasm
- pulmonary collapse

Page 169 By Heyson Chan


Added sounds
8. Expiratory wheezE (rochi): COPD, asthma
- higher the pitch, a higher velocity of the air jet. Poor guide to the severity
- is a continuous sound (musical). Due to continuous oscillation of opposing airway walls and
imply significant airway narrowing
- must time the relation to the respiratory cycle
- tend to be louder in expiration because airways normally dilate during inspiration and
narrower during expiration. Therefore, inspiratory wheeze implies severe airway narrowing
9. inspiratory crackle (crepitation)
- results of loss of stability of peripheral airways which collapse on expiration. With high
inspiratory pressure, there is a rapid air entry into the distal airway. This causes abrupt
opening of alveoli which may contain secretion.
- Fine (caused by opening and closing of small airway): heart failure, fibrosing alveolitis,
pulmonary edema => distal airway
- coarse: bronchietasis => proximal airway
- early inspiratory: chronic bronchitis, asthma
- early and mid-inspiratory and recurring in expiration: bronchietasis (altered by coughing)
- mid/late inspiratory: restrictive lung disease eg fibrosing alveolitis (reduced if lean forward)
and pulmonary edema
10. stridor: obstruction of main airway. Croaking noise loudest on inspiration. Emergency. Large
airway (=extrathoracic)
11. pleural friction rub: when both surface are roughened eg inflammatory exudate
- indicate pleurisy which maybe secondarily caused by pneumonia, pulmonary infarction

Comparisons
Disorder Mediastinal Chest wall Percussion BS Added
displacement movement sounds
Consolidation / Reduced Dull *Bronchial Coarse
Crackles
Collapse *Towards Decreased Dull Absent or /
reduced
Pleural Away Reduced *Stony dull Absent May be
effusion over fluid; pleural
maybe rub
bronchial
at upper
border
PTX Away if Decreased *Hyperresonant Absent or /
under greatly
tension PTX reduced
Bronchial / Decreased Normal Normal Wheeze
asthma symmetrically
Interstitial / Decreased Normal Normal Fine
pulmonary symmetrically inspiratory
fibrosis crackle
(fibrosing over
alveolitis) affected
lobes
unaffected
by cough

Page 170 By Heyson Chan


DDx of clubbing and crackles
1. bronchietasis
2. CA lung
3. fibrosing alveolitis
4. lung abscess

DDx of dullness at lung bases


1. pleural effusion: stony dullness, trachea may deviated to the opposite side in large effusion
2. raised hemidiaphragm eg hepatomegaly, phrenic nerve palsy
3. basal collapse: trachea deviated to the affected side, absent of breath sounds
4. collapse/consolidation: vocal resonance increased, bronchial breath sounds and associated
crackle
5. pleural thickening eg old TB or old emphysema or asbestos induced with or without
mesothelioma: trachea not deviated; breath sound will be heard

Cor pulmonale
- respiratory disorder
1. obstructive: COAD, chronic persistent asthma
2. restrictive
 intrinsic: interstitial frbrosis, lung resention
 extrinsic obesity, muscle weakness, high altitude
- pulmonary vascular disorder: ARDS, pulmonary emboli

Page 171 By Heyson Chan


Scrotum examination 
1. Inspection
- skinthin, pigmented and crinkled
- left testis always lies lower than Right
- each testis is ovoid shape
- note: any obvious mass, infection, scar, inflammatory skin changes

2. Palpation: place the penis to the other side (use a blanket or hand but not finger to hold it)
- ensure warm hand
- gentle pressure that won’t cause pain
- watch patient’s facial expression
- feel testicle between thumb and first two fingers
- 4 x3x2 cm
- Ovoid shaped
- Consistency
- then palpate the epididymis =>elongated structure along the posterolateral surface of the
testis (soft)
- roll with finger and thumb of the spermatic cord at the base of scrotum (like a rubber band)
- Examine both side

If with scrotal swelling


1. differentiate is scrotal mass or indirect hernia
- “get above” or not
2. palpate swelling between thumb and fingers
- determine: solid or cystic
3. cystic swelling
- entrapment of fluid in tunica vaginalis (hydrocele) or accumulation of fluid in epididymal
cyst
- should be fluctuant
- steady the mass between thumb and first two fingers of one hand
- use the index finger of the other hand to invaginate the mass in the second plane
- +ve: tense fluid-filled cyst will fluctuate between finger and thumb in response to pressure
change
- Cystic usu transilluminate

Hydrocele
- perform transillumination
o off the light
o cone a piece of paper (concentrate the light)
o Trans => 180o to the light source
o Spread bright red glow into scrotum
- cannot perform if the cyst wall is too thick or blood stained

Page 172 By Heyson Chan


Thyroid Examination 
頭三個(diplopia, protosis, lid lag/lid retraction)手三個 (warm sweaty hand, pulse, fine
tremor)腳三個 (proximal muscle weakness, ankle jerk, pretibial myxoedema)

Clinical Examination
The examination of the thyroid comprises three parts:
 The thyroid gland
 Structures surrounding the thyroid gland
 The thyroid status
 If asked to “examine the thyroid” you should start with the hand etc. if asked to “examine the
neck” you should just examine the neck (find any lump, lymphadenopathy, skin lesion, neck
pulse, bruit, test sensation, neck movement) and only then do a more general examination of the
thyroid, including hands and eyes if you think the lumps is in thyroid gland

Routine Introductions
9 Introduce yourself and ask for consent from the patient
10 Proper position: sit the patient up on a chair where you can approach from behind
11 Proper exposure: expose the neck and upper chest
12 Comment on the general condition of the patient
a) Nervous / irritated patient?
b) Thin / obese
c) Respiratory distress

Inspection
Inspection mainly includes the general inspection of the patient, the hand signs, the eye signs and the
inspection of the neck
Normally: cannot see

1 General condition
a) Nervous / irritated patient/ slowliness?
b) Thin / obese
c) Clothing fit to weather or not
d) Any staring appearance
e) Any pale waxy skin / diffuse hair or eyebow?
f) Any hoarness of voice?
- Integrity recurrent laryngeal nerve or in hypothy

2 The hand
a) Sweating
b) Palmar erythema: thyrotoxicosis
c) onycholysis
d) Pulse – rate and rhythm (tachycardia or even AF in thyrotoxic patient vs bradycardia in
hypothyroidism)
e) Fine tremor– with outstretched hand facing downwards with a paper
f) Clubbing (acropachy in Graves)
g) Carpel tunnel syndrome: hypothy

Page 173 By Heyson Chan


3 The eye (NO SPECS)
0) periorbital edema: hypo/hyperedema
a) Opthalmoplegia
- Fix patient’s head, over upper and outer quadrant for double vision
- Stand on the side of the patient
- As in examination for CN 3,4,6
- Variable opthalmoplegia in Grave’s disease due to lymphatic infiltration in EOM muscles
and CN III

b) Exophthalmos (>18 mm protrusion)


- Both lid moved away from centre with sclera visible below or all around iris
- Sit patient in a chair
- Check from the top by looking over the foreheadand at the side
- Can record by a Hertel exophthalmometer
- Secondary to retrorbital inflammation and lymphocytic infiltration
- Because eyes are pushed forward => patient can look up without wrinkling the forehead
but with difficulty in converging
- More common in smoker

c) Lid retraction (over activity of involuntary part of levator palpebrae superioris)


- Look at the relationship between the eyelid and the iris
- Normally: upper lid halfway between pupil and superior limbus + lower lid at a tangent
to inferior limbus
- Abn: upper lid raised + lower lid normal

d) Lid lag (upper eyelid does not keep pace with eyeball as it follows finger moving downward)
- Ask patient to follow your finger up (above the head) and down (below the nose) in a
vertical plane
- Patient’seyelid movement lags behind movement of the eye ball: loss coordination
- Due to sympathetic overstimulation: muscle spasm of the upper eyelid
- Also known as von Graefe’s sign

e) Chemosis (edema of conjunctiva)


- Due obstruction of normal venous and lymphatic drainage of conjunctiva by increased
retro-orbital pressure

f) Pupil size (over sympathetic?)

Page 174 By Heyson Chan


4 The neck
0) special attention to fullness on either side of trachea below cricoid cartilage
a) Obvious mass (especially mid-line lump)
i) Location (which side?) => describe (midline, distinct, enlarged, nodular, which lobes,
symmetrical, regularity)

ii) Any distended vein: if outlet obstruction


iii) Ask patient to swallow (give her a glass of water): always ask for a glass of water
- sip little water and HOLD it in the mouth
- fix your eye on the lump
- give instruction to swallow
- watch any upward movement of the goiter as pharyngeal muscle contract

iv) Ask patient to opening her mouth first => lock the TMJ => hold the lump => protrude the
tongue out(if you suspect thyroglossal cyst: tongue tug test)

b) Scar: subtotal thyroidectomy


- Collar scar indicates previous thyroid surgery
-
c) Distended neck veins / JVP
- Suggestive of SVCO – check Pamberton’s sign

d) Supraclavicular fossa asymmetry

5 The leg
a) Test ankle jerks
- Ask patient to rest the leg on the side of the bed / chair
= feel and watch
- Slow relaxing in hypothyroid; brisk in hyperthyroid

b) Ask patient to rise from a squatting position without using hands for support
- Proximal myopathy, a sensitive indicator for hypo/hyperthyroidism (common in Oriental,
may even with periodic paralysis)

c) Myxoedema (pre-tibial): non-pitting edema


Palpation
Should be performed both from the front and from behind, be gentle!
Normally: cannot palpate
1 The thyroid gland: both lobe
- position the landmarks: hyoid bone, thyroid cartilage, cricoid cartilage
 Size
 Site: lobes
 Tenderness: if thyroditis
 Mobility
 Consistency: firm or hard
 Nodular (single or multiple) or diffusely enlarged
 Ask patient to swallow (with water) again to confirm that it is the thyroid
- sip -> hold in mouth -> give instruction
 Whether you can get Below the mass
2 The trachea

Page 175 By Heyson Chan


- May be deviated by large goiter

3 Regional lymph nodes: malignancy may spread


Note their consistency, number and fixation
 From the chin backwards for: Submental, submandibular
 Around the ear for: pre- and post- auricular nodes
 Anterior to SCM for: Anterior triangle nodes, jugulodigastric nodes
 Clavicular region for: Supraclavicular and infraclavicular nodes
 Posterior to SCM for: Posterior triangle nodes
 Back of the neck for: Occipital nodes

4 may palpate for carotid: may be encased by a malignant thyroid gland

Percussion (define the lower extent)


1 Retrosternal goiter (rare)
 Feel the lower boarder of the gland from behind (cup by little finger or not)
 Percuss the clavicle and from the sternum below the suprasternal notch
- suggest by dullness
 If there is retrosternal goiter, perform the Pemberton’s sign to detect any superior vena cava
obstruction
- ask patient to raise arm above head
- with retrosternal goiter may develop signs of compression
- suffusion of the face, syncope, giddiness

Auscultation
1 Bruit: diaphragm
a) Thyroid mass
- both lobes
- Due to hypervascular state of the thyroid (point to Grave’s disease)
b) Carotid bruit

Questions
1 What history would you like to elicit from the patient?
The lump
 How did you notice that?
 Onset, progression
 Tenderness
Thyroid symptoms
 General, eye, GI, cardiac, neurological, gynecological

Effect of the lump


 Any breathing difficulties
 Any cosmetic concern?
 Any hoarseness of voice? (suspect malignancy)

Page 176 By Heyson Chan


2 What are the possible causes for thyroid enlargement?

Diffuse enlargement
 Toxic: Grave’s disease
 Non-toxic: simple colloid goiter (secondary to defective thyroid hormone production),
thyroditis (tender)

Nodular enlargement
 Solitary nodule
- Prominent nodule of a multinodular goiter
- Cysts
- Adenoma (FNAC indistinguishable from follicular carcinoma)
- Carcinoma
- Thyrioditis
Note: a technetium scan differentiates hot and cold nodule. A cold nodule that are solid or
partly cycstic must be regarded as malignant until proven otherwise
 Multinodular goiter
Examine the cervical lymph nodes

3 How can you confirm thyroid swelling from thyroglossal cyst?


They both have a midline position, but

Thyroid swelling Thyroglossal cyst


Does not moves with tongue protrusion Moves with tongue protrusion
Moves with swallowing Not moves with swallowing

4 What are the possible causes of enlarged cervical lymph node(s)?

Localized
 Infective: tuberculosis, infection of the head and neck region (e.g. skin infections), tonsillitis
 Neoplastic (may attach to skin): CA thyroid, CA lung

Generalized
 Infective: septicaemia, HIV
 Neoplastic: lymphoma (Hodgkin / non-Hodgkin)

5 Define anterior and posterior triangle. Give the differential diagnosis if lumps are found in
therespective area.

Anterior triangle
Anterior boarder of SCM, midline and mandible
 Solid: lymphadenopathy, chemodectomy, goiter, thyroglossal cyst, carotid body tumor,
cardotic artery aneurysm
 Cystic: branchial cysts

Posterior triangle
Posterior boarder of SCM, anterior boarder of the trapezius and the clavicle
 Solid: lymphadenopathy
 Cystic: cold absecess (due to TB)

Page 177 By Heyson Chan


6 Compare hyperthyroidism and hypothyroidism

Hyperthyroidism Hypothyroidism
(increase metabolism) (decrease metabolism)
Loss of weight Gain in weight, despite poor appetite
Heat intolerance Cold intolerance
Increased sweating Dry skin
Proximal myopathy Muscle fatigue
Diarrhea Constipation
Trachycardia / AF Bradycardia
Nervous, easy irritability Slow thought, speech and action,
depression
Fine tremor Carpal tunnel syndrome

7 What is Pemberton’s sign?


 Compression on raising patient’s hand above head by enlarged goiter
 Patient will show signs of suffocation and dilated veins

8 How would you grade a goiter (WHO)?


 Grade 0 – no goiter
 Grade 1 – palpable, but not visible (larger than terminal phalanges of examiner’s thumb)
- 1A: goiter detectable only on palpation
- 1B: goiter palpable and visible with neck extended
 Grade 2 – visible from a close distance
 Grade 3 – visible from a distance

9 How would you further investigate the patient?


It involves triple assessment
 Thyroid function test
 USG neck
 FNAC

10 What are the indications for surgery?


 Control of thyroid symptoms
 Cosmetic
 Carcinoma
 Compression

11 What are the complications for thyroid surgery?


Early
 Wound complications
 Injury to recurrent laryngeal nerve (hoarness of voice)
 Reactive bleeding
 Respiratory compromise secondary to laryngeal edema
Late
 Hypothyroidism (need life-long thyroxine in total thyroidectomy)
 Hypoparathyroidism (transient / permanent)
 Keloid scar formation

Page 178 By Heyson Chan


12 What are the treatment options available for hyperthyroidism?
Treatment options can be divided into medical, radiotherapy and surgery.

Anti-thyroid drugs RAI Surgery


Indication  Young Graves  Relapse after  Compression
(<40) subtotal  Cosmetic
 Preparation for thyroidectomy  Control of
surgery  >40 symptoms
 Pregnancy  Cancer
(PTU)
 thyrotoxicosis

C/I  Known fatal  Pregnancy (b4 or  Thyrotoxicosis


allergy after 6m)  Dependent on
 Thyrotoxic crisis voice
 Large goiter coz
cause edema
SE  Rash,  Hypothyroidism  Hypothyroidism
 Hepatitis  Exaggerate of the  Hypocalcaemia
 agranulocytosis eye’s sign of  Transient
(fever, sore Graves (esp hypoparathyroid
throat) smoker) ism
 Hoarseness of
voice

N.B. Anti-thyroid drugs include carbamepazole and PTU; PTU is preferred in pregnancy

13 What would you tell the patient before starting RAI therapy?
 Advise contraception during the course of therapy and continue for 6 months after stopping
 Stop breast-feeding
 Do not have close contact with pregnant lady and young children
 Encourage fluid intake
 Void frequently and flush the toilet twice
 Avoid food with high iodine content for a few weeks before treatment
 Risk of hypothyroidism – 50% (?) in 5 years, may requires life-long thyroxine

14 What do you know about thyroid cyst?


 True cysts with a completely smooth wall are very rare
 Most are composite lesions with colloid degeneration, necrosis or heamorrhage in benign or
malignant tumors
 Only benign is completely abolished by aspiration
 Not that cytology may be false-negative in a third of malignant cells

15 How would you manage multinodular goiter?


a) Investigations
 Thyroid function test
 USG neck – dimensions of goiter and nodules, look for cysts that can be aspirated
 CXR – for retrosternal goiter which may compress on the trachea
 Technetium scintiscan – demonstrates hot / cold nodule
 Fine needle aspiration

Page 179 By Heyson Chan


b) Treatment
Non-surgical
 Remove goitrogens
 Thyroxine 0.1-0.3mg per day (decrease TSH level)
 Aspiration of cysts with cytology to exclude malignancy, and treat recurrent simple cysts
with instillation of tetracycline
 RAI – for elderly patient unfit for surgery
Surgical
 Bilateral subtotal thyroidectomy with post-operative replacement of thyroxine

16 Compare different kinds of thyroid carcinoma.

Papillary Follicular Medullary Anaplastic


Incidence All age 50-59 years 50-59 years Elderly
groups, peak
at 40-49 years
Histological Cells Carcinoma and Arise from No
features containing adenoma are calcitonin characteristic
psammoma indistinguishable secreting architecture,
bodies and on FNAC – parafolicular does not
clear, ground therefore C cells resemble
glass nuclei recommend normal thyroid
(orphan Annie lobectomy if tissue
nuclei) follicular cells
seen on FNAC
(for diagnostic
histology)
Presentation Solitary Solitary thyroid May be Aggressive
thyroid lump lump solitary or tumor with
multiple if early spread to
occurring as surrounding
part of MEN structures
IIa or IIb
Spread Mainly Mainly Lymphatic Local,
lymphatic to heamatogenous spread lymphatic and
regional nodes to bone and lung haematogenous
Investigations TFT, CXR, USG neck, CT neck (for staging), schitigraphy (cold
nodules), FNAC (for cytological diagnosis)
Prognosis Excellent Excellent Good Poor
Treatment Total Total Total Palliative
thryoidectomy thyroidectomy / thryoidectomy surgery
/ lobectomy; lobectomy +/- with neck
central radioiodine to dissection if
compartment ablate residual LN involved
LN dissection malignant cells

Page 180 By Heyson Chan


17 What adjuvant therapy do you know of?
Adjuvant therapy useful for well-differentiated thyroid cancer only (i.e. paillary and follicular)
 Thyroxine suppression therapy (to suppress TSH level)
 Radioiodine therapy (total thyroidectomy must be performed before start of treatment)
 External beam radiation (3rd line treatment, not effective)

Investigation:
1. TSH (more sensitive than T4 except in case of secondary pituitary problem, pregnancy, control
of primary hyperthyroidism)
2. FNAC
3. US
4. thyroid scintogram

Types
Generalized Solitary nodule Multiple nodule Not palpable
Hyperthyroid Graves, primary Toxic nodule Toxic nodule / Thyroxine
hyperthyroidism goiter, overdose
(smooth, soft secondary
with a bruit) hyperthyroidism
(Plummer’s
syndrome)
Euthyroid Physiological CA (papillary Non-toxic Normal
(puberty and or nodular goiter,
pregnancy), follicular)(LN anaplastic CA
iodine involvement), (fixed to
deficiency, hemorrhage surrounding),
goitrogens into cyst, medullary CA
dominant
nodule
Hypothyroid Hashimoto’s Hashimoto’s Multinodular Thyroiditis,
thyroiditis, de goiter with post-
Quervain’s (hard gross thyroidectomy ,
and tender) degeneration post-RAI

Hyperthyroidism
1. constitutional symptoms: heat intolerance, fatigue, thin even with good appetite, OP
2. eye: lid lag / retraction, periorbital edema, NO SPECS (in Graves)
3. neck: goiter with bruit
4. CVS: palpitation, dyspnea on exertion, tachycardia, AF (in elderly), cardiac failure (thyroid
storm)
5. GI: diarrhea
6. reproductive: amenorrhoea, loss of libido
7. nervous: irritability, nervous
8. muscle: proximal myopathy, paralysis, tremor
9. skin: pretibial edema (Graves), acropachy (Graves), sweating, pruritis, palmar erythema,
onycholysis

Page 181 By Heyson Chan


Hypothyroidism
1. constitutional symptoms: cold intolerance, fatigue, obese,
2. eye: loss of outer 2/3 of eyebrow, periorbital edema
3. neck: goiter, hoarsness
4. CVS: bradycardia, hypotension
5. GI: constipaiton
6. reproductive: menorrhagia, galactorrhoea
7. nervous: mental slowness, dementia, depression
8. muscle: proximal myopathy, muscle pain, carpal tunnel, Delay Relaxation of tendon reflex
9. skin: myxodema, alopecia, dry skin, dry hair, malar rash
10. hematological: anemia

References:
6 Passing Surgery MB by Dr. Paul Lai
7 Clinical cases and OSCEs in Surgery by Ramachandran and Poole
8 Lecture Notes on General Surgery by Ellis, Calne and Watson
9 Surgical Finals – Passing the Clinical by Kuperberg and Lumley

Page 182 By Heyson Chan


Vascular Examination 
Routine Introduction
1 Introduce yourself and ask for consent from the patient
2 Proper exposure: expose from the groin to the toe, keeping the underwear
3 Proper position: ask the patient to stand up for inspection, then lye flat of the bed for palpation,
you may need to ask for permission to lift the patient’s leg up in order to inspect the heel
(important)

Inspection
 To avoid missing any pathologies, inspect from the front and from behind, paying special
attention to the webs between the toes
1 General: leg size, venous guttering (due to venous collapse secondary to ischemia), scars (pay
attention to the groin and popliteal fossa)

2 Color
 Pallor (chronic ischemia)
 Red shiny appearance (chronic ischemia)
 White (acute ischemia)

3 Trophic changes
 Loss of hair
 Nail changes
 Shiny skin

4 Loss of digits
 Due to previous amputation

5 Ulcer / gangrene
Pay special attention to the pressure area – lateral side of foot, head of 1st metatarsal, heel and
malleoli, between the toes and tips of the toes
 Comment on
a) Site, size, shape, color, depth
b) Edge (contour)
 Sloped – healing skin ulcer
 Punched out (no attempt to heal) – ischemic, DM, trophic, syphilitic
 Undermined – TB ulcer
 Rolled – basal cell carcinoma
 Everted – squamous cell carcinoma
c) Ask the patient whether the ulcer is painful(tenderness)
d) Base
 Necrotic / dry / infected / covered / discharge ( infection)
e) Surrounding tissues
 Pigmentation / induration / cellulites
f) Ask for permission to palpate the regional lymph nodes

Page 183 By Heyson Chan


Palpation
1 Temperature
 Use the back of your hand to feel the temperature
 Always compare both sides
2 Capillary refill
 Press hard on the pulp of the big toe
 Normally, the toe blanches and return to normal within two seconds

Peripheral pulses
1 Radial pulse
 Comment on the rate and rhythm (pay attention to AF)
2 Femoral pulses
 Below the mid-point of the inguinal ligament
 Compare both sides
3 Popliteal pulses
 Flex patient knee at 30 degree, cup with popliteal fossa with both hands
 Feel the popliteal pulse below the knee, medial to the center of the fossa
 Compare both sides
4 Dorsalis pedis
 Between the tendon of extensor hallisus longus and extensor digitorum longus
 Can feel both sides at the same time
 You may ask the patient to bring his big toe to his head to show the tendon
5 Posterior tibial
 1cm below and medial to the medial malleous
 Can feel both sides at the same time

Buerger’s test
The Bueger’s test is divided into two parts
(ake this opportunity to observe any venous guttering when the leg is elevated to about 15 degree)

First part
 Raise the patient leg until pallor develop
 Note the angle between the leg and the horizontal at which pallor develops

Second part
 Rest the patient leg on the edge of the bed, note any purplish discoloration (reactive hyperemia),
known as dependent rubor, in severe ischemia
 Confirms absence of auto-regulation

Routine Conclusion
4 Always tell the examiner that you would like to conclude the examination by
a) Auscultate along the course of all major arteries for bruit (e.g. AV fistula / plaque) and the
heart for murmurs (evidence for valvular disease)
b) Checking the ankle-brachial index (details will be discussed below)
c) Perform an abdomen examination for abdominal aorta aneurysm
d) If neuropathic foot is suspected  examine the lower limb neurologically
(i.e. test proprioception, jerks, tuning fork for vibration)
e) Examining the groin lymph nodes
5 Cover the patient up and make sure he feels comfortable

Page 184 By Heyson Chan


6 Summarize your findings and suggest possible differential diagnosis

Questions
1 What is the ankle-brachial index?
 Systolic pressure measured at the brachial artery by a Doppler probe, divided by systolic
pressure measured at the dorsalis pedis, with the cuff placed over the calf
 Normal index = 1; decrease if perfusion in the leg decrease
 Claudication (Fontaine class II) = 0.5
 Rest pain (Fontaine class III) = 0.3
 Impending gangrene (Fontaine class IV) = 0.2
Note: Fontaine class I = asymptomatic; class III, IV = critical ischemia

2 What are the differential diagnosis of a leg ulcer?


 Venous – Doppler ultrasound
 Arterial – ABPI
 DM – check urine sugar
 Neoplastic – BCC, SCC, melanoma
 Infective – TB, syphillus, AIDS
 Trauma
 Insect bite, chemical contact, radiation

3 What are the possible causes of ischemic ulcer?


Large artery obliteration
 Atherosclerosis
 Embolism
Small artery obliteration
 Raynaud’s disease (overactive alpha receptors  vasoconstriction)
 Buerger’s disease (collagen vascular disease, caused by infiltrate of plasma cells into arterial
wall, usually in heavy smokers)
 Embolism
 Diabetes (combination of neuropathy, peripheral vascular disease & infection)
 Physical agents, e.g. pressure, radiation, trauma, electric burns
 Scleroderma

4 What further investigations would you like to perform?


Blood investigations
 Urine for sugar and blood for glucose to exclude DM
 CBC to exclude polycythemia or anemia
 Serum cholesterol, increased risk for atherosclerosis
Non-invasive tests
 Stress ECG to exclude associated coronary artery disease and determine ET
 Echocardiogram to confirm vavular lesions
 Doppler ultrasound for measuring ABPI
Imaging
 Angiography to determine the site and extent of the blockage
 Duplex ultrasound to quantify the degree of stenosis
 Magnetic resonance angiography (alternative to angiography)

Page 185 By Heyson Chan


5 What is the principle of treatment in peripheral artery disease?
Conservative
 Smoking cessation, weight loss
 Exercise to increase the collateral circulation
 Foot care to prevent minor trauma which may lead to gangrene
 Treat co-existing conditions, e.g. DM, hyperlipidemia, hypertension
Interventional
 Angioplasty (good for small focal lesions_
 Endoluminal stenting
 Bypass surgery (indicated in severely handicapped)
i) Anatomical
(1) Femoral-popliteal bypass
 Bypass SFA occlusion
 Graft from femoral artery to popliteal artery
 Autologous or synthetic graft
(2) Femoral-distal bypass (autologous graft only)
 Bypass more distal occlusion
 Graft from femoral artery to more distal artery (e.g. peroneal)
 Autologous graft only (since high chance of failure)

ii) Extra-anatomical
(1) Axillo-femoral bypass
 Used if original graft becomes infected
 Graft from axillary artery to femoral artery
(2) Femoral-femoral bypass
 Used in conjunction with axillo-femoral bypass
 Graft goes from one femoral artery to another

6 What are the indications for surgical interventions?


 Rest pain >2weeks
 Gangrene (unhealed tissue loss) >2weeks
 Infection
 Severe claudication refractory to conservative measures

7 What are the symptoms and signs in acute ischemia?


 Pain
 Parathesia
 Paralysis
 Pallor
 Pulselessness
 Perishingly cold

8 Why are diabetic patients more prone to foot pathology?


 Neuropthay (microvascular)
 Peripheral vascular disease (macrovascular)
 Infection (both)
Note: DM patients may have preserved pulses due to calcification of walls of the vessels
preserved the pulses until late in the natural history of the disease

Page 186 By Heyson Chan


9 What are the risk factors for arterial disease?
 Smoking
 DM, HT, cholesterol
 Previous history of stroke and IHD
 Family history
 Renal failure

10 Differentiate intermittent claudication and rest pain

Intermittent claudication Rest pain


Site Thigh – external iliac Least purfused area (e.g.
Calf – superficial femoral toes or forefoot)

Intensity Due to increased oxygen Very severe, aching in


demand, pain due to nature, may wake the
anoxia, acidosis and build patient up
up of metabolites Naturopathic pain

Precipitating or relieving Increased in exercise Comes on when patient


factors relieved by resting lies flat, relieved by
Reproducible standing or walking

Note: Spinal claudication can be relieved by squatting

11 What is intermittent claudication?


 Ischemic muscular pain brought on by exercise at a reproducible walking distance, and
relieved by a period of rest
 Pain usually in the calves, but may be present in the buttocks and thighs (Leriche’s syndrome,
caused by aorto-iliac disease; also includes impotence)

12 What is critical ischemia?


 The presence of rest pain (pain in the foot at rest, particularly at night, relieved by hanging
foot over edge of bed), ulceration or gangrene indicates critical limb ischemia
 Pain worse at night because reduce blood supply to the distal aspects of the limb due to
decrease perfusion when patient is lying down, because:
i) Reduce cardiac output at night
ii) Reduce effect of gravity
iii) Relative dilatation of skin vessels due to warmth of blankets

13 Differentiate wet gangrene and dry gangrene.


Wet gangrene
 Ill-defined, spreading edge
 Skin blistering may occur
 Usually due to acute ischemia or local trauma, may be complicated by infection
Dry gangrene
 Line of demarcation between viable and dead tissue
 Indicates dead tissue may eventually fall off (auto-amputation)

Page 187 By Heyson Chan


14 Differentiate ischemic, venous and neuropathic ulcer.
Venous Ischemic Neuropathic
(trophic)
Site Gaiter’s area (over Tips of toes and Heel, underneath
medial malleolus) pressure area metatarsal heads
Lateral side and esp first toed
anterior part of the (pressure bearing
leg area) which subject
to repeated trauma

Shape Variable, usually Regular outline Regular outline,


irregular following skin
contour

Size Can be very large Varying, few mm Several cm


to few cm

Edge Sloping, pale Punched out, clean Deep punched out,


purple/brown Clean
Base Pink granulation Bone may be Often exposing
tissue or white exposed, NO bone becoz no
fibrous tissue granulation tissue feeling
Surrounding skin Chronic venous Grey/blue Normal
signs e.g.
lipodermatosclerosis
Skin temperature May be warmer Cold Normal
Pulses Normal Absent Normal
Pain Nil Moderate to severe Nil
Edema Present Absent
Ulcer feature Ragged edge Punched out
Superficial Often deep,
Ooze involving deep
fascia
Dry
Associated limb Varicosities Cold
features Leg warm, red, Ischaemic change
edematous Diminished or
Varicose dermatitis absent pulse
Haemosiderin Thin, shiny, dry
deposit skin
Atrophine blanche
(porcelain white
scar with rim of
telangectasia)
History Limb edema PVD: claudication,
Past DVT rest pain
Failed graft DM
Smoker
ABI >0.9 <0.5

Page 188 By Heyson Chan


Edge Example
Punched Arterial, trophic
Undermined Bed sore
Everted SCC
Rolled BCC

Floor and base Example


Dry or extended base or necrotic eschar Ischaemic
Superficial and tend to have fibrinous Venous
exudate and ooze sometimes purulent
fluid
Black Necrosis
Yellow Slough
Red Granulation
Pink epithelium

15 Differentiate between leg and foot ulcer


- leg: most are venous
- foot: most are arterial or DM
16 What are the indications for surgery in abdominal aorta aneurysm?
 Symptomatic (regardless of size)
 Asymptomatic >6cm in size
 Rapidly growing in size
 Strong family history

17 What are the surgical options for abdominal aorta aneurysm?


 Endovascular aortic repair
 Surgery (grafting / stenting)

18 What are the complications of surgery for abdominal aorta aneurysm?


Early
 Hypertension
 Ischemic bowel
 Leakage
 Compartment syndrome of the lower limb
 Renal failure (if supra-renal involvement)
Late
 Graft infection
 Re-rupture
 Aorto-enteric fistula
 Impotence

19 How are you going to follow-up patients with asymptomatic AAA?


 3cm: USG every 12 months
 4-5.5cm: USG every 6 months
 >5.5 cm: intervention needed

Page 189 By Heyson Chan


20 In general, what are the causes of aneurysm?
 Trauma
 Infection (e.g. mycotic, syphilitic)
 Inflammation (e.g. artherosclerotic)
 Congenital (e.g. connective tissue disorder, Berry)
 Degeneration

21 What is the treatment plan for a patient presenting with acute limb ischemia?
 Immediate heparinization
 Physical examination to locate the site of occlusion
- Above knee: embolectomy with a Fogarty catheter (under LA or GA)
- Below knee: thrombolysis (use of Streptokinase)
 Then get an angiogram to determine the run-off (indication of how much blood is purfusing
distal to the obstruction via collateral circulation)
- Run-off present: by-pass surgery
- Run-off absent: thrombolysis +/- amputation

22 Take a history from a patient presenting with intermittent claudication, varicose vein, acute
limbischemia and gangrenous toe.
Intermittent claudication
Varicose vein
Gangrenous toe
Acute limb ischemia

References:
1 Passing Surgery MB by Dr. Paul Lai
2 Clinical cases and OSCEs in Surgery
3 Lecture Notes on General Surgery by Ellis, Calne and Watson
4 Surgical Finals – Passing the Clinical by Kuperberg and Lumley

Page 190 By Heyson Chan


Acoustic Neuroma 
Physical Examination
 Inspection: loss of wrinkle on affected side (lower motor neuron type); loss of nasal-labial fold
and drooling of mouth angle on affected side (both)
 Unable to shut the eye tightly on affected side (look for Bell’s phenomenon)
 Unable to move the angle of mouth on affected side
 Reflex: loss of corneal reflex
 Ipsilateral sensory-hearing loss
 Ipsilayeral cerenellar signs may be positive

Conclusion
This elderly patient presented with (left) sided lower motor neuron type facial nerve palsy and left
sided sensory-neural hearing loss together with left sided cerebellar sign is most likely suffering from
acoutic neuroma. He is currently having problem in hearing with the left ear.

Questions
1 Name some tumors commonly arising from the cerebellopontine angle?
 Acoustic neuroma
 Meningioma
 Epidermoid cysts

2 What is the pathology for acoustic neuroma?


 Vestibular schawnoma

3 What cranial nerves are commonly affected in acoustic neuroma?


 CN7
 CN8 (unilateral sensory-neural hearing loss)

4 What is the investigation of choice?


 Audiogram to document sensory-neural hearing loss
 MRI brain

5 What is the treatment plan?


 Observation
 Radiotherapy to stop tumor growth
 Surgery
 Follow up MRI brain 3-6 months and repeat at 5 and 10 years

Page 191 By Heyson Chan


Acromegaly 
Presentation
Bony overgrowth: 
 Frontal bossing 
 Mandibular enlargement 
 Increase hand and foot size 
 
Soft tissue swelling 
 Increase heel pad thickening 
 Ring tightening 
 Coarse facial features 
 Large fleshy nose 
 Carpal tunnel symptoms 
 Visceromegaly 
 
Local tumor effects 
 Visual field defects 
 Headache 
 Double vision 
 Facial pain 
 Hypopituitarism 
 
Metabolic complications 
 Hypertension 
 Diabetes Mellitus 
 
Other symptoms 
 Cardiovascular complications 
 Deep and hollow‐sounding voice 
 Oily skin 
 Acanthosis nigricans 
 Colon polyps or cancer 
   

Page 192 By Heyson Chan


Investigation Results
To confirm the diagnosis 
1 Extended OGTT 
 GH not suppressed by glucose loading 
2 Plasma IGF‐1  
 Elevated 
 For screening and monitoring responses to treatment 
 
Other investigations 
 Anterior pituitary function 
 Prolactin level 
 Assessment of glucose intolerance 
 Serum calcium and renal function 
Radiological investigations 
 MRI / CT 
 
Questions
1 What are the causes of acromegaly? 
a) Primary GH excess 
 GH ademona 
 Mixed GH and prolactin adenoma 
 MEN I 
 GH cell cacinoma with metatasis 
b) Primary GHRH excess 
i) Central 
 Hypothalamic  
ii) Peripheral 
 Thyroid carcinoma 
 SCLC 
 Adrenal adenoma 
 
2 How would you treat the patient? 
 Transspehnoidal surgery is the treatment of choice 
a) Rapid theraputic effect 
b) Curative, particularly when the tumor is small 
c) Side effects: hypopituitarism, CSF leak, permanent DI 
 Radiotherapy 
 Medical: Octreotide sc – well tolerated, 60% control GH, 30% decrease tumor size 

Page 193 By Heyson Chan


Acute Coronary Syndrome 
Myocardial infarction
1 How do you diagnose a myocardial infarction?
Any two of the following three features 
 Typical chest pain >20 minutes 
 Recent ECG changes (ST changes in >2 leads, new‐onset LBBB) 
 Non‐equivocal cardiac markers  
 
2 What is your management plan in a patient presented with myocardial infarction? 
General
 Admit CCU 
 Close monitor vitals 
 O2 supplement with 4‐6L/min 
 Relieve anxiety with adequate explanation an support 
 CBC, L/RFT, lipid profile, CXR 
 Serial ECG and cardiac markers (TnT, CPK) for 3 days 
 Clotting profile to prepare for thrombolysis 
 
Specific
 Give adequate anagelsic (morphine 2‐5mg iv with RR monitor) 
 Give stool softener 
 Give aspirin 300 mg chewed 
 Give ACEI captopril 6.25mg Q8H or enalapril 5mg bd or acertil 2mg daily 
 Give beta‐blocker (avoid in inferior infarct due to possible involvement of AV node as AV 
node is supplied by right coronary artery) 
 Give statin  
 Consider streptokinase 1.5 MU if ST elevated MI presented within 3 hours 
 Consider CABG if thromoblytics contraindicated 
 
3 What are the contraindications of thrombolysis? 
 Active internal bleeding (incl haemorrhagic CVA, active GIB) 
 Suspected aortic dissection or pericarditis 
 Prolonged or traumatic CPR 
 Recent head trauma or intracranial tumour 
 Haemorrhagic eye condition (e.g. DM haemorrhagic retinopathy) 
 Blood pressure > 200/120 mmHg 
 
4 What are the high‐risk features? 
 Ongoing or recurrent rest pain 
 Hypotension, bradycardia or tachycardia, VT 
 ST segment changes >0.1 mV; new bundle branch block 
 Elevated Troponin > 0.1 mg/mL) 
 CHF 
 
 
 

Page 194 By Heyson Chan


5 What is the after‐care for uncomplicated MI? 
 Advise on risk factor modification and treatment(Smoking, HT, DM, hyperlipidaemia, 
exercise) 
 Stress test (Pre‐discharge or symptom limited stress 2‐3 wks postMI) 
 Angiogram if + ve stress test or post‐infarct angina 
 Drugs for Secondary Prevention of MI 
 Beat‐blocker: Metoprolol 25‐100 mg bd 
 Aspirin: 75‐300 mg daily 
 ACEI (esp for large anterior MI, recurrent MI, impaired LVsystolic function or CHF)e.g. 
Lisinopril 5‐20 mg daily; Ramipril 2.5‐10 mg daily;Acertil 2‐8 mg daily 
 Lipid lowering drugs (statin) – keep LDL <2.6 
 
Unstable Angina
1 How to diagnose unstable angina? 
Any two of the following three features: 
 Typical chest pain >20 minutes 
 Rest pain 
 Worsening pain 
2 What is the management plan for patients with unstable angina? 
 
General
 Admit CCU 
 Close monitor vitals 
 O2 supplement with 4‐6L/min 
 Relieve anxiety with adequate explanation an support 
 CBC, L/RFT, lipid profile, CXR 
 Serial ECG and cardiac markers (TnT, CPK) for 3 days 
 Correct underlying causes (e.g. anemia) 
 Consider IABP, urgent coronary angiogram / revascularisation in cases refractory to medical 
therapy 
 
Specific drug treatment:
 Nitrates (Oral first, iv if unstable esp. with CHF or marked HT) 
Begin with lowest dose, step up till pain is relieved  
Watch BP/P,  keep SBP > 100 mmHg 
 ß‐blockers (if no C/I) 
Metoprolol (Betaloc) 25‐100 mg bd 
Atenolol (Tenormin) 50‐100 mg daily 
 
 Aspirin (soluble or chewed) 160 mg stat & daily 
Ticlopidine 250mg bd or Clopidogrel 75mg daily if aspirin is contraindicated 
 Calcium Antagonists ( third line agent, non‐dihydropyridine type is preferred) 
Verapamil 40‐120 mg tds  
Nifedipine (Adalat) 5‐20 mg tds (to be considered only if patient is already on ß‐blockers)  
 Heparinisation (if pain is not controlled)   
SC low molecular weight (LMW) heparin 
 

Page 195 By Heyson Chan


Aortic Regurgitation 
Physical Examination
 Peripheral Signs: regular pulse (AF relatively uncommon in pure AR), collapsing pulse; Corrigan 
sign; nailbed pulsation (Quincke) 
 Carotid Pulse: jerky upstroke, collapsing   
 JVP: elevated if patient in heart failure 
 Apex Beat: volume overloaded; displaced if chronic AR  
 Auscultation: dual heart sound with ?loud S2; early diastolic murmur best heard at LLSB with 
the patient leaning forward and holding his breath at expiration 
 Lung Base: basal creptitation if patient in heart failure 
 Blood pressure: wide pulse pressure 
 
Conclusion 
This patient has aortic regurgitation likely due to (chronic rheumatic heart disease). The patient is 
clinically in (heart failure and is in shortness of breath). 
I would like to take the blood pressure of this patient for wide pulse pressure; check the pupils for 
Argyll Robertson pupil; look for stigmata of Marfan syndrome and check the joints for ankylosing 
spondylitis and rheumatoid arthritis. 
 
Questions 
1 What are the causes of aortic regurgitation? 
 Chronic rheumatic fever 
 Infective endocarditis 
 Hypertension 
 Syphilitic aortitis 
 Aortic dissection 
 Marfan syndrome 
 Ankylosing spondylitis  
 Rheumatoid arthritis 
 Trauma  
 
2 What investigations would you like to perform? 
 ECG may shows LVH and strain 
 CXR for pulmonary congestion and increased heart size 
 Echocardiogram to confirm the diagnosis and for the severity 
 Exercise testing for functional capacity 
 Cardiac catheterization 
 CT thorax for aortic root size 
 
3 How would you grade the severity of the disease? 
Clinical
 Wide pulse pressure 
 Collapsing pulse 
 ?Soft second heart sound 
 Duration of the diastolic murmur 
 

Page 196 By Heyson Chan


4 What medical treatment options are available for patients with aortic regurgitation? 
 Diuretics 
 Vasodilator (nitrate or prazosin) 
 ACE inhibitors (imroveing heart failure and maintain systolic function and prevent LV 
remodeling) 
 
5 What are the indications for aortic valve replacement in patients with aortic regurgitation? 
 Acute aortic regurgitation (SBE  needs urgent surgery) 
 Severe AR with class III symptoms 
 Early and progressive left ventricular dysfunction (before ejection fraction <60%) 
 Frequent ventricular ectopic beats 
 
6 What is Austin‐Flint murmur? How can in be differentiated from mitral stenosis? 
 A mid‐diastolic rumbling murmur present at apical area in AR 
 It differs from mitral stenosis that it is associate with 
 Sinus rhythm 
 Normal S1 
 Volume overloaded apex beat 
 Collapsing pulse 
 
7 What are the differential diagnosis if a patient presents with multiple vavular regurgitation?  
 Dilated cardiomyopathy 
 Infective endocarditis  ask for the temperature chart 

Page 197 By Heyson Chan


Aortic Stenosis 
Physical Examination
 Peripheral Signs: regular pulse (AF relatively rare in pure AS) 
 Carotid Pulse: small volume, slow rising 
 JVP: elevated if patient in heart failure 
 Apex Beat: pressure overloaded, mildly (or not) displaced apex beat 
 Auscultation: dual heart sound, soft S2; ejection systolic murmur best heard at aortic area 
radiating to the neck 
 Lung Base: basal creptitation if patient in heart failure 
 Blood pressure: narrow pulse pressure 
 
Conclusion 
This patient has aortic stenosis likely due to calcified aortic valve. The patient is clinically in (heart 
failure and is in shortness of breath). 
Questions 
1 What symptoms does patient with aortic stenosis suffer from? 
 Angina 
 Syncope 
 SOB on exertion 
 Fatigue  
Due to low cardiac output, coronary insufficiency and heart failure 
2 What are the causes of aortic stenosis? 
 Rheumatic heart disease 
 Calcified bicuspid aortic valve (fusion of aortic cusps which become thickened with varying 
degree of calcification) 
 Degenerative calcification 
 
3 How would you differentiate aortic stenosis from aortic sclerosis? 
 Aortic sclerosis is seen in the elderly; it will give rise to an ejection systolic murmur but with 
normal volume, undisplaced apex beat and localized murmur 
 
4 What are the complications for aortic stenosis? 
 Left ventricular failure 
 Sudden death 
 Arrythmia and conduction abnormalities (ventricular arrythmia, heart block) 
 Systemic embolization 
 Infective endocarditis  
 
5 What investigations would you like to perform? 
 ECG may shows LVH with strain pattern of increased R wave on V4‐V6 
 CXR for pulmonary congestion and increased heart size 
 Echocardiogram to confirm the diagnosis and for the severity 
 Exercise testing for functional capacity 
 Cardiac catheterization 
   

Page 198 By Heyson Chan


 
6 How would you grade the severity of the disease? 
Clinical Signs
 Narrow pulse pressure 
 Soft second heart sound 
 Narrow or reverse split second heart sound 
 Systolic thrill and heaving apex beat 
 Fourth heart sound 
 Cardiac failure 
Investigations 
 Aortic valve area <1cm2 
 Systolic gradient across the aortic leaflet 
 
7 What are the treatment options available for aortic stenosis? 
 General: avoid strenuous exercise 
 Medical: diuretics, nitrate and vasodilator for relieve of sx of heart failure 
 Surgical: balloon valvuloplasty (not effective) or aortic valve replacement 
   

Page 199 By Heyson Chan


Arrhythmia 
Tachycardia
Regular narrow complex tachycardia
 Sinus tachycardia 
 Atrial flutter with fixed block 
 Supraventricular tachycardia 
Irregular narrow complex tachycardia
 Atrial fibrillation 
 Atiral flutter with variable block 
Regular wide complex tachycardia
 Monomorphic VT 
 VT/SVT/AFL with abnormal conduction 
Irregular wide complex tachycardia
 Ventricular fibrillation 
 Polymorphic VT 
 AF/AFL with abnormal conduction 
Summary
  Regular  Irregular 
Narrow complex   Sinus tachycardia   Atrial fibrillation 
 Atrial flutter with fixed   Atiral flutter with variable 
block  block 
 Supraventricular 
tachycardia 
 Multifocal atrial 
tachycardia 
Wide complex   Monomorphic VT   Ventricular fibrillation 
 VT/SVT/AFL with abnormal   Polymorphic VT 
conduction   AF/AFL with abnormal 
  conduction 
   
Bradycardia
 Sinus bradycardia 
 Sick sinus syndrome 
 1st degree heart block 
 2nd degree heart block 
 3rd degree heart block 

Page 200 By Heyson Chan


Supraventricular Tachycardia
Management
 Check the haemodynamic of the patient 
If the patient is haemodynamically unstable: 
 Synchronize DC version 50J 
If the patient is haemodynamically stable: 
 Vagal maneuver 
 Unilateral carotid massage 
 Valsava maneuver (ask patient to pretend to strain during contipation) 
 Adenosine 10 mg i.v. bolus push (after ruling out asthma) 
 Warn the patient about chest patient and flushing before giving 
 If failed adenosine, consider verapamil i.v. 
 
How to differentiate between SVT with abnormal conduction pathway and VT? 
 Variability seen in SVT (e.g. AF with abnormal conduction) 
 Capture beat or fusion beat seen in VT (patholomonic)  
 Wider QRS (usually >140ms) and quicker heart rate (>150) seen in VT 
 
 

Page 201 By Heyson Chan


Polymorphic Ventricular Tachycardia
 
Key points
 Sustained polymorphic VT (>30s) rapidly degenerates to VF and required prompt 
unsynchronized cardioversion 
 Polymorphic VT with long QTc (>460ms) is known as Torsade de point 
 Polymorphic VT in the absence of prolonged QTc is ischemic in origin until proven otherwise 
 VF is usually more sustained 
 
Causes of prolonged QTc
 Electrolyte abnormalities 
 Hypokalemia 
 Hypomagnesaemia  
 Hypothermia 
 Drugs 
 Class I and III anti‐arrhythmics 
 Antibiotics (macrolides, ketoconazole) 
 TCA 
 Intracranial bleeding 
 Heart block with bradycardia 
 
Management of Torsade de point
 Admit CCU / ICU for close monitoring 
 Review old notes for precipitating factors and withdraw if possible 
 Review drug chart for anti‐arrhythmics and possible causes of electrolyte disturbance. Stop 
these medications, if any 
 Draw blood for electrolytes, send urgent (by ABG machine if necessary); correct electrolyte 
disturbance, if any 
 IV magnesium 5‐10 mmol over 15 minutes irrespective of serum level 
 Isoproterenol infusion or rapid ventricular pacing (defibrillation 360J) 
 No anti‐arrhythmics 
 No synchronized cardioversion 

Page 202 By Heyson Chan


Ventricular Fibrillation 
Management
 Shout for help 
 Check response and carotid pulse 
 Call cardiac arrest team (ext 2468) 
 Starts CPR  
 When cardiac arrest team arrive –check rhythm 
 Defibrillate if VF/VT, monophasic 360J 
 Immediately continue CPR x 2 min 
 Look for and treat causes 
 Review old notes 
 Urgent ABG, L/RFT (acidosis / electrolyte) 
 Listen to chest for air entry 
 Check JVP (if possible for temponade)  
 ECG for ischemia 
 Obtain iv access and give adrenaline 1mg Q3H 
 Consider amiodarone 300mg if VF/VT after 3 shocks, followed by infusion 
 Recheck rhythm and defibrillate if necessary every 2 minutes 
 
Reversible causes of cardiac arrest
 Hypovolemia 
 Hypoxia 
 Hypokalemia / hyperkalemia 
 Hypothermia 
 Tension pneumothorax 
 Temponade 
 Thromboembolism (PE) 
 Acidosis 
 Acute coronary syndrome 

Page 203 By Heyson Chan


Bradycardia
Causes
 Vagal overstimulation  
 Degeneration  
 Drugs (beta blocker, calcium channel blocker)  
 Electrolyte disturbance (especially hypokalemia) 
 Hypothyroidism 
 Hypothermia 
 Sepsis 
 Acute myocardial infarction (usually transient AV node dysfunction in posterior MI but 
irreversible in anterior MI) 
 
Management
 Rule out possible causes of SA dysfunction 
 Review drug chart for drug‐induced causes 
 RFT to rule out hypokalemia 
 TFT to rule out hypothyroidism 
 Measure body temperature to rule out hypothermia 
 Renal function test 
 ECG +/‐ Holter monitoring 
If patient symptomatic 
 Atropine i.v. may transiently increase heart rate 
 Also consider atropine or adrenaline 
 Pacing if patient haemodynamically unstable 
 
Indications for urgent transcutaneous pacing
 Asystole  
 Sinus bradycardia with symptoms 
 Mobitz type II 2nd degree heart block 
 3rd degree heart block 
 Bilateral bundle branch block 
 Newly acquired bifascular block 
 
Complications of temporary transvenous pacing
 Pneumothorax / Hemothorax (CXR pre‐ and post‐pacing) 
 Arrhythmia 
 Myocardial perforation 
 Infection of puncture site 
 Thrombosis  

Page 204 By Heyson Chan


Ascites 
Physical Examination 
 General inspection: look for signs of chronic liver disease, umbilicus may everted 
 Lymph nodes: pay special attention to the Virchow’s node 
 General palpation: no T/G/R 
 Liver: may be enlarged 
 Spleen: may be enlarged if complicated with portal hypertension  
 Kidney: normal 
 Ascites: shifting dullness +ve (always percuss with your fingers // to fluid) 
 Auscultation: normal 
 
Conclusion 
This patient has marked ascites and (leg edema with splenomegaly) due to (portal hypertension). 
The most likely cause is (chronic liver disease). The patient is currently (not in shortness of breath).  
 
Questions
1 What are the causes of ascites? 
Definition: pathological accumulation of fluid in the peritoneal cavity 
Exudate (protein > 25g/l)
 Liver cirrhosis  
 Abdominal malignancy 
Transudate (protein < 25g/l)
 Congestive heart failure 
 Hypoalbuminaemia  
 Liver failure 
 Nephrotic syndrome 
Note: send ascitic tap fluid for cytology, protein and culture 
 
2 How would you manage a patient with ascites? 
Investigations
 Send ascitic fluid for microscopy, white cell count (total and PMN), biochemistry, C/ST (use 
blood culture bottle) and cytology 
Treatment
Conservative Rx (aim to reduce body weight by 0.5 kg/day) 
 Low salt diet (1‐2 g salt per day) 
 Restriction of fluid intake (< 1L/day) 
 Monitor input/output, body weight, urine sodium 
 Spironolactone 100 mg daily (max 400 mg daily) 
 Furosemide (40‐120 mg per day) if necessary 
 Therapeutic paracentesis can be used in refractory ascites 
Therapeutic Paracentesis 
 Exclude spontaneous bacterial peritonitis before paracentesis 
 Give albumin at 6 g per litre of ascitic fluid removed  
 Caution in patients with hypotension and raised serum creatinine, monitor vital signs during 
paracentesis 
 

Page 205 By Heyson Chan


3 Define spontaneous bacterial peritonitis. What is the treatment plan? 
Definition
 Ascitic fluid neutrophil> 500/mm3with clinical symptoms or neutrophil > 250/mm3with 
evident of clinical manifestation  
(diagnostic paracentesis should be performed on hospital admission of all cirrhotic patient 
with ascites) 
Treatment 
 Give empirical antibiotics. Do not wait for culture result 
 Choice of antibiotics:  2nd or 3rd generation cephalosporin (cefotaxime – good coverage 
and penetration with low toxicity) orimipenem 
 Rx duration: 
5‐7 days if patient becomes asymptomatic and 2nd paracentesis on day 5 becomes normal 
10 days or till afebrile for > 3 days for complicated cases   
 
4 What is the mechanism of ascites formation in cirrhosis?  
 Peripheral arterial vasodilatation theory 
 Decreased filling of arterial tree  rises in renin‐aldosterone‐NA system 
 Renal vasoconstriction 
 Salt and water retention 
(Therefore spironolactone – aldosternone antagonist is effective) 
 
5 What is heapto‐renal syndrome? 
 Development of renal failure in patients with severe liver disease in the absence of any 
other identifiable cause of renal pathology 
 Hypopurfusion of the kidney is the hallmark of HRS 
 Type 1: associate with SBP, rapid rise of Cr >200, poor prognosis 
 Type 2: moderate and stable reduction of GFR, associate with refractory ascites, liver 
transplant for cure, possible use of terlipressin and albumin 
   

Page 206 By Heyson Chan


Atrial Fibrillation 
Physical Examination
 Peripheral Signs: irregularly irregular pulse 
 Carotid Pulse: normal   
 JVP: elevated without a waves 
 Apex Beat: may be displaced (mitral valve disease or chronic hypertension) 
 Auscultation: pulse deficit, pay attention to any underlying mitral valve disease 
 Lung Base: basal creptitation if patient in heart failure 
 
Conclusion 
This patient has an irregularly irregular pulse associate (with a tapping apex beat and early diastolic 
rumbling murmur best heard at the apex). My diagnosis is (mitral stenosis complicated with) atrial 
fibrillation. The most likely cause is (chronic rheumatic heart disease) and the patient is (currently 
stable). 
I would like to check the blood pressure and the thyroid status for possible underlying cause of the 
atrial fibrillation. 
 
Questions 
1 What are the causes of atrial fibrillation? 
 Aging  
 Mitral valve disease 
 Ischemic heart disease 
 Thyrotoxicosis 
 Constrictive pericarditis 
 Chronic pulmonary disease 
 
2 What are the complications of atrial fibrillation? 
 Palpitation 
 Thromboemolism 
 Heart failure 
 
3 How would you investigate the patient? 
 Electrolyte for hypokalemia and hypomagnesaemia 
 ECG for loss of P waves and irregularly irregular heart rate 
 Echocardiogram for LV function and underlying valvular disease 
 Thyroid function test to rule out thyrotoxicosis 
 Consider treadmill and Holter if indicated 
 
4 When will you consider warfarin in patients with atrial fibrillation? 
 Patients undergoing cardioversion 
 Patients with underlying mitral valve disease 
 Patients in left ventricular failure 
 Patients with cardiomyopathy 
 Patients above 60 years of age 
 
   

Page 207 By Heyson Chan


5 How do you choose between rate control and rhythm control? 
  Rate Control  Rhythm Control 
Advantages   Digoxin particularly useful   Prevents complications of 
in patients with AF and CHF AF by early termination 
 Beta‐blocker useful in   Anti‐coagulation is not 
patients with  needed 
thyroidtoxicosis 
Disadvantages    Narrow theraputic window   Significant side‐effects 
and slow onset for digoxin from amiodarone 
 Need of anticoagulation   Long term maintenance 
only achieved in 50% after 
one year 
 Need life‐style 
modification as well 
(abstinence from alcohol, 
avoid big meals and stress)  
Rate control
 Digoxin 
 Beta‐blocker (esmolol / metoprolol)  
 CCB (verapamil) 
Rhythm control 
 Electrical: DC cardioversion after anti‐coagulation with warfarin for 3‐4 weeks (if AF onset 
>48 hours) or TEE to rule out thrombus + LMWH 
 Pharamacological: amiodarone or class IA, IC, III drugs (procainamide, ibutilide) 
 
6 What is your management plan for a patient on long‐tern warfarin, preparing for dental 
extraction? 
 Admit the patient 5 days prior to surgery 
 Stop warfarin after admission, start full dose low molecular weight heparin injection 
(weight adjusted) 
 INR should be normalized in 5 days 
 Start usual dose of warfarin after operation and low molecular weight heparin until target 
INR is reached 
 
7 What is the target INR for patients with AF? 
 INR 2‐3 is recommended for patients with mitral valve disease with AF 
 Aspirin for low risk patient under 60 years of age 
 
8 Name some medications known to interact with warfarin? 
 Metronidazole 
 Macrolides (e.g. erthyromycin) 
 
9 If a patient on warfarin presented with abnormal clotting profile and acute bleeding, what 
will you do? 
 Stop warfarin 
 Give FFP infusion 
 Give oral vitamin K supplement 

Page 208 By Heyson Chan


Bronchiectasis 
Physical Examination
 Sputum mug (thick, green/yellowish sputum), bronchodilators on bedside 
 Inspection: clubbing, otherwise normal 
 Lymph node:  negative 
 Palpation: trachea central, apex beat not deviated, normal chest expansion Percussion: normal  
 Auscultation: vesicular breath sound, (may associate with prolonged expiratory phase), 
bilateral basal coarse inspiratory crepitations, changes with coughing; normal vocal resonance 
 
Conclusion
This lady with clubbing and bilateral basal coarse crackles which changes with coughing has 
bronchiectasis likely due to (previous chest infection) is currently in shortness of breath requiring 
bronchodilators. 
Questions
1 What is bronciectasis? 
 It is characterized by chronic bronchial supprative inflammation, dilatation and thickening 
airways, and airflow obstruction 
 
2 What are the causes of bronchiectasis? 
Post-infective
 Pneumonia 
 Tuberculosis 
 Viral 
 Measles 
 Pertuisis   
Post-obstructive
 Tumor 
 Foreign body 
 Post‐TB endobronchitis 
 LN compression 
Congenital
 Cystic fibrosis 
 Primary ciliary dyskinesia 
 Kartagener syndrome (associate with dextrocardia and sinus invertus) 
 RA, Sjogren syndrome 
 
 
3 What are the common organisms responsible for acute exacerbation? 
 H influenza 
 Strept pneumoniae 
 Staph aureus 
 Pseudomonas aeruginosa 
 Moraxella catarrhalis 
   

Page 209 By Heyson Chan


4 What is your treatment plan for acute exacerbation? 
Investigations 
 CBC, L/RFT 
 ABG if severe airway obstruction 
 Save sputum x C/ST 
 CXR 
 Diagnosis by high‐resolution CT thorax 
 
Treatment
 Chest physio‐therapy including postural drainage 
 Short acting beta‐agonist (salbutamol) 
 Antibiotics during early phase of acute exacernation (need anti‐Pseudomonal)  
 According to previous sensitivity results, or 
 3rd generation cephalosporin (cefoperazone) + quinolones (levofloxacin) 
 If severe haemoptysis  BAE + urgent bronchoscopy 
 Inhaled steroid (if airway reversibility following bronchodilator) 
 
5 Describe the typical chest X‐ray findings of bronchiectasis? 
 Tramlines 
 Cystic shadows  
(due to thickened airways and dilated bronchi) 
Note: plain CXR is not sensitive in the diagnosis of bronciectasis, use HRCT instead 
   

Page 210 By Heyson Chan


Brown‐Sequard Syndrome 
Physical Examination
 Inspection: normal 
 Tone: normal  
 Power: ipsilateral decreased power at and below the level of the lesion 
 Reflexes: ipsilateral hyper‐reflexia at the level of leision and hypo‐relexia below the level of the 
lesion 
 Sensory: presence of a sensory level: contralateral loss of pain and temperature sensation 
below the level of the lesion and ipsilateral loss of joint position and vibration sense 
 Cerebella signs: nil 
 Gait: hemiplegic / monoplegic gait  
 
Conclusion 
This patient has paraplegia with a sensory level has Brown‐Sequard Syndrome due to (trauma). He 
is limited by weakness in one limb and requires walking aid. 
I would like to examine the spine of the patient and look at the fundi for optic atrophy (multiple 
sclerosis). I would also like to ask for bladder and bowel symptoms. 
 
Questions 
1 What are the causes of Brown Sequard Syndrome? 
Causes involves hemisection of the cord 
 Multiple sclerosis 
 Trauma 
 Syringomyelia 
 Cord tumor 
 Heamatomyelia 
 Degenerative disease of the spine 
   

Page 211 By Heyson Chan


Bullous Eruption 
Description
 Comment on distribution of blisters 
 Comment on superficial / crusts 
 Comment on integrity of blisters 
 Look into mouth for ulceration 

Questions
1 What is bulla? 
 Bulla is a circumscribed elevation of the skin larger than 0.5cm, containing fluid 
 
2 How would you confirm the diagnosis? 
 Biopsy of a fresh blister (less than 12 hours old) with a portion of perlesional skin of 
histology and immunofluorescence studies 
 
3 What is the characteristic of pemphigus vulgarus? 
 Blisters affecting skin (epidermis) (usually over trunk and face, with extremeties relatively 
spared) and mucosa (e.g. oral mucosa, conjunctiva) 
 Intact blisters seldomly seen because the roof of each blister is thin and easily ruptured 
 Ruptured bullae lead to formation of erosions 
 Nikolsky’s sign – rubbing of normal skin may cause new lesions to form (superficial 
separation of skin after gentle pressure) 
 Biopsy shows suprabasal blisters with acantholysis (disruption of epidermal intercellular 
connections 
 IgG deposition within epidermis 
 
4 How would you manage a patient with pemphigus vulgaris? 
 Admit the patient if extensive disease (since potentially lethal) 
 Barrier nursing 
 Intravenous fluid 
 Oral steroid (high dose prednisolone)  
 Immunosupressants like azathioprine and cyclophamide can be added for their steroid‐
sparing effects 
 
5 What are the characteristics of bullous pemphigoid? 
 Three times as common as pemphigus vulgaris 
 Associate with penicillin, frusemide and captopril 
 Large intact blisters, commonly seen on the lower abdomen, inner thighs, groin, axillar and 
flexures 
 Early lesions in the form of urticarial plagues may precede bulla formation 
 Skin biopsy – subepidermal blisters with mixed infiltrates 
 IgG deposition in the dermal‐epidermal junction , eosinophil rich 
   

Page 212 By Heyson Chan


6 How would you manage patients with generalized forms of autoimmune bullous disease?  
 Treatment depends on severity of disease 
 Mild localized lesions – potent topical steroid 
 Generalized disease – systemic steroid needed 
 For recalcitrant cases, immunosuppressants may be added but should be used with cautions 
as most of the patients are elderly and are more susceptible to the side effects of 
immunosuppressants  
 Self‐limiting disease but spontaneous remission usually takes 3‐6 years 
 Chronic course marked by exacerbation and remission 
 
7 Mention some other causes of blistering diseases. 
Intraepidermal 
 Acute dermatitis e.g. eczema, contact dermatitis 
 Herpes infections 
 Friction blisters 
 Erythema multiforme 
 Pemphigus vulgaris 
Subepidermal  
 Bullous pemphigoid 
 Erythema multiforme (dermal type) 
 
8 What are the common causes of erythema multiforme? 
Typical target lesions with discoid well‐demarcated erythermatous plaques with central blisters, 
can be found in all four limbs (Steven Johnson syndrome if the mucosal is involved – sever form!) 
 Hypersensitivity reaction to infections – herpes simplex, streptococcal, mycoplasma 
 Drugs – penicillin, sulfonamides, barbiturates, piroxicam 
   

Page 213 By Heyson Chan


Cerebellar Disease 
Physical Examination

General
 Speech: scanning speech, explosive speech 
 Truncal ataxia 
 
Cranial Nerves
 EOM: horizontal jerky nystagmus in both eyes on looking towards the affected side with fast 
gaze towards affected side 
 
Upper Limb
 Finger nose test: past‐pointing 
 Rebound phenomenon: with arms outstretched and give a small push downward 
 Intentional tremor 
 Dysdiadochokinesia: rapid alternating movement of hands 
 Hypotonia 
 Abdent or pendular reflexes (legs swings forwards and backwards when knee jerk is elicited) 
 
Lower Limb
 Heel‐shin test 
 Gait: broad based, inability to do tandem walking 
 
Conclusion 
This patient has a cerebellar syndrome associated with (acute onset of vertigo), likely due to 
(cerebellar stroke). He is currently (bed‐bound and demonstrates marked ataxia). 
 
Questions
1 What are the causes of cerebellar syndrome? 
 Brianstem vascular lesion 
 Demyelination 
 Phenytoin toxicity 
 Alcholic cerebellar degeneration 
 Space occupying lesion in the posterior fossa 
 Hypothyroidism 
 
2 How are cerebellar signs localized? 
 Gait ataxia: anterior lobe 
 Truncal ataxia: posterior lobe 
 Limb ataxia, esp upper limb and hypotonia: lateral lobes 
   

Page 214 By Heyson Chan


3 What is the difference between sensory ataxia and cerebellar ataxia? 
 
  Cerebellar ataxia  Sensory ataxia 
Site of lesion  Cerebellum  Posterior column or 
peripheral nerves 
Deep tendon reflexes  Unchanged or pendular   Lost or diminished 
Deep sensation  Normal  Decreased or lost 
Sphincter disturbance  None  Decreased when posterior 
column involved, causing 
over flow incontinence 
 
4 What is nystagmus? 
 Nystagmus is a slow drift in one direction with fast correction in opposite direction. It is 
conventional to describe the nystagmus in the direction of fast phase. (pendular nystagmus 
if speed in both direction is the same) 
Vertical nystagmus (rare)
 Brainstem disease 
a) Upbeat = upper brainstem / anterior vermis of cerebellum  
b) Downbeat = medullary‐cervical junction 
Horizontal nystagmus (common) 
a) Ataxic nystagmus 
b) Multidirectional gaze‐evoked nystagmus 
c) Unidirectional nystagmus 
 
5 What is the investigation of choice for suspected cerebellar lesion? 
 MRI brain 
 
   

Page 215 By Heyson Chan


Charcot Marie Tooth Disease 
Physical Examination
 Inspection: Pes Caveous, inverted champagne bottle, distal muscle atrophy 
 Tone: Normal  
 Power: Distal limb weakness 
 Jerks: Absent ankle and knee jerk, downgoing planta 
 Cerebellar Signs: Nil 
 Sensory: Stock and gloving sensory loss 
 Gait: High stepping gait, foot drop 
 
Conclusion
This patient has inverted champagne bottle legs with sensory impairment, which is due to 
hereditary Charcot‐Marie‐Tooth disease. She has severe foot‐drop and requires calipers.  
I would like to enquire the family history, look at the spine for scoliosis, look at the hand for small 
hand muscle wasting and feel the lateral popliteal nerve for thickenings.  
 
Questions
1 In what conditions will pes caveous be seen? 
 Friedreich ataxia 
 
2 What subtypes of Charcot‐Marie‐Tooth disease do you know of? What is the mode of 
inheritence? 
 Type 1: demyleinating neuropathy, autosomal dominant  
 Type 2: axonal neuropathy  
 
3 How to establish the diagnosis?  
 Positive family history 
 Typical phsycial findings 
 Nerve conduction study 
 
4 What is the prognosis?  
 Good, degree of disability is minimal in spite of marked deformity 
 
5 Name a few causes of motor neuropathy. 
 Guillain‐Barre syndrome 
 Peroneal muscular atrophy 
 Lead toxicity 
 
6 Name a few causes of predominantly sensory neuropathy. 
 DM 
 Alcoholism 
 Vitamin V12 deficiency 
 Chronic renal failure 
 Leprosy 
   

Page 216 By Heyson Chan


CN3 Palsy 
Physical Examination
 Inspection: Ptosis in the affected side, “down and out”, dilated pupil on affected side (note: 
pupil is spared in ischemic causes)  
 EOM: the affected eye can only abduct and slightly depress; diplopia in all directions except on 
lateral gaze to the side of the third nerve lesion 
 Light reflexes: pupil on the affected side is unreactive (or slowly reactive) to light reflexes 
 Accommodation reflex: failed conjugation and pupil fixed on affected side 
 
Conclusion
This patient has a third nerve palsy, due to (diabetes mellitus) is experiencing severe diplopia and 
ptosis. 
I would like to take the blood pressure and check the urine for sugar for this patient. 
 
Questions

1 What are the causes of third nerve palsy? 
Medical
 Diabetes (most common cause) 
 Hypertension 
 Multiple sclerosis 
 Basal meningitis 
 Opthalmoplegic migraine  
Surgical
 Posterior communicating artery aneurysm (painful) 
 Herniating uncus 
 Tumors 
 Meningioma at the wing of sphenoid  
 Basal skull malignancy 
 
2 Describe the course of third nerve. 
 Nucleus at midbrain  tegmentum of the midbrain and emerges into the interpeduncular 
fossa  apex of the petrous temporal bone  cavernous sinus  superior orbital fissure 
 EOM muscles (SR, IR, MR, IO) 
 
3 What do you worry if a comatose patient presented to the A&E has third nerve palsy? What 
is your initial management plan? 
 Expanding supratentorial lesions leading to herniating uncus  
 Urgent CT brain after stabilization 
 
4 What is Weber syndrome?  
 Ipsilateral third nerve palsy with contralateral hemiplegia 
 The lesion is in the midbrain 
 
 
   

Page 217 By Heyson Chan


5 What do you know about the muscles of extra‐ocular movement? 
Third nerve 
 Medial rectus: adduct the eye 
 Superior rectus: elevate and abduct the eye 
 Inferior rectus: depress and abduct the eye 
 Inferior oblique: elevate and adduct the eye  
Fourth nerve 
Superior oblique: depress and adduct the eye 
Sixth nerve 
 Lateral rectus: abduct the eye 

Page 218 By Heyson Chan


CN6 Palsy 
Physical Examination 
 Inspection: eye of the affected side is slightly deviated medially 
 EOM: diminished lateral gaze on the affected side, horizontal diplopia on lateral gaze; 
peripheral image disappeared when affected eye being covered 

Conslusion
This patient has a sixth nerve palsy, due to (diabetes mellitus) is experiencing severe diplopia. 
I would like to take the blood pressure and check the urine for sugar for this patient. 
 
Questions
1 What are the causes of CN6 Palsy? 
Medical
 Hypertension 
 Diabetes  
 Multiple sclerosis 
 Basal meningitis 
Surgical 
 Raised intracranial pressure (false localizing sign) 
 Acoustic neuroma 
 Basal skull malignancy 
 
2 Describe the course of CN6. 
 Necucleus at pons  braintsem  base of skull  petrous tip  cavernous sinus  
superior orbital fissure  lateral rectus  
   

Page 219 By Heyson Chan


CN7 Palsy 
Physical Examination
 Inspection: loss of wrinkle on affected side (lower motor neuron type); loss of nasal‐labial fold 
and drooling of mouth angle on affected side (both)  
 Unable to shut the eye tightly on affected side (look for Bell’s phenomenon) 
 Unable to move the angle of mouth on affected side 
 Reflex: loss of corneal reflex 
 
Conclusion
This patient has a (lower) motor neuron type seventh nerve palsy, the most common cause is (Bell’s 
palsy) and the patient (cannot close his eye completely). 
I would like to look into the external auditory meatus and the oral cavity for vesicles and examine 
the parotid gland. I would also like to ask for disturbance of taste in the anterior 2/3 of the tongue 
for the involvement of chorda tympani and for hyperacusis for the involvement of stapedius muscle 
in the inner ear.  
I would like to check the urine for sugar. 
 
Questions
1 What are the causes of seventh nerve palsy? 
Upper motor neuron type
  Stroke 
Lower motor neuron type 
 Bell’s palsy 
 Ramsay‐Hunt syndrome 
 Cerebellopontine angle tumor 
 Old polio 
 Otitis media 
 Skull fracture 
 
2 Describe the course of seventh nerve. 
 Nucleus at the pons 
 Cerebello‐pontine angle 
 Internal acoustic meatues 
 Giving off chorda typani then enters stylomastoid foramen 
 Facial canal 
 Through parotid glands 
 Giving off the terminal branches 
 
3 How to differentiate between upper and lower motor neuron type CN7 palsy? 
 Upper motor neuron type: upper face spared  
 Lower motor neuron type: whole half of the face affected 
   

Page 220 By Heyson Chan


4 What is the treatment for Bell’s palsy? 
 Eye protection with eye drops and patch during sleep 
 Dexamethasone 40 mg (high dose) per day for 5 days 
 70% of patients will recover spontaneously without deficits 
 
5 What is Ramsay‐Hunt syndrome? What is the treatment? 
 Herpes zoster infection at the geniculate nuclei 
Treatment
 Eye protection 
 Oral steriod 
 Oral acyclovir  
 Analgesics 
 
6 Briefly describe how to examine different branches of the facial nerve. 
 Wrinke forehead by looking upwards for frontalis (temporal) 
 Close eyes while examiner attempts to open them for orbicularis oculi (zygomatic) 
 Purse lips while examiner presses cheeks for buccinator (buccal) 
 Show teeth for orbicularis oris (massater) 
   

Page 221 By Heyson Chan


CNS Infection 
1 What are the common organisms causing meningitis? 
 H influenza 
 Klebsiella pneumoniae 
 MTB 
 Meningococcal  
 Streptococcus pneumoniae  
 
2 What are the possible sauces of infection? 
 Head trauma (skull fracture) 
 Previous neuro‐surgery 
 Otitis media  
 Sinusitis 
 Dental caries 
 Chest infection (especially TB) 
 
3 What are the salient points to be noted during physical examination in a patient with 
suspected meningitis? 
 Note the temperature 
 Any neck stiffness / Kernik’s sign (meningism) 
 Any focal signs 
 Any pailloedema 
 Any paupular rash (suggestive of meningococcal infection) 
 
4 How would you manage a patient with suspected meningitis? 
 Admit the patient 
 Bed rest, DAT 
 BP/P/T Q4H, GCS 
 CBC with D/C, L/RFT, glucose, total protein 
 Blood for C/ST (before giving antibiotics) 
 CT brain (before LP) 
 LP for WCC, G stain, C/ST, glucose, total protein, AFB, PCR (quicker results for indentifying TB 
 need 1 year anti‐TB treatment) 
 Start empirical antibiotics (IV amoxicillin ( vancomycin in neurosurgical or IC patients), 
ceftriazone) 
 Change to appropiate antibiotics once the organism is identified 
 Antibiotics should be continued for 10 days for pneumococci and 3 weeks for G‐ve bacteria; 
don’t change to oral therapy 
 May consider systemic steroid with antibiotics to decrease the rate of complications 
 
   

Page 222 By Heyson Chan


5 What are the contraindications to LP? 
 Focal signs (or anything suggestive of increased ICP) 
 Seizures 
 Impairment of consciousness 
 Infection at site for LP 
 
6 How would you interpret the following LP results? 
 
  Appearance  Cell  Protein  Glucose  
Normal  Clear  <5 (lymphocytes)  0.2‐0.4  2.5‐4.5 (>50% of 
blood)  
Aseptic   Slightly turbid  10‐500  Slightly raised Normal 
(lymphocytes) 
Bacterial   Turbid  100‐1000  0.8‐4.0  Reduced (<50%) 
(polymorphs) 
TB  Slightly turbid  50‐100  >4.0  Reduced (<50%) 
(lymphocytes, 
polymorphs initially)
 
7 What are the complications of meningitis? 
 Brain edema 
 Electrolyte disturbance 
 Cerebral infarction 
 Hydrocephalus 
 Subdural empyema 
 Cranial nerve palsy 
 
8 Will you consider antibiotic prophylaxis to persons with close contact with patients suffering 
from meningococcal meningitis? 
 Yes, consider rifampicin prophylaxis 
 
9 What is the most common cause of viral encephalitis? What is the treatment for 
viralencephalitis? 
 HSV encephalitis (identified by PCR DNA assay) 
 IV acylovir (given empricially for all patients suspecting viral encephalitis) 
 
10 What are the characteristics of Taenia solium infection? 
 S/s: hydrocephalus, increased ICP, dementia, stroke, epilepsy 
 CT shows characteristic cysts that may be partly calcified 
 Deafness as major complication 
11 Name some fungus that can cause CNS infection. What is the treatment? 
 Cryptococcus neofromans can cause meningitis in IC patients 
 Latex aggrutilation test in CSF to detect cryptococcal antigen 
 Candida albicans causes multiple microabscess 
 Treatment by IV amphotericin B 
 
   

Page 223 By Heyson Chan


12 What is the typical EEG change of CJD? 
 Oth CJD and vCJD are caused by prions 
 Causes dementia, myoclonus, pyramidal and cerebeallar and extrapyramidal signs 
 CJD linked to progressive multifocal leukoencephalopathy 
 Variant CJD linked to bovine spongiform encephalopathy 
 Sporadic CJD shoes generalized spike waves  
 Death with 1 year in 90% of cases 
 
13 What are the causes of aseptic meningitis? 
 Partially treated bacterial meningitis 
 TB / fungal / viral meningitis 
 Neoplastic meningitis (e.g. lymphomatous or carcinomatous meningitis) 
 Parameningeal infections 
 Connective tissue disease (e.g. SLE) 
   

Page 224 By Heyson Chan


Conn’s Syndrome 
 
Presentation
 Hypertension – aldosterone acts on renal collecting ducts causing salt retention 
 Hypokalemia – aldosterone causing sodium retention and potassium excretion 
 Fatigue, muscle weakness, thirst 
 Polyuria, nocturia 
 
Investigation Results
 Renal function: high Na, low K 
 Captopril test: decrease aldosterone levels in normal patient by decreasing the simulatory 
angiotensin II but no effect in Conn’s syndrome 
 Imaging: CT abdomen and cholesterol scan (cholesterol needed in minerolo and glucocorticoid 
synthesis, high uptake in Conn’s syndrome) 
 
Questions
1 What are the causes of Conn’s syndrome? 
 Adrenal adenoma (90%) 
 Adrenal hyperplasia (10%) 
 
2 What is the treatment for Conn’s syndrome? 
 Adenoma: adrenalectomy 
 Hyperplasia: if unilateral –adreanlectomy; if bilateral – give apironolactone 
 All patients must be covered with spironolactone pre‐operatively 
   

Page 225 By Heyson Chan


COPD 
Physical Examination 
 Note any O2 supplement, SaO2 monitor, bronchodilator on bedside 
 Peripheral: note any clubbing, nicotine stain, purse lip breathing, use of accessory muscle, 
signs of CO2 retention 
 Inspection: symmetrical chest expansion, increased AP diatmeter 
 Lymph node: nil 
 Palpation: trachea, apex beat not deviated, diminished chest expansion 
 Percussion: normal / hyper‐resonance 
 Auscultation: vesicular breath sound, diminished air entry, bilateral expiratory wheeze with 
prolonged expiratory phase, vocal resonance normal  
 
Conclusion
This patient has. He is clinically in (shortness of breath with the use of accessory muscle and is on 
oxygen supplement of 2L/min). 
I would like to examine the heart for loud P2 suggesting cor pulmonale. 
 
Questions
1 What is the acute management of acute exacerbation of COPD? 
 Controlled low dose oxygen administration (start with 24% Venturi mask or 1‐2L/min by 
nasal prongs).   
 Check ABGs within 30‐60 mins of starting oxygen, modify flow rate according to PaO2 and 
pH 
 Other Ix: CBC with D/C, L/RFT, sputum for C/ST, ECG 
 CXR (pay attention to any penumothorax) 
 2 agonist (salbutamol 5mg Q4H puff) and ipratropium bromide alone or in combination 
 If no response, consider iv aminophylline 
 Corticosteroids (hydrocortisone 100 mg iv Q6‐8 hours or Prednisolone 30‐40 mg orally per 
day). Steroid should be quickly discontinued after the acute episode and long term use 
determined by steroid trial when patient is stable  
 Prescribe an antibiotic (augmentin 1mg po) if two or more of the followings: 
i) Increased breathlessnesss; 
ii) Increased sputum volume; 
iii) Development of purulent sputum 
 If pH is < 7.26 and PaCO2 is rising, consider ventilatory support.  
 
 
   

Page 226 By Heyson Chan


2 What is the chronic management of COPD? 
 The management of COPD is according to the lung function (GOLD guideline) 
 Mild, variable symptoms (FEV1>80%) 
Short acting 2 agonist or inhaled anticholinergic as required 
 
 Mild to moderate continuing symptoms (EFV1>50% but <80%) 
Regular therapy with bronchodilators, rehabilitation.  
Steroid if significant symptoms and lung function response 
 Severe disease (FEV1<30% or <50% with respiratory failure or cor pulmonale) 
Regular therapy with bronchodilators, rehabilitation.  
Steroid if significant symptoms and lung function response 
 Long term O2 to keep SaO2 >90% if respiratory failure 
Consider surgical treatment 
 For all stages 
Smoking cessation 
Educational programfor patient and family includingsmoking cessation and inhaler technique 
Influenza vaccine 
Assessment of reversibility and give steroid if reversible component +ve 
 
3 What are the common organisms causing acute exacerbation of COPD? 
 H influenza 
 Streptococcus penumoniae 
 Moraxella catarrhalis  
 
4 What are the indications of NIPPV in acute exacerbation of COPD? 
 Moderate to severe SOB with use of accessory muscles 
 Moderate acidosis and hypercapnia 
 Respiratory rate >25 per minute 
 
5 What are the contraindications of NIPPV in acute exacerbation of COPD? 
 Unconscious/uncooperative patient
 Upper airway obstruction
 Facial trauma / deformity / surgery
 Inability to clear respiratory secretions
 Haemodynamic instability 
 High risk of aspiration 
 
6 What are the complications of NIPPV? 
 Pulmonary barotraumas 
 Pneumothorax 
 Reduced cardiac output from positive pressure 
   

Page 227 By Heyson Chan


7 What are the indications of long term O2 supplement in COPD patient? 
 PaO2< 7.3 kPa or SaO2< 88% on room air at rest, or 
 In the presence of cor pulmonale when PaO2 7.3‐7.9 kPa or SaO2 > 89%, andcor pulmonale 
in ECG, haematocrit > 55%, congestive heart failure 
 In some specific situations, when PaO2> 8.0 kPa or SaO2> 90% in the presence of lung 
disease and other clinical needs, such as sleep apnoea with nocturnal desaturation not 
correctable by CPAP 
 
8 When would you consider discharging the patient? 
Consider discharge if patient, 
 Able to walk, eat, sleep
 Inhaled beta‐2‐agonist not > every 4 hours 
 ABG, clinically stable for 12‐24 hours 
 Understands use of medications 
 
9 How would you follow‐up the patient after discharge? 
 Follow‐up the patient 4‐6 weeks after discharge
 Assess: 
–Ability of cope with usual environment 
–FEV1 
–Inhaler technique 
–Understanding of treatment 
–Need for long‐term oxygen therapy 
   

Page 228 By Heyson Chan


CT Brain 
1 What are the typical CT brain features of epidural heaorrhage?  
 Extra‐axial  
 High attenuation, fusiform or biconvex collection (tough dural attachments tend to keep it 
more localized) 
 Limited anteriorly by coronal suture, posteriorly by lambodoid suture 
 Note any obstructive hydrocephalus and mass effect 
 Note any surrounding edema 
 
2 What are the causes of epidural hemorrhage? 
 Fractured temporal or parietal bone causing laceration of the middle meningeal artery and 
vein (look for skull fracture) 
 Suspect epidural hemorrhage if conscious level falls after a head injury (lucid interval); note: 
patient may develop signs of increased ICP rapidly  
 
3 What is the treatment for epidural hemorrhage? 
 Immediate neurosurgical consultation for clot evacuation +/‐ ligation of bleeding vessels 
 Maintain patent airway, especially in unconscious patients 
 
4 What are the typical CT brain features of subdural hemorrhage? 
 Cresentric shaped blood (may be new or old blood) 
 Note any blood in falx cerebri 
 Note any blood collected in the tentorium 
 Note any mass effect and hydrocephalus 
 
5 What are the causes of subdural hemorrhage? 
 Trauma with injury to the bridging veins (may be long time ago, common in elderly, 
alcoholics) 
 
6 What is the treatment for subdural hemorrhage? 
 Surgical evacuation of clot through blur hole 
 
7 What are the typical CT brain features of subarahoid hemorrhage? 
 Blood in sylvian fissure 
 Blood in ventricles 
 Notes any mass effect and hydrocephalus 
 Note any aneurysm identified 
 
 
8 What are the causes of subarachoid hemorrhage? 
 Ruptured aneurysm  
 Trauma 
   

Page 229 By Heyson Chan


9 What are the typical CT brain features of an acute infarct? 
Features of infarct may not be apparent on CT brain until 6 hours post‐infarct 
 Loss of grey‐white differentiation, loss of ribbon sign (early MCA infarct) 
 Wedge shape hypointense lesion 
 Well demarcated border within a vascular territory 
 Comment on any mass effect, obstructive hydrocephalus 
 Look for dense MCA sign (MCA thrombosis) 
 
10 What is your immediate management plan for a patient presented with acute stroke? 
Investigations
 CBC with D/C, L/RFT, TFT, random glucose, lipid profile, clotting profile 
 CXR 
 ECG 
 Urgent non‐contrast CT brain 
 Transcranial Doppler 
 Carotid duplex ultrasound 
 MR / CT angiography, if indicated 
Management 
 Admit to stroke unit 
 BP/P/T Q4H, neuro observation 
 Monitor and control hyperglycemia, but avoid correcting BP unless systolic BP >220 or 
diastolic BP > 120 (use iv labetolol in this case) 
 Nil by mouth until patient has passed the swallowing test, then allow puree diet 
 Early physio / occupational and speech therapy 
For ischemic stroke
 Start aspirin if not contraindicated 
 May consider thromoblysis if presented within 3 hours 
 Anti‐coagulate with warfarin in special cases 
 Consults neurosurgery for craniotomy if large infarct with midline shift 
For intracranial haemorrhage
 Urgent consult neurosurgery if posterior fossa or cortical haemorrhage with progressive 
deterioration 
 Avoid use of anti‐coagulants in the future 
 
11 What is the CT brain feature of a brain abscess? 
 Rim enhancing lesion on contrast CT brain 
 
12 What are the common causes of brain abscess? 
 Penetrating wound of the skill with secondary stapholococcal infection / strept pneumo 
 Direct spread from middle ear or ethmoid / frontal sinus 
 Blood born spread e.g. lung abscess, bronchiectasis 
 
13 What is the management for patients with brain abscess? 
 Aspiration of abscess through blur hole 
 Systemic antibiotics 
 Anticonvulsants prophylaxis since one third may develop seconday seizure 

Page 230 By Heyson Chan


Cushing’s Syndrome 
Physical Examination
Head and neck
 Moon‐face appearance 
 Acne formation 
 Buffalo hump 
 Supraclavicular fat pads 
Trunk
 Hirsutism  
 Central obsesity  
 Striae 
Limbs
 Proximal myopathy 
 Atrophic skin 
 Easy bruising 
 
I would also like to examine the patient for 
 Visual field for bitemporal hemianopia suggesting pituitary macroadenoma 
 Virilization suggesting adreanal carcinoma 
 Blood pressure for hypertension 
 Urine for sugar (associate with DM since cortisol is glucogenic) 
 Ask the patient for decrease libido / impotence / oligomenorrhea (gonadal dysfunction) and 
easy infection (immunocompromise) 
 Possible secondary causes: abdominal exam, chest exam (CA lung, use of steroid due to fibrosis 
/ asthma), joint (chronic use of steroid in arthritis)  
 
Questions 
1 What are the causes of Cushing’s syndrome? 
Exogenous
 Corticosteroid 
 ACTH 
Endogenous
ACTH dependent 
 Pituitary ACTH 
 Ectopic ACTH e.g. bronchial carcinoid, pancreatic islet cell tumor, medullary thyroid 
carcinoma 
ACTH independent 
 Adrenal neoplasia 
 Macronodular hyperplasia 
 Micronodular dysplasia 
 Sporadic 
 Familial (Carney’s syndrome)

Page 231 By Heyson Chan


2 What investigations would you like to perform? 
 
Screening 
 24 hr urine cortisol  
 9am and midnight plasma cortisol for loss of circadian rhythm (since cortisol usually high in 
morning – circadian rhythm) 
 Overnight dexamethasone suppression test (exclude iatrogenic first) 
False positive in acute illness, depression, alcoholism, enzyme inducing drugs 
 1mg dexamethasone at 11pm, morning blood at 8‐9am 
 >50nmol/L +ve 
 
Confirmation 
 Low dose dexamethasone (a synthetic glucocorticoid which is more potent than cortisol) 
suppression test 
 0.5mg Q6H x 2days  
 >50nmol/L = +ve 
 
Establish cause 
 High dose dexamethasone 
 2mg Q6H x 2days 
 > 50nmol/L  +ve 
 Adrenal hyperplasia secondary to pituitary ACTH 
 Adrenal hyperplasia secondary to ectopic ACTH or adrenal neoplasia (cortisol level not 
suppressed since ACTH level still too high to be overridden and too much cortisol to be 
suppressed) 
 Cortisol level suppressed in Cushing’s disease (pituitary cause, high dose 
dexamethasone overrides excess ACTH production by pituitary, causing negative 
feedback) 
 Plasma ACTH (in chilled bottles) 
 Low  adrenal tumour (benign or malignant) 
 High  ectopic ACTH 
Radiological localization 
 CT abdomen for adrenal adenoma and small carcinoid tumor 
 MRI pituitary for pituitary adenoma 
 CXR for CA lung (small cell carcinoma of bronchus  ACTH secreting) 
 Bilateral inferior petorsal venous sampling, coupling with administration of corticotropin 
releasing hormone 
   

Page 232 By Heyson Chan


3 What are the treatment options for Cushing’s syndrome? 
 Depends on the etiology 
Iatrogenic
 Stop the offending drugs (e.g. steroid) 
Adrenal adenoma / carcinoma 
 Surgical resection of tumor (open posterior approach / laparoscopic) / adreanlectomy 
 Post‐op hydrocortisone replacement for 2 years with extra doses during illness or 
psychological stressors 
Pituitary adenoma 
 Transphenoidal adenomectomy 
 Total 10 year remission: 60% 
 Hypopituitarism up to 50% 
 Pituitary irradiation 
 Indication: persistent disease after surgery 
 Side effects: visual loss, panhypopituitarism 
Ectopic ACTH producing carcinoma
 Excision if appropiate 
Medical
 Metyrapone (blocks cortisol synthesis) 
 Ketoconazole  
 

Page 233 By Heyson Chan


Diabetes Insipidus 
Presentation
 Excessive thirst 
 Excessive amount of urine 
 Hyponatremia  
 
Investigation Results
 Random blood glucose to rule out DM first (may give rise to polyuria and polydipsia!) 
 Paired plasma and urine osmolality test shows abnormally dilute urine to confirm the diagnosis 
of DI 
 Water deprivation test to confirm the diagnosis 
 Desmopressin injection to differentiate cranial DI and nephrogenic DI (if responsive, i.e. increase 
in urine osmolality, cranial DI as the kidney is still responsive to effects of ADH) 
 
Questions
1 What is the differential diagnosis of polyuria? 
Disorder of water intake or output
 Primary polydipsia 
 Increased water output, e.g. cranial / nephrogenic DI, drug induced inhibition of AVP release 
Disorder of renal solute excretion 
 DM 
 Diurectics 
 Chronic renal disease 
 
2 What are the causes of DI? 
Cause of DI 
Cranial  Nephrogenic 
 Idiopathic   X‐link 
 Neurosurgery, trauma   Low [K] 
 Hypoxic encephalopathy   High [Ca] 
 Anorexia nervosa   Lithium 
 Malignancy   Sjögren’s, amyloid, sickle cell 
 Histiocytosis X 
 
3 What is the treatment for DI? 
 Desmopressin (twice daily) – oral / intranasal / parenteral 
 Identification card indicating the diagnosis and need for treatment 
   

Page 234 By Heyson Chan


DM Retinopathy 
Fundi Examination
 Red reflex: present (or absent if patient has cataract vitreous hemorrhage) 
 Optic disc: normal margin, color and cup‐disc ration 
 Vessels: no AV nipping, neovascularization around the disc (in proliferative phase) 
 Back ground: dots and blots hemorrhages, hard exudates, cotton wool spots, note any laser 
burns 
 Macular: note any hard exudates or edema involving the macular 
 
Conclusion 
This patient has dots and blots hemorrhages, hard exudates and neovascularization. He is suffering 
from (proliferative) DM retinopathy with (no previous photocoagulation). He is currently 
asymptomatic. 
 
Questions 
1 How to grade DM retinopathy? 
 Non‐proliferative: at least one microaneurysms (which represents arterial dilatation) 
 Maculopathy: retinal thickening or hard exudates at or within one‐third disc diameter from 
the center of the macula 
 Pre‐proliferative: venous irregularities, multiple hemorrhages, multiple cotton wool spots, 
intrarentinal microvascular abnormalities 
 Proliferative: neovascularization on the optic disc or vitreous or pre‐retinal hemorrhage 
 
2 How to differentiate between DM retinopathy and hypertensive retinopathy? 
 
  DM retinopathy  HT retinopathy 
AV nipping  Absent  Present  
Exudates   Hard   Hard / soft 
Hemorrhage   Dots and blots  Flame shape 
Microaneurysm  Present  Absent  
Neovascularization   Present in proliferative stage Absent  
 
 
 
3 What are the complications of proliferative retinopathy? 
 Vitreous hemorrhage 
 Traction retinal detachment 
 Rubeotic glaucoma 
 
4 What is the use of photocoagulation? 
 Leads to regression of neovascularization by ablating protion of hypoxic retina to reduce 
angiogenic factors and reduce oxygen demand of retina 
 Also useful in treatment of microaneurysms, hemorrhages and edema 
   

Page 235 By Heyson Chan


5 Explain the pathophysiology of the signs seen in diabetic retinopathy. 
 DR is a microvascular process in which high retinal blood flowinduces microangiopathy in 
capillaries, arterioles and venules,causing vessel occlusion and leakage of plasma contents 
into theretina itself. 
 
 Microaneurysms are bulges in weakened vessel walls and appearas red dots scattered 
throughout the fundus. Microaneurysmsarise as a direct result of capillary occlusion. 
 
 Leakage exudates are yellow‐white lipid deposits. They representlipid which has leaked 
from vessels and been engulfed bymacrophages. 
 
 Haemorrhages from microaneurysms and weakened vessels appearred but their shape 
depends upon their site. Deep haemorrhagesare round (blots) because they are confined by 
the tightly packedlayers of the deep retina. Superficial haemorrhages are larger andblotchy 
as they follow the pattern of nerve fibres. 
 
 Cotton wool spots indicate ischaemic areas of retina. They arisebecause of arteriolar 
occlusion. To refer to CWS as soft exudatesis incorrect. 
 
 Venous dilatation is a sign that retinal blood supply is trying tokeep up with demand and 
venous beading results from sites ofcomplete vessel closure. 
 
 The term intraretinal microvascular abnormalities (IRMA) refersto dilated capillaries. 
Microaneurysms and IRMAs are best seenas black dots and lines against a green 
background when theophthalmoscope’s green filter is used to eliminate the redness ofthe 
choroid. 
 
 Neovascularisation occurs in response to occlusive ischaemia(which stimulates angiogenic 
factors) and affects veins, notarteries. New vessels are usually ‘wild’in appearance. They 
tendto loop off veins and can look like fronds of seaweed. 
 
 Subhyaloid haemorrhages are retained by the hyaloid membranebut may blow into the 
vitreous, threatening vision completely inthe affected eye because they are then restrained 
only by thelimits of the vitreous cavity. 
 
 Retinal detachment can occur when new vessels exert traction onthe retina. Rubeosis iridis 
(new vessels on the iris) may lead torubeotic glaucoma. 
6 Is retinopathy more likely in Type 1 or Type 2 diabetes mellitus? 
 Diabetic eye disease is the commonest cause of blindness in the UKfor patients of working 
age, and is related to the duration of andlevel of control of disease.  
 Type 1 diabetic patients generally have5–10 years of disease before developing eyes signs 
but eventuallynearly all succumb.  
 Type 2 diabetic patients may have eye signs atdiagnosis and are more prone  to 
maculopathy. 
   

Page 236 By Heyson Chan


7 What do you know about screening for diabetic retinopathy (DR)? 
 Screening has been shown to prevent loss of sight in 80% of patientswith PDR and 50–60% 
of patients with maculopathy. Routinescreening of all diabetic patients is mandatory at 
least annually. 
 Screening is performed by digital retinal photography. 
 
8 What are the management principles in DR? 
 NPDR with microaneuryms alone requires annual screening withgood overall diabetic 
control. All other grades of NPDR warrantophthalmological referral (non‐urgently except for 
very severe). Inaddition to strict diabetic control, a decision must be made 
aboutprophylactic laser photocoagulation therapy. 
 PDR requires urgent ophthalmological referral for laserphotocoagulation therapy, which 
aims to destroy ineloquent areasof retina (not concerned with vision) and thus attenuate 
synthesisof angiogenic factors. 
 Vitreous haemorrhage is difficult to treat, but requires urgentreferral. Advanced 
microsurgical techniques can sometimesevacuate vitreous blood. 
 Maculopathy requires ophthalmological referral. Focal lasertherapy may be indicated but 
must be balanced against potentialrisk to eloquent tissue. 
 
9 What other eye complications occur in diabetes mellitus? 
 Cataracts (early onset age related variety or snowflake cataracts inpoorly controlled Type 1 
diabetes), or visual changes due toosmotic alteration in lens shape 
 Ocular nerve palsies, especially a sixth nerve palsy 
 Increased risk of infection (conjunctivitis, styes, herpes zoster) 
 Central retinal artery and vein occlusion are also more common. 
 
 
 

Page 237 By Heyson Chan


Notes on ECG 
Routine Introduction
1 Name and HKID of the patient  
2 Date when ECG taken 
3 Review of medical history, old ECGs and the indication 
 
The ECG
1 Rate and rhythm  
a) Rate 
 Normal: between 60 and 100 bpm 
 Count the number of beats on the tracing and multiply this number by 6 to yield the 
heart rate per minute 
b) Rhythm  
i) Primary rhythm 
(1) Supraventricular (sinus, atrial, junctional)  
 Sinus rhythm: a regularly recurring P wave of the same morphology before each 
QRS complex, and the axis of these P waves should be within normal limits (i.e. 
between 15o and 75o) 
 Atrial: P wave that looks different from the P wave produced by the sinus beat; 
normal QRS 
 Junctional: P wave is usually not visible; normal QRS 
(2) Ventricular  
 No P wave and QRS > 120ms 
 
ii) Secondary rhythm 
(1) Heart block 
 If PR interval > 200ms 
(2) Escape beat 
 Always late, coming after a missed sinus beat 
(3) Pre‐mature beat 
 Always early, before the sinus beat 
 
2 P wave 
a) Axis  
 Normal: Upright in Lead I and aVF (i.e. between 15o and 75o) 
b) Morphology 
 Normal: less than 120 ms wide and less than 2.5 mm high (left panel) 
 Right atrial enlargement: peaked P wave taller than 2.5 mm in Lead II, III, or aVF(P 
pulmonale) or biphasic P wave in V1 
 Left atrial enlargement: Double peak m‐shape P wave in Lead II, III, or aVF (left panel) 
that is more than 120 ms wide (P mitrale) 
 
 
 
   

Page 238 By Heyson Chan


3 QRS complex 
a) Axis 
 Normal: Upright in Lead I and aVFor negative in aVF but upright in Lead I and Lead II 
(between ‐30o and 90o) 
 Left axis deviation: positive in Lead I but negative in both Lead aVF and Lead II (between 
‐30o and ‐90o) 
 Right axis deviation: negative in Lead I but positive in aVF (+90o to 180o) 
b) Amplitude  
 Normal: In V1 & V2 the QRS complex is predominantly negative with a small R wave and 
a relatively deep S wave; In V5 & V6 the QRS complex is predominantly positive with a 
tall R wave 
 RVH: In V1 the QRS complex is positive with a tall R wave; S wave is unusually deep in V6 
 LVH: The sum of the S wave in V1 and the R wave in V5 or V6 > 35 mm 
c) Morphology  
 Normal: < 120ms wide 
 RBBB: QRS > 120 ms, RR’ in V1 & V2 and wide S wave in V5 & V6 
 LBBB: QRS > 120 ms, wide monophasic R wave or with M pattern in V5 & V6, deep and 
wide S wave in V1 & V2 
d) Pathological Q waves 
 Present in transmural myocardial infarction 
 Q waves that are 2 mm deep and / or deeper 40 ms wide or wider 
4 ST segment 
 Normal: isoelectrical 
 ST segment depression (>1mm): subendocardial ischemia (reversible) 
 ST segment elevation with or without T wave inversion (>1mm): myocardial injury 
(irreversible) 
o V1‐V4: anterior (left anterior descending coronary artery) 
o Lead I, Lead aVL, V5 and V6: lateral (circumflex artery) 
o Lead II, Lead III, and Lead aVF: inferior (right coronary artery) 
5 T wave 
a) Peaked T wave– hyperkalemia (may shows widened QRS) 
b) U wave after T wave ‐ hypokalemia 
   

Page 239 By Heyson Chan


Fibrosing alveolitis 
Physical Examination 
 Patient may be in severe shortness of breath and on high flow oxygen 
 Note any sputum mug / O2 supplement, SaO2 monitor 
 Peripheral: clubbing, note any central cyanosis 
 Inspection: symmetrical chest expansion  
 Lymph node: nil 
 Palpation: trachea, apex beat not deviated, diminished chest expansion 
 Percussion: normal 
 Auscultation: bronchial breath sound, diminished air entry, bilateral basal end‐inspiratory fine 
crackles not cleared up on coughing, vocal resonance normal (c.f. bronciectasis: no wheeze) 
 
Conclusion
This patient has fibrosing alveolitis, the most likely cause in this locality is past pulmonary 
tuberculosis. He is clinically in (shortness of breath and is on oxygen supplement of 2L/min). 
I would like to examine the hand for RA and systemic sclerosis, the face for malar rash and the 
mouth for aphthous ulcer. I would also like to auscultate the heart for loud P2 seen in pulmonary 
hypertension. 
Questions
1 What are the causes of fibrosing alveolitis? 
Primary 
 Idiopathic pulmonary fibrosis 
Secondary
Connective tissue disorder
 SLE, RA, scleroderma, ankylosing spondylitis 
Radiation
Drugs
 Amiodarone (deposition of intracellular phospholipid; related to maintainence dose, 
>400mg/day) 
 Methrotrexate, busulphan, bleomycin (direct cytotoxic effects on the alveolar membrane; 
not dose dependent; Rx: stop using the drug + early steroid) 
Pneumoconiosis
 Silicosis – upper zone; related to mining, quarrying, stoneworks; risk of TB 
 Asbestosis – lower zone; pleural plaques, related to  
Granulomatous inflammation 
 Sacrcoidosis (multi‐system inflammatory disorder associate with noncaseating granuloma; 
CXR – bilateral hilar lymphadenopathy) 
 Wegener’s syndrome (systemic vasculitis of small‐sized vessels characterized by necrotizing 
granulomatousinflammation, especially in kidneys, upper and lower respiratory tract) 
Infection  
 TB, fungal, viral (CMV), protozoa (PCP) 
Extrinsic allergic alveolitis 
 Hypersensitive pneumonitis (IgG to antigens related due to repeated exposure to various 
organic dust can be identified by serological studies) 
Lymphangitis carcinomatosis 
   

Page 240 By Heyson Chan


2 How does the patient usually present with? 
 Insidious onset of pregressive dypsnia and dry cough for >3months 
 Associate with hypoxemia, cyanosis, cor pulmonale, premature death 
 
3 What are the diagnostic criteria for idiopathic pulmonary fibrosis? 
Major (all are required) 
 Exclusion of known causes of pulmonary fibrosis 
 Restrictive pattern on lung function test with hypozemia at rest or a reduced diffusion 
capacity 
 Bibasilar reticular opacity with minimal ground glass changes on HRCT 
 Transbronchial lung biopsy and BAL showing no evidence of alternative diagnosis 
Minor (need 3 out of 4) 
 Age > 50 years old 
 Insidious onset of SOB on exertion 
 During of illness >3months 
 Bibasilar inspiratory dry crackles 
 
4 How would you investigate the patient? 
 CBC with D/C, L/RFT 
 CXR for bilateral basal reticulonodular shadows and honeycombing in advance cases 
 High resolution CT thorax for pattern and extent of disease 
 ABG for hypoxemia and hypocapnia 
 Lung function test for restrictive pattern 
 Rheumatological Ix: ESR, RA, ANA 
 Lung biopsy through bronchoscopy (if failed  VATS guided) 
 
5 What are the good prognostic factors? 
 Short duration of disease 
 Young age of onset 
 Predominately ground‐glass shadowing on CXR   
 Little fibrosis on lung biopsy 
 
6 How would you manage the patient with idiopathicpulmonary fibrosis? 
Pharmacological
 Corticosteriod (use intermittent, high dose, pulse parental steroid) 
 Cyclophosphamide (2mg/kg/day) or azathioprine 
 Colchicine (anti‐fibrotic agent) 
 Pirfenidone (interferon‐ which is an anti‐fibrotic agent) 
 Lung transplantation is possible 
Follow‐up 
 CXR 
 Lung function test 
 Identify the cause and treat accordingly 
 
7 What are the causes of death in these patients? 
 Lung failure 
 Cor pulmonale 
 CA lung (10X risk) 

Page 241 By Heyson Chan


Gout 
Questions 
 
1 Apart from examining the affected joints, what else would you like to perform during 
physicalexamination? 
 Search for tophi which indicates chronic gout with inadequate treatment 
 Assess the hydration status 
 Palpate the abdomen for polycystic kidney 
 
2 What is your immediate management plan? 
Confirm the diagnosis 
 Joint aspiration for identification of crystals under polarized microscope 
Treatment 
 Ice pad on affected joints 
 Bed rest 
 Adequate hydration 
 NSAIDs – Preferred drugs of choice. 
 Use full therapeutic dose. 
 Indomethacin ‐ 50 mg 3‐4x/day for48 hours, then 50 mg 3x/day or less if attack 
hassubsided. 
 Do not use high dose for > 1 week. 
 Care in elderly, decrease dose. 
 Do not use aspirin or other salicylates. 
 Contraindicated in renal failure patient 
 Oral colchicine – Less favoured because of slow onset of 
 Action and invariably causing diarrhoea 
 1 mg stat, then 0.5 mg 2 hourly until relief of pain orside‐effects (nausea, vomiting, 
diarrhoea, abdominalpain). Maximum dose < 8‐10 mg in 24 hours. Do notrepeat within 3 
days. 
 Reduce dose in renal impairment 
 Corticosteroids – If NSAID or colchicine contraindicated 
 Intra‐articular steroid – Effective in monoarthritis. 
 Oral steroid – Prednisolone 30‐40 mg daily, tapered in7‐10 days. Rebound attacks may 
occur on withdrawal.   

Page 242 By Heyson Chan


3 What is the chronic management plan? 
 CBC, L/RFT, fasting glucose, lipid profile, uric acid level (after acute attack) 
 Avoid purine rich foods, alcohol 
 Reduce weight 
 Avoid thiazides, low dose aspirin, pyrazinamide 
 Maintain good urine output (>1.5‐2L per day) 
 Do not start treatment until several weeks after anacute attack has settled completely 
 If indicated, allopurinal (xanthine oxidase inhibitor) 300 mg daily if RFT normal (start at low 
dose 100mg, then titrate up gradually). (200 mg daily if GFR <60ml/min, 100 mg daily if 
GFR<30 ml/min) 
 If patient failed to achieve target uric acid level, check compliance; if high uric acid level 
despite good compliance, consider increasing dose of allopurinol or adding probenecid 
 
4 What are the indications for allopurinol? 
 After 2 attacks of acute gout 
 Tophaceous gout 
 Cytotoxic therapy 
 Gouty changes on XR 
 History of renal stone 
 Evidence of primary purine over‐production andexcretion 
 Young patients with high uric acid level and familyhistory of heart or renal disease
   

Page 243 By Heyson Chan


Heart Failure 
Physical Examination
 Peripheral signs: any oxygen supplement, angle of bed 
 Carotid pulse: normal 
 JVP: elevated  
 Apex beat: pay attention to any LVH 
 Auscultation: pay attention to any murmur and intenstity of heart sounds, S3/S4 
 Lung base: bilateral basal fine crepitation clears up on coughing 
 Bilateral pitting ankle edema / sacral edema 
 Note any tender hepatomegaly 
 Blood pressure: look for hypertention 
 
Conclusion 
This patient has congestive heart failure due to (hypertension) and is currently on (oxygen 
supplement and is in shortness of breath even at rest). He is in (NYHA class IV) failure. 
 
Questions 
 
1 What are the causes of heart failure? 
Systolic heart failure
 Ischaemic heart disease  
 Hypertension 
 Idiopathic dilated cardiomyopathies 
 Valvular heart disease, e.g. CRHD 
 Other cardiomyopthies, e.g. restrictive, infective, familial 
 Secondary to tachyarrhythmia, e.g. AF, VT   
 Infiltrate conditions, e.g. amyloid, iron overload, sarcoid 
 Senile and endomyocardial fibrosis 
Diastolic heart failure
 Hypertension ‐ commonest
 Left ventricular hypertrophy 
 Ischaemic heart disease ‐ myocardial ischaemia,  
 Aging 
 
2 What are the precipitating causes of acute exacerbation of heart failure? 
 Arrhythmia (e.g. AF, atrial flutter) – common  
 IHD (especially silent MI in elderly) – common  
 Uncontrolled HT– common  
 Anemia – common  
 Chest infection– common  
 Thyrotoxicosis 
 Poor compliance to medication 
   

Page 244 By Heyson Chan


3 How would you investigate the patient? 
Aim: a) confirm HF, differentiate systolic and diastolic HF b) assess severity c) look for etiology 
d) prognosis and guide treatment plan 
 CBC for anemia 
 TFT to rule out thyroid disease 
 RFT to prepare the patient for ACEI therapy 
 CXR to confirm the diagnosis (cardiomegaly, congestion, Kerley B lines, upper lob diversion, 
pleural effusion, consolidation of lower lung field, bat‐wing appearance due to interstitial 
edema) 
 ECG for myocardial ischemia 
 Echocardiogram to confirm the diagnosis, assess severity and look for valvular lesions 
 Exercise tests to identify IHD 
 Coronary angiogram to identify IHD 
 
4 What is the immediate management plan for a patient with acute heart failure? 
 Complete bed rest, prop up 
 Monitor BP/P, I/O, SaO2, CVP, RR, clinical status every 30‐60 mins 
 Chart I/O, body weight daily 
 Oxygen (may require high flowrate / concentration) 
 Low salt diet + fluid restriction (1L/day) 
 Investigations 
 If unstable BP, giveInotropic agents (dopamine 2.5‐10 g/kg/min or dobutamine 2.5‐15 
g/kg/min) 
 Frusemide(Lasix) 40‐120 mg ivfor symptomatic relief  
 IV nitrate e.g. isoket 2‐10 mg/hr 
 Morphine 2‐5 mg slow iv 
 If the patient has arrhythmia, use amiodarone or consider implantable defibrillator 
 Consider ventilatory support in case of desaturation, patient exhaustion, cardiogenic shock 
 
 
 
   

Page 245 By Heyson Chan


5 What is the long‐term management for heart failure? 
General
 Gentle exercise according to capacity 
 Weight reduction 
 Low salt diet 
Pharmacological 
 Diurectics – for symptomatic relieve 
Frusemide (judicious use in diastolic failure) 
 ACEI – improves signs and symptoms, improves ET, reduce mortality 
Captopril 6.25mg, enalapril 2.5mg 
S/E: symptomatic hypotension, deterioration of renal function (check renal function before 
giving ACEI), hyperkalemia, cough (common in Chinese) 
Check RFT 2‐3 weeks after starting ACEI 
 Aldosterone antagonist – reduce mortality and reduce salt and water retention 
Spironolactone, safe in combination with ACEI but potential hyperkalemia 
 Beta‐blocker – improves survival, imporves LC ejection fraction 
Carvedilol, metoprolol, bispolol – start at very low dose, improves slowly 
 Vasdilator – nitrates for APO 
 Digoxin – reduce sympathetic activity, mild positive inotropic agent 
 Amiodarone – only antiarrythatics suitable for heart failure patients 
 Inotropic support for cardiogenic shock 
Non‐pharmacological 
 Surgical 
 Implantable defibrillator 
 Biventricular pacing 
 Cardiac transplantation 
 
6 How to differentiate between right heart failure and left heart failure? 
Left heart failure
 SOB on exertion, orthopnea, PND 
 Left ventricular hypertrophy 
 Loud P2, pathaological S3 gallop, S4 
 Bilateral basal fine crepitations 
Right heart failure 
 Peripheral edema, ascites, nocturia 
 Raised JVP 
 Right ventricular heave 
 Hepatomegaly 
 Hepatojugular reflux 
 
7 What is the New York Heart Association classification? 
 Class I: no limitation on physical activities, no SOB even on exertion 
 Class II: slight limitation on physical activities, SOB on heavy exertion 
 Class III: marked limitation on physical activities, SOB on light exertion 
 Class IV: SOB at rest, unable to carry out physical activities 
 
   

Page 246 By Heyson Chan


Hepatomegaly 
Physical Examination
 General inspection: abdominal distention
 Signs of chronic liver disease: clubbing, palma erythema, spider navei, loss of axillary hair,
gyanecomastia, caput medusa
 Signs of hepatic failure: hepatic flap, drowsy patient
 Lymph nodes: pay special attention to the Virchow’s node
 General palpation: no T/G/R
 Liver: feel the texture and surface, measure liver span at MCL
 Spleen: may be enlarged if complicated with portal hypertension
 Kidney: normal
 Ascites: shifting dullness may be present
 Auscultation: listen for liver bruit / venous hum; normal bowel sound

Conclusion
This patient has (stigmata of chronic liver disease) together with a heaptomegaly. The likely
diagnosis includes a liver tumor, liver cirrhosis and chronic hepatitis. He is complicated with (ascites
but he doesn’t look drowsy at the moment).

Questions
1 What are the differential diagnoses for a sole hepatomegaly?
Tumors
Irregular boarder, firm to hard, nodular, may associate with bruit
High in the list of DDx if there is lymph node involvement
 Primary: hepatocellular carcinoma
 Secondary
 Lymphoma, leukemia
Infections
May be tender in acute hepatitis (stretch on its capsule due to recent enlargement)
 Viral: Hepatitis B / C infections
 Bacterial: liver abscess
Alcoholic liver diseases
Non-tender, firm
 Fatty liver
 Cirrhosis (Early stage, firm, non-tender, may associate with a bruit)

Metabolic disease
 Haemachromatosis – iron metabolism problem
 Wilson’s disease – copper metabolism problem, K-F ring
Congestive heart diseases
Smooth, firm, tender liver
 Right heart failure
 Tricuspid regurgitation (pulsatile liver – ask the patient to hold the breath)
“Pseudo-hepatomegaly”
 Hyper-expanded chest

Page 247 By Heyson Chan


2 What are the differential diagnoses for a heaptomegaly associated with splenomegaly?
 Lymphoproliferative disorder, e.g. lymphoma, leukemia (important)
 Portal hypertension secondary to cirrhosis (important)
 Infectious hepatitis
 Systemic disease: amylordosis, sarcoidoisis

3 What investigations would you like to perform if you detect hepatomegaly?


Blood investigation
 CBC with D/C
 L/RFT
 Hepatitis serology
 AFP, CEA
Imaging
 USG abdomen

4 What does a hepatic bruit indicates?


 It is heard in alcoholic hepatitis, primary or secondary carcinoma

5 What does an abdominal venous hum indicate?


 It is diagnostic of portal venous hypetension

6 What are the salient points in the history you would like to know from the patient?
7 What are the complications of chronic liver disease?
8 What is the normal liver span?
 Normal liver span = 13cm at the MCL

9 What is Child’s grading?


 It is the grading of severity of liver disease
 It consists of two clinical components (encephalopathy and ascites) and three laboratory
components (bilirubin, albumin and PT)
 Child A: score <7 Child B: 7-9 Child C: >10

Score 1 2 3
Encephalopathy None I and II III and IV
Ascites Absent Slight Moderate
Bilirubin (mol/l) <35 35-50 >50
for PBC (mol/l) <70 70-170 >170
Albumin (g/l) >35 28-35 <28
Prothrombin time (sec prolonged) 1-4 4-6 >6

10 How would you grade hepatic encephalopathy?


Hepatic encephalopathy
 I: Euphoria, mild confusion, mental slowness, slurred speech
 II: Grade I + inappropriate behaviour, drowsiness, flapping tremor
 III: Sleeps most of the time but arousable, incoherent speech, confused
 IV: Not arousable

Page 248 By Heyson Chan


11 What is the treatment plan for a patient with hepatic encephalopathy?
Identify and correct precipitating factors
 Watch out for gastrointestinal bleeding
 Avoid sedatives, diuretics and hepatotoxic drugs
 Treat infection

Treatment
 Monitor blood glucose, haemoglucostix q2-6h
 Check PT, blood ammonia level (good for monitoring progress)
 Low protein diet 20 - 40 g/day
 High calorie intake
 Prophylactic antibiotics after sepsis workup – 2nd/3rd generation cephalosporin
 Fleet enema and lactulose 10-20 ml tds orally, aim at bowel motion 2-3/day
 Neomycin (1g q4-6h) per oral

   

Page 249 By Heyson Chan


Hypercalcemia 
 
Presentation
 Polyuria 
 Polydipsia 
 Abdominal pain 
 Convulsion 
 Impaired sensorium 
For hyperparathyroidism: 
 Bones: bone pain, osteolastomas 
 Moans: Impaired sensorium, convulsion 
 Stones: renal stones 
 Abdominal groans: peptic ulcer, constipation, pancreastitis 
 
Questions
1 What are the causes for hypercalcemia? 

Increased Bone Absorption


 Hyperparathyroidism 
•  Ambulatory patients: ~90% 
•  Hospital patients: ~25% 
 Cancer 
•  Hospital patients: ~65% 
 Granulomatous reaction  
 Hyperthyroidism 
 Immobilisation 
 Paget’s disease 
 Estrogen / Tamoxifen 
 Hypervitaminosis A 

Increased Calcium Absorption


 Increased calcium intake 
 Hypervitaminosis D 
Other Causes 
 Lithium 
 Pheochromocytoma (rare, PTH‐related protein) 
 Adrenal insufficiency 
 Acute renal failure / rhabdomyolysis 
 Theophylline toxicity 
 Familial hypocalciuric hypercalcemia  

Rare
 Autosomal dominant disorder 
 Loss‐of‐function mutation in the calcium‐sensing sensor on the parathyroid cells and in the 
kidneys  
 
   

Page 250 By Heyson Chan


2 What is your immediate management plan for a patient presented with severe hypercalcemia? 
Rehydration  frusemide  calcitonin bisphosphonate 
 Off calcium / vitamin D supplement if any. 
 Volume repletion with NS at 150‐600 mL/hr infusion(guided by CVP / urine output);  
 Furosemide afterrehydration 20‐40 mg IV Q 2‐12 H; aim at a urine output of ~200 mL/Hr; 
close monitoring of Na K Ca Mg level. 
 Pamidronate / Zometa (a bisphosphonate) 30‐90 mg in 250‐500 mL NS infused over 4‐6hrs; 
maximum effect is not seen for several days; repeatanother dose after a minimum of 7 days 
if necessary. 
 Salmon calcitonin 4 IU/kg IMI / SC Q 12 H; Ca level beginsto fall within 2‐3 hrs; tachyphylaxis 
occurred within 2‐3 days. 
 Mitramycin: 25 μg/kg IV in 50 mL D5 over 3‐6 hrs infusion;Ca begins to decrease in 12 hrs; 
peak action at 48 hrs; repeatdose at 3‐7 days interval if necessary (usually reserve 
formalignancy‐related hypercalcaemia ). 
 Hydrocortisone 5 mg/kg IV Q 8 H then prednisolone 40‐100mg QD (onset: 3‐5 days; useful 
in haematologicalmalignancy, vitamin D intoxication, some CA breast) 
 Haemodialysis with zero or low Ca dialysate. 
 
3 How would you subsequently investigate the patient? 
 Corrected calcium level 
 Ionised calcium level 
 Paired sampling of serum calcium and parathyroid hormone 
 (Serum phosphate) 
 (Urinary calcium excretion) 
–High: hyperparathyroidism, malignancy 
– Low: milk‐alkaline syndrome, thiazide, Familial hypocalciuric hypercalcemia  
 For hyperparathyroidism – localization 
–Ultrasound 
–99m‐technetium sestamibi radio‐isotope scanning 

Page 251 By Heyson Chan


 
4 What is the subsequent medical management for patients with primary hyperparathyroidism? 
 Avoid thiazide diuretic and lithium carbonate therapy, volume depletion, prolonged bed 
rest or inactivity, and a high calcium diet.  
 Encourage physical activity to minimize bone resorption. 
 Encourage adequate hydration (at least six to eight glasses of water per day) to minimize 
the risk of nephrolithiasis. 
 Maintain a moderate calcium intake (1000 mg/day). A low calcium diet may lead to 
increased PTH secretion and aggravate bone disease. On the other hand, a high calcium diet 
may exacerbate hypercalcemia or hypercalciuria, particularly in patients with high serum 
calcitriol concentrations.  
 Maintain moderate vitamin D intake (400 to 600 IU daily). Vitamin D deficiency stimulates 
PTH secretion and bone resorption, and therefore is deleterious in patients with primary 
hyperparathyroidism. 
 Bisphoshponates 
–  Potent inhibitors of bone resorption 
– Long‐term control of osteopenia in patients with untreated primary hyperparathyroidism 
 Raloxifene–a selective estrogen receptor modulator 
 
5 In what situation is steroid useful in hypercalcemia? 
 Haematologicalmalignancy 
 Vitamin D intoxication 
 Some CA breast 
 
6 What are the indications for surgical treatment for primary hyperparathyroidism? 
Adenoma: remove affected gland and biopsy the other three 
Hyperplasia (also seen in tertiary hyperPTH): remove all four glands and replace 30mg of 
parathyroid tissue in the forearm 
 Patients with a serum calcium concentration of 0.25 mmol/L or more above the upper limit 
of normal 
 Patients with hypercalciuria (urinary calcium excretion greater than 10 mmol/day while 
eating their usual diet) 
 Patients with a creatinine clearance that is 30 percent or lower than that of age‐matched 
normal subjects 
 Patients with osteoporosis (bone density T score <2.5 in hip / spine) 
 Patients who are less than 50 years old 
 Patients in whom periodic follow‐up will be difficult  
 
 
 
 

Page 252 By Heyson Chan


Hyperkalemia 
Investigation Results
Check RFT and venous HCO3
 if Cr > 300  renal failure
(look for source of K intake, bleeding, rhabdomyolysis, acidosis)
 if Cr < 300  consider minerocorticoid deficiency
calculate trans-tubular potassium gradient (TTKG)
urine [ K ] serum osm
TTKG  
serum [ K ] urine osm
 expected TTKG > 10
 if TTKG < 7, suggest minerocorticoid defect
Repeat TTKG after fludrocortisone to distinguish aldosterone deficiency vs resistance

Questions
1 What are the causes of hyperkalemia?
 False / hemolysis / rhabdomyolysis / rapid transfusion of old blood
 Transcellular shift: acidosis, insulin deficiency
 Renal failure (rare if GFR > 20 ml/min)
 Reduced aldosterone activity (e.g. Addison’s)

2 How would you manage the patient?


Need treatment if K > 6.0 or ECG changes
10 mL 10% Ca resonium, 20U insulin, 50mL 50% glucose (10-10-20-50-50)

Acute treatment if K > 6.0 or ECG changes


a) 10% Ca gluconate 10-30 ml iv over 2-5 min with cardiac monitor
 if digoxin toxicity suspected, give over 30 min or omit
 onset 1-3 min; duration 30-60 min

b) 50 ml D50 solution + actrapid 10 units iv over 30 min


 can repeat 4 hours later
 reduce dose of actrapid if renal failure
 onset 30 min; duration 4-6 hours

c) 8.4% NaHCO3 50-100 ml iv over 30-60 min


 give after Ca infusion in separate iv line
 watch out for fluid overload
 onset 10 min; duration 2 hours

d) resonium C 15-45 gm PO q4-6h


 give in 100-200 ml 10% mannitol as laxative
 also as retention enema
 1 gm resonium binds 1 mmol K
 onset 1 hour; duration 6 hours

e) ventolin 10-20 mg in 3 ml NS by nebulizer


 onset 15 min; duration 3 hours
f) lasix

Page 253 By Heyson Chan


g) dialysis

Chronic treatment
 low K diet (< 2 g/day)
 diuretics: lasix or thiazide
 oral NaHCO3 300-900 mg tds (~10-30 mmol/day)
 fludrocortisone 0.1-0.2 mg daily

3 What are the typical ECG changes of hyperkalemia?


 Tenting of T wave
 Widen QRS
 Reduction and disappearance of P wave
 Bradycardia, sinusoidal wave, asystole

Page 254 By Heyson Chan


Hyperlipidemia 
Presentation
 Premature corneal arcus may be associated with hypercholesterolemia 
 Xanthelasmata may be associated with hypercholesterolemia 
 Tendon xanthoma are typical of familial hypercholesterolemia 
 Eruptive xanthomata (small lumps occur with severe hypertriglyceridaemia or 
chylomicronaemia) 
 
Questions
1 How to estimate the level of LDL? 
LDL = TC – HDL – TG/2.2 
 
2 What is the main lipid increased in the following situations? 
 DM – TG, decrease HDL 
 Obsesity – TG +/‐ cholesterol, decrease HDL 
 Nephritic syndrome – cholesterol +/‐ TG, decrease HDL 
 Chronic renal failure – TG +/‐ cholesterol, decrease HDL 
 Hypothyroidism – cholesterol +/‐ TG 
 Anti‐hypertensives – TG 
 Steroid – cholesterol 
 
3 What is the function of HMG‐CoA reductase inhibitor? 
 Inhibit the endogenous syntheis of cholesterol 
 Main site of action in the liver 
 Decreased cholesterol availability results in upregulation of the LDL receptors which take up 
LDL particles from the plasma so plasma LDL‐cholesterol decreaes 
 Most statins metabolized by CYP450 (except pravastatin) 
 Atorvastatin is the most potent statin (metabolized by CYP4503A4) 
 
4 What is the side effect of different lipid lowering agents? 
HMG CoA reductase inhibitors (e.g. simvasstain, pravastatin – taken with empty stomach,
lovastatin – taken with food)
For decreasing LDL (up to 50%), TG and increasing HDL, well tolerated 
Also have anti‐oxidant, anti‐inflmmatory and anti‐proliferative properties 
 GI side effects (nausea), insomnia, fatigue, headache 
 Liver enzyme abnormalities 
 Myopathy  
 
 
Bile acid sequestrants 
For hypercholesterolemia (but rarely used nowadays) 
 Intestinal bloating, obstruction 
 Nausea, flatulence, constipation  poorly tolerated 
 Hypercholermic acidosis 
 Malabsorption of anionic drugs 
 Malabroprtion of fat soluble vitamins 

Page 255 By Heyson Chan


Fibrates (gemfibrozil) 
Reduce TG up to 50%, increase HDL, s/e similar to statin (well tolerated), excreted by kidneys 
 GI side effects (nausea), insomnia, fatigue, headache 
 Liver enzyme abnormalities 
 Myopathy 
Nicotinic acid
Reduces VLDL and LDL, TG (up to 40%), increase HDL by 20% 
 Cutaneous flushing, nausea, diarrhea, pruritis, rashes  poorly tolerated 
 Acathosis 
 Exacerbates DM  not for DM patients! 
Note: fish oil containes polyunsaturated omega‐3 fatty acids and can be used to reduced TG 
levels with increased VLDL or chylomicrons 
 
5 What are the conservative measures in the management of hyperlipidemia? 
 Lifestyle advise on diet (usually adequate for patients with cholesterol level between 6.5 – 8) 
and exercise 
 Stop smoking and drinking 
 Control of other CVD risk factors including DM and HT 
 Lipid lowering drugs if conservative measures failed 
 
6 What is the drug of choice in the following situations? 
 Familial hypercholesterolemia (AD trait with excess LDL due to defect pr deficiency of LDL 
cell receptors)– statin  
 Hypercholesterolemia – statin 
 Hypertriglyceridaemia – niacin or gemfibrozil 

Page 256 By Heyson Chan


Hypernatremia 
Presentation
 Mental confusion 
 Seizures 
 Muscle irritability 
 Check the hydration status of the patient 
 
Investigation Results
 Check plasma and urine osmolarity 
 
Questions
1 What are the causes of hypernatremia?  
Common cause 
 GI, especially viral GE 
 Renal – diabetes insipidus, osmotic diuresis 
 Insensible loss (fever, respiratory infections, UTI – most common cause) 
 Addison’s disease 
 
2 How would you manage the patient? 
 Treat hypovolemia first 
 If the patient is hypovolemic, use NS initially  
NB. Many hypernatremic patients are frail elderly with weak heart 
May need frusemide cover 
 [Na] should not be reduced faster than 12 mmol/l per day (risk of cerebral edema) 

Page 257 By Heyson Chan


Hypocalcemia 
Presentation
 Parathesia 
 Cramps 
 Psychosis 
 Trosseau’s sign (spasm of the fingers when BP cuff is inflated around arm) 
 Chvostek’s sign (twitching of facial muscles when tapping on the facial nerve) 
 
Questions
1 What are the causes for hypocalcemia? 
 Removal of the parathyroid gland 
 Acute pancreatitis 
 Massive transfusion (citrate chelate calcium) 
 Vitamin D deficiency 
 Chronic renal failure 
 
2 What is your management plan for a patient presented with hypocalcemia? 
 10mL 10% iv calcium gluconate over 10 minutes 
 Oral vitamin D  
 Oral calcium preparations 

   

Page 258 By Heyson Chan


Hypokalemia 
Presentation
 Cramping, muscle weakness, hypotonia 
 Cardiac arrhythmia  
 Small or inverted T waves and U waves on ECG 
 
Questions
1 What are the causes of hypokalemia? 
Potassium depletion
 Renal loss of potassium (loop / thiazide diuretics, renal tubular acidosis, vomiting, Conn’s, 
Cushing’s) 
 GI loss of potassium(poor intake; vomiting, diarrhea, fistulas) 
 Dietary deficiency 
Transcellular shift of potassium 
 Metabolic alkalosis 
 Insulin 
 Beta‐2 agonist 
 Hypokalemic periodic paralysis 
 
2 How would you manage a patient with hypokalemia? 
Ix: 
 Serum RFT, total CO2 content, chloride, magnesium; 
 Simultaneous blood and urine x TTKG (trans‐tubularpotassium gradient) 
 Check baseline ECG (esp. those patients on digoxintherapy) 
Mx: 
If serum K > 2.5 mM & ECG changes are absent: oral supplement 
 KCl 20‐30 mmol/hour in saline infusion (up to 60‐80mM) as continuous IV infusion;  
 May combine with oralKCl 30‐40 mmoles (3‐4 gm syr KCl) Q4H;  
 Maximumtotal treatment dose: 100–200 mmoles per day (~ 3mmoles/kg/day). 
If serum K < 2.5 mM &/or ECG changes present: slow IV infusion 
 Consult ICU / cardiac monitor; 
 Keep KCl 80 mmol/L NS  give at 40 mmol/hour 
 Maximum total treatment dose: 100–200 mmoles per day (~ 3 mmoles/kg/day). 
Hypokalaemia associated with metabolic acidosis 
 Give potassium citrate solution (1 mmole/mL) 15‐30 mLQID in juice after meals;  
 Start K replacement beforebicarbonate therapy in separate IV line if indicated. 
NEVER give a bolus dose of potassium 

Page 259 By Heyson Chan


Hyponatremia 
Questions
1 What are the causes of hyponatremia? 
Determine the volume status of the patient first. 
Hypovolemia 
Renal loss 
 Thiazide / other diurectics 
 Salt‐losing nephropathy 
Extra renal loss 
 GI loss (e.g. vomiting / diarrhea / fistula / obstruction) 
 Sequestration to third space (e.g. peritonitis / pancreatitis / burns) 
Edematous state 
 CHF, cirrhosis, nephrotic 

Euvolemic
 SIADH  
 lung disease 
 neurologic disease 
 ectopic ADH 
 drugs: carbamazepine, omeprazole, NSAID, cisplatin, cyclophosphamide 
 post‐surgery, oxytocin 
 Hypothyroid 
 Addison (adrenal insufficiency)  
 Drugs (e.g. NSAID, omeprazole, Vasopressin, carbamazepine) 
 
2 How would you assess the patient? 
First, determine the volume status of the patient 
 Hypovolemic – clinical history to distinguish renal vs extra‐renal loss 
 Edematous state – CHF, cirrhosis, nephrotic 
 Euvolemic – SIADH, hypothyroid, Addison’s 
If clinically euvolemic, check the following 
 Plasma osmolarity – to exclude pseudo‐ / osmotic causes 
 Spot urine osmolarity  
 In SIADH, expect spot urine Na > 40  
 If spot urine Na < 20, suggest occult hypovolemia as the cause of increase ADH  
 With low plasma osmolarity, normal kidney should maximally dilute the urine 
 Urine is “abnormally concentrated” if osmolarity > 100 mOsm/kg 
 Spot urine sodium 
 
 A direct assessment of ADH activity 
 Acid‐base, potassium, uric acid level 
If suspecting SIADH – exclude other causes first! 
 Adrenal function 
 Renal function test 
 Thyroid function test 

Page 260 By Heyson Chan


 
3 How would you manage the patient? 
 Water restriction 1L/day 
 Hypovolemic causes: intravenous normal saline 
 Edematous state: fluid restriction + treat underlying cause +/‐ frusemide 
NB. hyponatremic usu. indicate advanced underlying disease and poor Px 
 SIADH 
 Clarify and treat underlying cause 
 Restrict water 800 to 1000 ml/day 
 NaCl supplement (1 gram = 17 mmol): 0.9 to 1.8 gm 2 to 3 times/day 
 Rapid correction by hypertonic saline infusion 
Indications (very rare)– repeat until Na>120 or patient asymptomatic 
 Acute  
 Neurological disturbance 
o Sodium deficit (mmol) = body weight (kg)  0.5  (140 – current [Na]) 
o Aim: [Na] should rise less than 12 mmol/l per day (important!) 
 
NB. Too rapid correction causes osmotic demyelination (central pontine myelinolysis)
 Regimen:  use  23.4%  NaCl  (1  ml  =  4  mmol)amount  (in  mmol)  over  24  hours  =  body 
weight (kg)  0.5  12 
   

Page 261 By Heyson Chan


L5 Root Compression 
Physical Examination
 Inspection: normal 
 Tone: normal 
 Power: weakness of big toe dorsiflexion, eversion and inversion of foot (c.f. intact foot 
inversion in common peroneal nerve palsy)  
 Jerks: loss of ankle jerk (c.f. common peroneal nerve palsy with intact ankle jerk) 
 Cerebellar Signs: Nil 
 Sensory: along S1 dermatone– outer calf (signiture zone at the dorsum of foot at the third 
MCP) (c.f. common peroneal nerve palsy – loss of sensation on the lateral aspect of the leg and 
the dorsum of the foot) 
 Gait: foot drop and high stepping gait 
 
Conclusion
This patient has weakness of (left‐sided) toe dorsiflexion and inversion with absent ankle jerk. The 
most likely diagnosis is L5 root compression due to L4/L5 disc protrusion. The patient has to wear 
caliper shoe. 
 
Questions
1 How to differential between lateral peroneal nerve palsy and L4 L5 nerve root compression? 
 Ankle inversion preserved in lateral peroneal nerve palsy 
 Ankle jerk preserved in lateral peroneal nerve palsy 
 
2 What are the causes of foot drop? 
 Peripheral neuropathy 
 L4, L5 root lesion 
 Lateral / common peroneal nerve palsy 
 Motor neuron disease 
 Sciatic nerve palsy 
 Lumbosacral plexus lesion 
 
3 What are the causes of L5 nerve root compression? 
 Posterior disc prolapse of L4/L5 (causing L5 symptoms) 
(May cause sciatica demonstrated by straight leg raising test) 
 
4 What investigations would you like to perform? 
 Spine X‐ray (AP and lateral) 
 MRI spine 
 Nerve conduction study 
   

Page 262 By Heyson Chan


5 How would you manage a patient with L4/L5 nerve root compression? 
Conservative
 Bed rest 
 Physiotherapy 
 Education on proper posture and avoid lifting up heavy objects  
 Analgesic for pain relief 
 Oral steroid for reliving inflammation 
Surgical 
 Surgical decompression 
 
6 What are the causes of lateral peroneal nerve palsy? 
 Compression (e.g. from application of a tourniquet or plaster of Paris cast) 
 Direct trauma 
 Leprosy 
 Ganglion arising from the syperior tibiofibular joint may compress the nerve 
 
7 How would you manage a patient with lateral peroneal nerve palsy? 
 Mild disease: conservative – maintain muscle bulk, caliper shoes 
 Severe disease: surgery 
 

Page 263 By Heyson Chan


Lateral Medullary Syndrome 
Physical Examination
 CN 2: VA, visual field – normal 
 Direct and indirect light reflex– normal 
 Partial ptosis on ipsilateral side with constricted pupil (Horner’s) 
 CN 3,4,6: EOM – nystagmus, no diplopia 
 CN 5: ipsilateral loss of pain and temperature sensation of face 
 CN 7: (no) facial weakness (depending on extent) 
 CN 8: normal, Rinne’s +ve, Webers not localizing 
 CN 9‐10: impaired gag reflex 
 CN 11: normal 
 CN 12: no tongue weakness 
 
Also note: 
 Intractable hiccups, dysphagia 
 Ipsilateral limb ataxia 
 
Conclusion
This patient with ipsilateral Horner’s syndrome with contralateral loss of pain and temperature 
sensation has lateral medullary syndrome due to (stroke) and has (dysphagia) 
 
Questions 
1 Which vessel is occluded? 
Any of the following: 
 Posterior inferior cerebellar artery 
 Vertebral artery  
 Superior, middle or inferior lateral medullary artery 
 
2 What tracts are affected in lateral medullary syndrome? 
 Cerebellum – dysarthia, ipsilateral limb ataxia, vertigo, nystagmus  
 Sympathetic tract – Horner’s syndrome 
 CN 5 tract – ipsilateral loss of pain and temperature of face 
 CN 10 tract – ipsilateral pharyngeal and laryngeal paralysis 
 Spinothalamic tract – contralateral sensory loss (pain andtemperature)  
 
3 How may the patient present? 
 Sudden onset of vertigo, vomiting 
 Ipsilateral ataxia 
 Contralateral loss of pain and temperature sensation 
 
4 What are the features of Horner’s syndrome? 
 Miosis 
 Partial ptosis 
 Enophthalmos 
 Loss of sweatings if the lesion is below superior cervical ganglion 
 

Page 264 By Heyson Chan


5 What are the causes of Horner’s syndrome? 
Results from the involvement of the sympathetic pathway which starts in the sympathetic 
nucleus and travels through the brainstem and spinal cord to the level of C8/T1/T2 to the 
sympathetic chain, satellite ganglion and carotid sympathetic plexus. Causes include: 
 Brainstem infarct 
 Synringomelia 
 Supraclavicular tumor 
 Apical lung CA / TB 
 Cervical rib 
 Carotid aneurysm 
 Neck surgery / CA (e.g. thyroid) 
Note: migrane may cause intermittent Horner’s syndrome 

Page 265 By Heyson Chan


Lung Collapse 
Physical Examination 
 
 Note any sputum mug / O2 supplement, SaO2 monitor 
 Peripheral: note any clubbing, nicotine stain, count the respiratory rate 
 Inspection: diminished chest expansion on affected side; note any scar 
 Lymph node: if positive  points towards carcinoma as the cause 
 Palpation: trachea deviated to the affected side, apex beat not deviated, diminished chest 
expansion on affected side 
 Percussion: dull percussion note on affected side  
 Auscultation: vesicular breath sound, diminished air entry on affected side, decreased vocal 
resonance on affected side 
 
Conclusion
This patient has a collapsed left lung and I would like to exclude malignancy. He is clinically in 
(shortness of breath and is on oxygen supplement).  
 
Questions

1 What are the causes of a collapsed lung? 
 Bronchogenic carcinoma 
 Previous penumectomy (with thoracotomy scar) 
 Mucus plugs 
 Extrinsic compression from hilar adenopathy 
 Tuberculosis (Brock’s syndrome – compression of R middle lobe by ab enlarged lymph 
node) 
 Other intra‐bronchial carcinoma 
 
2 What investigations would you like to perform? 
 CBC, L/RFT 
 Sputum for cytology, acid fast bacilli and culture 
 Bronchoscopy + biopsy of lesion indentified 
 Chest X‐ray 
 CT thorax (may perform CT‐guided biopsy for peripheral lesion) 
 Percutaneous fine‐needle aspiration for lung nodule indentified 
 May condier mediatinoscopy for staging before radical surgery 
 (if confirmed CA lung, PET scan for metastasis) 
 
3 What are the chest X‐ray findings lung collapse? 
 Raised hemidiaphragm on the affected side 
 Loss of lung volume on the affected side 
 Double heart boarder in left lower lobe collapse  
 Indistinct diaphragm indicated lower lobe collapse  
 Loss of right heart border reflects collapse affecting right middle lobe 
 
 

Page 266 By Heyson Chan


4 What are the causes of small hand muscle wasting? 
Unilateral
 Brachial plexus trauma 
 Pancoast’s tumor 
 Cervical cord lesion 
 Malignant infiltration of the brachial plexus 
Bilateral 
 Rheumatoid arthritis 
 Old age 
 Cervical spondylosis 
 Bilateral cervical ribs 
 Motor neuron disease 
 Syringomyelia 
 Charcot‐Marie‐Tooth disease 
 GBS 
 

Page 267 By Heyson Chan


Lung Mass 
1 What are the features that need to be described for a lung mass? 
 Site, size, shape 
 Content (homogenous / heterogenous) 
 Border (ill‐defined) 
 Any cavitation 
 Any hilar lympadenopathy 
 Any pleural extension 
 Any calcification 
 Any similar lesions 
 Any bony destruction 
 
2 What investigations would you like to perform for a patient with an incidental finding of a 
lungmass on CXR? 
 Bronchoscopy 
 CT thorax +/‐ CT guided biopsy (for better delineation of the nodule and to look for other 
small lesions not visible on CXR) 
 
3 What are the differential diagnoses for multiple nodules on a CXR? 
 Metastasis until proven otherwise 
 Military TB 
 Fibrosis  
 
4 Which type of cancer typically metastasis as multiple cavitations nodules? 
 Squamous cell carcinoma (e.g. CA cervix, head and neck primary) 

Page 268 By Heyson Chan


Mitral Regurgitation 
Physical Examination
 Peripheral Signs: AF (of chronic MR) 
 Carotid Pulse: normal   
 JVP: elevated if patient in heart failure 
 Apex Beat: volume overloaded; displaced  
 Auscultation: dual heart sound with soft S1; pan‐systolic murmur best heard at apex radiating to 
the axilla 
 Lung Base: basal creptitation if patient in heart failure 
Conclusion 
This patient has mitral regurgitation likely due to (chronic rheumatic heart disease). The patient is 
clinically in (heart failure and is in shortness of breath). 
Questions 
1 What are the causes of mitral regurgitation? 
 Rhaumatic heart disease 
 Infective endocarditis 
 Left ventricular dysfunction 
 Mitral valve prolapse 
 Cadiomyopathy 
 Connective tissue disorder 
 Post‐myocardial infarction 
 
2 What are the complications? 
 Infective endocarditis 
 Atrial fibrillation 
 Systemic embolization 
 
3 What investigations would you like to perform? 
 ECG for P mitrale, AF and LVH 
 CXR for pulmonary hypertension and LVH 
 Echocardiogram to confirm the diagnosis and for the severity 
 Cardiac catheterization 
4 What is the medical treatment for mitral regurgitation?  
 Asymptomatic patient: antibiotic prophylaxis for endocarditis 
 If atrial fibrillation: digoxin to slow ventricular response 
 Heart failure: diurectics and inotropes, but major consideration should be given to surgery 
 Urgent surgery for acute mitral regurgitation  
5 What are the indications for surgery in mitral regurgitation? 
 Moderate to severe symptoms despite medical treatment 
 Asymptomatic patient with ejection fractions falls to 60% on 6‐monthly echo  
 
6 What are the factors suggesting severe mitral regurgitation? 
Clinical
 Large left ventricle 
 S3 
Investigations
 Ejection fraction <30% 

Page 269 By Heyson Chan


Mitral Stenosis 
Physical Examination
 Peripheral Signs: AF  
 Carotid Pulse: normal   
 JVP: elevated if patient in heart failure 
 Apex Beat: tapping (palpable S1); may be displaced  
 Auscultation: loud S1, mid‐diatolic rubling murmur best heard at apex with the bell and patient 
lying on left lateral position 
 Lung Base: basal creptitation if patient in heart failure 
Conclusion 
This patient has mitral stenosis most likely due to (chronic rheumatic heart disease) complicated 
with atrial fibrillation and (congestive heart failure). 
 
Questions 
1 What are the causes of mitral stenosis? 
 Rheumatic heart disease 
 
2 What is severe mitral stenosis on echocardiogram? 
 Mitral valve area <1cm2 
 
3 What are the complications? 
 Left atrial enlargement and atrial fibrillation 
 Systemic embolization 
 Pulmonary hypertension 
 Tricuspid regurgitation 
 Right heart failure  
 
4 What investigations would you like to perform? 
 ECG may shows P mitrale, right axis deviation, RVH 
 CXR for pulmonary congestion and left atrial size 
 Echocardiogram to confirm the diagnosis and for the severity 
 Exercise testing for functional capacity 
 Cardiac catheterization 
 
5 What are the treatment options for mitral stenosis? 
 Asymptomatic patient in sinus rhythm: antibiotic prophylaxis for endocarditis 
 If atrial fibrillation: digoxin to slow ventricular response and anti‐coagulation 
 Heart failure: diurectics and inotropes, but major consideration should be given to surgery 
 Moderate to severe symptoms or pulmonary hypertension – consider: (1) balloon 
valvotomy or valvuloplasty or percutaneous transluminal mitral commussurotomy, or (2) 
surgery 
 
6 What are the indications for surgery? 
 Severe symptoms of pukmonary congestion and significant mitral stenosis 
 Pulmonary hypertension or haemoptysis, even asymptomatic 
 Recurrent thromboemolism despite therapeutic anti‐coagulation  

Page 270 By Heyson Chan


Mixed Aortic Valve Lesion 
Physical Examination
 Peripheral Signs: collapsing pulse if AR dominant 
 Carotid Pulse: bisferious, small volume or large volume (depending on dominant lesion) 
 JVP: elevated if patient in heart failure 
 Apex Beat: displaced if chronic AR; hypervolemic or hyperkinetic depending on dominant lesion 
 Auscultation: dual heart sound (soft S2 if AS dominant; loud S2 if AR dominant); early diatolic 
murmur best heard at LLSB (AR); ejection systolic murmur radiating to the neck (AS) 
 Lung Base: basal creptitation if patient in heart failure 
 Blood pressure: depending on the dominant lesion 
 
Conclusion 
This patient has aortic stenosis and aortic regurgitation likely due to chronic rheumatic heart 
disease. The dominant lesion is (aortic regurgitation), evidenced by (collapsing pulse and wide pulse 
pressure). The patient is clinically in (heart failure and is in shortness of breath). 
 
Questions 
1 How to determine the dominant lesion in mixed aortic valve lesion? 
Dominant aortic stenosis
 Small volume carotid pulse 
 Normal or narrow pulse pressure 
Dominant aortic regurgitation 
 Collapsing pulse 
 Large volume carotid pulse 
 Wide pulse pressure 
 
2 What are the causes for mixed aortic valve lesion? 
 Chronic rheumatic heart disease 
 Bicuspid aortic valve 
 
3 What investigations would you like to perform? 
 Chest X‐ray 
 Electrocardiogram 
 Echocardiogram 
   

Page 271 By Heyson Chan


Mixed Mitral Valve Lesion 
Physical Examination
 Peripheral Signs: AF 
 Carotid Pulse: normal 
 JVP: elevated if patient in heart failure 
 Apex Beat: displaced; hypervolemic or tapping depending on dominant lesion 
 Auscultation: dual heart sound (loud S1 if MS dominant; soft S1 if MR dominant); pan‐systolic 
murmur best heard at apex, radiating to the axilla (MR); mid‐diatolic rubling murmur best heard 
at apex with the bell and patient lying on left lateral position (MS);  
 Lung Base: basal creptitation if patient in heart failure 
 
Conclusion 
This patient has mitral stenosis and mitral regurgitation likely due to chronic rheumatic heart 
disease. The dominant lesion is (mitral regurgitation), evidenced by (volume overloaded, displaced 
apex with soft S1). The patient is clinically in (atrial fibrillation and heart failure). 
 
Questions 
1 How to determine the dominant lesion in mixed mitral valve lesion? 
Dominant mitral stenosis
 Tapping apex beat, minimally displaced 
 Loud first heart sound 
Dominant mitral regurgitation 
 Volume overloaded, displaced apex 
 Soft first heart sound 
 Third heart sound present 
 
2 What are the causes for mixed mitral valve lesion? 
 Chronic rheumatic heart disease 
 

Page 272 By Heyson Chan


Motor Neuron Disease 
Physical Examination
 Inspection: muscle wasting may be present in some muscle groups 
 Tone: spastic / flaccid (mixed) 
 Power: painless, asymmetrical weakness of muscles  
 Jerks: mixed hyper‐reflexia and hypo‐reflexia (e.g. brisks reflexes from a wasted muscles / 
fasciculation of the tongue with a brisk jaw jerk) 
 Cerebellar Signs: Nil 
 Sensory: normal (must be normal) 
 Gait: maybe wheelchair bound 
 Cranial nerves: fasciculation of tongue, bulbar palsy, dysarthia, dysphagia  
Conclusion
This patient has mixed upper and lower motor neuron signs without sphincter disturbance. I would 
like to rule out cervical cord compression. My other differential diagnosis is motor neuron disease.  
The patient is wheelchair bound as a result of the disease. 
Questions
1 What are your differential diagnoses? 
 Cervical cord compression 
 Motor neuron disease (after ruling out cord compression by MRI) 
 Multifocal motor neuropathy 
 Spinal muscular atrophy (genetic disease with loss of motor neuron in spinal cord and 
brainstem) 
2 What are the characteristics of motor neuron disease? 
 Rarely begins the age of 40 years 
 Presence of upper and lower motor involvement of a single spinal segment, and motor 
dysfunction involving at least two limbs or one limb and bulbar muscles 
 Sensory must be intact 
 Ocular movements not affects 
 No cerebellar or extrapyrimadal signs 
 Sphincters are involved late, if at all 
 Disease is fatal within 5‐7 yeas (due to bronchopneumonia) 
3 How many subtypes are there in motor neuron disease?  
 Progressive muscular atrophy (loss of anterior horn cells) 
 Primiary lateral sclerosis (loss of cortical Betz’s cells) 
 Amyotrophic lateral sclerosis (combined features of progressive muscular atrophy and 
primary lateral sclerosis) 
 Progressive bulbar palsy (loss of lower‐brainstem motor nuclei cells – CN 9, 10, 12 palsy) (c.f. 
pseudobulbar palsy associate with paralysis) 
4 What is the prognosis?  
 Poor, disease is fatal within 5‐7 years  
5 Mention a few causes of absent knee jerks with upgoing planta. 
 Peripheral neuropathy in a stroke patient 
 Motor neuron disease 
 Upper motor neuron disease in a DM patient 
 Tabes dosalis 
 Subacute combined degeneration of cord 

Page 273 By Heyson Chan


Myasthenia Gravis 
Physical Examination
 Inspection: note any bilateral partial ptosis; worsened ptosis after sustained upward gaze for at 
least 45 seconds; nasal speech 
 Cranial nerve: variable diplopia and variable squint 
 Tone: normal / decreased 
 Power: normal, decreased after repetitive movement (e.g. ask patient to repeated abduct and 
adduct shoulder  weakness in shoulder abduction) 
 Reflexes: normal / hyporeflexia, downgoing planta 
 Sensory: normal 
 Cerebella signs: nil 
 Gait: normal  
 
Conclusion 
 
This patient has variable diplopia and proximal weakness associate with fatiguability especially on 
repeated exertion of the muscle. The most likely diagnosis is myasthenia gravis.  
 
Questions 
 
1 What is the differential diagnosis? 
 Myasthenia gravis 
 Botulism 
 Lambert‐Eaton syndrome 
 
2 What investigations would you like to perform in this patient? 
 Tensilon test 
 Vital capacity 
 CXR / CT thoraz 
 Serum acetylcholine receptor antibodies 
 Thyroid function test (to rule out associate thyroid disorder) 
 EMG 
 
3 How would you manage a patient with myasthenia crisis? 
 Admit ICU 
 Watch out for respiratory failure in any patient withprogressive weakness 
 Regularly monitor FVC (peak flow rate, SaO2, ABG not useful)–Intubate and initiate 
mechanical ventilation if FVC < 15‐20ml/kg or patient exhausted 
 Stop anticholinesterase 
 Start prednisolone 50‐70 mg/day, early steroid‐induceddeterioration may occur. 
 Perform plasma exchange 50 ml/kg daily or on alternate daysuntil adequateresponse 
achieved (usually after 2‐5exchanges) or administer IVIG 0.4 g/kg/day for 5 days 
 Resume anticholinesterase at a smaller dose 48‐72 hours afterstabilization and titrate 
according to response. 
 Identify and treat any precipitating conditions 
 

Page 274 By Heyson Chan


4 What is the general management plan for myasthenia? 
Depends on mode of presentation 
Pure ocular MG
 Anti‐cholinesterase (pyrodpstigmine) 
 Steroid may help 
Mild generalized MG 
 Anti‐cholinesterase (pyrodpstigmine) 
 Young patient may benefit from thymectomy in long term 
Moderately severe MG 
 Anti‐cholinesterase are usually inadequate 
 Steroid and azathoprine are usually required 
 Early thymectomy recommended 
Fulminating MG 
 Elective ventilation 
 Plasma exchange 
 IVIG 
 Thymectomy  
 
5 What drugs may precipitate myasthenia? 
 Aminoglycosides 
 Quinine 
 Quinidine (“gin‐tonic” effect) 
 Procainamide 
 Beta‐blockers 
 Musclerelaxants
 Penicillamine
 
6 What may the patient experience in overdose of anti‐cholinesterase? 
 Abdominal cramps 
 Diarrhea 
 Myasthenia crisis 
 
7 What is the underlying pathophysiology? 
 Antibodies directed against the postsynaptic acetyl‐choline receptors 

Page 275 By Heyson Chan


Myositis 
Physical Examination
 Inspection: normal in polymyositis; dermatomyositis: purplish‐blue rash around the eyes 
(Heliotrope rash); erythema f knuckles (Gottron’s sign) 
 Tone: normal / decreased 
 Power: proximal muscle weakness and tenderness of muscle 
 Reflexes: normal / hyporeflexia, downgoing planta 
 Sensory: normal 
 Cerebella signs: nil 
 Gait: high steping gait 
 
Conclusion 
This patient has symmetrical proximal muscle weakness associated with heliotrope rash and 
gottron’s papules. The most likely diagnosis is dermatomyositis. (He has difficulty in climbing stairs 
and combing hairs). (He is also experiencing dysphagia and dysphonia).  
I would like to look for underlying malignancy. 
Questions 
1 What is the differential diagnosis of symmetrical proximal muscle weakness? 
 Inherited: muscular dystrophies 
 Inflammation: polymyositis, dermatomyositis, polymyalgia rheumatica 
 Endocrine disorders: thyroid problem, Cushings, DM 
 Electrolyte disorders: hypoklameia, hypocalcemia, hypomagnesemia 
 Toxic / drug‐induced: steroid, statins, alcohol 
 Neuromuscular: MG, myopathy, motor neuron disease, muscle dystrophy 
 Others: paraneoplastic syndrome 
 
2 What investigations would you like to perform in patients with lower motor neuron lesions? 
Routine
 L/RFT, glucose for electrolyte disturbance 
 TFT for metabolic myopathy 
 CPK for myositis 
 CXR for CA lung causing paraneoplastic syndrome 
 ECG for cadiomyopathy associated with myopathy 
 FVC for monitoring lung function (respiratory reserve) 
Specific
 EMG 
 Nerve conduction study 
 Muscle biopsy 
 Nerve biopsy 
 MRI for muscle or nerve swelling and inflammation 
 
3 If the patient has dermatomyositis, what are the expected investigation findings? 
 CPK elevated (mirror disease activity) 
 EMG shows myopathic changes 
 Muscle biopsy reveals infiltration of inflammatory cells 
 

Page 276 By Heyson Chan


4 How would you manage the patient? 
 Bed rest at acute stage 
 Investigations as above 
 Prenisolone 1mg/kg daily for 5‐8 weeks (then maintenance dose for ~1year)  
 If steroid resistant, consider methotrexate, azathioprine and high dose IVIG 
 Treat underlying malignancy 
 
5 What is the underlying pathophysiology? 
 Autoimmunity 
 Polymyositis: cellular immunity mediated by lymphocytes 
 Dermatomyositis: humoral immunity with deposition of Ig and complements in vessels 
causing necrosis and thrombosis 

Page 277 By Heyson Chan


Neurological Localization
Limb Weakness
1 Weakness in all four limbs 
a) With increased reflexes and upgoing plantar 
 Cervical cord lesion or bilateral pyramidal lesions (use sensory testing and cranial nerve 
sign to discriminate) 
b) With absent reflexes 
 Polyradiculopathy or peripheral neuropathy or myopathy (sensory testing should be 
normal in myopathy) 
c) Mixed upper motor neuron (legs) and lower motor neuron (arms) 
 Motor neuron disease (no sensory loss) or mixed cervical myelopathy and radiculopathy 
(with sensory loss) 
d) Normal reflexes 
 Fatiguable weakness, particularly with cranial nerve abnormalities  
 
2 Weakness in both legs 
a) With increased reflexes and upgoing plantar 
 Suggests a lesion in the spinal cord. The lesion must be above the root level of the 
highest motor abnormality. Sensory level present. 
b) With absent reflexes in the legs 
 Polyradiculopathy, cauda equina lesions or peripheral neuropathy  
3 Unilateral arm and leg weakness 
 Contralateral sensory findings (pain and temperature loss) indicate lesion of half ipsilateral 
cervical cord lesion (Brown‐Sequard) 
 Contralateral cranial nerve lesions or brainstem signs indicate brainstem affected 
 Ipsilateral facial or tongue weakness indicate lesion above brainstem 
4 Single limb weakness 
a) Upper limb 
 Median nerve: weakness and wasting of thenar muscles; sensory loss over thumb, index 
and middle finger 
 Ulnar nerve: weakness with or without wasting of all muscles in the hand except LOAF; 
sensory loss over little and half ring finger 
 T1 root: wasting of all small hand muscles; sensory changes confined to the median 
forearm 
 Radial nerve: weakness of finger extension, wrist extension; minimal sensory changes at 
anatomical snuff box; loss of reflex of supinator 
 Bilateral wasting of small muscles: peripheral neuropathy (with sensory loss) or motor 
neuron disease (without sensory loss) 
 C5 root: weakness of shoulder abduction and external rotation and elbow flexion; loss of 
bicep jerk; sensory loss of outer aspect of upper arm 
 C6 root: weakness of elbow flexion, pronation; loss of supinator jerk; sensory loss over 
lateral aspect of forearm and thumb 
 C7: weakness of elbow and wrist extension; loss of triceps jerk; sensory loss of middle 
finger 
 C8 root: weakness of finger flexion; loss of finger jerk; sensory loss over the medial 
aspect of forearm 

Page 278 By Heyson Chan


b) Lower limb 
 Common peroneal nerve palsy: weakness of foot dorsiflexion and eversion with 
preserved inversion; sensory loss over lateral shin and dorsum of foot 
 L4 root: weakness of knee extension and foot dorsiflexion; loss of knee jerk; sensory loss 
over medial shin 
 L5 root: weakness of foot dorsiflexion, inversion and eversion, extension of the big toe 
and hip abduction; sensory loss over lateral shin and dorsum of foot 
 S1 root: weakness of plantar flexion and foot eversion; loss of ankle jerk; loss of sensory 
over lateral border of foot and sole of foot 
 
5 Variable weakness 
 Worse after exercise or evening, consider myasthenia gravis 
 
Question
1 What are the common causes of the following conditions? 
a) Myopathy 
 Inherited: muscular dystrophies 
 Inflammation: polymyositis, dermatomyositis, polymyalgia rheumatica 
 Endocrine disorders: thyroid problem, Cushings, DM 
 Electrolyte disorders: hypoklameia, hypocalcemia, hypomagnesemia 
 Toxic / drug‐induced: steroid, statins, alcohol 
 Neuromuscular: MG, myopathy, motor neuron disease, muscle dystrophy 
 Others: paraneoplastic syndrome 
b) Myasthenic syndromes 
 Myasthenia gravis: usually idiopathic, occasionally drug induced (peniciilamine) 
 Lambert‐Easton syndrome (paraneoplastic) 
c) Mononeuropathies 
 Compression 
 Entrapment  
d) Radiculopathy 
 Cervical or lumbar disc protrusion (most common: L5/S1  S1 root) 
e) Peripheral neuropthy 
 Acute motor neuropathy: GBS 
 Subacute sensorimotor: vitamin deficiency, heavy metal toxicity, drugs (isoniazid), 
uraemia  
 Chronic: DM, hypothyroidism, charcot‐marie‐tooth 
f) Mononeuritis multiplex 
 Inflammatory: polyarteritis nodosa, RA, SLE 
g) Polyradiculopathy 
 Indicate lesion to many roots, produced more proximal weakness than peripheral 
neuropathy 
h) Spinal cord syndrome 
i) Brainstem lesions 
 Infarction, hemorrhage, rare: multiple sclerosis, tumor, trauma 
j) Hemisphere lesions 
 Infarction, hemorrhage, tumor, trauma or multiple sclerosis 
 

Page 279 By Heyson Chan


Old Polio 
Physical Examination 
 Inspection: muscle atrophy on affected side, leg length discrepancy 
 Tone: normal / hypotonic on affected side 
 Power: ipsilateral decreased power  
 Reflexes: absent knee jerk and ankle jerk on affected side, downgoing plantar 
 Sensory: normal 
 Cerebella signs: nil 
 Gait: High stepping gait, foot drop 
 
Conclusion
This patient has unilateral wasting and deformity of the left leg due to old polio and is walking with 
a high stepping gait. 
I would like to examine the spine for kyphoscoliosis (seen in polio with involvement if trunk muscles) 
and tuft of hair (seen in spina bifida) 
 
Questions
1 What is the differential diagnosis? 
 Old polio 
 Spina bifida ( ask about bladder function) 
 
2 Name other causes of lower motor neuron lesions in the legs. 
Anterior horn cell
 Polio 
Nerve Root 
 Prolapsed intervetrbral disc 
 Cauda equina syndrome 
Peripheral nerve 
 Diabetic neuropathy  
 Charcot‐Marie‐Tooth disease 
 Vitamin B12 deficiency 
Neuro‐muscular junction 
 Myasthenia gravis 
Muscles
 Polymyositis / dermatomyoisitis 
 Muscle dystriphy 
 
3 How is polio prevented? 
 Oral live attenuated vaccine 
 
4 What is spina bifida?  
 Incomplete closure of the bony vertebral canal and is commonly associated with a similar 
anomaly of the spinal Cord 
 Commonest site at lumbosacral region 
 Associate with cutaneous changes e.g. lipoma, sinus, scarring 
 Associate with neuropathic bladder 
 

Page 280 By Heyson Chan


5 What are the possible causes for spina bifida? 
 Maternal diabetes 
 Sodium valporate during pregnancy  
 
6 Mention some prenatal screening tests for spina bifida. 
 Amniotic AFP level 
 Amniotic acetylcholinesterase estimation 
 High‐resolution diagnostic ultrasounography 
   

Page 281 By Heyson Chan


Optic Atrophy 
Description
 Pale disc with sharp margin

I would also like to


 Examine the papillary light reflex for Marcus-Gunn pupil
 Look for cerebellar signs in multiple sclerosis

Conclusion
This patient has optic atrophy. I would like to rule out multiple sclerosis.

Questions
1 What are the causes of optic atrophy?
 Demyelinating disorders (e.g. multiple sclerosis) (common)
 Opric nerve compression by tumor or aneurysm (common)
 Glaucoma (common)
 Syphilis (Tabes dorsalis – Argyll Robertson pupil)
 Toxins: methanol, tobacco, lead
 Ischemia, including central retinal artery occlusion due to thromoboemolism, temporal
arteritis, idiopathic acute ischemia optic neuropathy,
 Hereditary disorders: Leber’s optic atrophy
 Paget’s disease
 Vitamin B12 deficiency
 Secondary to retinitis pigmentosa

2 How would you investigate the patient?


 CBC, ESR
 Blood glucose
 Serology for syphilis
 Vitamin B12 levels
 CT brain and orbit
 ECG
 Pattern-stimulated visual evoked responses
 Electroretinography

3 How would you investigate a patient with multiple sclerosis?


 Spinal radiography, including both cervical and thorasic agents
 Lumbar puncture: total protein concentration may be raised (in60% of cases), with an increase
in the level of IgG (in 40%) and oligoclonal bands (in 80%) on electrophoresis
 Visual evoked potentials: despite normal visual function there may be prolonged latency in
cortical response to a pattern stimulus indicating a delay in conduction in the visual pathways
 MRI brain: about 50% of patients with early multiple sclerosis in the spinal cord show
abnormal areas in the periventricular white matter
 Serum vitamin B12 – to exclude subacute degeneration of the spinal cordially

Page 282 By Heyson Chan


4 How would you diagnose multiple sclerosis?
 Classically defined as disseminated in time and place
 Basically, there are 3 types of presentation of MS: >=2 attacks, 1 attack, Insidious
neurological progression suggestive of MS
 “2 attacks+ 2 lesions”" approach
 objective clinical evidence of lesions: not just by the description of the patient, e.g. INO in this
caseAdditional data needed for MS Dx:
 dissemination in time
-detection of gadolinium enhancement at >=3 months after the onset of the first clinical event
or -detection of a new T2 lesion appearing at any time compared with a reference scan done at
least 30 days after the onset of the initial clinical event.
 dissemination in space
3 of the following:
•>=1 gadolinium-enhancing lesions or 9 T2 hyperintense lesions;
-one or more infratentorial lesions;
-one or more juxtacortical lesions;
-three or more periventricular lesions.
 visual evoked potential
-evoked potential by sensory stimulation of a subject's visual field
-e.g. flashing lights
-changes in EEG
-diagnosis of optic neuritis : delayed signal
-classic finding in Multiple Sclerosis : delayed signal
-blindness : no changes in EEG potentials -Positive CSF-Oligoclonal bands; increased IgG
index
5 What is the pathophysiology of multiple sclerosis?

6 How may multiple sclerosis present?


 Spinal cord (50%)
Sensory / motor / sphincter
 Optic neuritis (25%)
Central scotoma
 Brainstem / cerebellum lesion (20%)
Diplopia, vertigo
 Other (5%)

7 What are the clinical categories of multiple sclerosis?


 Relapsing-remitting: episodes of acute worsening with recovery and a stable course between
relapses
 Secondary progressive: gradual neurological deterioration with or without superimposed
acute relapses in a patient who previously had relapsing-remitting multiple sclerpsos
 Primary progressice: gradual, almost continous neurological deteroation from the onset of
symptoms
 Progressive relapsing: gradual neurological detrioration from the onset of symptoms but with
subsequent superimposed relapses

Page 283 By Heyson Chan


8 What are the treatments for multiple sclerosis?

Acute attack
 High dose iv methyprednisolone
 Shorten relapse but not affect final outcome
Relapse prevention
 Interferon (1beta & 1alpha)
Pros: Prevent inflammation
Cons: Does not prevent axonal injury, expensive
S/E: Flu-like symptoms, depression, abortion
C/I: Depression, liver derangement, pregnancy etc
 Glatiramer acetate (iv)
No effect on primary progressive MS
?long term benefit

9 What is internuclear ophthalmoplegia?


 Normal lateral gaze on affected eye
 When looking to the contralateral side of lesion, eye of the affected side failed to adduct and
eye of the unaffected side shows horizontal nystagmus
 Convergence in both eyes are normal

10 Where is the lesion in internuclear ophthalmoplegia?


 Lesion in the MLF

11 What is the differential diagnosis?


Lesion in the medial longitudinal fasiculas (MLF)
 Multiple sclerosis
 Brainstem lesion
 Tumor
 AVM
 Vascular (stroke)

Page 284 By Heyson Chan


Parkinsonism 
Physical Examination
Inspection 
 Pill‐rolling tremor of the hand (maybe asymmetrical or symmetrical) 
 Mask face appearance 
 Talk to the patient and note any monotonous speech 
Hands
 Pill‐rolling tremor (4‐5 Hz) 
 Cog‐wheel rigidity 
 Touch thumb with fingers in turn as fast as possible (bradykinesia) 
 Open and close fingers as fast as possible (bradykinesia) 
 Ask the patient to write and notice any mircographia 
Freezing 
 Ask the patient to rise from sitting position 
 Note any difficulty in initiating gait, turning 
Gait 
 Small‐stepping gait 
 Loss of arm swing 
Postural instability 
 Note any postural tremor 
 Retropulsion – warn and show once to the patient before performing 
Ask for 
 Note any gaze disturbance for supranuclear palsy 
 Listen to the chest for wheeze in striatal nigral degeneration 
 Measure postural BP for autonomic dysfunction 
 Perform an MMSE to screen for dementia 
 
Conclusion 
This patient with marked pill‐rolling tremor in his right hand, bradykinesia, walks with small‐
stepping gait has parkinsonism. The most likely cause is idiopathic Parkinson’s disease. He is limited 
by (bradykinesia and is experiencing difficulty in initiating gait). He currently (requires a walking 
frame). 

Page 285 By Heyson Chan


Questions 
1 What are the causes of Parkisonism? 
Idiopathic
 Parkinson’s disease (most common) 
Secondary 
 Drugs – anti‐psychotics, metoclopramide 
 Vascular – multi‐infarct, Binswanger’s disease 
 Trauma – boxer’s injury 
 Infections – e.g. postencephalitic, past‐typhoid fever, AIDS, CJD 
 Toxins – e.g. manganese, methanol, carbon monoxide 
 Wilson’s disease 
Parkinson’s plus 
 Progressive supranuclear palsy 
 Multiple system atrophy 
 Dementia syndromes 
 
2 What investigations would you like to perform? 
 Parkinson’s disease is a clinical diagnosis. Investigations may be performed to rule out 
secondary causes 
 CT brain to rule out normal pressure hydrocephalus or vascular parkinsonism 
 In patients younger than 50 years old, search of K‐F ring and measure serum ceruloplasmin 
and copper level for Wilson’s disease 
 
3 What are the treatment options for Parkinson’s disease? 
Treatment of Parkinson’s disease should be individualized according to patient’s functional 
status, age, stage of disease and response to treatment. 
General
 Encourage exercise 
 Support group, physiotherapy, physical aids 
 Consider social and caring issues 
 May not need medical treatment until disturbing symptoms occur 
Medical 
If patient’s age > 60 years old 
 Start L‐dopa +/‐ carbidopa (peripheral decarboxylase inhibitor) 
S/E: gradual shortening of drug effect (wearing off) and development of chorea after each 
dose of L‐dopa (peak‐dose dyskinesia)  adjusts dosage 
Good response to L‐dopa suggests Parkinson’s disease 
 
If patient’s age <60 years old 
 Dopamine agonist (ropinirole) – good for young patient with mild disease 
S/E: nausea, hypotension, sleep attacks (don’t drive) 
Bromocriptine is a weakest dopamine agonist 
 Selegiline (MAO‐B inhibitor) 
 Amantidine  
Antiviral agent with anticholinergic and possible dopaminergic and antiglutaminergic 
(therefore in higher dose, effective in reducing L‐dopa induced dyskinesia) effect 

Page 286 By Heyson Chan


Surgical 
To be considered if disabling motor complications persist despite adjustment of dosage
 Deep brain stimulation 
 Thalamotomy  
 
4 What do you know of Parkinson’s plus? What are their characteristics? 
Multiple system atrophy
 Less tremor 
 Symmetrical onset 
 Gait disturbance (+++ for SND; + for SDS, SOPCA) 
 Autonomic dysfunction (++ for SND; +++ for SDS; + for SOPCA) 
 Cerebellar signs (+ for SND, SDS; +++ for SOPCA)  
 Pyramidal signs (+++ for SND; ++ for SDS, SOPCA) 
 Respiratory stridor (+++ for SND) 
 Poor response to L‐dopa 
Progressive supranuclear palsy 
 Less tremor 
 Symmetrical onset 
 Severe gait disturbance  
 Moderate dementia 
 Mild cerebellar signs  
 Vertical gaze palsy  
 Axial dystonia (rigidity) 
 Poor response to L‐dopa 
Dementia syndrome 
 Dementia 
 Poor response to L‐dopa 
 

Page 287 By Heyson Chan


Phaeochromocytoma 
Presentation
Classic traid of: 
 Palpitation 
 Headaches 
 Periodic excessive sweating 
Other symptoms 
 Anxiety 
 Weight loss 
 Flushing 
 Angina / stroke 
 
Investigation Results
 Increase random sugar (catecholamines are glucogenic) 
 24 hour urine for VMA 
 Imaging: CT abdomen, MIBG scan (radioisotope‐labeled catecholamine analog), IVC venous 
sampling if above fails 
 
Questions
1 What is phaeochromocytoma? 
 An adrenal medulla tumor of the chromaffin cells which produces excess catecholamines 
(NA > adreanlines) 
 
2 What is the rule of 10 in phaeochromocytoma? 
 10% bilateral 
 10% malignant 
 10% familial (von Hipple‐Lindau syndrome, MEN I/II) 
 10% extra‐adrenal 
 10% multiple 
 10% in children 
 
3 What is the treatment? 
 Conservative (before operation) – alpha blockade by prazosin 
 Surgical (ensure adequate alpha and beta blockade before surgery) – excision of the tumor 
with early ligation of venous drainage  
 
4 Should you give alpha blocker before beta blocker before surgery? Why? 
 Heart muscles are solely supplied by beta receptors 
 If all beta receptors in the heart are block, main effect of catecholamines will go to the alpha 
receptors in peripheral vessels 
 Extensive vasospasm leading to overshoot of hypertension will occur 
   

Page 288 By Heyson Chan


5 What is MEN syndrome?  
Mutiple endocrine neoplasia is an autosomal dominant predisposition to develop multiple 
endocrine tumor 
MEN I
 Parathyroid hyperplasia 
 Pancreatic islet cell tumor 
 Pituitary tumor 
MEN IIa 
 Medullary thyroid carcinoma 
 Pheochromocytoma 
 Hyperparathyroidism  
MEN IIb 
 Medullary thyroid carcinoma 
 Mucosal neuroma 
 Pheochromocytoma  
 Marfanoid body habitus  
 

Page 289 By Heyson Chan


Pleural Effusion 
Physical Examination
 Inspection: diminished chest expansion on affected side, note pleural tap 
 Lymph node: if positive  points towards TB / carcinoma as the cause 
 Palpation: trachea central, apex beat not deviated, diminished chest expansion on affected 
side 
 Percussion: stony dull percussion note on affected side  
 Auscultation: vesicular breath sound, diminished air entry on affected side, decreased vocal 
resonance on affected side 
 
Conclusion
This patient has pleural effusion and is clinically in shortness of breath. I would like to further 
investigate for the cause of the pleural effusion. 
 
Questions
1 What are the causes of pleural effusion? 
Exudative
 Pneumonia 
 Tuberculosis 
 Malignancy 
 Pulmonary infarction 
 Connective tissue disorder (e.g. SLE, RA) 
 Sympathetic effusion (e.g. intra‐abdominal inflammation – pancreatitis) 
 Chylothoarx – disruption of lymphatic drainage via the thorasic duct with characteristic 
milky appearence 
 Hemothorax –trauma, tumor or iatrogenic 
Transudative 
 Congestive heart failure 
 Liver failure 
 Renal failure 
 Nephritic syndrome 
 Hypthyroidism 
2 How to differentiate between transudative and exudative pleural effusion? 
Light’s criteria 
 Ratio of the pleural fluid to serum protein >0.5 
 Ratio of pleural fluid to serum LDH >0.6 
 Pleural fluid LDH is greater than two‐third of the upper limit for blood LDH 
3 How would you manage this patient? 
 Give O2 
 Blood Ix: CBC w/ D/C, L/RFT, blood glucose, clotting profile 
 Sputum for C/ST, AFB and cytology 
 Imaging: CXR (pre and post‐pleural tap) 
 Pleural tap, send pleural fluid for cell count, cytology, G‐stain, C/ST, AFB, pH, glucose, 
protein, LDH, cholesterol   
 Pleural biopsy if indicated 
 Note: for theraputic tap; do NOT tap >1.5L/time 

Page 290 By Heyson Chan


 
4 What is the management for recurrent malignant pleural effusion? 
 Tube thoracostomy and chemical pleurodesis (Tetracycline 1 g in 50 ml NS) Chest tube 
kept unclamped thereafter for drainage until daily output < 150 ml/day and CXR shows 
the lung to be re‐expanded and most of the effusion drained 
 Surgical pleurodesis ‐ Thoracoscopic (simple and safe, applicable to most patients even if 
lung function is compromised) or thoracotomy approach     
 Supportive care 
 
5 Which organisms are commonly associated with pleural effusion? 
 Pyogenic bacterial infections (S. aureus, Strept, G‐ bacilli, anerobes) 
 Atypical pneumonia 
 Actinomycosis 
 Nocardiosis 
 Viral infections 
 Tuberculosis  
 
6 What are the complications of pleural tap? 
 Trauma: Pneumothorax, haemothorax, haemoptysis, air embolism, damage to liver and 
spleen 
 Re‐expansion pulmonary edema from too rapid removal of fluid (<1L at a time) 
 Pleural infection/empyema 
 Vagal shock 
 Seeding of mesothelioma (avoid biopsy if this is suspected) 
 
7 What are the CXR features suggestive of pleural effusion? 
 Homogenous opacity 
 Meniscus 
 CP angle blunted 
 If fluid level ‐‐> think of loculated pleural effusion or previous pleural tap 
8 What is the reason for the presence of pleural effusion without mediastinal shift? 
 Pressure on lung by pleural effusion 
 Lung collapse 
 No mediastinal shift 
 
9 What is the reason for the presence of pleural effusion with mediatinal shift? 
 Think of empyema, check fever and WCC 
 
10 What are the common organisms causing empyema? 
 S aureus 
 Streptococcus 
 G‐ve bacilli 
 Anerobes  
 Primary tuberculosis 

Page 291 By Heyson Chan


Pneumonia 
Chest X-ray
Technical
1 Identity: check name, ID of the patient 
2 Date of film taken 
3 Correctly mounted 
4 PA / AP film 
5 Exposure 
Airway
6 Rotation 
7 Trachea 
Breathing
8 Degree of inspiration 
9 Lung field: air bronchogram  
10 Pleural spaces: note any pleural effusion 
Circulation
11 Heart size and great vessles 
12 Hilum, mediatinum  
Others
13 Gas under diaphragm 
14 Bony structure 
15 Soft tissues 
 
Questions

1 What are the common organisms responsible for community‐acquired pneumonia? 
Typical
 Strept pneumoniae 
 Staph aureus 
 H influenza 
 G‐ve organisms  
Atypical
 Myciplasma pneumiae 
 Legionellae 
 Chlamydia 
 Viral (including SARS) 
Specific risk factors for specific pathogens 
 Penicillin resistant penumococcal: >65 years old, alcoholics, immunocompromised, medical 
co‐morbidities, recent use of antibiotics  
 Enteric gram‐negative organisms: nursing home, cardiopulmonary  
 Pseudomonas aerogenosa: structural lung disease, chronic steroid use, malnutrition 
 
   

Page 292 By Heyson Chan


2 What are your criteria for determining whether the patient needs admission? 
Low risk patient if 
 Age <50 
 Absence of the following conditions 
 Active neoplastic disease at the time or within 1 year of presentation 
 Congestive heart failure 
 Cerebrovascular disease 
 Chronic renal disease 
 Chronic liver disease 
 Absence of the following abnormal physical signs 
 Altered mental state 
 Pulse >125 
 Respiratory rate >30 
 Systolic blood pressure <90 
 T <35 or >40 
 Absence of the following features in investigations  
 Arterial pH <7.35 
 Blood urea 
 Na <130 
 Glucose >14 
 Hematocrit <30% 
 PO2 <8kPa 
 Pleural effusion 
 
3 What investigations would you like to perform for a patient presented with pneumonia? 
 CXR 
 Complete blood count/Diff WCC 
 Renal/Liver function tests 
 Sputum MCS/Tb 
 SaO2 or blood gases 
 Atypical serology 
 NPA for flu, RSV, etc. If flu A +ve: H5 by RT‐PCR ormonoclonal antibody (IF) 
 SARS work‐up (NPA, serum RNA, stool, etc for CoV) 
 Blood culture 
 Urinary antigens for pneumococcus/legionella 
 Bronchoscopy 
 Chest CT 
 Tracheal aspirate/ lung bx 
 
   

Page 293 By Heyson Chan


4 What is your choice of antibiotics? 
Outpatient pneumonia: non‐smokers of any age with nocardiopulmonary disease 
 Augmentin+/‐Macrolide 
Outpatient pneumonia with cardiopulmonary disease / smokers 
 Rx: Augmentin +/‐Macrolide 
 Those with DRSP risk# ‐ Rx: Augmentin/Unasyn +Amoxil +/‐Macrolide OR Fluoroquinolone 
Hospitalised patients with mild to moderate infection (these 
patients have risk factors requiring hospitalisation) 
 Without DRSP risk# ‐ Rx: augmentin +/‐Macrolide OR Fluoroquinolone 
 With DRSP risk# ‐ Rx: Augmentin/Unasyn + Amoxil +/‐ Macrolide OR Cefotaxime/Ceftriaxone 
+/‐ Macrolide 
Severe hospitalised community‐acquired pneumonia 
(Either 1 out 3 major OR 2 out of 6 minor) 
Major criteria: a) ARF, b) Septic shock, c) Require MV 
Minor criteria: a) RR>30/min, b) PaO2/FiO2<250,c) SBP<90 mmHg or DBP<60 mmHg, d) Urea>7 
mmol/L,e) Mental confusion, f) Multilobar involvement 
 Without pseudomonas risk: 
Rx: Cefotaxime/Ceftriaxone +/‐Macrolide or Piperacillin/Tazobactam +/‐ Macrolide 
 With pseudomonas risk 
Rx: 2 antipseudomonal agents +/‐ Macrolide, Fluoroquinolone + antipseudomonal agents 
* Modify antibiotics according to C/ST when available 
 
5 Compare the difference in signs and symptoms of typical and atypical pneumonia. 
 
Typical  Atypical 
Acute onset  Gradual onset 
Fever > 39 C  Lower fever 
Chills and rigors  Absent 
Purulent sputum  Dry cough 
Rales  Absent 
Leukocytosis  Normal WCC 
Lobar distribution  Diffuse 
Cavitation  Reticular 
Frank effusion  Bronchovascular 
Bulging fissure  Perihilar 
 

Page 294 By Heyson Chan


Pneumothorax 
Chest X-ray
Technical
1 Identity: check name, ID of the patient 
2 Date of film taken 
3 Correctly mounted 
4 PA / AP film 
5 Exposure: spine just seen under cardiac shadow 
Airway
6 Rotation: distance between the clavicle and spinous process 
7 Trachea: deviated in tension pneumothorax 
Breathing
8 Degree of inspiration: suggests underlying COPD / asthma if overinflated 
9 Lung field 
10 Pleural spaces: not fully expanded on side of lesion(air in pleural space) 
Size: determined by lung apex‐to‐thoracic cupola distance in upright CXR. 
Small < 3cm &large >3cm 
Circulation
11 Heart size and great vessles 
12 Hilum, mediatinum  
Others
13 Gas under diaphragm 
14 Bony structure: note any bone fractures which suggests trauma 
15 Soft tissues 
 
Causes 
 Idiopathic 
 COPD 
 CA lung 
 Asthma 
 Mechanical ventilation 
 Iatrogenic (e.g. CVP insertion, pleural tap) 
 PCP  
 
Management 
General
 Monitor vital signs (BP/P/RR/SaO2; whether patient can speak full sentence)  
 Give O2 supplement 
 Bed rest 
 Give analgesic prn 
Specific
Tension pneumothorax
 Medical emergency 
 Insert large bore (16 gauge) cannula in 2ndintercostals space, mid clavicular line to relive the 
tension 
 Insert chest drain afterwards 

Page 295 By Heyson Chan


Primary spontaneous pneumothorax (no underlying lungabnormalities) 
Clinically stable with small pneumothorax
 Conservative: monitor symptom and CXR 
Clinically stable with large pneumothorax
 Small bore catheter (14F) or 16‐22F chest drain* 
Clinically unstable with large pneumothorax
 16‐22F chest drain*. 24‐28F if bronchopleural fistula or mechanical ventilation anticipated 
Persistent air leak > 4 days, surgical referral for thoracoscopy# 
Risk of recurrence = 30% in 5 years after 1st episode, with greatest risk in the first 6 months; 
recurrence even higher after the 2nd episode and reaches 50% 
 
Secondary spontaneous pneumothorax (underlying lung disease) 
 Should be hospitalized even if clinically stable 
Clinically stable with small pneumothorax 
 Conservative or chest drain depending on symptom and courseof pneumothorax 
Clinically stable with large pneumothorax
 16‐22F chest drain* 
Clinically unstable with large pneumothorax
 24‐28F chest drain* 
Note: 
 Chest drain should be placed until no more air leak 
 Persistent air leak > 5 days, surgical referral for thoracoscopy 
 Attached to water‐seal device. Suction should be applied if lung fails tore‐expand 
 Chemical pleurodesis (e.g. by doxycycline) can be considered if surgery contraindicated 
orpatient refuses operation or poor prognosis from patient’s underlyingdisease
Procedure of chest drain
 Review latest CXR to confirm diagnosis, location and extent 
 Guided by percussion, incise skin along anterior or mid‐axillary line in 5th or 6th intercostal 
space (or at 2nd intercostal space, mid‐clavicular line of anterior chest wall uncommonly used)  
 Anaesthetise all layers of thoracic wall including pleura 
 Blunt dissection of intercostal muscle with artery forceps down to parietal pleura 
 Preferred insertion method: Double‐clamp outer end of Argyle drain (24 Fr to drain air/fluid, 28 
Fr to drain blood/pus). Apply artery forceps in parallel with tip of drain. Breach pleura with 
finger. Insert drain tip, release forceps and use them to direct drain into pleural cavity. 
 Direct drain apically to drain air and basally to drain fluid  
 Attach chest drain to 2 cm underwater seal. Ensure fluid level swings with respiration and 
coughing 
 Take CXR to confirm tube position and detect complications 
 
Complications of chest drain
 Trauma: Pneumothorax, haemothorax, haemoptysis, air embolism, damage to liver and spleen 
 Pleural infection/empyema 
 Vagal shock 
Indications of pleurodesis
 Lung failed to re‐expand and persistent air leak for 3 days 
 Repeated episodes of primary pneumothorax 
 First episode of secondary pneumothorax 
Surgical pleurodesis is preferred if patient can tolerate 

Page 296 By Heyson Chan


Polycystic Kidney Disease 
Physical Examination
 General inspection: pallor, AV fistula if patient is on heamo‐dialysis 
 Lymph nodes: nil 
 Inspection of abdomen: abdominal distention 
 General palpation: no T/G/R, the kidney/pancrease may be palpated  
 Liver: normal (or enlarged due to liver cysts) 
 Spleen: not enlarged 
 Kidney: ballotable (enlarged bilaterally but may be palpable unilaterally only) 
 Ascites: nil 
 Auscultation: normal bowel sound 
 
Conclusion 
This patient has bilaterally enlarged kidneys suffers from adult onset polycystic kidney disease. He is 
complicated with (end‐stage renal failure) currently undergoing (peritoneal dialysis). I would like to 
measure the blood pressure, examine the patient neurologically and ask the family history. 

Page 297 By Heyson Chan


Questions 
1 Is this a genetic disease? What is the mode of inheritance? 
 Autosomal dominant disease with 100% penetrance 
 
2 What are the possible presentations of these patients? 
 Haematuria 
 Hypertension 
 Urinary tract infection 
 Abdominal pain / distention 
 Uraemic symptoms 
 Stroke due to berry aneurysm  
 Complications related to renal cysts 
 
3 What is the prognosis? 
 1/3 die of renal failure; 1/3 die of hypertension and related complications and 1/3 die of 
other diseases  
 
4 What are the possible complications? 
Renal complications
 Hypertension (ACEI as first line for controlling BP) 
 Back pain or abdominal pain 
 Haematuria 
 Cysts infection / rupture 
 Urinary tract infection 
 Proteinuria 
 Renal failure 
Extra‐renal complications 
 Cysts in the liver, ovary, pancreas, spleen and central nervous system 
 Mitral valve prolapse / aortic regurgitation  
 Berry aneurysm 
 Colonic diverticulum 
 Hernia of anterior abdominal wall 
 
5 What is your management plan? 
Investigations
 CBC, L/RFT, urine mircosopy, urine x C/ST 
 USG kidneys (may be equivocal under 20 years old) 
 Contrast‐enhanced CT or MRI brain as screening for berry aneurysm 
 Renal biopsy to confirm the diagnosis 
Management 
 Monitor and control blood pressure 
 Treat urinary tract infections promptly by iv ciprofloxacin 
 Dialysis if end stage renal failure 
 Nephrectomy indicated if severe uncontrolled urinary infection / haematuria, malignant 
transformation or big cysts causing symptoms 

Page 298 By Heyson Chan


Prolactinoma 
Presentation
 Amenorrhea and galatorrhea in women 
 Decreased libido and impotence in men 
 
Questions
1 What are the causes of hyperprolactinemia? 
Physiological
 Pregnancy 
 Lactation and nursing 
 Stress (including venepuncture  overcome by serial measurement over 60 minutes via an 
indwelling cannula) 
Drug effect
 Estrogen 
 Dopamine agonist (e.g. haloperidol, methyldopa) 
Pathological
 Pituitary adenoma 
 Hypothalamic‐pituitary stalk lesions (loss of dopamine inhibtion) 
 Hypothyroidism (loss of  
 Chronic renal failure 
 Severe liver disease 
 
2 What is the definition of secondary ameorrhea? 
 Absence of menses for six months (or greater than three times the previous cycle interval) in a
woman whohas menstruation before
 
3 What are the causes of amenorrhea?  
Physiological 
 Pregnancy (most common cause in women in reproductive age) 
 Menopause (most common cause of peri‐menopausal women) 

Failure of hypothalamus-pituitary-ovarian axis


 Hypothalamus: anorexia nervosa, Kallman syndrome, etc 
 Pituitary: prolactinoma 
 Ovary: Turner syndrome, polycystic ovarian disease (PCOD), Testicular feminisation 
syndrome (patientswith testicular feminisation do not have ovaries and uterus), premature 
menopause, ovarian dysgenesis 
 Others: other endocrinopathologies such as hyperprolactinemia, and drug effects (e.g. 
haloperidol, methyldopa, tricyclic anti‐depressants, tranquilizers 
Absence of normal endometrium 
 Uterine agenesis 
 Asherman syndrome 

Obstruction of outflow tract


 Vulval: Imperforate hymen 
 Vaginal: transverse vaginal septum, vaginal agenesis or hypoplasia 

Page 299 By Heyson Chan


4 What are the points to note during clinical evaluation of a patient with secondary 
amenorrhea? 
 
History
 Age of menarche and menstrual history 
 Exclude pregnancy 
 Growth and development, including that of secondary sexual characteristics 
 History of virilization or hirsutism 
 Hot flushes 
 Sense of smell 
 History of weight change and exercise 
 Breast discharge 
 Emotional crisis or psychological crisis 
 Symptoms of other endocrine disorders and systemic illness  
 
Physical examination 
 Height and weight 
 Arm span 
 Secondary sexual characteristics: breast, pubic hair development 
 Features of Turner syndrome 
 Features of virilization (clitoromegaly, hirsutism, temporal balding, acne, deepening of voice, 
pectoral muscle enlargement) 
 Feature of PCOS (obesity, hirsutism) 
 Breast examination for galatorrhea 
 Evidence of other endocrine disease 
 Abdominal and pelvic examination 
 
5 How would you investigate a patient with secondary amenorrhea? 
 LH, FSH level 
 Prolactin level  
 TSH 
 Testosterone in men 
 L/RFT 
 Pregnancy test to rule out pregnancy 
 
6 What are the indications for prolactin measurement? 
 Galactorrhea 
 Enlarged sella turcica 
 Suspected pituitary tumor 
 Hypogonadotrophic hypogonadism in both men and women 
 Gynecomastia in male   

Page 300 By Heyson Chan


7 What are the treatment options for prolactinoma? 
Treatment essential for all macroadenoma (risk of further tumor expansion, hypopituitarism 
and visual impairment) and is recommended for microadenomas (restore fertility, prevent 
osteoporosis and to treat symptoms) 

Medical
 Bromocriptine  
 Side effects: nausea, vomiting, dizziness, postural hypotention  can be avoided by starting 
at low dose, then gradually increase dosage over weeks; patient should be advised to take 
the drug at night or with meals 
 Both microadeoma and macroadenoma (even with visual field impairment) responds well 
with medical treatment  
 
Surgical / radiotherapy 
 Surgical therapy by transpheoidal surgery reserved for patient fails or cannot tolerate 
medical treatment 
 Radiotherapy reserved for patients with prolactin‐secreting macroadenomas who have 
failed to respond adequately to medical and surgical treatment 

Page 301 By Heyson Chan


Psoriasis 
Physical Examination
Hand 
 Joint deformity: DIP joint may be involved (c.f. RA) 
Nail 
 Pitting, dystrophy 
 Oncholysis 
Body, scalp, trunk (on the extensor surface, sparing of face and sun exposed area) 
 Multiple erythematous silvery plaques of irregular size with well‐demarcated edge  
 
Conclusion 
This patient has multiple erythematous silvery plaques of irregular size asymertically distributed 
over the extensor surface of his body. It is associated with joint deformity. The most likely diagnosis 
is plaque‐type psoriatic arthritis. He is limited by and requires walking aid. 
I would like to examine his scalp and the trunk. I would also like to examine the joints of this patient. 
 
Questions 
1 What are the typical causes of flare up in psoriasis?  
 Drugs: beta‐blocker, ACEI, alcohol, indomethacin, lithium, anti‐malarial 
 Stress 
 Infection: streptococcal infection, staphylococcal, HIV 
 Injury to skin 
 
2 What are the treatment options for psoriasis?  
General
 Stress management 
 Avoid aggravating factors 
 Counseling and education – no cure, but control 
Topical
 Sunlight  
 Phototherapy (especially narrow‐band ultraviolet B) 
S/E: dry skin, cataract, photoaging and increased risk of skin cancer 
 Topical steroid 
S/E: possible rebound following discontinuation, skin atrophy 
 Tar  
Safe and cheap but unpleasant 
Systemic 
 Synthetic retinoids (acintertin or etretin) 
Side effects: teratogenic, hyperlipidemia, impaired liver function, xerosis 
 Methrotrexate 
Side effects: bone marrow suppression and liver fibrosis 
 Low‐dose cyclosproin  
 

Page 302 By Heyson Chan


Pulmonary Fibrosis 
Physical Examination 
 Note any sputum mug / O2 supplement, SaO2 monitor 
 Peripheral: note any clubbing, nicotine stain, count the respiratory rate 
 Inspection: diminished chest expansion on affected side; note any radiation scar 
 Lymph node: nil 
 Palpation: trachea deviated to the affected side, apex beat not deviated, diminished chest 
expansion on affected side 
 Percussion: dull percussion note on affected side  
 Auscultation: bronchial breath sound, diminished air entry on affected side, localized mid‐
inspiratory fine crackles not cleared up on coughting, decreased vocal resonance on affected 
side 
 
Conclusion
This patient has left upper lobe fibrosis, the most likely cause in this locality is past pulmonary 
tuberculosis. He is clinically stable without distress.  

Page 303 By Heyson Chan


Questions

1 What are the causes of apical fibrosis? 
 Tuberculosis 
 Ankylosing spondylitis 
 Radiation‐induced fibrosis  
 
2 What is the treatment regime for pulmonary TB? 
 DOTS for 6 months 
 Initial phase: HRZE for 2 months 
 Continuation phase: HR for 4 months 
 (prolong treatment if extrapulmoary TB – 9 to 12 months) 
 MDR‐TB – use 2nd line agents or according to C/ST 
 2nd line agents e.g. fluoroquinolone, aminoglycosdie, ethambutol 
 
3 Name some common side effects of different anti‐TB drugs 
a) Isoniazid 
 Hepatitis (stop if ALT >3 times, re‐introduce if ALT <2 times) 
 Rash 
 Peripheral neuropathy 
 Optic neuritis 
 Aplastic anemia 
b) Rifampin 
 Hepatitis 
 Rash 
 GI: nausea, vomiting, abdominal pain 
 Thrombocytopenic purpura 
c) Pyrazinamide 
 Anorexia 
 Nausea 
 Hepatitis 
 Arthralgia 
d) Streptomycin 
 Rash 
 Vestibular and auditory nerve damage 
 Renal damage  
e) Ethambutol 
 Retrobulbar neuritis 

Page 304 By Heyson Chan


Renal Investigations 
1 Assessment of GFR
urine volume ( L / day )  [urine Cr (mmol / L)]
 CrCl (ml / min)   1440
[ plasma Cr ( mol / L)]
GFR Remarks
(ml/min)
> 90 kidney damage with normal or increased GFR
60 – 90 kidney damage with mild decrease in GFR
 only have problem when there is additional insult
30 – 60 moderate decrease in GFR
 abnormal serum creatinine
 can remain stable for extended period
 need evaluation and treatment of complications
15 – 30 severe decrease in GFR
 further progression inevitable
 need referral to nephrologist for evaluation and plan of dialysis
5 – 15 kidney failure
 not life threatening, may have benefit from dialysis
<5 end stage renal disease
 die soon if not dialyzed

2 Urine analysis and microscopy


 Urine pH >8 in UTI  proteus
 Early morning urine albumin-to-creatinine level to screen for microalbuminuria
 Dysmorphic red cells  glomerulonephritis
 White cell cast  pyelonephritis

3 Immunological tests
 Increased serum IgA in IgA nephropathy and HSP
 Paraproteinemia in multiple myeloma and amyloid
 ANA in SLE
 C3 decreased in acute post-infectious or post-streptococcal glomerulonephritis, SLE, sub-
acute bacterial endocarditis (note: complement level are of no value in assessing severity and
prognosis)

4 Renal USG
 First-line investigations for visualizing renal tract
 Kidney size <9cm or difference >1.5cm = abnormal
 Renal cysts >10 in one kidney, thick of polycystic kidney
 Doppler study to assess renal blood flow or detect renal vein thrombosis
 Note: DM nephropathy not associate with small kidney

Page 305 By Heyson Chan


5 Intravenous urography
Indications:
 Investigations for recurrent UTI
 Evaluation of macroscopic hematuria
 Detection and location of renal stone
 Detection of congenital anomalies
 Detection of functional obstruction in combination with furosemide
Decreased diagnostic yield and increased risk of contrast nephropathy in patients with pre-
existing renal impairment (Cr > twice normal)

6 Radionuclide imaging
 Tc-DTPA scan
 Estimate renal plasma flow, provide functional assessment of both kidneys
 Administation of captopril in hypertensive patient allows detection of renovascular disease
which has 90% sensitivity and specificity
 DTPA scan also useful for assessing renal perfusion in renal transplant patient
 Use of frusemide with DTPA scan useful in identifying true obstructive uropathy
 DMSA – renal cortical scanning agent, useful for detecting UTI and scarring

7 Angiography
 Definitive investigation for suspected renal artery stenosis
 Detect microaneurysm in renal vasculature for patient with polyarthritis nodosa
 Potential kidney donors to map renal artery system in potential kidney donor

8 Renal biopsy
Indications
 Nephrotic syndrome
 Nephritic syndrome
 Significant proteinuria or hematuria
 Acute renal failure with uncertain pathology and no recovery
 Rule out co-existing GN in patients with DM
 Transplant allograft dysfunction

Contraindications
 Uncooperative patient
 Solitary kidney
 Abnormal clotting profile
 Uncontrolled severe hypertension
 Hydronephrosis
 Polycystic kidneys
 Very small kidney (<8cm)
 Acute pyelonephritis
 Renal neoplasms

Complications
 Perinephric hematoma (reduce risk by complete bed rest for 24 hours and monitor regularly
to detect any signs of hemorrhage)
 Prevent heavy lifting and strenuous extertion

Page 306 By Heyson Chan


Retinitis Pigmentosa 
Description
 Peripheral retina shows widespread scattering of perivascular bone spicule pigmentation and 
arteriolar narrowing 
 Retina veins (but never arteries) has a sheath of pigmentation for part of their course 
 Optic disc is pale 
 Maculopathy, which is atrophic or cystoid 
 
Questions 
1 What do you know about retinitis pigmentosa? 
 Retinintis pigmentosa is a slow degenerative disease of the retina. It occurs in both eyes 
 Begins in early childhood, often results in the loss of sight by middle or advanced age 
 Degeneration mainly affects the rods and cones, in particular the rods 
 
2 What is the prognosis? 
 Most patients are registered blind by the age of 40 with central field less than 20degree in 
diameter 
 
3 How may it present? 
 Defective in night vision, which may occur several years before the pigment is visible in 
retina 
 Progresses with contriction in the visual field 
 
4 Name some conditions that it is associated with? 
 Cataracts 
 Deaf‐mutism 
 Mental deficiency 
 
5 How would you manage the patient? 
 Genetic counseling 
 Impaired vision training and aids for daily living 
 Refer for job training 
 Regular ophthalmology follow up including visual fields, electroretinogram 
 
 
 
 
 

Page 307 By Heyson Chan


Rheumatoid Arthritis 
Physical Examination
Ask the patient for permission to examine her hands and then ask whether the hands are sore 
before you proceed! 
Hands
 Swan neck deformity of PIP 
 Boutonniere deformity (hyperextension at the DIP and flesion at the PIP)  
 Ulnar deviation and subluxation of MCP 
 Subluxation of writs joint 
Others
 Nail fold infarcts and vasculitic skin lesions 
 Palmar erythema 
 Wasting of small hand muscles 
 Examine the elbow for rheumatoid nodules 
 
Conclusion 
This patient has symmetrical polyarthritis of small joints is suffering from rheumatoid arthritis. I 
would like to perform simple perform simple assessment on the functional status of this patient 
(e.g. buttoning her clothes). 
I would also like to examine the other joints. 
 
Questions 
1 How would you diagnose RA? 
Patient must have four out of the following sever 
 Morning stiffness lasting at least for 1 hour* 
 Simutaneous polyarthritis (three or more joints)* 
 Arthritis of hand joints (DIP usually spared)* 
 Symmetrical arthritis* 
 Rheumatoid nodules 
 Abnormal rheumatoid factor 
 Typical changes on X‐ray of hands and wrists 
* Must be present for at least 6 weeks 
 
2 What is a rheumatoid factor?  
 Rheuamtopid factor consists of autoantibodies of IgM type that is directed against the Fc 
portion of IgG 
 Presence in 80% of RA patients (if absent = seronegative RA) 
 Also present in SBE, COPD, liver cirrhosis, SLE, infection 
 
3 What is the management plan for RA? 
First-line
 NSAID 
 Ibuprofen 1200mg Q8H or indomethacin 200mg Q8H 
 Use minimal dose or consider omeprazole/misoprostol for PU prophylaxis  
 May also consider COX2 inhibitor which has less GI s/e (same renal s/e) 
 S/E: elevated liver enzyme, renal insufficiency, gastric bleeding 

Page 308 By Heyson Chan


DMARDS 
 Methrotrexate 
 Weekly injection, excreted by kidney, response noted within 3‐6 weeks (rapid onset 
with sustained effects) 
 Check RFT, LFT, HBsAg and HCV before starting Rx 
 S/E: nausea, vomiting, rashes, pancytopenia, fatal pneumonitis (3%), teratogenic  
 Sulfasalzine 
 Metabolized to yield 5ASA and sulfapyridine, response seen in 4‐6 weeks 
 Well‐tolerated with minimal GI side effect 
 Monitor CBC and LFT 
 C/I in G6PD deficiency 
 Corticosteriod 
 Use low dose before DMRSA takes effect (which usually takes a few days) 
 Reserved for patients who have very active disease and prolonged morning stiffness 
which affects morning functions 
 Biologic agents (e.g. TNF antagonist ‐ rule out infection before use!) 
 Etanrecept (monotherapy as effective as methrotrexate in improving joint site 
destruction; s/e: injection site reaction, infection, malignancy) 
 Infliximab (anti‐TNF alpha, significant response in combination with methrotrexate; 
adverse effect due to increased risk of infection) 
 
4 What are the risk factors for NSAID‐induced GIB? 
 Chronically disabled 
 Age > 60 years 
 Previous history of proven peptic disease  
 Co‐administration of prednisolone > 30 mg/day 
 Higher dosage of NSAIDs 
 Extent of inflammatory disease for which NSAIDs are prescribed 
 
 
5 What are the typical radiological appearance of RA? 
 Peri‐articular soft tissue swelling 
 Juxta‐articular osteoporosis 
 Uniformed joint space narrowing 
 Central and marginal erosions 
 Synovial cysts formation 
 Joint deformities and subluxation 
 Symmetrical small joint distribution 
 
6 What are the contraindications for DMARD? 
 
7 What are the salient features you would like to elicit from the history? 
Establish the diagnosis
 Course of disease 
 Treatment received and its side effects 
 Functional capacity 

Page 309 By Heyson Chan


Scleroderma 
Clinical Features
Hands  
 Thickening and tightening of skin over fingers 
 Sclerodactyly (finger pulp atrophy) 
 Beaking of nails 
 Atrophic nails 
 Telangiectasia (nail fold capillaries) 
 Raynaud’s phenomenon 
 Subcutaneous calcification (fingers, elbows and extensor aspect of the forearms) 
 Vitiligo and pigmentation 
Face  
 Microstomia 
 Difficulty in opening the mouth 
 Beak‐like or pinched appearance of nose 
 Blotchy telangiectasia 
Proceed to  
 Assessing hand function 
 Joint for arthralgia or arthritis 
 Abdomen for liver (primary biliary sclerosis) 
 
Conclusion
This patient (middle/old aged lady) has sclerodactyly, tightening of the skin over hands and face 
due to scleroderma. He has difficulty in buttoning his clothes and has marked dysphagia due to 
esophageal involvement. 

Page 310 By Heyson Chan


Questions
1 What other organs are involved? 
 Vascular – Raynaud’s phenomenon in 90% of patients (WBC white  blue  cyanosis); note: 
similar changes occur in internal organs, thereby causing renal crisis and pulmonary 
hypertension 
 Skin: pinched face, thin lips, radial furrows around the mouth; skin changes result in 
limitation of motion, impairment of hand function and cosmetic problem 
 Musculoskeletal: arthralgia, proximal muscle weakness, subcutaneous clacinosis 
 GI: esophageal dysmotility 
 Pulmonary: pulmonary fibrosis 
 Myocardial: myocardial fibosis 
 Renal: renal crisus 
 Weight loss, fatigue, reactive depression 
 
2 How would you investigate a patient with systemic sclerosis?  
Note: no diagnostic or specific investigations for systemic sclerosis 
 ANA present in more than 90% of patients with systemic sclerosis 
 Nail‐fold capillary microscopy to distinguish primary Raynaud’s from connective tissue 
disorder 
 
3 How would you manage the patient? 
 Education and psychological support 
 Treat vascular abnormalities such as Raynaud’s pehenomenon (with calcium channel 
blockers and other vasodilators) 
 Symptomatic treatment e.g. omepraxole for esophagitis 
 Early phase of diffuse form: immunosuppressive drugs (cyclophosphamide – for active 
interstitial lung disease, methrotrexate) 
 Later stages: anti‐fibrotic drugs (penicillamine, interferon) 
 NSAID for pericarditis; steroid for myocarditis 
 ACEI for renal crisis (any elevation of blood pressure) 
 
4 What are the phases of skin changes in scleroderma? 
 Early edematous phases (with pitting edema of hands and possible, forearms, legs and face) 
 Dermal phase 
 Atrophic phase, followed by contracture 
 Other skin changes may occur e.g. depigmentation 
 
5 What is CREST syndrome? 
 Calcinosis 
 Raynaud’s phenomenon 
 Esophageal dismobility 
 Sclerodactyly 
 Telangiectasia  
A more favorable prognosis with this subtype than systemic sclerosis 

Page 311 By Heyson Chan


Spinal Cord Compression 
Physical Examination
 Inspection: normal, patient on foley due to incontinence  
 Tone: increased 
 Power: decreased power at and below the level of the lesion 
 Reflexes: hypo‐reflexia at the level of leision and hyper‐relexia below the level of the lesion 
 Sensory: presence of a sensory level: loss of pain and temperature sensation below the level of 
the lesion and ipsilateral loss of joint position and vibration sense 
 Cerebella signs: nil 
 Gait: paraplegic  
 
Conclusion 
This patient has paraplegia with a sensory level has spinal cord compression due to (trauma). He is 
bed‐bound due to paraplegia and requires foley insertion due to incontinence. 

Page 312 By Heyson Chan


Questions 
 
1 What are the causes of spinal cord compression? 
Traumatic
 Injury to the spinal cord 
Non-traumatic
 Spinal cord compression (e.g. due to tumor) 
 Inflammatory (e.g. due to multiple sclerosis)  
 Infective  
 
2 What investigations would you like to perform? 
 XR spine 
 MRI spine of relevant level if immediately, otherwise myelogram and CT myelogram 
 Send CSF obtained during myelogram for microscopy,culture, biochemistry, Ig and cytology 
 Spinal angiogram, Vitamin B12 and folate 
 
3 How would you manage the patient with acute spinal cord syndrome? 
 Correct any compromised airway, breathing and circulation 
 
 Immobilize relevant level of spine in case of traumaticspinal cord injury or spine instability. 
 
 Initiate appropriate treatment for specific spinal cordlesions: 
 Neurosurgical / orthopaedic consultation for structural lesions 
 Antimicrobial therapy for abscess or other infections 
 Methylprednisolone 1 gm intravenously over one hour daily for 4 days, may be useful in 
non‐infectious inflammatory myelitis 
 Mono‐traumatic acute cord syndrome, start steroid within 8hrs: methylprednisolone 30 
mg/kg iv in 15 min, followed by 45‐minute pause, then 5.4 mg/kg/hr iv for 23 hrs 
 
 Institute general supportive care: 
 Proper positioning & splinting 
 Adequate hydration and nutrition 
 Bladder catheterization 
 Regular monitoring of vital signs 
 
 Close monitoring of respiratory function (FVC, respiratoryrate) in case of high cord lesions 
 
4 What is the management plan for a patient with acute cord compression due to tumor? 
 Immediate decompression by 
 Emergency surgery, or 
 Immediate radiotherapy  

Page 313 By Heyson Chan


Splenomegaly 
Physical Examination
 General inspection:  
 Signs of chronic liver disease: 
 Signs of hepatic failure: 
 Lymph nodes: nil 
 General palpation: no T/G/R 
 Liver: maybe palpable, depending on underlying cause 
 Spleen: palpable along Gardner’s line, dull on ACL, can’t get above it  
 Kidney: normal 
 Ascites: shifting dullness may be present 
 Auscultation: listen for liver bruit / venous hum; normal bowel sound 
 
Conclusion 
This patient has a sole splenomegaly, associate with stigmata of chronic liver disease, the most 
likely diagnosis is portal hypertension secondary to chronic liver disease. The patient is clinically 
stable.

Page 314 By Heyson Chan


Questions 
 
1 What are the causes of sole splenomegaly? 
Massive (beyond umbilicus)
 Myeloproliferative disorder (e.g. myelofibrosis, CML) 
 Malaria 
 Kala‐azar 
Moderate 
 Myeloproliferative disorder, e.g. myelofibrosis, chronic myeloid leukemia 
 Lymphoproliferative disorder, e.g. lymphoma, leukemia  
 Haemolytic anemia  
 Portal hypertension secondary to liver cirrhosis 
 Infective: SBE, infectious hepatitis 
 
2 How can you differentiate between a splenomegaly and an enlarged kidney? 
 
Splenomegaly Enlarged kidneys
Cannot reach over boarder  Can reach over upper boarder 
Dull on percussion  Resonant on percussion 
Moves with respiration along  Not moves with respiration 
Gardner’s line 
Notches can be felt (medial notch)  No notches 
Not ballottable  Ballottable 
 
3 How would you investigate the patient? 
 CBC w/ D/C for anemia, platelet count, white cell count 
 Liver function test for  
 Hepatitis serology 
 Ultrasound abdomen if chronic liver disease suspected 
 Bone marrow biopsy if haematological disease suspected 
 
4 What are the complications of hypersplenism? 
 Anemia 
 Low platelet count 
 Abdominal distention 

Page 315 By Heyson Chan


Stroke 
Physical Examination
 Inspection: normal  
 Tone: increased 
 Power: decreased power on affected side 
 Reflexes: hyper‐reflexia on affected side, up‐going planta on affected side 
 Sensory: maybe affected, depending on site of lesion 
 Cerebella signs: nil 
 Gait: hemiplegic gait 
 
Conclusion
This patient has hemiplegia likely to due cerebral vascular accident. He is walking with stick due to 
hemiplegia. 
I would like to listen to the carotid for bruit, check the blood pressure and urine for sugar. I would 
also like to examine the rest of the neurological system to localize the lesion. 
 
Questions
1 Define stroke. 
 Stroke is an acute focal or global disturbance of cerebral function of vascular origin lasting 
more than 24 hours. (c.f. TIA which resolves within 24 hours) 
 
2 What is your management plan for a patient presented with acute onset hemiplegia? 
Investigations
 CBC with D/C, L/RFT, TFT, random glucose, lipid profile, clotting profile 
 CXR 
 ECG 
 Urgent non‐contrast CT brain 
 Transcranial Doppler 
 Carotid duplex ultrasound 
 MR / CT angiography, if indicated 
Management 
 Admit to stroke unit 
 BP/P/T Q4H, neuro observation 
 Monitor and control hyperglycemia, but avoid correcting BP unless systolic BP >220 or 
diastolic BP > 120 (use iv labetolol in this case) 
 Nil by mouth until patient has passed the swallowing test, then allow puree diet 
 Early physio / occupational and speech therapy 
For ischemic stroke
 Start aspirin if not contraindicated 
 May consider thromoblysis if presented within 3 hours 
 Anti‐coagulate with warfarin in special cases 
 Consults neurosurgery for craniotomy if large infarct with midline shift 
For intracranial haemorrhage
 Urgent consult neurosurgery if posterior fossa or cortical haemorrhage with progressive 
deterioration 
 Avoid use of anti‐coagulants in the future 

Page 316 By Heyson Chan


3 What are the measures for secondary prevention of stroke?
 Aggressive control of blood pressure (use diurectics or ACEI) after the acute period (if no arterial 
stenosis/occlusion)
 Control DM, lipids (statin, maintain LDL <2.6) 
 Stop smoking; regular exercise 
 Antiplatelet (aspirin 80‐325mg/day or clopidogrel 75mg/day) for ischaemic stroke 
 Warfarin for atrial fibrillation (keep INR 2‐3) if not contraindicated 
 Consider endarterectomy/angioplasty if symptomatic extracranial carotid stenosis>60‐70% in a 
center with low perioperative risk 

4 What are the risk factors for stroke? 
General
 Male 
 Advanced age 
 Family history 
 Previous stroke 
Ischemic 
 Hypertension 
 Diabetes 
 Atrial fibrillation 
 Hyperlipidemia  
 Smoking 
 Ischemic heart disease 
 Carotid stenosis 
 Intracranial artery stenosis 
Heamorrhagic 
 Hypertension 
 Use of anti‐thrombotics  
 Drugs  
 Hypercholestronemia 
 Amyloid angiography 
 
5 State some clinical features of a middle cerebral artery infarct. 
 Contralateral hemiplegia (leg relatively spared)  
 Contralateral hemianopia 
 Aphasia (dominant) 
 Neglect of contralateral limb, dressing difficulty (non‐dominanat) 
   

Page 317 By Heyson Chan


6 State some localizing features of anterior and posterior circulation. 
Anterior circulation
 Monocular blindness 
 Dysphasia (Broca’s in frontal lobe, Wernicke’s in parietal‐temoral area) 
 Gerstmann syndome (finger agnosia, acalculia, agraphia – lesion in dominant parietal lobe) 
Posterior circulation
 Ataxia 
 Nystagmus 
 Vertigo 
 Cranial nerve palsy (excluding I and II and UNM VII) 
 Internuclear ophthalmoplegia 
 Horner’s syndrome 
 
7 What is Weber syndrome? 
 Ipsilateral third nerve palsy with contralateral hemiplegia 
 Indicating lesion at the mid‐brain 
 
8 What is the significance of homonyomous hemianopia in a stroke patient?  
 Indicating the lesion involvement of the optic radiation 
 Suggestive of involvement of MCA  
Note: other causes include lesions involving the optic tract, behind the lateral geniculate body 
and the occipital cortex (macular‐sparing) 
 
9 What is internuclear ophamoplegia? What are the possible casues? 
 Lesion in the medial longitudinal bundle which connects the sixth nerve on one side to the 
third nerve on the opposite side of the brainstem 
 Affected eye failed to adduct when looking towards the unaffected side while the 
unaffected eye can abduct with a horizontal nystagmus. However, eye on the affected can 
adduct when the other eye is covered. 
 Nystagmus is more prominent on the abducting eye (Harris’ sign) 
 Normal eye movement when looking towards affected side. Normal convergence. 
 Possible causes: multiple sclerosis, brainstem lesion (e.g. tumor, stroke), drugs (phenytoin, 
carbamazeoine) 
 
10 What are the symptoms suggestive of haemorrhagic stroke?  
 Association with headache and vomiting at onset 
 Rapid deterioration of consiousness 
Note: However, a CT brain is required to ascertain haemorrhagic stroke 
 
11 What is lacunar infarct? Briefly describe different types of lacunar infarct. 
 Five main types: pure motor, pure sensory, senorimotor, ataxic,  
   

Page 318 By Heyson Chan


12 What are the complications of stroke? 
 Edema (occurs 2‐3 days after infarct, peaks at 4‐5 days; osmotic agent or hyperventilation 
for temporary relieve, steroidshould NOT be used) 
 Haemorrhagic transformation of infarct (usually after large infarct, usually occur 1 week (3 
weeks for cerebellar) after infarct) 
 Seizure (occurs in 10%, need to consider anti‐epileptic after first seizure) 
 Depression 
 Vascular dementia 
 
13 What are the features of ischemic stroke on CT brain? 
 Early infarct: loss of grey‐white differentiation 
 Then, edema (loss of sulci / loss of insular ribbon) 
 Look for dense MCA sign (indicating thromboemolic event  blood clot at MCA) 
 Comment on any mid‐line shift 
Note: CT brain maybe normal in first 24‐48 hours or if the infarct is small or within the posterior 
fossa 
 
14 What are the features of hemorrhagic stroke on CT brain? 
 Irregularly shaped hyperdense lesion suggestive of acute hemorrhage (chronic hemorrhage 
is isodense)  
 Comment on the site and extent (vol = height x width x length /2) 
 Comment on any mid‐line shift 
 Comment on any surrounding edema 
 Comment on any compression to the ventricles (be specific, e.g. anterior horn of the right 
lateral ventricle) 
 Suggest the possible etiology (HT in classical area) 
 Suggest your immediate management plan 
 
15 What is your immediate management plan for a patient with atrial fibrillation, on 
warfarinpresented with hemiplegia, with CT brain confirming intracranial hemorrhage and 
INR 2.0? 
 Re‐check clotting profile 
 Stop warfarin 
 IV FFP to reverse warfarin action 
 Consult neurosurgery 
 
16 What are the common sites for hypertensive hamorragic stroke? 
 Basal ganglia 
 Cerebellar 
 Thalamic  
 Potine 

Page 319 By Heyson Chan


Transplanted Kidney 
Physical Examination
 General inspection: may be pallor, no J/C; scar in right iliac fossa 
 Previous heamodialysis: AV fistula in the arm 
 Signs of chronic renal failure: scratch marks, pigmentation, flapping tremor 
 Lymph nodes: nil 
 General palpation: no T/G/R, mass in the right iliac fossa, “kidney‐shaped”, ~5‐8 cm in diameter, 
dull on percussion, can get above the boarder 
 Liver: normal 
 Spleen: normal  
 Kidney: normal (enlarge if transplant due to CRF secondary to polycystic kidney) 
 Ascites: nil, note any edema 
 Auscultation: listen for liver bruit / venous hum; normal bowel sound 
 
Conclusion 
This patient has a transplanted kidney. The likely reason is chronic renal failure due to (adult 
polycystic kidney disease) evidenced by (bilateral ballotable kidneys). He had previously (undergone 
haemodialysis evidenced by the AV fistula in his arm). 
I would like to check the blood pressure, urine for dipsticks and look for evidence of peripheral 
neuropathy. 
   

Page 320 By Heyson Chan


Questions 
1 What are the differential diagnoses for a mass in the right iliac fossa? 
Arising from the GI system
 CA caecum (more common) 
 Crohn’s disease (common)  
 Appendix mass (rarer)  
 Ileoceacal TB (rarer) 
Arising from the gynaecological system
 Ovarian mass 
 Fibroid uterus 
Arising from the urological system
 Transplant kidney 
Arising from the skin and soft tissue
 Sebaceous cysts 
 Lipoma 
 
 
2 What are the mainstays of immunosuppresion therapy for prophylaxis against rejection in 
renal transplant patient? What are their common side effects? 
 Corticosteriod: Cushion’s, acnes, DM, HT, gastritis, osteoporosis, AVN 
 Azathioprine: bone marrow suppression, liver dysfunction and alopecia 
 Cyclosporin A: nephrotoxic, hypertension, tremor, dyslipidemia, malignancy 
Maintenance dose of these drugs should be tapered gradually, according to patient’s condition 
 
 
3 What are the complications of kidney transplant? 
Surgical
 Postoperative urinary leakage, lymphocele, hematoma 
 Renal artery thrombosis, stenosis 
 Wound infection 
 Ureteric stenosis 
Medical
 Opportunistic infections ‐Pneumocystis carinii, cytomegalovirus, herpes, tuberculosis, 
fungal  
 Drug toxicity – steroid, azathioprine, cyclosporin A 
 Rejection –hyperacute, acute and chronic 
 Metabolic ‐ dyslipidemia, Cushing’s, diabetes, obesity 
 Recurrent glomerulopathy : common ones being, IgA nephropathy, focal glomerulosclerosis 
 Cardiovascular ‐ hypertension, ischemic heart disease 
 Malignancy ‐ lymphoma, skin malignancy and Kaposi’s sarcoma 
 Bone ‐ avascular necrosis of bone, osteopenia 
 Eye ‐ cataract, glaucoma 
 

Page 321 By Heyson Chan


Tricuspid Regurgitation 
Physical Examination
 Peripheral Signs: look for needle marks, parasternal heave 
 Carotid Pulse: normal   
 JVP: elevated with prominent V wave 
 Apex Beat: normal  
 Auscultation: dual heart sound with loud P2 (if pulmonary HT); pan‐systolic murmur best heard 
at apex radiating to the axilla 
 Lung Base: basal creptitation if patient in heart failure; may be able to heard wheeze in COPD 
 
Conclusion 
This patient has tricuspid regurgitation secondary due to (chronic lung disease and cor pulmonale). 
The patient is clinically in (heart failure and is in shortness of breath). 
 
Questions 
1 What are the causes of tricuspid regurgitation? 
 Functional: pulmonary hypertension 
 Chronic rheumatic heart disease  
 Right heart endocarditis (in IVDA) 
 
2 How to differentiate between mitral regurgitation and tricuspid regurgitation? 
 
  Mitral regurgitation  Tricuspid regurgitation 
Pulse  Jerky or normal  Normal 
JVP  Normal   Prominent V wave 
Palaption  Volume overload apex  Left parasternal heave 
Auscultation  Pansystolic murmur  Pansystolic murmur, 
radiating to axilla  intensity increased with 
inspiration  
Others    Pulsatile liver 
 
 
 

Page 322 By Heyson Chan


Urinary Tract Infection 
1 What are the risk factors for urinary tract infections? 
 Uropathogen virulence factors, e.g. P fimbriated strains of E. coli a/w pyelonephritis 
 Host factors 
 Behavioral – sexual intercourse, use of spermicidal products, voiding habits 
 Genetic – epithelial cell adherence, mucosal anti‐bacterial factors 
 Biologic – estrogen status, immune response 
 Factors a/w possible occult renal or prostatic involvement 
 DM, male, elderly, pregnancy 
 Urinary tract instrumentation, childhood UTI, recent antimicrobial use 
 Factors a/w complication 
 Urinary tract obstruction 
 Functional abnormality – neurogenic bladder, VUR 
 Foreign bodies 
 Renal failure, transplant, immunosuppression 
 
2 What are the common pathogens associated with urinary tract infections? 
 Klebsiella 
 E coli (most common) 
 Enterobacters 
 Proteus (commonly associate with stone, urea splitting alkali) 
 Staphylococcus 
 
3 What is the typical presentation of lower urinary tract infection? 
 Frequency 
 Dysuria 
 Hematuria 
 
4 What is the typical presentation of upper urinary tract infection? 
 Consititutional upset 
 Fever 
 Chills, rigors 
 Loin pain 
 Urinary retention / incontinence  
 HT and impaired renal function if chronic pyelonephritis 
 
5 How would you investigatea patients with complicated urinary tract infection? 
 IVU with post‐micturating film to dectect devemental abnormalities, stones and tumors 
 MCU for vesicoureteric refluc 
 USG or MIBG for renal scarring 
 Urodynamic  study  for  cases  with  urinary  retention  and  retrograde  nephrogram  for 
suspected constriction or obstruction 
 
 
 

Page 323 By Heyson Chan


6 How would you manage a case of lower urinary tract infection? 
Uncomplicated 
 No urine culture needed;  
 Empirical treatment with 3‐day regimen: 
 Fluoroquinolone; or 
 Cotrimoxazole (if local resistance to cotrimoxazole < 20%) 
Complicated
 Need urine culture plus 7‐day regimen: 
 Pregnant 
 Possible a/w occult renal or prostatic involvement (see above) 
 With complication factors (see above) 
 For pregnant cases, avoid quinolone, repeat urine culture 2 weeks later to document 
eradication even if responded clinically  
 In all case, if symptom persists or recur in 2 weeks, repeat urine culture and treat 
accordingly by 2‐week regimen 
 

7 How would you manage a case of upper urinary tract infection? 
 Urine culture  blood culture 
 Assess for need of hospitalization 
 If not, treat with oral fluoroquinolone; if yes, treat with iv antibiotics  
(e.g. fluoroquinolone, cephalosporin, ampicillin / gentamicin) 
 If good response, need 10 to 14‐day regimen  
 If poor response, need review sensitivity and urologic evaluation 
 
8 How would manage a case of TB kidney? 
 15% TB are extra‐pulmonary and TB kidney consists 30% of all extra‐pulmonary TB  
 TB bacilli reach the kidney through blood stream 
 Inflammation and fibrosis causing stricture and narrowing of ureters 
 Patients may present with PUO or symptoms of obstruction or constitutional symptoms or 
HT or CRF 
 Diagnosis by early morning urine x3 for AFB and USG kidney 
 Treatment by conventional anti‐TB treatment for 9 months – 1 year 
 Steroid may be used to prevent possible fibrosis once the urine is clear of AFB 
 
9 What are the causes of sterile pyuria?  
 TB kidney 
 Partially treated UTI 
 
10 How would you manage recurrent uncomplicated cystitis in young women? 
 Treat acute infection 
 Consider behavioural modification, e.g. avoid spermicide, postcoital urination 
 Urologic evaluation not routinely indicated 
 Consider topical estriol if postmenopausal 
 If  3 episode in 1 year and patient desires management, consider the followings: 
1. UTI related to coitus – post‐coital antibiotics  
2. UTI not related to coitus – daily or thrice‐weekly antibiotic prophylaxis  
3. NB. In either case, stop after 6 months; consider longer course if UTI pattern recur 
 Patient‐initiated treatment for symptomatic episodes 
 Cranberry juice may also reduce the frequency of recurrent UTI 

Page 324 By Heyson Chan


 

Valvular Replacement 
Physical Examination
 Peripheral: AF (if long standing mitral valve problem); metallic heart sound may be heard 
without the stethoscope  
 JVP: not elevated 
 Carotid pulse: normal  
 Precordium: sternotomy / thoracotomy scar 
 Apex beat: displaced (if long standing mitral valve problem) 
 Auscultation: metallic S1 (MVR); S2 (AVR); systolic flow murmur may be heard 
 Lung base: clear  
 Note: presence of early diastolic murmur indicates malfunctioning aortic valve 
Proceed to the following examination 
 Oral examination for dental care 
 Limb power for stroke (complication) 
 Pressure point in lower limb for bruises (signs of over‐warfarinization) 
 Look for jaundice (hemolytic anemia or valvular failure) 
 
Conclusion
This patient with metallic first and second heart sound has double valve replacement. He is in atrial 
fibrillation, likely due to previous long‐standing mitral valve problem.  
 
Questions
1 What are the complications of metallic heart valve? 
Due to metallic heart valve
 Thromboembolism 
 Valve dysfunction 
 Haemolysis 
 Endocarditis  
Due to anti‐coagulation 
 Bleeding  
 
2 What are the advantages xenograft? 
 No need for chronic anti‐coagulation  
 
3 What are the complications of xenograft? 
 Degeneration with time 
 Calcification 
 Higher risk of higher rate of re‐operation  
 
4 What is the usual life‐span of a procine valve? 
 7 years for mitral procine prothesis 
 10 years for aortic procine prothesis 
 
5 When would you consider xenograft? 
 Patients unable to take anti‐coagulants  
 Patients not expected to live longer than the predicted lifespan of the prosthesis 

Page 325 By Heyson Chan


VSD 
Physical Examination
 Peripheral Signs: regular pulse 
 Carotid Pulse: normal   
 JVP: elevated if patient in heart failure 
 Apex Beat: thrill may be palpable at LLSB; apex beat is volume overloaded and is displaced 
 Auscultation: note any loud P2 (complicated with pulmonary hypertension) pan‐systolic 
murmur best heard at LLSB of high grade and it may be so loud that it is audible all over the 
precordium 
 Lung Base: basal creptitation if patient in heart failure 
 
Conclusion 
This patient has ventricle spetal defectlikely due to. The patient is clinically in (heart failure and is in 
shortness of breath). 

Page 326 By Heyson Chan


Questions 
 
1 What are the causes of VSD? 
 Congenital 
 Rupture of the interventricular septum as a complication of MI (emergency!) 
 
2 Where is the defect usually situated? 
 In the membranous portion of the interventricular septum 
 
3 Can VSD close spontaneously? 
 Yes. Usually occurs in small effect, in early childhood in about 50% of the patients. 
 
4 What are the complications? 
 Congestive heart failure 
 Infective endocarditis 
 Pulmonary hypertension (Eisenmenger syndrome) 
 Right ventricular outflow obstruction 
 
5 What investigations would you like to perform? 
 ECG (for severity) for left atrial and ventricular enlargement; if pulmonary hypertension 
develops, right atrial and ventricular enlargement 
 CXR (for severity) for pulmonary hypertension and LVH 
 Echocardiogram to confirm the presence and location of the VSD; Doppler ultrasound to 
determine the magnitude of shunting and the pulmonary vascular resistance 
 Cardiac catheterization and angiography can confirm the presence and location of the VSD 
as well as determine the magnitude of shunting and the pulmonary vascular resistance 
6 What is the management for patients with VSD?  
The natural history of VSD depends on the size of the defect and the pulmonary vascular 
resistance  
 Asymptomatic patient: antibiotic prophylaxis for endocarditis 
 Patients with large VSD who survive to adulthood usually have left ventricular failure or 
pulmonary hypertension with associated right heart failure. Surgical closure is 
recommended. 
 
7 What other patients need surgical closure of VSD? 
 Recurrent endocarditis 
 Development of aortic regurgitation due to prolapse of the right coronary cusp through 
septal defect 
 Progressive left ventricular dilatation due to volume overload imposed by the shunt 
 When the defect is due to an acute rupture of the ventricular spetum 

Page 327 By Heyson Chan

Potrebbero piacerti anche